Você está na página 1de 741

CAPÍTULO I.

EL CUERPO ORDENADO
DE LOS NÚMEROS
REALES

SECCIONES
A. Elementos notables en R.
B. Congruencias. Conjuntos numerables.
C. Método de inducción completa.
D. Desigualdades y valor absoluto.
E. Ejercicios propuestos.

1
A. ELEMENTOS NOTABLES EN R.

Sea S 6= ∅ un subconjunto del conjunto R de números reales.


a) Diremos que S está acotado superiormente por u, o que u es cota superior
de S, cuando x ≤ u, ∀x ∈ S.
Una cota superior u de S se llama supremo de S si ningún número
menor que u es cota superior de S, es decir cuando u es la menor de
las cotas superiores de S.
Si u es el supremo de S y u ∈ S, entonces u se llama máximo de S.
b) Análogamente a los anteriores se pueden definir los conceptos siguientes:
El conjunto S está acotado inferiormente por i, o bien i es cota inferior
de S, cuando x ≥ i, ∀x ∈ S.
Una cota inferior i de S se llama ı́nfimo de S si ningún número mayor
que i es cota inferior de S, es decir cuando i es la mayor de las cotas
inferiores de S.
Si i es el ı́nfimo de S e i ∈ S, entonces i se llama mı́nimo de S.
Una propiedad fundamental de los números reales la constituye el axioma
del supremo, el cual establece que todo subconjunto de R no vacı́o y acotado
superiormente posee supremo. De él se deduce otra condición análoga para
la existencia de ı́nfimo. Además estas propiedades son caracterı́sticas del
conjunto R.

PROBLEMA 1.1.

Hallar el supremo, ı́nfimo, máximo y mı́nimo (cuando existan) de


los siguientes conjuntos:
n sen nπ o
a) C = |n∈N .
n
n cos nπ o
b) D = |n∈N .
n

2
Solución

a) Como sen nπ = 0 si n ∈ N, entonces C = {0}, de modo que


sup C = ı́nf C = máx C = mı́n C = 0.
(
1 si n es par
b) Como cos nπ = entonces
−1 si n es impar,
D = {−1, 1/2, −1/3, 1/4, . . . } = {−1, −1/3, −1/5, . . . }∪{1/2, 1/4, 1/6, . . . }.
Luego ı́nf D = mı́n D = −1 y sup D = máx D = 1/2.

PROBLEMA 1.2.

Hallar el supremo y el ı́nfimo, cuando existan, de los siguientes


conjuntos de números reales, especificando cuáles tienen elemento
máximo o mı́nimo, es decir, cuándo el supremo o el ı́nfimo perte-
necen al conjunto.
 
1
a) A = |n∈N .
n
 
1
b) B = | n ∈ Z, n 6= 0 .
n
 
1
c) C = x | x = 0 ó x = , n ∈ N .
n
n √ o
d) D = x | 0 ≤ x < 2, x ∈ Q .

e) E = x | x2 + x + 1 ≥ 0 .


f) F = x | x2 + x − 1 < 0 .


g) G = x | x < 0, x2 + x − 1 < 0 .


Solución

a) Como A = {1, 1/2, 1/3, . . . }, es claro que


sup A = máx A = 1 e ı́nf A = 0 6∈ A,
por lo que no existe mı́n A.

3
b) B = {1, 1/2, 1/3, . . . } ∪ {−1, −1/2, −1/3, . . . } por lo que

máx B = sup B = 1 e ı́nf B = mı́n B = −1.

c) Podemos escribir C = A ∪ {0} y tenemos

máx C = sup C = 1 e ı́nf C = mı́n C = 0.


d) Como sup D = 2 6∈ D, no existe máx D. Sin embargo ı́nf D = mı́n D =
0.

−1 ± 1−4
e) Al resolver la ecuación x2 + x + 1 = 0 tenemos x = y las
2
raı́ces son imaginarias, por lo que x2 + x + 1 ≥ 0, ∀x ∈ R, es decir
E = R que no está acotado ni superior ni inferiormente.

2 −1 ± 5
f) Las raı́ces de la ecuación x +x−1 = 0 son x = . Luego la inecua-
2 √ √ !
−1 − 5 −1 + 5
ción x2 +x−1 < 0 sólo se verifica en el intervalo F = , .
2 2
Para este intervalo tenemos
√ √
−1 + 5 −1 − 5
sup F = 6∈ F e ı́nf F = 6∈ F,
2 2
por lo que no existen ni el máximo ni el mı́nimo de F .

g) Como G = F ∩ {x | x < 0} = {x | (−1 − 5)/2 < x < 0}, resulta que

−1 − 5
sup G = 0 6∈ G e ı́nf G = 6∈ G
2
y en este caso tampoco el conjunto posee máximo ni mı́nimo.

PROBLEMA 1.3.

Hallar el supremo, ı́nfimo, máximo y mı́nimo del conjunto


 
1 n
I= + (−1) | n ∈ N .
n

4
Solución

Descomponemos el conjunto como

I = {1 + 1/n | n es par} ∪ {−1 + 1/n | n es impar}


= {3/2, 5/4, 7/6, . . . } ∪ {0, −2/3, −4/5, . . . }.

De aquı́ es fácil ver que

sup I = máx I = 3/2 e ı́nf I = −1 6∈ I,

por lo que no existe el mı́nimo de I.

PROBLEMA 1.4.

Calcular el supremo y el ı́nfimo de los conjuntos


∞  
[ 1 1
A= − , .
n n
n=1
∞  
\ 1 1
B= − , .
n n
n=1
∞  
\ 1 1
C= , .
2n 2n − 1
n=1

Solución

a) Como cada intervalo (−1/n, 1/n) está contenido en el anterior, resulta


que
∞  
[ 1 1
A= − , = (−1, 1)
n n
n=1
y sup A = 1, ı́nf A = −1.
b) Como 0 es el único punto que pertenece a todos los intervalos anteriores,
tenemos que
∞  
\ 1 1
B= − , = {0}
n n
n=1
con lo que sup B = ı́nf B = 0.

5
c) Como ninguno de los intervalos [1/2n, 1/(2n − 1)] tiene puntos en común,
resulta que
∞  
\ 1 1
C= , =∅
2n 2n − 1
n=1
y no posee supremo ni ı́nfimo.

PROBLEMA 1.5.

Sean A y B dos conjuntos no vacı́os de números tales que x ≤


y, ∀x ∈ A, y ∈ B .
a) Demostrar que sup A ≤ y, ∀y ∈ B y que x ≤ ı́nf B, ∀x ∈ A.
b) Demostrar que sup A ≤ ı́nf B .

Solución

a) Sea a = sup A. Por definición, i) a ≥ x, ∀x ∈ A (a es cota superior de


A), y
ii) si a0 ≥ x, ∀x ∈ A, entonces a ≤ a0 (a es la cota superior más
pequeña de A).
Por ii), como todo y ∈ B es cota superior de A, a = sup A ≤ y.
Análogamente, si llamamos ı́nf B = b, también la definición indica que
i) b ≤ y, ∀y ∈ B y ii) si b0 ≤ y, ∀y ∈ B, entonces b ≥ b0 .
Como todo x ∈ A verifica ii), tenemos que ı́nf B ≥ x.
b) Debido a que todo elemento de A es cota inferior de B, es claro que
sup A ≤ ı́nf B puesto que ı́nf B es la máxima cota inferior.

PROBLEMA 1.6.

Sea S un subconjunto no vacı́o de R y denotamos por s a una cota


superior de S . Demostrar que

s = sup S ⇐⇒ ∀ε > 0, ∃a ∈ S : s − ε < a ≤ s.

6
Solución

=⇒: Si s = sup S, entonces ∀x ∈ S, s ≥ x y para todo ε > 0, s − ε


no es cota superior de S, porque s − ε < s. Esto quiere decir que existe
a ∈ S tal que a > s − ε. Pero también a ≤ s pues s = sup S. En resumen,
∀ε > 0, ∃a ∈ S : s − ε < a ≤ s.
⇐=: Si λ = sup S, no puede ser λ < s pues eligiendo ε < s − λ, tendrı́amos
que λ < s − ε y no podrı́a existir a ∈ S tal que s − ε < a en contra de la
hipótesis. Luego ha de ser λ ≥ s y como sup S es la mı́nima cota superior,
sup S = s.

PROBLEMA 1.7.

a) Sean A y B dos conjuntos no vacı́os con A ⊂ B . Probar que sup A ≤


sup B e ı́nf A ≥ ı́nf B .
b) Sean A y B dos conjuntos acotados superiormente, y definimos
A + B = {x + y | x ∈ A, y ∈ B}. Demostrar que sup(A + B) =
sup A + sup B , ı́nf(A + B) = ı́nf A + ı́nf B .
c) Si A = {xi }i∈I , B = {yi }i∈I y C = {xi + yi }i∈I , demostrar que
sup C ≤ sup A + sup B e ı́nf C ≥ ı́nf A + ı́nf B .

Solución

a) Si llamamos b = sup B, entonces b ≥ y, ∀y ∈ B. Por tanto b ≥ y, ∀y ∈ A,


lo que quiere decir que b es cota superior de A. Como sup A es la
mı́nima cota superior, se deduce que sup A ≤ sup B.
Análogamente, ı́nf B es una cota inferior de A y como ı́nf A es la mayor
de todas ellas, resulta que ı́nf A ≥ ı́nf B.
b) Sean a = sup A y b = sup B. Entonces a ≥ x, ∀x ∈ A y b ≥ y, ∀y ∈ B.
Por tanto a + b ≥ x + y, ∀x ∈ A, y ∈ B y a + b es cota superior de
A + B. Probemos que es la mı́nima:
Como a = sup A, por el problema anterior, ∀ε > 0, ∃a0 ∈ A : a−ε/2 <
a0 ≤ a, y como b = sup B, ∀ε > 0, ∃b0 ∈ B : b − ε/2 < b0 ≤ b.

7
Sumando miembro a miembro deducimos que existe a0 + b0 ∈ A + B
tal que a + b − ε < a0 + b0 ≤ a + b, lo que indica que a + b es la mı́nima
cota superior de A + B, es decir, sup(A + B) = a + b = sup A + sup B.
La prueba de que ı́nf(A + B) = ı́nf A + ı́nf B es análoga a la anterior,
con las modificaciones obvias.
c) Obsérvese que C = {xi + yi }i∈I es un subconjunto de A + B pues
A + B = {xi + yj | xi ∈ A, yj ∈ B}. Por los apartados a) y b) se
deduce que

sup C ≤ sup(A+B) = sup A+sup B e ı́nf C ≥ ı́nf(A+B) = ı́nf A+ı́nf B.

PROBLEMA 1.8.

Probar que R es arquimediano, es decir que verifica la propiedad:

∀a, b ∈ R, a > 0, b ≥ 0, ∃n ∈ N tal que na > b.

Solución

Si fuera falso, na ≤ b, ∀n ∈ N. Por tanto, n ≤ b/a, n = 1, 2, . . . lo que


quiere decir que N está acotado superiormente.
Por el axioma del supremo, debe existir k = sup N, lo que a su vez implica
que k − 1 no es cota superior. Entonces existe m ∈ N tal que k − 1 < m de
donde k < m + 1 y m + 1 ∈ N lo que es absurdo.

PROBLEMA 1.9.

Sean a, b ∈ R con a < b. Probar que existe r ∈ Q tal que a < r < b.

8
Solución

Si aplicamos la propiedad arquimediana de los números reales con 1 y b − a


demostrada en el problema anterior, tenemos que existe n ∈ N tal que n(b −
a) > 1.
Por una parte, si b ≥ 0, existe p ∈ N tal que p · 1/n ≥ b; llamamos m al
menor de tales enteros. Entonces
m−1 m
<b≤ .
n n
m−1 m m−1 1
Si fuera ≤ a, entonces b − a ≤ − = , lo que contradice la
n n n n
elección de n. Por tanto
m−1 m−1
a< <by ∈ Q.
n n
Por otra parte, si b ≤ 0, entonces −a > −b ≥ 0 y, por la primera parte,
existe r ∈ Q tal que −b < r < −a, es decir, a < −r < b y −r ∈ Q.
Observación. La propiedad anterior indica que, a pesar de que el conjunto
de números racionales es mucho menor que el de los reales, está uniforme-
mente distribuido entre estos, pues siempre hay algún racional comprendido
entre dos reales.

B. CONGRUENCIAS. CONJUNTOS NUMERABLES.

Recordamos el concepto de aplicación biyectiva entre conjuntos:


Una aplicación f : A → B entre dos conjuntos cualesquiera A y B es
inyectiva cuando dados dos elementos cualesquiera x1 , x2 ∈ A, se verifica
que f (x1 ) = f (x2 ) =⇒ x1 = x2 . Equivalente a la implicación anterior es
la contrarrecı́proca, es decir f es inyectiva cuando x1 6= x2 =⇒ f (x1 ) 6=
f (x2 ).
Por otra parte, la aplicación f es sobre cuando ∀y ∈ B, ∃x ∈ A tal que
f (x) = y, es decir cuando todos los elementos de B son imagen de algún
elemento de A.
Si una aplicación es a la vez inyectiva y sobre, se dice que es biyectiva.

9
Cuando una aplicación f : A → B es biyectiva, existe, y también es biyectiva,
la llamada aplicación inversa f −1 : B → A definida por f −1 (y) = x cuando
f (x) = y, para todo y ∈ B.
Decimos que dos conjuntos A y B son congruentes o equipotentes, y lo repre-
sentamos como A ∼ B, cuando existe una aplicación biyectiva f : A → B.
Es sencillo probar que la relación de congruencia ası́ definida es de equiva-
lencia.
Si llamamos cardinal de un conjunto al número de elementos o puntos que
posee, la definición anterior permite decir que dos conjuntos congruentes
tienen el mismo cardinal.
Si clasificamos los conjuntos de números reales con respecto a su cardinal,
tenemos:
- Conjunto finito es aquél cuyo cardinal es finito (tiene sólo un número finito
de elementos).
- Conjunto infinito numerable es aquel conjunto congruente con el conjunto
de números naturales. Por ejemplo, el conjunto de los números racionales es
numerable.
- Conjunto infinito no numerable es el conjunto que no es congruente con N
ni con cualquier subconjunto de N.
Un intervalo como (0, 1) no es numerable y su cardinal, llamado potencia
del continuo, se suele expresar con la letra c.

PROBLEMA 1.10.

Sean los cı́rculos concéntricos

C1 = {(x, y) | x2 + y 2 = a2 }, C2 = {(x, y) | x2 + y 2 = b2 }

con 0 < a < b. Establecer geométricamente una aplicación biyectiva


entre C1 y C2 .

Solución

Definimos la aplicación f : C2 → C1 que a cada x ∈ C2 le hace corresponder


el punto f (x) ∈ C1 de intersección del radio que va del centro 0 al punto x
con la circunferencia C1 , como se ilustra en la figura.

10
x

f (x)
O

C1

C2
Resulta que f es claramente inyectiva y sobre por lo que define una aplicación
biyectiva entre C1 y C2 .

PROBLEMA 1.11.

Demostrar que dos segmentos arbitrarios tienen tantos puntos el


uno como el otro.

Solución

Dados los segmentos AB y A0 B 0 , llamamos P al punto de intersección de


AB 0 y BA0 . A cada punto C de AB le hacemos corresponder el punto C 0 de
intersección de CP con A0 B 0 como indica la figura.
A C B

P
A0 C0 B0
La aplicación C 7→ C 0 es biyectiva pues cada punto de AB tiene una imagen
única en A0 B 0 y cada punto de A0 B 0 es la imagen de un único punto de AB.

PROBLEMA 1.12.

Demostrar las siguientes congruencias entre conjuntos:


a) [0, 1] ∼ (0, 1).
b) [0, 1] ∼ [0, 1).
c) [0, 1] ∼ (0, 1].

11
Solución

a) Si escribimos [0, 1] = {0, 1, 1/2, 1/3, . . . }∪A y (0, 1) = {1/2, 1/3, . . . }∪A,
donde A = [0, 1]\{0, 1, 1/2, 1/3, . . . } = (0, 1)\{1/2, 1/3, . . . }, podemos
definir la función f : [0, 1] → (0, 1) por

2
 si x = 0,
f (x) = 1/(n + 2) si x = 1/n, n ∈ N,

x si x 6= 0, 1/n, n ∈ N.

Esta función es biyectiva por lo que [0, 1] ∼ (0, 1).


b) La función f : [0, 1] → [0, 1) definida por
(
1/(n + 1) si x = 1/n, n ∈ N,
f (x) = es biyectiva por lo que [0, 1] ∼
x si x 6= 1/n, n ∈ N.
[0, 1).
c) Sea f : [0, 1) → (0, 1] la función definida por f (x) = 1 − x. Esta función
es biyectiva y por tanto [0, 1) ∼ (0, 1]. Como por b) [0, 1] ∼ [0, 1),
por la transitividad de la congruencia entre conjuntos se deduce que
[0, 1] ∼ (0, 1].

PROBLEMA 1.13.

Demostrar que cada uno de los intervalos [a, b], (a, b), [a, b), (a, b]
tiene la potencia del continuo, es decir tiene cardinal c.

Solución

La fórmula f (x) = a + (b − a)x define las funciones biyectivas

f : [0, 1] → [a, b], f : [0, 1) → [a, b), f : (0, 1) → (a, b) y f : (0, 1] → (a, b].

Como ya sabemos que [0, 1] ∼ [0, 1), [0, 1] ∼ (0, 1) y [0, 1] ∼ (0, 1], por la
transitividad se deduce que [0, 1] ∼ [a, b], [0, 1] ∼ (a, b), [0, 1] ∼ [a, b) y
[0, 1] ∼ (a, b].

12
PROBLEMA 1.14.

Demostrar que R tiene cardinal c.

Solución

Es fácil comprobar (ver gráfica adjunta) que la función f : (−1, 1) → R


x
definida por f (x) = es biyectiva por lo que (−1, 1) ∼ R.
1 − |x|
Como ya sabemos que (0, 1) ∼ (−1, 1), por la transitividad se deduce que
(0, 1) ∼ R, luego card (R) = c.

PROBLEMA 1.15.

Sea E = {2, 4, 6, . . . } el conjunto de los números pares. ¿Es biyec-


tiva la aplicación f : N → E definida por f (x) = 2x? ¿Es N ∼ E ?

Solución

La función f es inyectiva porque si f (x) = f (x0 ), entonces 2x = 2x0 , de


donde x = x0 .

13
También es sobre porque dado cualquier y ∈ E, existe x ∈ N tal que y = 2x,
es decir, y = f (x).
Como f es biyectiva, N ∼ E.

PROBLEMA 1.16.

¿Es numerable la sucesión {a1 , a2 , . . . } con ai 6= aj si i 6= j ?

Solución

Debido a que toda sucesión es una aplicación de N en el conjunto de sus


elementos, f (n) = an , si los an son distintos, la función es biyectiva y la
sucesión forma un conjunto numerable.

PROBLEMA 1.17.

Demostrar que A × B ∼ B × A, (A × B) × C ∼ A × (B × C), para


cualesquiera conjuntos A, B, C .

Solución

Para la demostración basta considerar las aplicaciones biyectivas siguien-


tes:
f : A × B → B × A definida por f (a, b) = (b, a), a ∈ A, b ∈ B;
f : (A × B) × C → A × (B × C) por f ((a, b), c) = (a, (b, c)), a ∈ A, b ∈ B, c ∈
C.

PROBLEMA 1.18.

Demostrar que N × N es numerable.

14
Solución

El conjunto N × N puede escribirse como una sucesión de elementos distintos


ası́: (1, 1), (2, 1), (1, 2), (1, 3), (2, 2), (3, 1), . . . según se ilustra en el diagrama.

0 1 2 3 4
Según el problema 1.16, se puede definir una biyección entre N × N y N.

PROBLEMA 1.19.

Demostrar que M × M es numerable, donde M = N ∪ {0}.

Solución

Como todo entero positivo a ∈ N puede escribirse de una sola manera en la


forma a = 2r (2s + 1) con r, s ∈ M , la función f : N → M × M definida por
f (a) = (r, s) es biyectiva. Luego M × M es numerable.

PROBLEMA 1.20.

Probar que Z es numerable.

15
Solución


0
 si n = 1,
La función f : N → Z definida por f (n) = n 0 si n = 2n0 , n0 ∈ N,

 0
−n si n = 2n0 + 1, n0 ∈ N,
(
−n/2 + 1/2 si n es impar
(que también se puede expresar como f (n) = )
n/2 si n es par
es evidentemente biyectiva, lo que muestra que Z es numerable.

PROBLEMA 1.21.

Demostrar que Q es numerable.

Solución

Llamamos Q+ al conjunto de los números racionales positivos y Q− al con-


junto de los números racionales negativos. Entonces Q = Q+ ∪{0}∪Q− .
Sea f : Q+ → N × N definido por f (p/q) = (p, q), donde expresamos el
número racional p/q como cociente de dos enteros positivos primos entre sı́.
Es inmediato comprobar que f es inyectiva y que, por tanto, Q+ es equipo-
tente a un subconjunto de N × N. Como este último conjunto es numerable,
también lo es Q+ .
Se comprueba asimismo que Q− es numerable (basta definir f (−p/q) =
(p, q)). Como Q es unión de dos conjuntos numerables y uno finito, Q es
numerable.

PROBLEMA 1.22.

Demostrar que el conjunto P de todos los polinomios p(x) = a0 +


a1 x + · · · + am xm con coeficientes enteros es numerable.

16
Solución

Para todo par de números naturales (m, n) sea P(m,n) el conjunto de los poli-
nomios p(x) = a0 +a1 x+· · ·+am xm de grado m para los que |a[ 0 |+|a1 |+· · ·+
|am | = n. Es claro que P(m,n) es finito. Como además P = P(m,n) es
(m,n)∈N×N
unión numerable de conjuntos finitos, se deduce que P es numerable.

PROBLEMA 1.23.

Un número real r se llama algebraico si es solución de una ecua-


ción polinómica p(x) = a0 + a1 x + · · · + an xn = 0 con coeficientes
enteros. Demostrar que el conjunto de los números algebraicos es
numerable.

Solución

Es evidente, según el problema anterior, que el conjunto de tales ecuaciones


polinómicas E = {p1 (x) = 0, p2 (x), p3 (x) = 0, . . . } es numerable. Si llama-
mos Ai = {x | x es solución de pi (x) = 0}, como todo polinomio de grado n
no puede
[ tener más de n raı́ces, todo Ai es finito. Por consiguiente, y como
A= Ai es unión numerable de conjuntos finitos, A es numerable.
i∈N

PROBLEMA 1.24.

Un número real se llama trascendente si no es algebraico (por


ejemplo, π y e son trascendentes). Demostrar que el conjunto de
los números trascendentes no es numerable.

Solución

Si A es el conjunto de los números algebraicos, R \ A es el conjunto de los


números trascendentes. Como R = A ∪ (R \ A), al ser A numerable y R

17
no numerable, R \ A no puede ser numerable (en caso contrario, R serı́a
numerable por ser unión de dos conjuntos numerables).

PROBLEMA 1.25.

Demostrar que un subconjunto de un conjunto numerable es finito


o numerable.

Solución

Sea A = {a1 , a2 , . . . } un conjunto numerable y sea B ⊂ A.


Si B = ∅, entonces B es finito.
Si B 6= ∅, sea an1 el primer elemento de la sucesión {a1 , a2 , . . . } tal que
an1 ∈ B. Llamamos an2 al primer elemento que sigue a an1 en la sucesión
anterior tal que an2 ∈ B. Entonces B = {an1 , an2 , . . . }. Si el conjunto de
enteros positivos {n1 , n2 , . . . } es acotado, B es finito. En caso contrario, B
es numerable.

PROBLEMA 1.26.
√ √
Demostrar que 2+ 3 no puede ser racional.

Solución

√ √ √ √
Si x = 2 + 3 =⇒ x2 = 2 + 3 + 2 6 =⇒ x√ 2 − 5 = 2 6 =⇒ x4 − 10x2 +

1 = 0, y la propia construcción prueba que 2 + 3 es raı́z de la última
ecuación.
Recordamos aquı́ el teorema de Ruffini que expresa que toda solución ra-
cional p/q de una ecuación a0 xn + a1 xn−1 + · · · + an = 0, con ai ∈ Z y
a0 6= 0, an 6= 0, verifica que p divide a an y q divide a a0 .

18
En nuestro caso, si p/q es solución racional de la ecuación de coeficientes
enteros x4 − 10x2 + 1 = 0, entonces p y q son divisores de 1. Por tanto, las
únicas soluciones racionales son 1 y -1, que no verifican la ecuación,
√ como

se puede comprobar inmediatamente. Como sabemos que x = 2 + 3 es
solución, no puede ser racional.

PROBLEMA 1.27.
√ √
Sean m y n dos números naturales primos entre sı́ y m, n
√ √
irracionales. Demostrar que también son irracionales i) m · n
√ √
y ii) m + n.

Solución

√ √
i) Si fuera m · n = p/q ∈ Q con p y q primos entre sı́, entonces m · n =
p2 /q 2 =⇒ p2 = m · n · q 2 , de modo que p2 es múltiplo de q 2 y en
consecuencia p es múltiplo de q. Como m.c.d.(p, q) = 1, sólo puede ser
√ √
q = 1 y m · n = p ∈ N.
Descomponemos ahora m, n, p en factores primos:
m = mα1 1 mα2 2 . . . mαr r , con mi primos;

n = nβ1 1 nβ2 2 . . . nβs s , con ni primos;


p = pγ11 pγ22 . . . pγt t , con pi primos;
donde mi 6= nj , ∀i, j, por ser m y n primos entre sı́.
Tenemos
√ √
q
m· n = mα1 1 . . . mαr r · nβ1 1 . . . nβs s = pγ11 . . . pγt t
=⇒ mα1 1 . . . mαr r · nβ1 1 . . . nβs s = p2γ 1 2γt
1 . . . pt .

Esto implica que cada uno de los factores del primer miembro es
igual a alguno de los factores del segundo miembro, de modo que
mα1 1 , . . . , mαr r , nβ1 1 , . . . , nβs s son cuadrados perfectos, ası́ como también
√ √
deben serlo m y n. Esto quiere decir que m y n son números na-
turales, lo que contradice la suposición inicial.

19
√ √
ii) Si fuera m + n = p/q ∈ Q, con m.c.d.(p, q) = 1, entonces, elevando
al cuadrado,

√ p2 √ p2 /q 2 − (m + n) p2 − q 2 (m + n)
m+n+2 mn = 2 =⇒ mn = = ∈Q
q 2 2q 2

pues p2 − q 2 (m + n) ∈ Z y 2q 2 ∈ Z.
√ √
Esto contradice el apartado i); luego m+ n es irracional.

C. MÉTODO DE INDUCCIÓN COMPLETA.

El método de inducción completa es un método de demostración de propie-


dades válidas en todo el conjunto N y se basa en el principio de inducción,
que dice lo siguiente:
Si S es un subconjunto de N con las dos condiciones siguientes:
i) 1 ∈ S;
ii) cada vez que k ∈ S, entonces k + 1 ∈ S;
entonces S = N.
De lo anterior se deduce que para que una propiedad P (n) que depende de
n sea cierta ∀n ∈ N, basta comprobar:
i) Que P (1) es cierta, es decir, la propiedad es cierta para n = 1.
ii) Que si suponemos P (k) cierta para cualquier k, entonces P (k + 1) es
cierta.

PROBLEMA 1.28.

Demostrar que 1 + 3 + · · · + (2n − 1) = n2 , para todo n ∈ N.

20
Solución

Para n = 1 el enunciado es cierto porque 2 · 1 − 1 = 12 .


Si suponemos que es cierto para n = k, es decir que 1+3+· · ·+(2k−1) = k 2 ,
debemos probar que también lo es para k + 1:

1+3+· · ·+(2k−1)+[2(k+1)−1] = k 2 +[2(k+1)−1] = k 2 +2k+1 = (k+1)2 ,

que corresponde precisamente a la propiedad para k + 1.

PROBLEMA 1.29.

Demostrar las siguientes fórmulas para todo n ∈ N:


n(n + 1)(2n + 1)
a) 12 + 22 + · · · + n2 = .
6
n2 (n + 1)2
b) 13 + 23 + · · · + n3 = .
4

Solución

1(1 + 1)(2 · 1 + 1)
a) Para n = 1: 12 = .
6
k(k + 1)(2k + 1)
Suponemos que 12 + 22 + · · · + k 2 = y probaremos
6
que

(k + 1)(k + 1 + 1)[2(k + 1) + 1]
12 + 22 + · · · + k 2 + (k + 1)2 = :
6
Por hipótesis,

k(k + 1)(2k + 1)
12 + 22 + . . . + k 2 + (k + 1)2 = + (k + 1)2
6
k(k + 1)(2k + 1) + 6(k + 1)2 (k + 1)[k(2k + 1) + 6(k + 1)]
= =
6 6
(k + 1)[(k + 2)(2k + 3)]
= .
6

21
12 (1 + 1)2
b)Para n = 1: 13 = .
4
k 2 (k + 1)2
Suponemos que 13 + 23 + · · · + k 3 = .
4
(k + 1)2 (k + 1 + 1)2
Probemos que 13 + 23 + · · · + k 3 + (k + 1)3 = :
4
k 2 (k + 1)2 k 2 (k + 1)2 + 4(k + 1)3
13 + 23 + · · · + k 3 + (k + 1)3 = + (k + 1)3 =
4 4
(k + 1)2 [k 2 + 4(k + 1)] (k + 1)2 (k + 2)2
= = .
4 4

PROBLEMA 1.30.

Demostrar que 3 + 2 · 31 + · · · + 2 · 3n = 3n+1 , ∀n ∈ N.

Solución

Para n = 1, 3 + 2 · 31 = 31+1 .
Suponemos que 3 + 2 · 31 + · · · + 2 · 3k = 3k+1 para algún k ∈ N.
Probemos que 3 + 2 · 31 + · · · + 2 · 3k+1 = 3k+2 :

3 + 2 · 31 + · · · + 2 · 3k+1 = 3k+1 + 2 · 3k+1 = 3 · 3k+1 = 3k+2 .

PROBLEMA 1.31.

Demostrar que para todo n > 1,

1 + 1 · 1! + 2 · 2! + · · · + (n − 1) · (n − 1)! = n!

Solución

Para n = 2: 1 + 1 · 1! = 2!
Suponemos que 1 + 1 · 1! + 2 · 2! + · · · + (k − 1) · (k − 1)! = k! y debemos
probar que 1 + 1 · 1! + 2 · 2! + · · · + (k − 1) · (k − 1)! + k · k! = (k + 1)!.

22
En efecto: 1 + 1 · 1! + 2 · 2! + · · · + (k − 1) · (k − 1)! + k · k! = k! + k · k! =
k!(1 + k) = (k + 1)!

PROBLEMA 1.32.

Demostrar que para todo n ∈ N:


1 1 1 n
a) + + ··· + = .
1·2 2·3 n · (n + 1) n+1
1 1 1 n
b) + + ··· + = .
1·3 3·5 (2n − 1) · (2n + 1) 2n + 1

Solución

1 1
a) Para n = 1: = .
1·2 1+1
1 1 1 k
Suponemos que + + ··· + = .
1·2 2·3 k · (k + 1) k+1
1 1 1 1 k+1
Probemos que + + ··· + + = :
1·2 2·3 k · (k + 1) (k + 1) · (k + 2) k+1+1
1 1 1 1 k 1
+ + ··· + + = +
1·2 2·3 k · (k + 1) (k + 1) · (k + 2) k + 1 (k + 1) · (k + 2)
k(k + 2) + 1 k 2 + 2k + 1
= =
(k + 1) · (k + 2) (k + 1) · (k + 2)
(k + 1) 2 k+1
= = .
(k + 1) · (k + 2) k+2

1 1
b) Para n = 1: = .
1·3 2·1+1
Supongamos que
1 1 1 k
+ + ··· + = ,
1·3 3·5 (2k − 1) · (2k + 1) 2k + 1
y probemos que
1 1 1 k+1
+· · ·+ + = .
1·3 (2k − 1) · (2k + 1) (2(k + 1) − 1) · (2(k + 1) + 1) 2(k + 1) + 1

23
En efecto,

1 1 1
+ ... + +
1·3 (2k − 1) · (2k + 1) (2(k + 1) − 1) · (2(k + 1) + 1)
k 1 k(2k + 3) + 1
= + =
2k + 1 (2k + 1) · (2k + 3) (2k + 1) · (2k + 3)
2
2k + 3k + 1 (2k + 1)(k + 1) k+1
= = = .
(2k + 1) · (2k + 3) (2k + 1) · (2k + 3) 2k + 3

PROBLEMA 1.33.
n
X n2 + 3n + 5
Demostrar que (1 + h + h2 ) = · n, ∀n ∈ N.
3
h=1

Solución

1
(1+h+h2 ) = 1+1+12 =
P
Se verifica para n = 1 pues el primer miembro es
h=1
12 + 3 · 1 + 5
3, y el segundo, · 1 = 3.
3
k
X k 2 + 3k + 5
Supongamos que se verifica para n = k, es decir (1 + h + h2 ) = ·k
3
h=1
y probemos que también es cierto para n = k:

k+1
X k
X
(1 + h + h2 ) = (1 + h + h2 ) + [1 + (k + 1) + (k + 1)2 ]
h=1 h=1
k2 + 3k + 5
= · k + (k + 2) + (k + 1)2
3
k 3 + 3k 2 + 5k + 3k + 6 + 3k 2 + 3 + 6k
=
3
k 3 + 6k 2 + 14k + 9
= .
3

24
Por otra parte,

(k + 1)2 + 3(k + 1) + 5 (k + 1)3 + 3(k + 1)2 + 5(k + 1)


· (k + 1) =
3 3
k 3 + 3k 2 + 3k + 1 + 3k 2 + 6k + 3 + 5k + 5
=
3
k 3 + 6k 2 + 14k + 9
= .
3

PROBLEMA 1.34.

Probar que a2n − b2n es divisible por a + b, ∀n ∈ N.

Solución

Para n = 1: a2 −b2 es divisible por a+b, porque a2 −b2 = (a+b)(a−b).


Suponemos que a2k − b2k es divisible por a + b.
Probemos que a2(k+1) − b2(k+1) es divisible por a + b:

a2(k+1) − b2(k+1) = a2k+2 − b2k+2 = a2 a2k − b2 b2k


= a2 a2k − a2 b2k + a2 b2k − b2 b2k = a2 (a2k − b2k ) + b2k (a2 − b2 ).

Por hipótesis, existe C tal que a2k − b2k = C(a + b), de modo que:

a2(k+1) − b2(k+1) = a2 (a2k − b2k ) + b2k (a2 − b2 )


= a2 C(a + b) + b2k (a − b)(a + b) = (a + b)[a2 C + b2k (a − b)],

lo que prueba la tesis.

PROBLEMA 1.35.

Demostrar que x2n−1 + y 2n−1 es divisible por x + y , ∀n ∈ N.

25
Solución

Para n = 1, x2·1−1 + y 2·1−1 = x + y, que claramente es divisible por x +


y.
Suponemos que x2k−1 + y 2k−1 es divisible por x + y, es decir, existe C tal
que x2k−1 + y 2k−1 = C(x + y).
Probemos que x2(k+1)−1 + y 2(k+1)−1 es divisible por x + y:

x2(k+1)−1 + y 2(k+1)−1 = x2k+1 + y 2k+1 = x2 x2k−1 + y 2 y 2k−1


= x2 x2k−1 − y 2 x2k−1 + y 2 x2k−1 + y 2 y 2k−1
= (x2 − y 2 )x2k−1 + y 2 (x2k−1 + y 2k−1 )
= (x − y)(x + y)x2k−1 + y 2 C(x + y)
= (x + y)[(x − y)x2k−1 + y 2 C].

PROBLEMA 1.36.

Sean a, b ∈ R fijos. Demostrar que para todo n ∈ N,

an − bn = (a − b)(an−1 + an−2 b + · · · + abn−2 + bn−1 ).

Solución

Para n = 1, a1 − b1 = a − b.
Suponemos que ak −bk = (a−b)(ak−1 +ak−2 b+· · ·+abk−2 +bk−1 ) y debemos
probar que ak+1 − bk+1 = (a − b)(ak + ak−1 b + · · · + abk−1 + bk ):

ak+1 − bk+1 = a · ak − b · bk
= a · ak − a · bk + a · bk − b · bk = a(ak − bk ) + bk (a − b)
= a(a − b)(ak−1 + ak−2 b + · · · + abk−2 + bk−1 ) + bk (a − b)
= (a − b)(ak + ak−1 b + · · · + abk−1 + bk ).

26
PROBLEMA 1.37.

Demostrar la fórmula del binomio de Newton


       
n n n n n−1 n n−1 n n
(a + b) = a + a b + ··· + ab + b .
0 1 n−1 n

Solución
   
1 1 1 1 1
Para n = 1, (a + b) = a + b = a + b.
0 1
       
k k k k k−1 k k−1 k k
Suponemos que (a + b) = a + a b + ··· + ab + b
0 1   k− 1   k 
m m m+1
y probemos la fórmula para k+1 -recordemos que + = :
n n+1 n+1

(a + b)k+1 = (a + b)(a + b)k


        
k k k k−1 k k−1 k k
= (a + b) a + a b + ··· + ab + b
0 1 n−1 k
           
k k+1 k k k k k k k k+1
= a + + a b + ··· + + ab + b
0 0 1 k−1 k k
       
k + 1 k+1 k+1 k k+1 k + 1 k+1
= a + a b + ··· + abk + b .
0 1 k k+1

PROBLEMA 1.38.

Demostrar que n7 − n es múltiplo de 42, ∀n ∈ N.

Solución

Debido a que 42 = 2 · 3 · 7, debemos probar que n7 − n es múltiplo de 2, 3


y 7.

27
Para n = 1, 17 − 1 = 0 que es evidentemente múltiplo de 42 (en realidad es
múltiplo de cualquier número).

Suponemos que k 7 − k es múltiplo de 42, es decir, k 7 − k = 42c para algún


c ∈ Z.

Probemos que (k + 1)7 − (k + 1) es divisible por 42. Por una parte,

(k + 1)7 − (k + 1) = (k + 1)[(k + 1)6 − 1]


= (k + 1)[(k + 1)3 − 1][(k + 1)3 + 1]
= (k + 1)[k 3 + 3k 2 + 3k + 1 − 1][k 3 + 3k 2 + 3k + 1 + 1]
= (k + 1)k[k 2 + 3k + 3][k 3 + 3k 2 + 3k + 2]
= (k + 1)k(k + 2)[k 2 + 3k + 3][k 2 + k + 1],

lo que da lugar a una expresión múltiplo de 2 y de 3, para cualquier valor


de k, pues en la factorización intervienen tres números naturales consecuti-
vos.

Por otra parte, si aplicamos la fórmula del binomio de Newton,

(k + 1)7 − (k + 1) = k 7 + 7k 6 + 21k 5 + 35k 4 + 35k 3 + 21k 2 + 7k + 1 − k − 1


= (k 7 − k) + 7(k 6 + 3k 5 + 5k 4 + 5k 3 + 3k 2 + k)
= 42c + 7d = 7(6c + d),

y resulta una expresión múltiplo de 7. De los dos desarrollos se obtiene el


resultado deseado.

PROBLEMA 1.39.

Demostrar que 32n+2 + 26n+1 es múltiplo de 11, ∀n ∈ N.

Solución

Para n = 1, 32·1+2 + 26·1+1 = 81 + 128 = 19 · 11.

Suponemos que 32k+2 +26k+1 es múltiplo de 11, es decir, 32k+2 +26k+1 = 11c
para algún c ∈ Z.

28
Probemos que 32(k+1)+2 + 26(k+1)+1 es múltiplo de 11:

32(k+1)+2 + 26(k+1)+1 = 32k+4 + 26k+7 = 9 · 32k+2 + 26 · 26k+1


= 9 · 32k+2 + 64 · 26k+1 = 9(32k+2 + 26k+1 ) + 55 · 26k+1
= 9 · 11 · c + 5 · 11 · 26k+1 = 11(9 · c + 5 · 26k+1 )

como se querı́a probar.

PROBLEMA 1.40.

Probar que 22n + 15n − 1 es múltiplo de 9, ∀n ∈ N.

Solución

Para n = 1 se verifica porque 22·1 + 15 · 1 − 1 = 18 = 2 · 9.


Suponemos que 22k + 15k − 1 es múltiplo de 9, es decir, 22n + 15n − 1 = 9c
para algún c ∈ Z.
Probemos que 22(k+1) + 15(k + 1) − 1 es múltiplo de 9:

22(k+1) + 15(k + 1) − 1 = 4 · 22k + 15k + 14


= 4(22k + 15k − 1) − 45k + 18 = 4 · 9c − 9 · 5k + 9 · 2,

que es múltiplo de 9.

PROBLEMA 1.41.

Sean x1 , x2 , x3 tres números naturales consecutivos. Demostrar que


x21 + x22 + x23 no es múltiplo de 3 pero x31 + x32 + x33 es múltiplo de 9.

29
Solución

Si escribimos x1 , x2 , x3 como (n − 1), n, (n + 1), tenemos


(n − 1)2 + n2 + (n + 1)2 = 3n2 + 2 que para n = 2 da como resultado 14,
que no es múltiplo de 3.
Ahora bien, probaremos por inducción que (n−1)3 +n3 +(n+1)3 = 3n3 +6n
es múltiplo de 9, para todo n ≥ 2.
Para n = 2, 3 · 23 + 6 · 2 = 36 = 9 · 4.
Suponemos que 3k 3 + 6k es múltiplo de 9, es decir, 3k 3 + 6k = 9c para algún
c ∈ Z, y debemos probar que 3(k + 1)3 + 6(k + 1) es múltiplo de 9:
3(k+1)3 +6(k+1) = 3k 3 +9k 2 +9k+3+6k+6 = 3k 3 +6k+9(k 2 +k+1) = 9c+9d,
que es evidentemente múltiplo de 9.

PROBLEMA 1.42.

Demostrar que el producto de n factores, cada uno de los cuales


es suma de los cuadrados de dos números enteros, se puede expre-
sar como la suma de los cuadrados de otros dos números enteros.

Solución

Debemos demostrar que (a21 + b21 )(a22 + b22 ) . . . (a2n + b2n ) = A2 + B 2 , para todo
n ∈ N. Para n = 1 es evidente porque a21 + b21 = A2 + B 2 eligiendo A = a1 y
B = b1 .
Suponemos que (a21 +b21 )(a22 +b22 ) . . . (a2k +b2k ) = A2 +B 2 y probemos que
(a21 + b21 )(a22 + b22 ) . . . (a2k+1 + b2k+1 ) = A21 + B12 :
(a21 + b21 )(a22 + b22 ) . . . (a2k+1 + b2k+1 ) = (A2 + B 2 )(a2k+1 + b2k+1 )
= A2 a2k+1 + A2 b2k+1 + B 2 a2k+1 + B 2 b2k+1
= (Aak+1 )2 + (Abk+1 )2 + (Bak+1 )2 + (Bbk+1 )2
= (Aak+1 )2 + (Bbk+1 )2 − 2Aak+1 Bbk+1
+(Bak+1 )2 + (Abk+1 )2 + 2Aak+1 Bbk+1
= (Aak+1 − Bbk+1 )2 + (Bak+1 + Abk+1 )2
= A21 + B12 .

30
PROBLEMA 1.43.

Demostrar que para todo n ≥ 4 se verifica n! > 2n .

Solución

Para n = 4, 4! = 24 > 24 = 16.


Suponemos que k! > 2k y probaremos que (k + 1)! > 2k+1 :

(k + 1)! = (k + 1) · k! > (k + 1) · 2k ≥ 2 · 2k = 2k+1 .

PROBLEMA 1.44.

Demostrar la desigualdad de Bernoulli (1 + x)n > 1 + nx para n =


2, 3, . . . , si x > −1, x 6= 0.

Solución

Para n = 2: (1 + x)2 = 1 + 2x + x2 > 1 + 2x, porque x 6= 0.


Supongamos que (1 +x)k > 1+kx y probemos que (1+x)k+1 > 1+(k +1)x.
En efecto,

(1 + x)k+1 = (1 + x)(1 + x)k > (1 + x)(1 + kx) = 1 + x + kx + kx2


= 1 + (k + 1)x + kx2 > 1 + (k + 1)x.

Nótese que el resultado no es cierto para n = 1, pero serı́a cierto para todo
n ∈ N si modificamos el enunciado por (1 + x)n ≥ 1 + nx.

31
D. DESIGUALDADES Y VALOR ABSOLUTO.

Es sabido que la relación ≤ permite ordenar el cuerpo de los números reales.


Además la relación es de orden total, lo que quiere decir que dados dos
números reales cualesquiera x e y, se cumple que x ≤ y o bien y ≤ x.
Por otra parte, será importante a lo largo del curso la resolución de inecua-
ciones con soluciones en R. Para ello son de utilidad las siguientes propieda-
des:
a) x < y, z ∈ R =⇒ x + z < y + z.
b) x < y, z > 0 =⇒ x · z < y · z.
c) x < y, z < 0 =⇒ x · z > y · z. (Observa que cambia el sentido de la
desigualdad).
d) x < y, y < z =⇒ x < z.
e) x < y, u < v =⇒ x + u < y + v.
Todas estas propiedades tienen sus análogas si cambiamos el signo < por
cualquiera de >, ≤ ó ≥.
Otras propiedades de las desigualdades se obtienen a partir del concepto de
valor absoluto. Ası́, dado un número real x, su valor absoluto o módulo, que
representamos por |x|, se define como:
(
√ x si x ≥ 0,
|x| = + x2 = máx{x, −x} =
−x si x < 0.
Geométricamente representa la distancia del punto x al origen de coordena-
das.
Las propiedades básicas son las siguientes:
a) −|x| ≤ x ≤ |x|.
b) |x + y| ≤ |x| + |y| (desigualdad triangular).
c) |x| ≤ a ⇐⇒ −a ≤ x ≤ a.
d) |x| ≥ a ⇐⇒ x ≥ a ó x ≤ −a.

PROBLEMA 1.45.
a+b √
Si a, b ≥ 0, demostrar que ≥ ab.
2

32
Solución

√ √
Partimos de la desigualdad evidente ( a − b)2 ≥ 0 y obtenemos sucesiva-
mente:
√ a+b √
a − 2 ab + b ≥ 0 =⇒ ≥ ab.
2
Observación. Este resultado se puede generalizar al siguiente:
a1 + · · · + an √
≥ n a1 . . . an ,
n
lo que indica que la media aritmética de los números a1 , . . . an es mayor o
igual a la media geométrica de los mismos.

PROBLEMA 1.46.

Demostrar que, para cualquier a ∈ R, a ≤ máx{1, an }, siendo n ∈ N


fijo.

Solución

Probaremos la propiedad para los distintos valores de a.


Si a > 1, máx{1, an } = an y es evidente que a ≤ an .
Si −1 ≤ a ≤ 1, máx{1, an } = 1 y a ≤ 1.
Si a < −1 y n es par, máx{1, an } = an y a ≤ an . En cambio, si n es impar,
máx{1, an } = 1 ≥ a.

PROBLEMA 1.47.

Demostrar que si a y b son números reales positivos,


p p
a > b ⇐⇒ a2 > b2 ⇐⇒ a > b.

33
Solución

i) a > b =⇒ a2 > b2 :
Supongamos que a > b. Como a > 0, a · a > b · a.
Como también b > 0, a · b > b · b. Pero a · b = b · a. Luego, a · a = a2 >
b · a > b · b = b2 , es decir, a2 > b2 .
ii) a2 > b2 =⇒ a > b:
Si a2 > b2 , entonces a2 − b2 > 0 =⇒ (a − b)(a + b) > 0. Como a, b > 0,
a + b > 0, por lo que a − b > 0, es decir a > b.
p p
iii) a > b =⇒ a > b:
p p p p
Si,
p p por el contrario,
p p fuera
p p a ≤p pb, como a, b > 0, tenemosp 2que
p pa a ≤ p 2a b y a b ≤ b b. En definitiva, a = ( a) ≤
a b ≤ ( b) = b, lo que contradice la hipótesis.
p p
iv) La demostración de que a > b =⇒ a > b es completamente análoga
a la del apartado i).

PROBLEMA 1.48.

Demostrar que la suma de cualquier número positivo con su recı́pro-


co nunca es menor que 2.

Solución

Se trata de probar que a + 1/a ≥ 2, ∀a > 0. Inecuaciones equivalentes a la


primera son

a2 + 1
≥ 2 ⇐⇒ a2 + 1 ≥ 2a ⇐⇒ a2 − 2a + 1 ≥ 0 ⇐⇒ (a − 1)2 ≥ 0
a
lo que es siempre cierto por ser el cuadrado de un número real.

34
PROBLEMA 1.49.
1 1
Si a > 0, a 6= 1 y n ∈ N, demostrar que an+1 + > an + .
an+1 an

Solución

Como en problemas anteriores vamos escribiendo inecuaciones equivalentes


a la que queremos demostrar hasta llegar a una cuya demostración sea más
asequible. Tenemos entonces:

1 1 a2(n+1) + 1 a2n + 1
an+1 + > an + ⇐⇒ >
an+1 an an+1 an
2(n+1) 2n+1
⇐⇒ a +1>a +a
⇐⇒ a2n+2 − a2n+1 − a + 1 > 0
⇐⇒ (a2n+1 − 1)(a − 1) > 0.

En esta última inecuación, si a > 1, ambos factores son positivos por lo que
su producto también lo será. En cambio si 0 < a < 1, ambos factores son
negativos por lo que su producto será también positivo. En ambos casos se
prueba lo deseado.

PROBLEMA 1.50.

Demostrar que a2 + b2 + c2 > ab + bc + ca, ∀a, b, c ∈ R excepto para


a = b = c.

Solución

Como a2 + b2 > 2ab, b2 + c2 > 2bc, c2 + a2 > 2ca, sumando miembro a


miembro las tres inecuaciones resulta 2(a2 + b2 + c2 ) > 2(ab + bc + ca) y
dividiendo por 2 obtenemos lo deseado.

35
Si a = b = c se verifica la igualdad, por lo que en general podemos escribir
a2 + b2 + c2 ≥ ab + bc + ca.

PROBLEMA 1.51.

Si a, b, c, d ∈ R son tales que a2 + b2 = 1 y c2 + d2 = 1, demostrar


que ac + bd ≤ 1.

Solución

Sabiendo que (a−c)2 ≥ 0 y (b−d)2 ≥ 0, resultan las desigualdades a2 +c2 ≥


2ac y b2 + d2 ≥ 2bd. Sumando ambas, tenemos que a2 + c2 + b2 + d2 ≥
2(ac + bd), es decir 2 ≥ 2(ac + bd) o bien 1 ≥ ac + bd.

PROBLEMA 1.52.

Demostrar que x3 + y 3 > x2 y + y 2 x si x, y ∈ R+ (x 6= y).

Solución

La inecuación x3 + y 3 > x2 y + y 2 x es equivalente a (x + y)(x2 − xy + y 2 ) >


xy(x + y). Si dividimos por x + y, que es positivo, tenemos que también es
equivalente a x2 − xy + y 2 > xy, o bien x2 − 2xy + y 2 > 0 lo cual es siempre
cierto porque x2 − 2xy + y 2 = (x − y)2 y x 6= y.

PROBLEMA 1.53.

Demostrar que an + bn > an−1 b + abn−1 siempre que a, b ∈ R+ (a 6= b)


y n > 1.

36
Solución

Desigualdades equivalentes a la que queremos probar son


an + bn > an−1 b + abn−1 ⇐⇒ an − an−1 b − (abn−1 − bn ) > 0
⇐⇒ an−1 (a − b) − bn−1 (a − b) > 0
⇐⇒ (an−1 − bn−1 )(a − b) > 0.
Esta última inecuación es siempre cierta porque, si a > b, ambos factores
son positivos y, si a < b, ambos factores son negativos.

PROBLEMA 1.54.

Demostrar:
a) x2 − y 2 > x − y si x + y > 1 y x > y .
b) x2 − y 2 < x − y si x + y > 1 y x < y .

Solución

a) Si x > y, entonces x − y > 0. De la desigualdad x + y > 1 obtenemos


(x + y)(x − y) > (x − y), es decir x2 − y 2 > x − y.

b) Como x < y, x−y < 0. Multiplicando ambos miembros de la desigualdad


x + y > 1 por x − y se invierte el sentido de la desigualdad y se tiene:
(x + y)(x − y) < (x − y) ⇐⇒ x2 − y 2 < x − y.

PROBLEMA 1.55.
a+b
Sabiendo que la media aritmética de dos números a y b es ,
2
√ 2ab
la media geométrica es ab y la media armónica es , probar
a+b
a+b √ 2ab
que > ab > , si a y b son positivos y distintos.
2 a+b

37
Solución


i) La siguiente cadena de equivalencias prueba que (a + b)/2 > ab (otro
método se realizó en el problema 1.45):
a+b √ √
> ab ⇐⇒ (a + b)2 > (2 ab)2 ⇐⇒ a2 + b2 + 2ab > 4ab
2
⇐⇒ a2 − 2ab + b2 > 0 ⇐⇒ (a − b)2 > 0.

√ 2ab
ii) Veamos ahora por el mismo método que ab > :
a+b
√ 2ab (2ab)2
ab > ⇐⇒ ab > ⇐⇒ (a + b)2 > 4ab ⇐⇒ (a − b)2 > 0.
a+b (a + b)2

PROBLEMA 1.56.

Demostrar que si a, b, c, d son números reales positivos tales que


a c a+c c
> , entonces > .
b d b+d d

Solución

Sumando a ambos miembros de la desigualdad la misma cantidad c/b, tene-


mos:
a c a c c c a+c c(b + d) a+c c
> =⇒ + > + =⇒ > =⇒ > .
b d b b d b b bd b+d d

PROBLEMA 1.57.
1 1 2
Demostrar que + > si x, y ∈ R+ (x 6= y).
x y x+y

38
Solución

Escribamos expresiones equivalentes a la inecuación a probar:


1 1 2 x+y 2
+ > ⇐⇒ > ⇐⇒ (x + y)2 > 2xy
x y x+y xy x+y
⇐⇒ x2 + 2xy + y 2 > 2xy ⇐⇒ x2 + y 2 > 0,

lo cual es evidentemente cierto, pues x 6= y.

PROBLEMA 1.58.

a2 + b2
Demostrar que ≥ ab, ∀a, b ∈ R. ¿En qué caso se verifica la
2
igualdad?

Solución

Si partimos de la desigualdad a probar, mediante expresiones equivalentes


obtenemos:
a2 + b2
≥ ab ⇐⇒ a2 + b2 ≥ 2ab ⇐⇒ a2 + b2 − 2ab ≥ 0 ⇐⇒ (a − b)2 ≥ 0,
2
que es siempre cierto.
La igualdad se verifica cuando a = b.

PROBLEMA 1.59.

Si a1 , a2 , . . . , an y b1 , b2 , . . . , bn son números reales cualesquiera, pro-


bar la desigualdad de Schwarz:

(a1 b1 + a2 b2 + · · · + an bn )2 ≤ (a21 + a22 + · · · + a2n )(b21 + b22 + · · · + b2n ).

39
Solución

Si x ∈ R, se tiene

(a1 x + b1 )2 + · · · + (an x + bn )2 ≥ 0.

Desarrollando y agrupando términos, llegamos a:

A2 x2 + 2Cx + B 2 ≥ 0,

con A2 = a21 + · · · + a2n , B 2 = b21 + · · · + b2n , C = a1 b1 + · · · + an bn .


La inecuación anterior se puede escribir, completando cuadrados, como (Ax+
C/A)2 + B 2 − C 2 /A2 ≥ 0. Si elegimos x = −C/A2 , resulta B 2 − C 2 /A2 ≥ 0,
es decir, A2 B 2 ≥ C 2 , que es la desigualdad buscada.

PROBLEMA 1.60.

Resolver las siguientes inecuaciones:


a) |x − 3| ≤ 1.
b) |x2 − 1| < 1/2.
c) (x + 1)(x − 1)(x − 2) > 0.

Solución

a) |x − 3| ≤ 1 ⇐⇒ −1 ≤ x − 3 ≤ 1 ⇐⇒ 2 ≤ x ≤ 4 ⇐⇒ x ∈ [2, 4].
b)

|x2 − 1| < 1/2 ⇐⇒ −1/2 < x2 − 1 < 1/2 ⇐⇒ 1/2 < x2 < 3/2
√ √ √
⇐⇒ 1/2 < |x|2 < 3/2 ⇐⇒ 1/ 2 < |x| < 3/ 2
√ √ √
⇐⇒ 1/ 2 < x < 3/ 2 (para x ≥ 0)
√ √ √
ó − 3/ 2 < x < −1/ 2 (para x < 0)
√ ! √ !
3 1 1 3
⇐⇒ x ∈ − √ , − √ ∪ √ ,√ .
2 2 2 2

40
c) Como (x + 1)(x − 1)(x − 2) = 0 cuando x = −1, x = 1, x = 2, podemos
escribir la siguiente tabla de signos:

x < −1 −1 < x < 1 1<x<2 x>2


x+1 – + + +
x−1 – – + +
x−2 – – – +
(x + 1)(x − 1)(x − 2) – + – +

Entonces (x + 1)(x − 1)(x − 2) > 0 cuando −1 < x < 1 ó x > 2.

PROBLEMA 1.61.

Hallar el conjunto de los valores de x para los que se verifica:


1 3
a) + ≥ 5.
x 2x
b) x(x + 2) ≤ 24.
c) |x + 2| < |x − 5|.
x x+3
d) > .
x+2 3x + 1

Solución

1 3 5x
a) + ≥ 5 ⇐⇒ 2 ≥ 5 ⇐⇒ 5x ≥ 10x2 ⇐⇒ 2x2 − x ≤ 0 ⇐⇒ x(2x − 1) ≤ 0.
x 2x 2x
Hacemos la tabla de signos de los factores, teniendo en cuenta que
x(2x − 1) = 0 ⇐⇒ x = 0 ó x = 1/2:

x<0 0 < x < 1/2 x > 1/2


x – + +
2x − 1 – – +
x(2x − 1) + – +

La solución de la inecuación es pues el intervalo (0, 1/2] (téngase en


cuenta que para x = 0 no tiene sentido la inecuación).

41
b) x(x + 2) ≤ 24 ⇐⇒ x2 + 2x − 24 ≤ 0 ⇐⇒ (x − 4)(x + 6) ≤ 0.
Como el primer miembro de la inecuación se anula en los puntos x = 4
y x = −6, la correspondiente tabla de signos es:

x < −6 −6 < x < 4 x>4


x+6 – + +
x−4 – – +
(x + 6)(x − 4) + – +

La solución de la inecuación es el intervalo cerrado [−6, 4].


c) Para resolver la inecuación |x + 2| < (
|x − 5| debemos eliminar los valores
x+2 si x + 2 ≥ 0
absolutos, sabiendo que |x + 2| = y |x − 5| =
−x − 2 si x + 2 < 0
(
x−5 si x − 5 ≥ 0
. Por ello descomponemos la recta real en los
−x + 5 si x − 5 < 0
siguientes intervalos:

x < −2 −2 ≤ x < 5 5≤x


|x + 2| −x − 2 x+2 x+2
|x − 5| −x + 5 −x + 5 x−5

y resolvemos las inecuaciones que resultan en cada intervalo.


Si x < −2: −x − 2 < −x + 5 ⇐⇒ −2 < 5 lo cual es siempre cierto. El
intervalo (−∞, −2) es solución de la inecuación.
Si −2 ≤ x < 5: x + 2 < −x + 5 ⇐⇒ 2x < 3 ⇐⇒ x < 3/2. La solución
en este caso es [−2, 3/2).
Si 5 ≤ x: x + 2 < x − 5 ⇐⇒ 2 < −5 lo cual es siempre falso.
Uniendo las soluciones parciales tenemos que la solución completa de
la inecuación es el intervalo (−∞, 3/2).
d)

x x+3 x x+3 x(3x + 1) − (x + 3)(x + 2)


> ⇐⇒ − > 0 ⇐⇒ >0
x+2 3x + 1 x + 2 3x + 1 (x + 2)(3x + 1)
2x2 − 4x − 6 2(x − 3)(x + 1)
⇐⇒ > 0 ⇐⇒ > 0.
(x + 2)(3x + 1) (x + 2)(3x + 1)

Los puntos donde puede cambiar de signo la expresión son los que
anulan numerador o denominador, es decir x = −2, −1, −1/3, 3. La

42
tabla de signos queda de la forma:

−1 −1
x < −2 −2 < x < −1 −1 < x < 3 3 <x<3 x>3
x+2 – + + + +
x+1 – – + + +
3x + 1 – – – + +
x−3 – – – – +
2(x−3)(x+1)
(x+2)(3x+1) + – + – +

La tabla indica que la solución de la inecuación es (−∞, −2)∪(−1, −1/3)∪


(3, ∞).

43
E. EJERCICIOS PROPUESTOS.

1.- Sea S un subconjunto no vacı́o de R y denotamos por i a una cota


inferior de S . Demostrar que

i = ı́nf S ⇐⇒ ∀ε > 0, ∃b ∈ S : i ≤ b < i + ε.

Sugerencia: Ver problema 1.6.

2.- Probar, por el método de inducción completa, que 34n+2 + 52n+1


es múltiplo de 14.
Sugerencia: Ver problema 1.39.

3.- Demostrar que xn − y n es divisible por x − y , ∀n ∈ N.


Sugerencia: Ver problema 1.36.

4.- Probar las siguientes propiedades de los números reales:


(a) |x − y| ≤ |x| + |y|.
(b) |x| − |y| ≤ |x − y|.
Sugerencia: Elevar al cuadrado los dos miembros de la desigualdad.

(c) |x| − |y| ≤ |x − y|.

5.- Resolver las siguientes desigualdades:


1 1
(a) + > 0.
x 1−x
Resp.: x ∈ (0, 1).

(b) 5 − x2 < −2.


√ √
Resp.: x ∈ (−∞, − 7) ∪ ( 7, ∞).

(c) x2 + x + 1 ≤ 0.
Resp.: ∅.

(d) |1 + 2x| ≤ 1.
Resp.: x ∈ [−1, 0].

(e) |x + 2| ≥ 5.

44
Resp.: x ∈ (−∞, −7] ∪ [3, ∞).

(f) |x − 5| < |x + 1|.


Resp.: x > 2.

(g) x < x2 − 12 < 4x.


Resp.: x ∈ (4, 6).


x+1
(h) x−1 ≤ 1.
Resp.: x ≤ 0.

x + y + |y − x| x + y − |y − x|
6.- Probar que máx(x, y) = y mı́n(x, y) = ,
2 2
∀x, y ∈ R.
Sugerencia: Tener en cuenta que, cuando y ≥ x, |y − x| = y − x y
máx(x, y) = y.

7.- Encontrar el error en los siguientes razonamientos:


x+1
(a) Para resolver la desigualdad ≥ 1, procedemos ası́:
x−1
x + 1 ≥ x − 1, de donde 1 ≥ −1, lo que indica que la desigualdad
inicial es válida para todos los números reales. En particular lo
será para x = −1, lo que conduce a
−1 + 1
≥ 1,
−1 − 1
es decir, 0 ≥ 1.
(b) De la desigualdad evidente 8 < 16, deducimos la siguiente
cadena de desigualdades:
1 1
> ⇐⇒ (1/2)3 > (1/2)4
8 16
⇐⇒ 3 log(1/2) > 4 log(1/2) ⇐⇒ 3 > 4.

45
CAPÍTULO II.
FUNCIONES DE
VARIABLE REAL

SECCIONES
A. Dominio e imagen de una función.
B. Representación gráfica de funciones.
C. Operaciones con funciones.
D. Ejercicios propuestos.

47
A. DOMINIO E IMAGEN DE UNA FUNCIÓN.

Una relación entre dos conjuntos X e Y de números reales que hace corres-
ponder a cada elemento ”x” del primer conjunto un solo elemento ”y” del
segundo conjunto se llama función de ”y” respecto a ”x”. Dicha relación
viene expresada por una ecuación en dos variables y = f (x).
El conjunto de números reales ”x” para los cuales la fórmula que define la
función produce valores también reales se llama dominio de la función. En
sı́mbolos,
D(f ) = {x ∈ R : ∃y ∈ R, y = f (x)}
El conjunto de valores ”y” que se obtienen como resultado de aplicar la
fórmula que define la función a los valores del dominio se llama imagen o
rango de la función.

R(f ) = {y ∈ R : ∃x ∈ D(f ), y = f (x)}

Gráficamente, el dominio corresponde a los valores del eje de abscisas (X)


en los cuales la función se puede representar.
La imagen corresponde a los puntos del eje de ordenadas (Y ) para los que
existe gráfica.

PROBLEMA 2.1.

Determinar las funciones a las que da lugar la ecuación de la


circunferencia x2 + y 2 = r2 .

Solución

La ecuación√ x2 + y 2 = r2 no corresponde a una función. Pero si escribi-


mos y = ± r2 − x2 , obtenemos dos funciones cuyo dominio es el intervalo
cerrado [−r, r] para ambas, mientras que las imágenes son diferentes: para
la primera función es [0, r] y para la segunda, [−r, 0]. Las gráficas son las
que se muestran a continuación.

48
√ √
y= r2 − x2 y = − r2 − x2

PROBLEMA 2.2.

¿Cuál (o cuáles) de las ecuaciones y = x2 , x = y 2 corresponde a


una función de y respecto a x?

Solución

Las ecuaciones y = x2 , x = y 2 representan dos parábolas. La primera de


ellas es una función pero la segunda no es función. Sin embargo, da lu-
√ √
gar a dos funciones y = x e y = − x. Las gráficas son las siguientes:

√ √
y = x2 y= x y=− x

De las gráficas se observa que el dominio de y = x2 es todo R y la imagen


√ √
el conjunto [0, ∞). El dominio de y = x e y = − x es el intervalo [0, ∞),
la imagen de la primera es también el intervalo [0, ∞) y la de la segunda
(−∞, 0].

PROBLEMA 2.3.


Encontrar el dominio y el rango de la función y = 1 − x.

49
Solución.

Para poder efectuar la raı́z, el radicando debe ser no negativo. Es decir,


tenemos que resolver la inecuación 1 − x ≥ 0. La solución es 1 ≥ x, o bien
x ∈ (−∞, 1].
El
√ rango o imagen corresponde a los posibles resultados de la operación
1 − x. Como 1 − x ≥ 0, las raı́ces de números positivos dan números
positivos y no falta ninguno. Ası́ que la imagen es el intervalo [0, ∞).

PROBLEMA 2.4.

√ 2
Encontrar el dominio y el rango de la función y = x+2 .

Solución.

De nuevo necesitamos efectuar una raı́z cuadrada, para lo cual plantearemos


la inecuación x + 2 ≥ 0. La solución es x ≥ −2, o bien, x ∈ [−2, ∞).
El
√ rango2o imagen corresponde a los posibles resultados de la operación
x + 2 . La raı́z cuadrada da resultados positivos y al elevarlos al cua-
drado el resultado también es positivo. De nuevo la imagen es el intervalo
[0, ∞).
Observación: No se puede confundir la función anterior con la función y =
x + 2, pues el dominio de esta última son todos los reales. La simplificación
de la raı́z con el cuadrado sı́ es posible pero sólo para los valores de x ≥ −2,
que son los del dominio de la función.

PROBLEMA 2.5.

1
Encontrar el dominio y el rango de la función y = .
cos(x2 )

Solución.

En este caso aparece una división, que es una operación válida para números
reales no nulos. Debemos plantear la inecuación cos(x2 ) 6= 0.

50
Como la función coseno se anula en los valores ±π/2,p ±3π/2, ±5π/2,
p . . .p
, de-
be ser x2 6= ±π/2, ±3π/2, ±5π/2, . . . , es decir, x 6= ± π/2, ± 3π/2, ± 5π/2, . . .
Todos estos valores formarán el dominio.
Como la función coseno toma valores comprendidos entre −1 y 1, la división
de 1 entre números x ∈ [−1, 1] resultan números mayores que uno, o menores
que -1.
El rango o imagen será entonces la unión de los intervalos [−∞, −1] y [1, ∞).

PROBLEMA 2.6.
r
x−3
Calcular el dominio de la función y = .
2x + 1

Solución.

En este caso aparece una división dentro de una raı́z cuadrada. Debemos
plantear dos inecuaciones, una que permita la división y otra que permita
la raı́z.
(a) 2x + 1 6= 0; (b) (x − 3)/(2x + 1) ≥ 0.
Al despejar x de (a), resulta x 6= −1/2.
Para resolver (b) es conveniente construir una tabla de signos para el nume-
rador y el denominador y operarlos de acuerdo a las reglas de productos (o
cocientes) de signos. En resumen tenemos:

x < −1/2 −1/2 < x < 3 x>3


x−3 – – +
2x + 1 – + +
(x − 3)/(2x + 1) + – +

Esto nos dice que (b) es cierto cuando x ≤ −1/2 o cuando x ≥ 3.


Reuniendo (a) y (b) queda en definitiva que el dominio de la función es la
unión de los intervalos (−∞, −1/2) y [3, ∞).

PROBLEMA 2.7.

p
Calcular el dominio de la función y = |x| − 2x.

51
Solución.

En este caso aparece un valor absoluto dentro de una raı́z cuadrada. Al


plantear la inecuación |x| − 2x ≥ 0, habrá que separarla en dos casos de
acuerdo al signo de la expresión que aparece dentro del valor absoluto.
(a) x − 2x ≥ 0 si x ≥ 0 =⇒ −x ≥ 0 si x ≥ 0 =⇒ x = 0.
(b) −x − 2x ≥ 0 si x < 0 =⇒ −3x ≥ 0 si x < 0 =⇒ x < 0.
En definitiva, la solución será la unión de los valores obtenidos en (a) y en
(b), es decir, el intervalo (−∞, 0].

PROBLEMA 2.8.

Encontrar el dominio de las funciones definidas por las siguientes


fórmulas:

a) f (x) = 1 − x2 .
p √
b) f (x) = 1 − 1 − x2 .
1 1
c) f (x) = + .
x−1 x−2
√ √
d) f (x) = 1 − x2 + x2 − 1.

e) f (x) = 2x2 − x − 1.

Solución


a) f (x) = 1 − x2 está definida cuando 1−x2 ≥ 0. Al resolver la inecuación
tenemos:

1 ≥ x2 ⇐⇒ |x|2 ≤ 1 ⇐⇒ |x| ≤ 1 ⇐⇒ −1 ≤ x ≤ 1.

En definitiva, D(f ) = {x : |x| ≤ 1} = [−1, 1].


p √ √
b) f (x) = 1 − 1 − x2 tiene sentido para 1 − 1 − x2 ≥ 0 y 1 − x2 ≥ 0.
De aquı́ obtenemos:
p p
1− 1 − x2 ≥ 0 ⇐⇒ 1 ≥ 1 − x2 ⇐⇒ 1 ≥ 1−x2 ⇐⇒ 0 ≥ −x2 ⇐⇒ x ∈ R;

1 − x2 ≥ 0 ⇐⇒ 1 ≥ x2 ⇐⇒ |x| ≤ 1.
Se obtiene de lo anterior que D(f ) = {x : |x| ≤ 1} = [−1, 1].

52
1 1
c) f (x) = + no está definida únicamente cuando x = 1 y x = 2,
x−1 x−2
por lo que D(f ) = {x : x 6= 1, x 6= 2} = R \ {1, 2}.
p p
d) f (x) = 1 − x2 + x2 − 1 está definida cuando 1−x2 ≥ 0 y x2 −1 ≥ 0,
es decir, cuando 1 ≥ |x| y |x| ≥ 1, lo que da en definitiva |x| = 1, o
bien, el conjunto {−1, 1}.
e) El dominio de la función es el conjunto de puntos para los que 2x2 − x −
1 ≥ 0. Como las raı́ces del polinomio son x = 1, x = −1/2, debemos
resolver la inecuación 2(x − 1)(x + 1/2) ≥ 0. Haciendo una tabla de
signos como en el problema 2.6 tenemos que D(f ) = (∞, −1/2]∪[1, ∞).

PROBLEMA 2.9.

Encontrar el dominio de las siguientes funciones:


√ √
a) f (x) = 1 − x + x − 2.
x2
b) f (x) = .
1 − cos x
x
c) f (x) = .
1 + x2
d) f (x) = ln(arc sen x).
ln x
e) f (x) = .
sen(ln x)
1
f) f (x) = .
x − |x|
√ 1
g) f (x) = −x + √ .
2+x

Solución

√ √
a) El dominio de f (x) = 1 − x + x − 2 será el conjunto de puntos para
los que 1 − x ≥ 0 y x − 2 ≥ 0.
Por una parte, 1 − x ≥ 0 ⇐⇒ x ≤ 1 y por otra, x − 2 ≥ 0 ⇐⇒ x ≥ 2;
la intersección de ambos conjuntos es el vacı́o. Por tanto, D(f ) = ∅.
x2
b) Si f (x) = , debe ser 1 − cos x 6= 0, es decir cos x 6= 1. Esto da
1 − cos x
lugar al conjunto D(f ) = R \ {2kπ | k ∈ Z}.

53
x
c) Como el denominador de la función f (x) = nunca se anula, el
1 + x2
dominio es todo R.
d) Si f (x) = ln(arc sen x), ha de ser arc sen x > 0 lo cual ocurre cuando
0 < x ≤ 1. Luego D(f ) = (0, 1].
ln x
e) El dominio de la función f (x) = será el conjunto intersección
sen(ln x)
de x > 0 y sen(ln x) 6= 0. Pero

sen(ln x) = 0 ⇐⇒ ln x = kπ con k ∈ Z ⇐⇒ x = ekπ .

En definitiva, D(f ) = {x | x > 0 y x 6= ekπ con k ∈ Z}.


1
f) Para que f (x) = esté definida, debe ser x − |x| 6= 0, es decir
x − |x|
x 6= |x|, lo cual ocurre si x < 0. Por tanto, D(f ) = (−∞, 0).
√ 1
g) El dominio de f (x) = −x + √ se obtiene como solución de las
2+x
inecuaciones −x ≥ 0 y 2 + x > 0. Esto da el conjunto x ≤ 0, x > −2,
es decir el intervalo (−2, 0].

B. REPRESENTACIÓN GRÁFICA DE FUNCIONES.

Se llama gráfica de una función al conjunto de puntos en el plano que verifi-


can la fórmula que define dicha función. La abscisa de los puntos corresponde
a la variable independiente y la ordenada a la variable dependiente.

G(f ) = {(x, y) ∈ R2 : x ∈ D(f ), y = f (x)}

Para dibujar una función hay que tener en cuenta la forma en que está de-
finida. Ası́, si la función es de la forma
(a) y = ax + b, será una recta y bastan dos puntos de la misma.
(b) y = ax2 + bx + c, será una parábola y se necesita el vértice y los puntos
donde corta a alguno de los ejes.
Otra forma será escribirla como y = a(x − h)2 + k y deducir su gráfica
de la de y = x2 como veremos en los ejemplos siguientes.

54
ax + b
(c) y = representa una hipérbola, la cual se podrá escribir como
cx + d
n
y =m+ y dibujarla mediante transformaciones de y = 1/x.
x+p

(d) y = |f (x)| corresponde al valor absoluto de la función f (x). Se repre-


senta la función y = f (x) y de la parte que quedó debajo del eje Y se
toman los puntos simétricos respecto a este eje.

(e) Si la función está definida de diferentes maneras en distintos inter-


valos I1 ,
. . . , In de números reales, es decir tiene la forma general
f1 (x) si x ∈ I1

f (x) = . . . habrá que dibujar por separado la fun-

fn (x) si x ∈ In ,

ción que corresponde a cada uno de los intervalos, para después reunir
sus partes.

PROBLEMA 2.10.

Dibujar la gráfica de la función f (x) = | − x + 1/4|.

Solución

En primer lugar dibujamos la gráfica de y = −x + 1/4 para después tra-


zar la gráfica simétrica respecto al eje X de la parte negativa. Resulta:

y = −x + 1/4 y = | − x + 1/4|

55
PROBLEMA 2.11.

Trazar la gráfica de las siguientes funciones:


a) f (x) = [x] (donde [x] representa el mayor entero que es menor o
igual a x).
b) f (x) = [x] − x.
c) f (x) = {x} (donde {x} es la distancia de x al entero más próximo).

Solución

a) La función [x] (llamada parte entera de x) es constante en cada intervalo


de la forma [n, n + 1) donde n es un número entero. Tenemos:

-2 -1

0 1 2 3

b) Debido al apartado anterior, [x] − x = n − x si x ∈ [n, n + 1), resultando


siempre la parte decimal del número x cambiada de signo (pues n
representa la parte entera que se resta). La gráfica se repite en todos
los intervalos [n, n + 1), con n ∈ Z, lo que quiere decir que la función
es periódica de perı́odo 1.

c) En este caso también la función es periódica y la gráfica queda de la


forma:

56
PROBLEMA 2.12.
(
x2 si |x| < 2,
Dibujar la gráfica de la función y =
1 si |x| ≥ 2.

Solución

La parábola y = x2 se debe representar en el intervalo (−2, 2) que equivale


a |x| < 2. Fuera de este intervalo, la gráfica es una recta horizontal y = 1.

PROBLEMA 2.13.

1
Dibujar la gráfica de la función y = 2 − .
x

Solución

Esta gráfica se obtiene a partir de la gráfica de y = 1/x mediante dos


transformaciones: un cambio de signo que produce una simetrı́a respecto
al eje X y la suma de dos unidades lo que origina que la gráfica suba dos
unidades respecto al eje Y . La sucesión de gráficas es la siguiente:

57
y = 1/x y = −1/x y = 2 − 1/x

PROBLEMA 2.14.

Dada la función f (x) = −(x2 − x), dibujar la gráfica de la función


y = |f (x)|.

Solución

Como | − (x2 − x)| = |x2 − x|, vamos a dibujar y = x2 − x, y después tomar


su valor absoluto.
Podemos escribir, completando cuadrados, x2 − x = (x2 − x + 1/4) − 1/4 =
(x − 1/2)2 − 1/4.
La secuencia de gráficas a dibujar será (a) y = x2 ; (b) y = (x − 1/2)2 ; (c)
y = (x − 1/2)2 − 1/4.
La parte (b) se obtiene trasladando el vértice media unidad a la derecha, y
la parte (c) bajando la gráfica 1/4 respecto al eje Y .

y = x2 y = (x − 1/2)2 y = (x − 1/2)2 − 1/4

En definitiva, tenemos:

58
y = |x2 − x|

PROBLEMA 2.15.

|x|
Representar gráficamente la función f (x) = .
1 + |x|

Solución

(
x
1+x si x ≥ 0,
Escribimos la función según el signo de x como f (x) = −x
1−x si x < 0.
x −x
Por tanto basta dibujar las funciones f1 (x) = y f2 (x) = y ob-
1+x 1−x
tener a partir de ellas la función f (x).
1 1
Si escribimos f1 (x) = 1 − y f2 (x) = 1 + , podemos representar
1+x x−1
la siguiente secuencia de gráficas:

−1 1
y = −1/x y= y =1−
1+x 1+x

59
1 1
y = 1/x y= y =1+
x−1 x−1

Reuniendo las dos gráficas en una, resulta en definitiva

|x|
f (x) =
1 + |x|

PROBLEMA 2.16.

Dibujar el conjunto de los puntos (x, y) que satisfacen las siguientes


relaciones
a) |x| + |y| = 1.
b) |x| − |y| = 1.
c) |x − 1| = |y − 1|.
d) x2 + y 2 = 0.
e) xy = 0.
f) x2 − 2x + y 2 = y .
g) x2 = y 2 .
h) x = y 2 .
i) x = |y|.

60
Solución

a) En cada uno de los cuadrantes la relación es de la forma:

x, y ≥ 0 : x + y = 1, es decir y = 1 − x;
x ≥ 0, y < 0 : x − y = 1, es decir y = x − 1;
x, y < 0 : −x − y = 1, es decir y = −1 − x;
x < 0, y ≥ 0 : −x + y = 1, es decir y = 1 + x.

Resulta en definitiva,

-1 1

-1

b) Análogamente al apartado anterior, descomponemos la ecuación según


los signos de x e y:

x, y ≥ 0 : x − y = 1, es decir y = x − 1;
x ≥ 0, y < 0 : x + y = 1, es decir y = 1 − x;
x, y < 0 : −x + y = 1, es decir y = 1 + x;
x < 0, y ≥ 0 : −x − y = 1, es decir y = −1 − x.

-1 1

61
c) En este caso descomponemos la ecuación según los signos de x − 1 e
y − 1:

x, y ≥ 1 : x − 1 = y − 1, es decir y = x;
x ≥ 1, y < 1 : x − 1 = 1 − y, es decir y = 2 − x;
x, y < 1 : 1 − x = 1 − y, es decir y = x;
x < 1, y ≥ 1 : 1 − x = y − 1, es decir y = 2 − x.

0 1 2

d) x2 + y 2 = 0 ⇐⇒ x = y = 0. La solución es el origen.
e) xy = 0 ⇐⇒ x = 0 ó y = 0. La solución está formada por los ejes de
coordenadas.
f) x2 − 2x + y 2 = y es la ecuación de una circunferencia. Para determinar
el centro y radio, la escribiremos como suma de cuadrados:
x2 − 2x + y 2 − y = 0 ⇐⇒ (x − 1)2 + (y − 1/2)2 = 1 + 1/4 = 5/4.
Se trata
√ pues de la ecuación de una circunferencia de centro (1, 1/2) y
radio 5/2.

62
g) x2 = y 2 ⇐⇒ |x| = |y|.

En el primero y tercer cuadrantes, x = y, mientras que en el segundo


y cuarto, x = −y.

h) x = y 2 es la ecuación de una parábola cuya gráfica es la siguiente:

(
x=y si y ≥ 0,
i) x = |y| ⇐⇒
x = −y si y < 0.

63
C. OPERACIONES CON FUNCIONES.

Las operaciones comunes con números reales se pueden definir para las fun-
ciones. Tenemos lo siguiente:
1. (f + g)(x) = f (x) + g(x) y D(f + g) = D(f ) ∩ D(g).
2. (f − g)(x) = f (x) − g(x) y D(f − g) = D(f ) ∩ D(g).
3. (f · g)(x) = f (x) · g(x) y D(f · g) = D(f ) ∩ D(g).
4. (f /g)(x) = f (x)/g(x) y D(f /g) = D(f ) ∩ D(g) ∩ {x : g(x) 6= 0}.
5. (f n )(x) = [f (x)]n y D(f n ) = D(f ), si n ∈ N.
Además se define la composición entre dos funciones ası́:
6. (f ◦ g)(x) = f [g(x)], siendo D(f ◦ g) = {x ∈ D(g) : g(x) ∈ D(f )}.
Cuando al componer dos funciones el resultado es la identidad (y = x), se
dice que las funciones son inversas.

PROBLEMA 2.17.

Una función f es par si f (x) = f (−x), ∀x ∈ D(f ), e impar si


f (x) = −f (−x), ∀x ∈ D(f ).
a) Determinar si f + g es par, impar o no necesariamente ninguna
de las dos cosas, en los cuatro casos obtenidos al tomar f par o
impar y g par o impar (las soluciones pueden estar conveniente-
mente dispuestas en una tabla 2 × 2).
b) Hágase lo mismo para f · g .
c) Idem para f ◦ g .
d) Demostrar que toda función par f puede escribirse de la forma
f (x) = g(|x|) para una infinidad de funciones g .

Solución

a)
g\f PAR IMPAR
PAR par ni par ni impar
IMPAR ni par ni impar impar

64
b)
g\f PAR IMPAR
PAR par impar
IMPAR impar par

c)
g\f PAR IMPAR
PAR par par
IMPAR par impar

d) Dada una función par f , basta definir g(x) = f (x) para x ≥ 0 y g


arbitraria para x < 0 para que f (x) = g(|x|).

PROBLEMA 2.18.

Determinar cuáles de las siguientes funciones son pares y cuáles


son impares:
ax + a−x
a) f (x) = .
2
√ √
b) f (x) = 1 + x + x2 − 1 − x + x2 .
1+x
c) f (x) = ln .
1−x
p p
d) f (x) = 3 (x + 1)2 + 3 (x − 1)2 .

e) f (x) = ln(x + 1 + x2 ).

Solución

a) Como f (−x) = (a−x + ax )/2 = f (x), la función es par.

b) La función es impar porque


p p
f (−x) = 1 − x + (−x)2 − 1 + x + (−x)2
p p
= 1 − x + x2 − 1 + x + x2 = −f (x).

65
1+x
c) Si escribimos f (x) = ln = ln(1 + x) − ln(1 − x), entonces
1−x
1−x
f (−x) = ln = ln(1 − x) − ln(1 + x) = −f (x).
1+x
La función es impar.
p p p p
d) Como f (−x) = 3 (−x + 1)2 + 3 (−x − 1)2 = 3 (x − 1)2 + 3 (x + 1)2 =
f (x), la función es par.
p √
e) Podemos escribir f (−x) = ln(−x + 1 + (−x)2 ) = ln(−x + 1 + x2 ).
Como
p p
f (x) + f (−x) = ln(x + 1 + x2 ) + ln(−x + 1 + x2 )
p p
= ln(x + 1 + x2 )(−x + 1 + x2 )
p 
= ln ( 1 + x2 )2 − x2 = ln 1 = 0,

se deduce que f (x) = −f (−x) y la función es impar.

PROBLEMA 2.19.

1
Sea f (x) = . Interpretar lo siguiente, averiguando los valores
1+x
de x para los que tiene sentido:
a) f (f (x))
b) f (1/x).
c) f (cx).
d) f (x + y).
e) f (x) + f (y).
f) ¿Para qué números c existe x tal que f (cx) = f (x)?

Solución

 
1 1 1 x+1
a) f (f (x)) = f = 1 = x+1+1 = .
1+x 1 + 1+x 1+x
x+2
Tiene sentido para x 6= −1 y x 6= −2.
1 1 x
b) f (1/x) = 1 = x+1 = .
1+ x x
x+1
Tiene sentido para x 6= 0 y x 6= −1.

66
1
c) f (cx) = .
1 + cx
Tiene sentido para x 6= −1/c si c 6= 0.
1
d) f (x + y) = .
1 + (x + y)
Tiene sentido cuando x + y 6= −1.
1 1 x+y+2
e) f (x) + f (y) = + = .
1+x 1+y (x + 1)(y + 1)
Tiene sentido para x 6= −1 e y 6= −1.
1 1
f) f (cx) = f (x) =⇒ = .
1 + cx 1+x
Para que tenga sentido, debe ser x 6= −1/c si c 6= 0 y x 6= −1. En esta
situación,

1 1
= =⇒ 1 + x = 1 + cx =⇒ x = cx.
1 + cx 1+x

Si c = 1, se verifica para todo x ∈ R \ {−1} y si c 6= 1, se verifica sólo


para x = 0.

PROBLEMA 2.20.


Dadas las funciones f (x) = x + 1/x y g(x) = x + 1, encontrar la
función compuesta.

Solución

A falta de información más precisa, vamos a realizar las dos composiciones


posibles dependiendo del orden en que se escriban las funciones (los resulta-
dos serán diferentes pues la composición no es conmutativa). Resulta:

√  √ 1
(f ◦ g)(x) = f x+1 = x+1+ √ ;
x+1
p
(g ◦ f )(x) = g(x + 1/x) = x + (1/x) + 1.

67
PROBLEMA 2.21.
(
0 si x ∈ Q,
Sea g(x) = x2 y sea h(x) =
1 si x ∈ R \ Q.
a) ¿Para qué valores de y es h(y) ≤ y ?
b) ¿Para cuáles es h(y) ≤ g(y)?
c) ¿Qué es g(h(z)) − h(z)?
d) ¿Para cuáles w es g(w) ≤ w?
e) ¿Para cuáles t es g(g(t)) = g(t)?

Solución

a) Si y ∈ Q, h(y) = 0; en este caso, h(y) ≤ y cuando 0 ≤ y.

Si y 6∈ Q, h(y) = 1; en este caso, h(y) ≤ y cuando 1 ≤ y.

En definitiva, h(y) ≤ y en el conjunto

{y ∈ Q : y ≥ 0} ∪ {y ∈ R \ Q : y ≥ 1}.

b) Si y ∈ Q, h(y) ≤ g(y) si y sólo si 0 ≤ y 2 , lo cual es cierto para todo y.

Si y 6∈ Q, h(y) ≤ g(y) si y sólo si 1 ≤ y 2 , es decir, |y| ≥ 1.

En definitiva, h(y) ≤ g(y) en Q ∪ {y ∈ R \ Q : |y| ≥ 1}.

c) Si z ∈ Q, g(h(z)) − h(z) = g(0) − 0 = 0.

Si z 6∈ Q, g(h(z)) − h(z) = g(1) − 1 = 1 − 1 = 0.

Luego, g(h(z)) − h(z) = 0, ∀z ∈ R.

d) g(w) ≤ w cuando w2 ≤ w, o bien w(w − 1) ≤ 0, cuya solución es [0, 1].

e) g(g(t)) = g(t) equivale a

g(t2 ) = t2 ⇐⇒ t4 = t2 ⇐⇒ t2 (t2 − 1) = t2 (t − 1)(t + 1) = 0.

La solución de esta ecuación da los puntos {−1, 0, 1}.

68
PROBLEMA 2.22.

a) Supóngase que H es una función e y un número tal que H(H(y)) =


y . ¿Cuál es el valor de H(H(H(. . . (H(y) . . . ))) (80 veces)?
b) La misma pregunta sustituyendo 80 por 81.
c) La misma pregunta de a) si H(H(y)) = H(y).

Solución

a) Por hipótesis H 2 (y) = H(H(y)) = y. Procediendo por recurrencia, ob-


tenemos sucesivamente que

H 3 (y) = H(H 2 (y)) = H(y);


H 4 (y) = H(H 3 (y)) = H(H(y)) = y;
...
2n−1
H (y) = H(y);
2n
H (y) = y.

de modo que H(H(H(. . . (H(y) . . . ))) = H 80 (y) = y.


b) Por el mismo razonamiento anterior, H 81 (y) = H(H 80 (y)) = H(y).
c) Si H 2 (y) = H(H(y)) = H(y), H 3 (y) = H 2 (y) = H(y), y ası́ sucesiva-
mente, se puede obtener por recurrencia que H n (y) = H 2 (y) = H(y)
y en particular, H 80 (y) = H(y).

PROBLEMA 2.23.

Dadas las funciones P (x) = 2x , Q(x) = x2 , R(x) = sen x, determi-


nar los siguientes valores (en cada caso la solución debe ser un
número):
a) (Q ◦ P )(y).
b) (Q ◦ R)(y).
c) (Q ◦ P ◦ R)(t) + (R ◦ P )(t).

69
Solución

a) (Q ◦ P )(y) = Q(P (y)) = Q(2y ) = (2y )2 = 22y .


b) (Q ◦ R)(y) = Q(R(y)) = Q(sen y) = sen2 y.
c) (Q ◦ P ◦ R)(t) + (R ◦ P )(t) = (QP̧)(R(t)) + R(P (t)) = Q(P (sen t)) + R(2t )
= Q(2sen t ) + sen(2t ) = (2sen t )2 + sen(2t ) = 22 sen t + sen(2t ).

PROBLEMA 2.24.

Expresar cada una de las siguientes funciones en términos de las


funciones P (x) = 2x , Q(x) = x2 , R(x) = sen x.
a) f (x) = 2sen x .
b) f (x) = sen 2x .
c) f (x) = sen x2 .
d) f (x) = sen2 x.
t
e) f (t) = 22 .
 2

f) f (u) = sen 2u + 2u .
   2sen y 
g) f (y) = sen sen sen 22 .
2 2 +sen a)
h) f (a) = 2sen a
+ sen a2 + 2sen(a .

Solución

a) f (x) = 2sen x = P (sen x) = (P Ŗ)(x) =⇒ f = P Ŗ.


b) f (x) = sen 2x = R(2x ) = (RP̧)(x) =⇒ f = RP̧.
c) f (x) = sen x2 = R(x2 ) = (RQ̧)(x) =⇒ f = RQ̧.
d) f (x) = sen2 x = Q(sen x) = (QŖ)(x) =⇒ f = QŖ.
t
e) f (t) = 22 = P (2t ) = (P P̧)(t) =⇒ f = P P̧.
 2
  2

f) f (u) = sen 2u + 2u = R 2u + 2u = R(P (u) + P (u2 ))

= R[P (u) + (P Q̧)(u)] = [R(¸P + P Q̧)](u) =⇒ f = R(¸P + P Q̧).

70
   2sen y 
g) f (y) = sen sen sen 22 = R (R (R (P (P (P (R(y)))))))

=⇒ f = RŖŖP̧P̧P̧Ŗ.
h) Como en los casos anteriores, podemos escribir
2 2 +sen a)
f (a) = 2sen a
+ sen a2 + 2sen(a = P (sen2 a) + R(a2 ) + P (sen(a2 + sen a))
= P (Q(sen a)) + R(Q(a)) + P (R(a2 + sen a))
= (P Q̧Ŗ)(a) + (RQ̧)(a) + (P Ŗ)(Q + R)(a)
=⇒ f = P Q̧Ŗ + RQ̧ + P Ŗ(¸Q + R).

PROBLEMA 2.25.

Demostrar o dar un contraejemplo de las siguientes proposicio-


nes:
a) f ◦ (g + h) = f ◦ g + f ◦ h.
b) (g + h) ◦ f = g ◦ f + h ◦ f .
1 1
c) = ◦ g.
f ◦g f
1 1
d) =f◦ .
f ◦g g

Solución

a) Si tomamos g(x) = h(x) = 1 y f una función para la que f (2) 6=


f (1) + f (1) (por ejemplo f (x) = x2 ), resulta que [f ◦ (g + h)](x) = f (2)
pero [f ◦ g + f ◦ h](x) = f (1) + f (1). La proposición es falsa.
b) [(g + h) ◦ f ](x) = (g + h)(f (x)) = g(f (x)) + h(f (x)) = (g ◦ f )(x) + (h ◦
f )(x) = (gf̧ + hf̧)(x), ∀x, por lo que la proposición es cierta.
c) Como
     
1 1 1 1
(x) = = (g(x)) = ◦ g (x),
f ◦g f (g(x)) f f
la proposición es cierta.
d) Sea g(x) = 2 y f una función para la que f (1/2) 6= 1/f (2) (cualquier
función f (x) = k con |k| =
6 1 lo cumple). Entonces
 
1 1 1
(x) = pero f ◦ (x) = f (1/2).
f ◦g f (2) g

71
La proposición es falsa.

PROBLEMA 2.26.

a) Sea f (x) = x + 1. ¿Existen funciones g tales que f ◦ g = g ◦ f ?


b) Sea f una función constante. ¿Para qué funciones g se cumple
f ◦ g = g ◦ f?
c) Supóngase que f ◦ g = g ◦ f, ∀g . Demostrar que f es la función
identidad.
d) Si f < g , ¿se cumple h ◦ f = h ◦ g ? ¿Es f ◦ h < g ◦ h?

Solución

a) Para que f ◦g = g ◦f debe ser f (g(x)) = g(f (x)), ∀x, es decir g(x)+1 =
g(x + 1). La función g debe ser solución de la ecuación g(x + 1) −
g(x) = 1. En particular es válida cualquier función g(x) = x + k con k
constante.

b) Si (f ◦ g)(x) = (g ◦ f )(x) y f (x) = k, entonces k = g(k) por lo que basta


tomar cualquier función g tal que g(k) = k.

c) Probaremos el contrarrecı́proco, es decir, si f 6= identidad, entonces


existe g tal que f ◦ g 6= g ◦ f :

f 6= identidad =⇒ ∃a, a0 (a 6= a0 ) : f (a) = a0 . Entonces (g ◦ f )(a) =


g(a0 ) y (f ◦ g)(a) = f (g(a)).

Elegimos g tal que g(a) = g(a0 ) = a. De este modo, f (g(a)) = f (a) =


a0 y g(f (a)) = g(a0 ) = a. Como a 6= a0 , entonces g ◦ f 6= f ◦ g, como
querı́amos probar.

d) Si f (x) < g(x), no tiene por qué cumplirse que h(f (x)) = h(g(x)) (basta
definir h = identidad).

Sin embargo, como f (h(x)) < g(h(x)), ∀h(x), sı́ es cierto que f ◦ h <
g ◦ h.

72
PROBLEMA 2.27.

Hallar la función inversa de f en los siguientes casos:


a) f (x) = 2x + 3
b) f (x) = ln(x/2).
c) f (x) = x2 − 1.
d) f (x) = arc tg 3x.

e) f (x) = 3 1 − x3 .
(
x si x ≤ 0,
f) f (x) = 2
x si x > 0.
¿En qué campos estarán definidas estas funciones inversas?

Solución

Utilizaremos en todos los casos la equivalencia y = f (x) ⇐⇒ x = f −1 (y).


y−3
a) f (x) = 2x + 3 ⇐⇒ y = 2f −1 (y) + 3 ⇐⇒ f −1 (y) = .
2
La función está definida en todo el campo real.

x f −1 (y) f −1 (y)
b) f (x) = ln ⇐⇒ y = ln ⇐⇒ ey = ⇐⇒ f −1 (y) = 2ey .
2 2 2
El dominio de definición es todo R.

c) f (x) = x2 − 1 ⇐⇒ y = [f −1 (y)]2 − 1 ⇐⇒ f −1 (y) = y + 1.
Está definida cuando y + 1 ≥ 0, es decir en [−1, ∞).
tg y
d) f (x) = arc tg 3x ⇐⇒ y = arc tg 3f −1 (y) ⇐⇒ f −1 (y) = .
3
Esta función está definida en R \ {y | cos y = 0} = R \ {(2k + 1)π/2 :
k ∈ Z}.

e) f (x) = 3 1 − x3 ⇐⇒ y = 3 1 − f −1 (y)3 ⇐⇒ y 3 = 1 − f −1 (y)3
p
p
⇐⇒ f −1 (y)3 = 1 − y 3 ⇐⇒ f −1 (y) = 3 1 − y 3 . La
función está definida en todo R.
(
x si x ≤ 0,
f) f (x) = 2
x si x > 0.

Si x ≤ 0, como y = x, también y ≤ 0 y f −1 (y) = y.

73
Si x > 0, como y = x2 también y > 0 y [f −1 (y)]2 = y es decir,

f −1 (y) = y.
(
y si y ≤ 0,
En definitiva, f −1 (y) = √ y el dominio es todo R.
y si y > 0,

PROBLEMA 2.28.

ax + b
Sea f (x) = , x 6= a/c. Probar que f −1 (x) = f (x).
cx − a

Solución

Para calcular la inversa de una función, despejamos x de la ecuación que


ax + b
define dicha función. Ası́ pues, de y = obtenemos
cx − a
y(cx − a) = ax + b =⇒ cyx − ay = ax + b =⇒ x(cy − a) = ay + b
ay + b ax + b
=⇒ x = =⇒ f −1 (x) = =⇒ f −1 (x) = f (x).
cy − a cx − a

74
D. EJERCICIOS PROPUESTOS.

1.- Determina de las siguientes ecuaciones cuáles corresponden a una


función. En caso de no serlo, encuentra las fórmulas de todas las
funciones a que dan lugar las ecuaciones correspondientes. En
cualquier caso encuentra el dominio y la imagen. (Se recomienda
dibujar la gráfica).
a) x · y = 1.
Resp.: Sı́ ; y = 1/x ; R − {0}; R − {0}.

b) x2 · y 2 = 1.
Resp.: No; y = 1/x, y = −1/x; R − {0}, R − {0}; R − {0}, R − {0}.


c) xy = 1.
Resp.: Sı́ ; y = 1/x; R − {0}; R − {0}.

2.- En los siguientes ejercicios se dan las ecuaciones de


p ciertas fun-

ciones. Encontrar su dominio y su rango. a) y = 1 + x.

Resp.: [0, ∞); [1, ∞).

b) y = cos 1/x.
Resp.: R − {0}; [−1, 1].

|x|
c) y = .
1 + |x|
Resp.: R; [0, 1).

p
3.- Encontrar el dominio de las siguientes funciones: a) f (x) = |x + 2| + x.

Resp.: [−1, ∞).

1
b) f (x) = √ .
3− x+5
Resp.: [−5, 4) ∪ (4, ∞).

75
r
x
c) f (x) = .
(x − 1)(x + 2)
Resp.: (−2, 0] ∪ (1, ∞).

4.- Si el dominio de la función f (x) es el intervalo [a, b], ¿cuál es el


dominio de la función g(x) = f (mx + n) con m > 0?
Resp.: D(g) = [(a − n)/m, (b − n)/m].

5.- Dibujar la gráfica de las siguientes funciones:


a) y = −|x| − 1.

Resp.:

b) y = |x − 1| + |x + 1|.
Resp.:

c) y = x − 1 + |4 − x2 |.

76
Resp.:

1
d) y = .
|x − 3| + |x + 4|
Resp.:

x−1
e) y = .
8x
Resp.:

6.- Dibujar la gráfica de las siguientes funciones:


a) y = signo(x2 − 4).

77
Resp.:

b) y = [1/x].
Resp.:

c) y = 3 − [3x].
Resp.:

p
d) y = x − [x].

78
Resp.:

7.- Sea la función f (x) = x2 . Representar la función y = f (x + 2) − 1.

Resp.:

8.- Dibujar la gráfica de la función



2x − 1
 si x < −1,
f (x) = x2 − 1 si − 1 ≤ x ≤ 2,

3 si x > 2.

Resp.:

9.- Dada la gráfica de f (x), dibujar f (x) + 2, 2f (x), 2 − f (x), [f (x)]:

79
y = f (x)

-2 1

Resp.:

3 y = f (x) + 2

2 y = 2f (x)

-2 1 -2 1

y = [f (x)]
y = 2 − f (x)

-2 1 -2 1

10.- Sea la función f (x) definida ası́:


−3x
 si x < 0,
f (x) = −x2 + 2x si 0 ≤ x ≤ 2,

1 si x > 2.

Dibujar la gráfica de f (x) y la de 2 − f (x).

80
Resp.:

11.- En el intervalo −4π ≤ x ≤ 4π , dibujar las siguientes gráficas:


a) f (x) = cos x.

b) f (x) = cos 2x.

c) f (x) = | cos x|.

d) f (x) = cos(x/2).

81
12.- ¿Cuál es la relación entre las siguientes funciones?
x − 2x2
a) f (x) = .
x
b) g(x) = 1 − 2x.
x2 − 2x3
c) h(x) = .
x2

d) i(x) = 1 − 4x + 4x2 .
(x3 + x)(1 − 2x)
e) j(x) = .
x(1 + x2 )
Resp.: f = h = j, |g| = i.

13.- La misma pregunta anterior para las funciones:


a) f (x) = |x|.

b) g(x) = x2 .
c) h(x) = x· signo x.
Resp.: Son todas iguales.

1 √
14.- Dadas f (x) = √ , g(x) = x − 1, probar que (f + g)(x) = 0
x−1
no tiene solución en su dominio.
x
Resp.: (f + g)(x) = √ .
x−1

15.- Construir una función polinómica de grado 3, sabiendo que es


simétrica respecto al origen y que pasa por (1, 0) y (-1, 0).
Resp.: f (x) = ax3 − ax, a 6= 0

16.- Dadas las funciones f (x) = −(x2 − x), g(x) = | − x + 1/4|, hallar
1
√ y su dominio.
g f̧
p 1
Resp.: (1/ gf̧)(x) = ; Dom = R − {1/2}.
|x − 1/2|


17.- Dadas f (x) = x, g(x) = x − 1,
(a) hallar f ◦ g y dibujarla.
(b) Idem para g ◦ f .

82
(c) Idem para f −1 (si existe).
√ √
Resp.: (f ◦ g)(x) = x − 1; (g ◦ f )(x) = x − 1; (f −1 )(x) = x2 para
x ≥ 0.

x−1 √
18.- Dadas f (x) = , g(x) = x,
x
(a) Calcular (f ◦ g)(x).
(b) Calcular (g ◦ g)(9).
(c) Probar que f (x) · f (1 − x) = 1.

x−1 √
Resp.: (f ◦ g)(x) = √ ; (g ◦ g)(9) = 3.
x

19.- Hallar la función inversa de f en los siguientes casos:


a) f (x) = (x − 1)3 .

Resp.: f −1 (x) = 3 x + 1.

(
x si x es racional,
b) f (x) =
−x si x es irracional.

Resp.: f −1 (x) = f (x).

c) f (x) = x + [x].
Resp.: f −1 (x) = x − n cuando x ∈ [2n, 2n + 1), ∀n ∈ Z.

x
d) f (x) = si −1 < x < 1.
1 − x2

−1 −1 + 1 + 4x2
Resp.: f (x) = .
2x

ax + b
20.- ¿Para qué valores de los parámetros a, b, c, d la función f (x) =
cx + d
es inversa de sı́ misma?
Resp.: a = −d.

21.- ¿En qué intervalo tiene inversa la función f (x) = y = x + |2 − x|.


Resp.: (2, ∞).

83
22.- Probar que si f es una función creciente, también lo es f −1 .

23.- Demostrar que si f y g son inyectivas, también lo es (f ◦ g) y


comprobar que (f ◦ g)−1 = g −1 ◦ f −1 .

84
CAPÍTULO III.
LÍMITES DE
FUNCIONES

SECCIONES
A. Definición de lı́mite y propiedades básicas.
B. Infinitésimos. Infinitésimos equivalentes.
C. Lı́mites infinitos. Ası́ntotas.
D. Ejercicios propuestos.

85
A. DEFINICIÓN DE LÍMITE Y PROPIEDADES BÁSICAS.

Un número real L se dice lı́mite de una función y = f (x) en un punto x = c


si los valores de la función se van acercando a L cuando x toma valores cada
vez más próximos a c. Simbólicamente se expresa por:

lı́m f (x) = L ⇐⇒ ∀ε > 0, ∃δ > 0 tal que |f (x) − L| < ε si 0 < |x − c| < δ.
x→c

Debemos tener en cuenta que no importa en este caso el comportamiento


de la función en el punto c; puede incluso no estar en el dominio. Lo que
sı́ debe ocurrir es que todos los puntos próximos a c estén en el dominio y
sus imágenes estén cada vez más cerca de L.
Análogamente, se dice que una función f tiene lı́mite infinito en x = c, y se
escribe como lı́m f (x) = ∞, cuando
x→c

∀M > 0, ∃δ > 0 tal que f (x) > M si 0 < |x − c| < δ.

Si únicamente interesa aproximarse a c por la derecha de él (es decir, para


valores mayores que c), se hablará de lı́mite lateral por la derecha, y análoga-
mente de lı́mite lateral por la izquierda (para valores x < c). Las notaciones
que se usarán son las de lı́m f (x) y lı́m f (x), respectivamente.
x→c+ x→c−

Un caso particular de lı́mites laterales son los lı́mites al infinito, es decir los
casos en que x = ±∞. Ası́ decimos que lı́m f (x) = L, cuando
x→∞

∀ε > 0, ∃k > 0 tal que |f (x) − L| < ε si x > k.

Los lı́mites conservan las operaciones básicas de funciones siempre que dichas
operaciones sean posibles en el punto donde se está calculando el lı́mite.

PROBLEMA 3.1.

Calcular lı́m 3(2x − 1)(x + 1)2 .


x→2

Solución

Basta sustituir el punto x = 2 en la función. Resulta que

lı́m 3(2x − 1)(x + 1)2 = 3(4 − 1)(3)2 = 81.


x→2

86
PROBLEMA 3.2.

3(2x − 1)
Calcular lı́m .
x→−1 (x + 1)2

Solución

Al intentar sustituir en la función el punto x = −1, se anula el denominador.


Esto quiere decir que cuanto más nos aproximamos a −1, más grande es el
cociente. Por eso el lı́mite es infinito (∞).

PROBLEMA 3.3.

x2 + x − 2
Calcular lı́m .
x→−2 x2 − 4

Solución

La situación es parecida al problema anterior. Sin embargo el numerador


también se anula en x = −2.
No podemos asegurar que el cociente se hace más grande cuando x se acerca a
−2. Pero si factorizamos numerador y denominador, podemos escribir

x2 + x − 2 (x + 2)(x − 1) x−1 −3
lı́m 2
= lı́m = lı́m = = 3/4.
x→−2 x −4 x→−2 (x + 2)(x − 2) x→−2 x − 2 −4

PROBLEMA 3.4.
√ √
x + 2 − 2x
Calcular lı́m .
x→2 x−2

87
Solución

También la situación es similar pero antes de factorizar debemos eliminar


las raı́ces del numerador, es decir, debemos racionalizar. Nos queda:
√ √ √ √
( x + 2 − 2x)( x + 2 + 2x) (x + 2) − 2x
L = lı́m √ √ = lı́m √ √
x→2 (x − 2)( x + 2 + 2x) x→2 (x − 2)( x + 2 + 2x)
−x + 2 −1
= lı́m √ √ = = −1/4.
x→2 (x − 2)( x + 2 + 2x) 2+2

PROBLEMA 3.5.

Resolver lı́m ([x] − x).


x→4

Solución

Como la función parte entera es escalonada, puede tomar diferentes valores


a la derecha y a la izquierda del punto x = 4. Debemos calcular los lı́mites
laterales separadamente.

lı́m ([x] − x) = lı́m (3 − x) = 3 − 4 = −1.


x→4− x→4−
lı́m ([x] − x) = lı́m (4 − x) = 4 − 4 = 0.
x→4+ x→4+

Al ser distintos los lı́mites laterales en x = 4, no existe el lı́mite de la función


en el punto.

PROBLEMA 3.6.

x2 − 2x
Calcular lı́m .
x→2 x2 − 4x + 4

Solución

Como el numerador y denominador tienden a cero, debemos factorizar ambos


y simplificar. Tenemos:
(x − 2)x x
L = lı́m = lı́m = ∞.
x→2 (x − 2)(x − 2) x→2 x − 2

88
PROBLEMA 3.7.

(x + h)3 − x3
Calcular lı́m .
h→0 h

Solución

También en este caso se anulan el numerador y el denominador. Desarrolla-


mos primero la potencia y luego simplificamos:
(x3 + 3x2 h + 3xh2 + h3 ) − x3 h(3x2 + 3xh + h2 )
L = lı́m = lı́m
h→0 h h→0 h
2 2 2
= lı́m (3x + 3xh + h ) = 3x .
h→0

PROBLEMA 3.8.

x − (n + 1)xn+1 + nxn+2
Calcular lı́m .
x→1 (1 − x)2

Solución

Si factorizamos el numerador como x−(n+1)xn+1 +nxn+2 = (x−1)2 [nxn +


(−1 + n)xn−1 + (−2 + n)xn−2 + · · · + x], tenemos:

(x − 1)2 [nxn + (−1 + n)xn−1 + (−2 + n)xn−2 + · · · + x]


L = lı́m
x→1 (1 − x)2
n n−1
= lı́m [nx + (−1 + n)x + (−2 + n)xn−2 + · · · + x]
x→1
n(n + 1)
= n + (n − 1) + · · · + 2 + 1 = .
2

PROBLEMA 3.9.
√ √
x2 − 2x + 6 − x2 + 2x − 6
Calcular lı́m .
x→3 x2 − 4x + 3

89
Solución

En primer lugar debemos racionalizar el numerador multiplicando y divi-


diendo la expresión por el conjugado de dicho numerador para después fac-
torizar y simplificar la expresión. Resulta:
√ √ √ √
( x2 − 2x + 6 − x2 + 2x − 6)( x2 − 2x + 6 + x2 + 2x − 6)
L = lı́m √ √
x→3 (x2 − 4x + 3)( x2 − 2x + 6 + x2 + 2x − 6)
(x2 − 2x + 6) − (x2 + 2x − 6)
= lı́m √ √
x→3 (x2 − 4x + 3)( x2 − 2x + 6 + x2 + 2x − 6)

−4(x − 3)
= lı́m √ √
x→3 (x − 3)(x − 1)( x2 − 2x + 6 + x2 + 2x − 6)
−4 −4 −1
= lı́m √ √ = = .
x→3 (x − 1)( x2 − 2x + 6 + x2 + 2x − 6) 2(3 + 3) 3

PROBLEMA 3.10.
p p
3
x2 − 2 3 x + 1
Calcular lı́m .
x→1 x−1

Solución

El numerador es un cuadrado perfecto y para racionalizarlo utilizamos la



fórmula a3 − b3 = (a − b)(a2 + ab + b2 ) donde en este caso a = 3 x y b = 1.
Resulta:
p p p p
( 3 x − 1)2 ( 3 x − 1)2 ( 3 x2 + 3 x + 1)2
L = lı́m = lı́m p p
x→1 x−1 x→1 (x − 1)( 3 x2 + 3 x + 1)2

(x − 1)2 x−1
= lı́m p p = lı́m p p = 0.
x→1 (x − 1)( x2 +
3 3
x + 1)2 x→1 ( x2 + 3 x + 1)2
3

PROBLEMA 3.11.

3
x−1
Calcular lı́m √ .
x→1 4
x−1

90
Solución

En este caso racionalizamos el numerador y el denominador utilizando la


fórmula general ap − bp = (a − b)(ap−1 + ap−2 b + · · · + abp−2 + bp−1 ) con
p = 3 para el numerador y p = 4 para el denominador. El proceso es el que
se indica a continuación:
p p p p p p
( 3 x − 1)( 3 x2 + 3 x + 1)( 4 x3 + 4 x2 + 4 x + 1)
L = lı́m p p p p p p
x→1 ( 4 x − 1)( 4 x3 + 4 x2 + 4 x + 1)( 3 x2 + 3 x + 1)
p p p
(x − 1)( 4 x3 + 4 x2 + 4 x + 1) 4
= lı́m p p = .
x→1 (x − 1)( 3 x2 + 3 x + 1) 3

PROBLEMA 3.12.
√ √
p
x− pc
Calcular lı́m .
x→c x−c

Solución

Si aplicamos la fórmula ap − bp = (a − b)(ap−1 + ap−2 b + · · · + abp−2 + bp−1 )


√ √
con a = p x y b = p c, tenemos:
x−c
L = lı́m p p p p p p
x→c (x − c)( xp−1 +
p p
xp−2 c + · · · + p x p cp−2 + p cp−1 )
p

1 1
= lı́m p p = p .
x→c p
xp−1 p
+ ··· + c p−1 p cp−1
p

B. INFINITÉSIMOS. INFINITÉSIMOS EQUIVALENTES.

Una función cuyo lı́mite es cero cuando la variable independiente x tiende


a un valor c recibe el nombre de infinitésimo en x = c. Se dice que un
f (x)
infinitésimo tiene orden n cuando existe y es no nulo el lı́mite lı́m .
x→c (x − c)n

91
Dos infinitésimos f (x) y g(x) en x = c son equivalentes cuando su cociente
f (x)
tiene lı́mite uno al tender x a c, es decir cuando lı́m = 1.
x→c g(x)

Un método común para calcular lı́mites consiste en sustituir infinitésimos por


otros equivalentes de modo que el cálculo resulte más sencillo. A continuación
damos una lista de las equivalencias más comunes entre infinitésimos:

sen f (x) ∼ f (x), cuando f (x) → 0;


2
1 − cos f (x) ∼ [f (x)] /2, cuando f (x) → 0;
tg f (x) ∼ f (x), cuando f (x) → 0;
f (x)
a − 1 ∼ f (x) ln a, cuando f (x) → 0;
(en particular ef (x) − 1 ∼ f (x))
ln(1 + f (x)) ∼ f (x), cuando f (x) → 0
(de otra forma ln f (x) ∼ f (x) − 1, cuando f (x) → 1).

Has de tener en cuenta que estas fórmulas sólo se pueden aplicar cuando
los infinitésimos aparezcan como factor en la función cuyo lı́mite se quiere
calcular. En otras palabras, la siguiente propiedad es válida: Si f y g son
infinitésimos equivalentes en x = c y h es cualquier función que tiene lı́mite
finito c, entonces f · h es un infinitésimo equivalente a g · h.

PROBLEMA 3.13.

Sabiendo que sen x ∼ x cuando x → 0, probar que tg x ∼ x y que


1 − cos x ∼ x2 /2.

Solución

Basta calcular el lı́mite del cociente:


tg x sen x 1
lı́m = lı́m · lı́m = 1.
x→0 x x→0 x x→0 cos x

Por otra parte,

1 − cos x (1 − cos x)(1 + cos x) sen2 x


L = lı́m = lı́m = lı́m
x→0 x2 /2 x→0 (1 + cos x) · x2 /2 x→0 (1 + cos x) · x2 /2
2
sen x 2
= lı́m 2
· lı́m = 1 · 1 = 1.
x→0 x x→0 1 + cos x

92
PROBLEMA 3.14.

1 − cos x
Resolver lı́m .
x→0 x

Solución

Análogamente al problema anterior, tenemos:


1 − cos x (1 − cos x)(1 + cos x) sen2 x
L = lı́m = lı́m = lı́m
x→0 x x→0 x(1 + cos x) x→0 x(1 + cos x)
sen x sen x
= lı́m · lı́m = 1 · 0 = 0.
x→0 x x→0 1 + cos x

PROBLEMA 3.15.

x − sen 3x
Resolver lı́m .
x→0 sen 5x

Solución

Separamos la expresión en dos fracciones y utilizamos las equivalencias


sen 3x ∼ 3x y sen 5x ∼ 5x:
x − sen 3x x sen 3x
L = lı́m = lı́m − lı́m
x→0 sen 5x x→0 sen 5x x→0 sen 5x
x 3x
= lı́m − lı́m = 1/5 − 3/5 = −2/5.
x→0 5x x→0 5x

PROBLEMA 3.16.

x sen x
Resolver lı́m .
x→0 x + sen x

Solución

Dividimos numerador y denominador por x y aplicamos la equivalencia


sen x ∼ x: x sen x
x sen x 0
L = lı́m x+sen x = lı́m sen x = = 0.
x→0 x→0 1 + 2
x x

93
PROBLEMA 3.17.

sen x − sen a
Calcular lı́m .
x→a x−a

Solución

x−a x+a
Aplicamos la fórmula sen x − sen a = 2 sen cos y tenemos:
2 2
2 sen x−a x+a
2 cos 2 sen x−a x+a 2a
L = lı́m = lı́m x−a2 · cos = 1 · cos = cos a.
x→a x−a x→a
2
2 2

PROBLEMA 3.18.

cos x − cos a
Calcular lı́m .
x→a x−a

Solución

x−a x+a
Aplicamos en este caso la fórmula cos x − cos a = −2 sen sen ,
2 2
con lo que:
−2 sen x−a x+a
2 sen 2 − sen x−a
2 x+a
L = lı́m = lı́m x−a · sen = − sen a.
x→a x−a x→a
2
2

PROBLEMA 3.19.

x − sen 2x
Calcular lı́m .
x→0 x + sen 3x

Solución

En primer lugar sacamos factor común 2x en el numerador y 3x en el de-


nominador y después aplicamos las equivalencias sen 2x ∼ 2x y sen 3x ∼
3x:
2x 21 − sen2x2x 1 sen 2x
2 12 − 1

2 2 − 2x −1
L = lı́m = lı́m = · 1 = .
x→0 3x 1 + sen 3x 1 sen 3x

3 3x
3 x→0
3 + 3x
3 3 +1 4

94
PROBLEMA 3.20.
√ √
1 + sen x − 1 − sen x
Calcular lı́m .
x→0 x

Solución

Si racionalizamos el numerador y tenemos en cuenta que sen x ∼ x, obtene-


mos:
√ √ √ √
( 1 + sen x − 1 − sen x)( 1 + sen x + 1 − sen x)
L = lı́m √ √
x→0 x( 1 + sen x + 1 − sen x)
1 + sen x − 1 + sen x
= lı́m √ √
x→0 x( 1 + sen x + 1 − sen x)
sen x 1
= 2 lı́m ·√ √ = 2 · 1 · 1/2 = 1.
x→0 x 1 + sen x + 1 − sen x

PROBLEMA 3.21.

ax − bx
Calcular lı́m .
x→0 cx − dx

Solución

Teniendo en cuenta la equivalencia de los infinitésimos x y ex − 1 cuando


x → 0, resulta:
ex ln a 1 − ex ln b−x ln a

ex ln a − ex ln b
L = lı́m x ln c = lı́m
x→0 e − ex ln d x→0 ex ln c (1 − ex ln d−x ln c )
ex ln a (x ln b − x ln a) ex ln a x ln(b/a) ln(b/a)
= lı́m x ln c = lı́m x ln c = .
x→0 e (x ln d − x ln c) x→0 e x ln(d/c) ln(d/c)

PROBLEMA 3.22.

ex − esen x
Calcular lı́m .
x→0 x − sen x

95
Solución

En primer lugar sacamos ex factor común en el numerador:

ex (1 − esen x−x )
L = lı́m .
x→0 x − sen x
Como sen x − x → 0, podemos aplicar la equivalencia de los infinitési-
mos
1 − esen x−x ∼ −(sen x − x). Tenemos ası́:

ex (x − sen x)
L = lı́m = lı́m ex = 1.
x→0 x − sen x x→0

PROBLEMA 3.23.

ey + sen y − 1
Calcular lı́m .
y→0 ln(1 + y)

Solución

Descomponemos en primer lugar la expresión en dos sumandos:


 y
ey − 1

e −1 sen y sen y
L = lı́m + = lı́m + lı́m .
y→0 ln(1 + y) ln(1 + y) y→0 ln(1 + y) y→0 ln(1 + y)

Debido a la equivalencia de los infinitésimos ey − 1 ∼ ln(1 + y) ∼ sen y,


resulta que L = 1 + 1 = 2.

C. LÍMITES INFINITOS. ASÍNTOTAS.

Anteriormente aparecieron lı́mites de la forma 0/0, los cuales forman un ca-


so particular de los llamados lı́mites indeterminados, pues su resultado no
se puede obtener en forma directa. Otros casos de indeterminación en los

96
lı́mites se presentan al considerar valores infinitos. Estos casos de indeter-
minación son ∞ − ∞, 0 · ∞, ∞/∞, 00 , ∞0 , 1∞ y para resolverlos se pueden
seguir las siguientes reglas (que completaremos en el capı́tulo 6 con otras
técnicas):

(a) Si la función es algebraica (sólo aparecen operaciones algebraicas y


no trigonométricas) y el lı́mite tiene la forma ∞/∞, se comparan los
grados del numerador y denominador.

a.1- Si el grado del numerador es mayor que el del denominador, el


resultado del lı́mite es ∞.

a.2- Si el grado del denominador es mayor que el del numerador, el


resultado es cero.

a.3- Si ambos tienen el mismo grado, el resultado es el cociente de los


coeficientes de los términos de mayor grado.

En resumen, en el caso ∞/∞ los términos que intervienen en el lı́mite


son los de mayor grado; el resto se puede desechar.

(b) Para las otras formas de indeterminación, es posible transformarlas en


alguna de las conocidas y utilizar las técnicas expuestas para ellas. Por
ejemplo, en las indeterminaciones con exponenciales (en particular de
la forma 1∞ ) se puede utilizar la fórmula f (x)g(x) = eg(x) ln f (x) , y el
exponente presenta ahora una indeterminación del tipo 0 · ∞.

Como aplicación de los lı́mites infinitos se pueden definir las ası́ntotas:

La recta x = h es ası́ntota vertical de la función y = f (x) cuando lı́m f (x) =


x→h
∞ o lı́m f (x) = −∞ (basta algún lı́mite lateral).
x→h

La recta y = k es ası́ntota horizontal de la función y = f (x) cuando


lı́m f (x) = k o bien lı́m f (x) = k.
x→∞ x→−∞

La recta y = mx + b es ası́ntota oblicua de la función y = f (x) cuando


existen los lı́mites que definen las constantes m y b ası́:

f (x)
m = lı́m , b = lı́m [f (x) − mx];
x→∞ x x→∞

o bien
f (x)
m = lı́m , b = lı́m [f (x) − mx],
x→−∞ x x→−∞

y m 6= 0 en ambos casos.

97
PROBLEMA 3.24.

(2x − 3)(3x + 5)(4x − 6)


Calcular lı́m .
x→∞ 3x3 + x − 1

Solución

Comparamos los grados del numerador y denominador y tenemos:


24x3 + . . . 24
L = lı́m 3
= = 8.
x→∞ 3x + . . . 3

PROBLEMA 3.25.
 
1 5
Calcular lı́m − 2 .
x→3 x − 3 x −x−6

Solución

Haciendo denominador común tenemos:


 
1 5
L = lı́m −
x→3 x − 3 (x − 3)(x + 2)
x+2−5 1 1
= lı́m = lı́m = .
x→3 (x − 3)(x + 2) x→3 x + 2 5

PROBLEMA 3.26.
 
1 3
Calcular lı́m − .
x→1 1 − x 1 − x3

Solución

Teniendo en cuenta que 1 − x3 = (1 − x)(1 + x + x2 ), hacemos denominador


común y resulta:
1 + x + x2 − 3 (x − 1)(x + 2) −3
L = lı́m 3
= lı́m 2
= = −1.
x→1 1−x x→1 (1 − x)(1 + x + x ) 3

98
PROBLEMA 3.27.

x
Calcular lı́m √
3
.
x→∞ x3 + 10

Solución

Dividimos numerador y denominador por x. Ası́:


x/x 1
L = lı́m √
3
= lı́m p = 1.
x→∞ x3+ 10/x x→∞ 1 + 10/x3
3

Este resultado se puede obtener directamente sabiendo que se trata de una


indeterminación del tipo ∞/∞ donde el numerador y denominador tienen
el mismo grado, siendo uno los coeficientes de los términos de mayor grado.

PROBLEMA 3.28.
p p
Calcular lı́m ( x2 + 4x − x2 − 4x).
x→∞

Solución

Tenemos una indeterminación del tipo ∞ − ∞. Multiplicando y dividiendo


por el conjugado resulta:
√ √ √ √
( x2 + 4x − x2 − 4x)( x2 + 4x + x2 − 4x)
L = lı́m √ √
x→∞ x2 + 4x + x2 − 4x
x2 + 4x − (x2 − 4x) 8x/x
= lı́m √ √ = lı́m √ √
x→∞ 2 2
x + 4x + x − 4x x→∞ ( x + 4x + x2 − 4x)/x
2

8 8
= lı́m p p = = 4.
x→∞ 1 + 4x/x2 + 1 − 4x/x2 2

PROBLEMA 3.29.
p p
Calcular lı́m ( n2 + an + b − n2 + cn + d).
n→∞

99
Solución

Procediendo análogamente al problema anterior tenemos:


√ √ √ √
( n2 + an + b − n2 + cn + d)( n2 + an + b + n2 + cn + d)
L = lı́m √ √
n→∞ n2 + an + b + n2 + cn + d
(n2 + an + b) − (n2 + cn + d) (a − c)n + (b − d)
= lı́m √ √ = lı́m √ √
n→∞ n2 + an + b + n2 + cn + d n→∞ n2 + an + b + n2 + cn + d
(a − c) + (b−d) a−c a−c
= lı́m p pn = = .
n→∞ 2
1 + a/n + b/n + 1 + c/n + d/n2 1+1 2

PROBLEMA 3.30.
√ √3
3x + 1 − x2
Calcular lı́m √
4
.
x→∞ x2 + 1

Solución


Si dividimos numerador y denominador por x se obtiene:
√ √ √
3 √ p p
3x + 1/ x − x2 / x 3 + 1/x − 3 x2 /x3/2
L = lı́m √
4 √ = lı́m p = −∞.
x→∞ x2 + 1/ x x→∞ 4
1 + 1/x2

Bastaba también en este caso comparar los grados del numerador y deno-
minador para obtener el resultado.

PROBLEMA 3.31.
p
3
p
3
Calcular lı́m ( x3 + ax2 − x3 − ax2 ).
x→∞

Solución

Si multiplicamos el numerador y denominador por el factor


p3
p p
(x3 + ax2 )2 + 3 (x3 + ax2 )(x3 − ax2 ) + 3 (x3 − ax2 )2 ,

100
tenemos
(x3 + ax2 ) − (x3 − ax2 )
L = lı́m p p p
x→∞ (x3 + ax2 )2 + 3 (x3 + ax2 )(x3 − ax2 ) + 3 (x3 − ax2 )2
3

2ax2
= lı́m p p p
x→∞ 3 (x3 + ax2 )2 + 3 (x3 + ax2 )(x3 − ax2 ) + 3 (x3 − ax2 )2

2a 2a
= lı́m p p p = .
x→∞ 3 (1 + a/x)2 + 3 (1 + a/x)(1 − a/x) + 3 (1 − a/x)2 3

PROBLEMA 3.32.
p
3
p
3
x+1−
x
Calcular lı́m p p .
x→∞ x+1− x

Solución

Multiplicamos numerador y denominador por los conjugados de ambos para


eliminar las raı́ces. Ası́ tenemos:
p p p p p p p
( 3 x + 1 − 3 x)( 3 (x + 1)2 + 3 x(x + 1) + 3 x2 )( x + 1 + x)
L = lı́m p p p p p p p
x→∞ ( x + 1 − x)( x + 1 + x)( 3 (x + 1)2 + 3 x(x + 1) + 3 x2 )
p p
x+1+ x
= lı́m p p p .
x→∞ 3 (x + 1)2 + 3 x(x + 1) + 3 x2

Tenemos una indeterminación del tipo ∞/∞, donde el grado del numerador
es 1/2 y el grado del denominador es 2/3. Como éste es mayor, el lı́mite es
cero.

PROBLEMA 3.33.
p p 
3
Calcular lı́m x3 + 2x2 − x2 − 1 .
x→∞

Solución

p p 
Si escribimos L = lı́m 6 (x3 + 2x2 )2 − 6 (x2 − 1)3 y aplicamos la fórmu-
x→∞
la
a6 − b6 = (a − b)(a5 + a4 b + a3 b2 + a2 b3 + ab4 + b5 ),

101
p p
con a = 6
(x3 + 2x2 )2 y b = 6
(x2 − 1)3 , tenemos que

(x3 + 2x2 )2 − (x2 − 1)3


L = lı́m
x→∞ (a5 + a4 b + a3 b2 + a2 b3 + ab4 + b5 )

x6 + 4x4 + 4x5 − (x6 − 3x4 + 3x2 − 1)


= lı́m .
x→∞ (. . . )

Como el denominador toma la forma


p p
a5 + a4 b + a3 b2 + a2 b3 + ab4 + b5 = 6 (x3 + 2x2 )10 + 6 (x3 + 2x2 )8 (x2 − 1)3
p p p
+ 6 (x3 + 2x2 )6 (x2 − 1)6 + 6 (x3 + 2x2 )4 (x2 − 1)9 + 6 (x3 + 2x2 )2 (x2 − 1)12
p p6
p
6
+ 6 (x2 − 1)15 = x30 + . . .+ .(6) . . + x30 + . . .,

resulta
4x5 + 7x4 + . . . 4 2
L = lı́m √ √ = = .
x→∞ 6
x30 + . . .+ .(6) 6
. . + x30 + . . . 6 3

PROBLEMA 3.34.

1
Calcular lı́m x sen .
x→0 [x]

Solución

Como el intervalo [0, 1) no está en el dominio de la función, no existe


1
lı́m x sen .
x→0 + [x]
1
Por otra parte, si x → 0− , [x] = −1 y lı́m x sen = lı́m x sen(−1) = 0.
x→0− [x] x→0−

PROBLEMA 3.35.

πx
Calcular lı́m (1 − x) tg .
x→1 2

102
Solución

Si hacemos el cambio de variable z = x − 1 y utilizamos la equivalencia de


los infinitésimos tg z y z, cuando z → 0, obtenemos:
π(z + 1)  πz π 
L = lı́m −z tg = lı́m −z tg +
z→0 2 z→0 2 2
πz z z 2
= lı́m z cotg = lı́m πz = lı́m = .
z→0 2 z→0 tg
2
z→0 πz/2 π

PROBLEMA 3.36.

tg πx+1
2x
Calcular lı́m √ .
x→∞ 4 x4 + 2

Solución

 
πx + 1 π 1 1 1
De la identidad tg = tg + = − cotg =− y de
2x 2 2x 2x tg(1/2x)
1 1
la equivalencia entre infinitésimos tg ∼ , pues 1/2x → 0, podemos
2x 2x
escribir:

−1 −2x
L = lı́m √ = lı́m √ = −2,
x→∞ 1 4 4
x +2 x→∞ 4
x4 + 2
2x
pues los grados del numerador y denominador coinciden.

PROBLEMA 3.37.

5
ln x3 − 2x + 7
Calcular lı́m .
x→∞ ln x2

Solución

Debido a las propiedades de los logaritmos, tenemos:


(1/5) ln(x3 − 2x + 7) 1 ln x3 (1 − 2/x2 + 7/x3 )
L = lı́m = lı́m
 2 ln x 10 x→∞  ln x
x→∞
1 ln x ln(1 − 2/x2 + 7/x3 ) 1 3
= lı́m 3 + = (3 + 0) = .
10 x→∞ ln x ln x 10 10

103
PROBLEMA 3.38.

Calcular lı́m x[ln(x + 1) − ln x].


x→∞

Solución

Como 1/x → 0, resulta que ln(1 + 1/x) ∼ 1/x con lo que:


 
x+1 1 1
L = lı́m x ln = lı́m x ln 1 + = lı́m x · = 1.
x→∞ x x→∞ x x→∞ x

PROBLEMA 3.39.
r
1 1+x
Calcular lı́m ln .
x→0 x 1−x

Solución

 
2x 2x 2x
Cuando x → 0, → 0 por lo que ln 1 + ∼ . Enton-
1−x 1−x 1−x
ces:
 
1 1 1+x 1 2x
L = lı́m · ln = lı́m ln 1 +
x→0 x 2 1 − x x→0 2x 1−x
1 2x 1
= lı́m · = lı́m = 1.
x→0 2x 1 − x x→0 1 − x

PROBLEMA 3.40.
√ 
x
Calcular lı́m x 2−1 .
x→∞

104
Solución

Debido a la equivalencia 21/x − 1 ∼ (ln 2) · 1/x, tenemos:


  1
L = lı́m x 21/x − 1 = lı́m x · · ln 2 = ln 2.
x→∞ x→∞ x

PROBLEMA 3.41.

√  1
1 + x ln(1+x)
Calcular lı́m √ .
x→0 1+x

Solución

Como la base es siempre uno, el lı́mite de la exponencial es

L = lı́m 1f (x) = 1.
x→0

Observación. No hay en este caso ninguna indeterminación porque la fun-


ción es constante.

PROBLEMA 3.42.
 n
n+1
Calcular lı́m .
n→∞ n−1

Solución

Se trata de una indeterminación del tipo 1∞ . Llamando L al lı́mite y to-


mando logaritmos, tenemos:
n+1
ln L = lı́m n ln .
n→∞ n−1
   
n+1 n+1 n+1
Como → 1, tenemos la equivalencia ln ∼ − 1 , con
n−1 n−1 n−1
lo que resulta:
 
n+1 2
ln L = lı́m n − 1 = lı́m n · = 2.
n→∞ n−1 n→∞ n−1

105
En definitiva, L = e2 .

PROBLEMA 3.43.

Calcular lı́m (1 + sen x)1/x .


x→0

Solución

De nuevo la indeterminación es de la forma 1∞ y aplicamos el mismo pro-


cedimiento del problema anterior. Ası́:
1 1 sen x
ln L = lı́m ln(1 + sen x) = lı́m (1 + sen x − 1) = lı́m = 1,
x→0 x x→0 x x→0 x
de donde L = e.

PROBLEMA 3.44.

2
Calcular lı́m (cos ax)1/x .
x→0

Solución

Al igual que los problemas anteriores podemos escribir ln L = lı́m (1/x2 ) ln cos ax.
x→0
Aplicando ahora las equivalencias entre infinitésimos ln cos ax ∼ cos ax−1 ∼
−a2 x2 /2, resulta:

1 −a2 x2
 
1
ln L = lı́m 2 (cos ax − 1) = lı́m 2 = −a2 /2.
x→0 x x→0 x 2
2 /2
En definitiva, L = e−a .

PROBLEMA 3.45.

h  π i sen1 x
Calcular lı́m tg x + .
x→0 4

106
Solución

Tenemos de nuevo una indeterminación ∞


π  delπtipo 1 y aplicamos las siguien-
tes equivalencias: ln tg x + ∼ tg x + − 1, sen x ∼ tg x ∼ x:
4 4
1  π 1 h  π i
ln L = lı́m ln tg x + = lı́m tg x + −1
x→0 sen x 4  x→0 sen x 4

1 tg x + 1 1 tg x + 1 − 1 + tg x
= lı́m − 1 = lı́m ·
x→0 sen x 1 − tg x x→0 sen x 1 − tg x
2 tg x 2x
= lı́m = lı́m = 2.
x→0 sen x(1 − tg x) x→0 x(1 − tg x)

Entonces L = e2 .

PROBLEMA 3.46.

 1+cotg2 x
x−2
Calcular lı́m 2
.
x→0 x +x−2

Solución

Como tenemos una indeterminación 1∞ , aplicamos las equivalencias


x−2 x−2
ln ∼ 2 − 1 y posteriormente sen2 x ∼ x2 :
x2
+x−2 x +x−2
 
2 x−2 2 x−2
ln L = lı́m (1 + cotg x) ln 2 = lı́m (1 + cotg x) −1
x→0 x + x − 2 x→0 x2 + x − 2
cos2 x −x2 −x2
     
1
= lı́m 1 + 2 2
= lı́m 2 2
x→0 sen x x +x−2 x→0 sen x x +x−2
2
 
1 −x −1
= lı́m 2 2
= lı́m 2 = 1/2.
x→0 x x +x−2 x→0 x + x − 2

Resulta entonces que L = e1/2 = e.

PROBLEMA 3.47.

 sec2 π
2 π 2−bx
Calcular lı́m sen .
x→0 2 − ax

107
Solución

Tenemos una indeterminación 1∞ y procedemos de forma similar a los pro-


blemas anteriores:
π π
ln L = lı́m sec2 ln sen2
x→0 2 − bx 2 − ax
π
− cos2 2−ax
 
π π
= lı́m sec2 sen2 − 1 = lı́m .
x→0 2 − bx 2 − ax x→0 cos2 π
2−bx
 
π 2 2 π πax πax
Ahora bien, si escribimos cos = cos + = sen2
2 − ax 2 2(2 − ax) 2(2 − ax)
πax πax πbx πbx
y usamos las equivalencias sen ∼ , sen ∼ ,
2(2 − ax) 2(2 − ax) 2(2 − bx) 2(2 − bx)
obtenemos:
 2
πax
π
− cos2 2−ax − 2(2−ax)
ln L = lı́m = lı́m 
x→0 cos2 π
2
x→0 πbx
2−bx
2(2−bx)

−(πax)2 4(2 − bx)2


−a2 (2 − bx)2 −4a2 −a2
= lı́m = lı́m = = .
x→0 (πbx)2 4(2 − ax)2 x→0 b2 (2 − ax)2 4b2 b2
2 /b2
En definitiva, L = e−a .

PROBLEMA 3.48.

Calcular lı́m [cos(x − 1) + a sen(x − 1)]1/ ln x .


x→1

Solución

Hacemos el cambio de variable z = x−1. De este modo z → 0 y ln(1+z) ∼ z,


con lo que tenemos:
1 1
ln L = lı́m ln(cos z + a sen z) = lı́m (cos z + a sen z − 1)
z→0 ln(1 + z) z→0 ln(1 + z)
 
cos z + a sen z − 1 cos z − 1 a sen z
= lı́m = lı́m + = 0 + a = a,
z→0 z z→0 z z

con lo que L = ea .

108
PROBLEMA 3.49.

p p !n
n n
a+ b
Calcular lı́m .
n→∞ 2

Solución

Tenemos nuevamente la indeterminación 1∞ y utilizamos las equivalen-


cias
pn
p
a − 1 ∼ (1/n) ln a y n b − 1 ∼ (1/n) ln b. Tenemos entonces:
p p !
n
a+ n b
ln L = lı́m n −1
n→∞ 2
p p ! p p
n n n n
a−1 b−1 a−1 b−1
= lı́m n + = lı́m n + lı́m n
n→∞ 2 2 n→∞ 2 n→∞ 2
n 1 n 1 ln ab p
= lı́m · ln a + lı́m · ln b = = ln ab.
n→∞ 2 n n→∞ 2 n 2
p
En definitiva, L = ab.

PROBLEMA 3.50.
" #n
21/n + 31/n + 41/n
Calcular lı́m .
n→∞ 3

Solución

De forma análoga al problema anterior tenemos:


" #
21/n + 31/n + 41/n
ln L = lı́m n · −1
n→∞ 3
" #
1 21/n − 1 31/n − 1 41/n − 1
= lı́m + +
3 n→∞ 1/n 1/n 1/n
 
1 (1/n) ln 2 (1/n) ln 3 (1/n) ln 4 1
= lı́m + + = [ln 2 + ln 3 + ln 4] .
3 n→∞ 1/n 1/n 1/n 3

109

Entonces L = e(1/3) ln(2·3·4) = 3
24.

PROBLEMA 3.51.
!nx
1/x 1/x 1/x
a1 + a2 + · · · + an
Calcular lı́m .
x→∞ n

Solución

En este caso, como la indeterminación es también de la forma 1∞ , el proce-


dimiento es análogo al realizado en los dos problemas anteriores:
!
1/x 1/x 1/x 1/x
a1 + · · · + an a + · · · + an − n
ln L = lı́m nx − 1 = lı́m nx · 1
x→∞ n x→∞ n
 
1/x
= lı́m x (a1 − 1) + · · · + (a1/x n − 1)
x→∞
1/x
= lı́m x(a1 − 1) + · · · + lı́m x(a1/x
n − 1)
x→∞ x→∞
1 1
= lı́m x · ln a1 + · · · + lı́m x · ln an = ln a1 + · · · + ln an .
x→∞ x x→∞ x
Resulta entonces que L = a1 · · · · · an .

PROBLEMA 3.52.

x2 − 1
Encontrar las ası́ntotas de la función f (x) = .
x−1

Solución

(a) Las ası́ntotas verticales sólo se pueden encontrar para valores de x donde
el lı́mite sea infinito. En este caso puede ser en x = 1 pues anula el denomi-
(x − 1)(x + 1)
nador. Pero como lı́m f (x) = lı́m = lı́m (x + 1) = 2, no hay
x→1 x→1 x−1 x→1
ası́ntotas verticales.
(b) Para ver si hay ası́ntotas horizontales, debemos calcular:

110
x2 − 1
lı́m f (x) = lı́m = ∞ (pues el grado del numerador es mayor que el
x→∞ x→∞ x − 1
grado del denominador).
x2 − 1
lı́m f (x) = lı́m = −∞ (por la misma razón que antes. Además el
x→−∞ x→−∞ x − 1
numerador es positivo y el denominador negativo para valores de x próximos
a −∞).
Esto quiere decir que no hay ası́ntotas horizontales.
(c) Para comprobar la existencia de ası́ntotas oblicuas calcularemos los si-
guientes lı́mites:
f (x) x2 − 1
m = lı́m = lı́m = 1,
x→∞ x x→∞ x(x − 1)

pues el grado del numerador es igual al del denominador y son uno los
coeficientes de x2 en ambos términos.
x2 − 1 x2 − 1 − x2 + x
b = lı́m [f (x) − mx] = lı́m − x = lı́m = 1.
x→∞ x→∞ x − 1 x→∞ x−1
La ası́ntota oblicua es la recta y = x + 1.
Análogamente se obtiene para −∞:
f (x) x2 − 1
m = lı́m = lı́m = 1,
x→−∞ x x→−∞ x(x − 1)

pues nuevamente el grado del numerador es igual al del denominador y son


uno los coeficientes de x2 en ambos términos.
x2 − 1 x2 − 1 − x2 + x
b = lı́m [f (x) − mx] = lı́m − x = lı́m = 1.
x→−∞ x→−∞ x − 1 x→−∞ x−1
La recta y = x + 1 también es ası́ntota oblicua para −∞.

PROBLEMA 3.53.

Comprobar que no existen los siguientes lı́mites:


lı́m sen x, lı́m cos x, lı́m tg x.
x→∞ x→∞ x→∞

Solución

Si existiera L = lı́m sen x, deberı́a ser |L| ≤ 1, porque | sen x| ≤ 1. Además,


x→∞
por definición,
∀ε > 0, ∃M > 0 tal que x > M =⇒ L − ε < sen x < L + ε.

111
Ahora bien, es evidente que existen infinitos valores de x, de las formas
kπ, π/2 + 2kπ, 3π/2 + 2kπ, cuyo valor absoluto es mayor que M , y para los
cuales la función sen x toma, respectivamente, los valores 0, 1, −1. Por tanto,
si fijamos ε > 0 de modo que fuera del entorno (L − ε, L + ε) queden por lo
menos dos de tales valores, la función alcanzarı́a valores no contenidos en el
entorno prefijado. Esto prueba que L no puede ser el lı́mite.
El mismo razonamiento sirve para probar que no existen lı́m cos x y lı́m tg x.
x→∞ x→∞

PROBLEMA 3.54.

Demostrar que no existen los lı́mites siguientes:


a) lı́m x sen x.
x→∞

b) lı́m x cos x.
x→∞

c) lı́m x tg x.
x→∞

d) lı́m e1/ sen x .


x→0

Solución

a) Veamos en primer lugar que el lı́mite no puede ser infinito:


En efecto, por definición, si lı́m x sen x = ∞, dado cualquier número
x→∞
positivo M , debe existir k > 0 tal que x sen x > M , para todo x > k.
Sin embargo, siempre es posible encontrar valores de x superiores a k,
para los cuales sen x, y por consiguiente, x sen x, es nulo, es decir, para
los cuales x sen x ≤ M .
A continuación mostraremos que dicho lı́mite no puede ser un número
finito:
Si fuese lı́m x sen x = L, a cada ε > 0 le corresponderı́a un k > 0 tal
x→∞
que |x sen x − L| < ε, para todo x > k. Ahora bien, si consideramos
un número k tal que |k − L| > ε, siempre es posible encontrar valores
x > k para los cuales sen x = 1 y x sen x = x > k pero |x sen x − L| >
|k − L| > ε, lo que contradice el hecho de que L es el lı́mite.
Los apartados b) y c) se prueban de manera completamente análoga, apo-
yándose exclusivamente en el concepto de lı́mite.

112
Por último, como lı́m e1/ sen x = +∞ y lı́m e1/ sen x = e−∞ = 0, no existe
x→0+ x→0−
el lı́mite por ser distintos los lı́mites laterales.

113
D. EJERCICIOS PROPUESTOS.

1.- Calcular (en caso de que existan) los lı́mites siguientes:


1/x − 1/3
a) lı́m .
x→3 x−3

Resp.: -1/9.

x−1
b) lı́m .
x→1 xn − 1
Resp.: 1/n.

x + x2 + · · · + xn − n
c) lı́m .
x→1 x−1
n(n + 1)
Resp.: .
2
√ √
x + 1 − 2x
d) lı́m .
x→1 x2 − x

Resp.: −1/2 2.


x− x
e) lı́m 4 .
x→1 x − 1

Resp.: 1/8.


(2x − 3)( x − 1)
f) lı́m .
x→1 2x2 + x − 3
Resp.: -1/10.


x
g) lı́m p √ .
x→0+ 4+ x−2
Resp.: 4.


x2 − x
h) lı́m √ .
x→1 x−1
Resp.: 3.
Sugerencia: Racionalizar numerador y denominador.

114
i) lı́m x cosec x.
x→0

Resp.: 1.

sen 3x
j) lı́m .
x→0 tg 5x

Resp.: 3/5.

sen 2x
k) lı́m .
x→0 2x2 + x
Resp.: 2.

l) lı́m (1 − cos x) cotg x.


x→0

Resp.: 0.

1 − cos x
m) lı́m .
x→0 x2 + sen 3x2
Resp.: 1/8.

cos3 x − 1
n) lı́m .
x→0 x3
Resp.: ∞.


o) lı́m x cosec2 2x.
x→0+

Resp.: 1/2.

x2 sen(1/x)
p) lı́m .
x→0 sen x
Resp.: 0.

sen x
q) lı́m .
x→∞ x
Resp.: 0.

r) lı́m (x − π) cotg x.
x→π

Resp.: 1.

1 1
s) lı́m − .
x→0 sen2 x 1 − cos x

115
Resp.: ∞.

sen(x − π/3)
t) lı́m .
x→π/3 1 − 2 cos x

Resp.: 1/ 3.

ln tg[(π/4) + ax]
u) lı́m .
x→0 sen bx
Resp.: 2a/b.

 x+2
x−1
v) lı́m .
x→∞ x−3
Resp.: e2 .

(n2 −1)/n
n2 + 3

x) lı́m .
n→∞ n2 + 4n
Resp.: e−4 .

2.- Calcular (en caso de que existan) los lı́mites siguientes:


( 3
x +1
si x > −1
a) lı́m f (x) con f (x) = x+1
2x 2
x→−1
1−x2
si x < −1.
Resp.: ∞ (por la izquierda); 3 (por la derecha).

x
b) lı́m .
x→1/2 [x]

Resp.: No existe porque el intervalo [0, 1) no está en el dominio de la


función.

[x]2 − 9
c) lı́m .
x→3 x2 − 9

Resp.: ∞ (por la izquierda); 0 (por la derecha).

x2 − 9
d) lı́m .
x→3 |x − 3|

Resp.: - 6 (por la izquierda), 6 (por la derecha).

116
|x|
x + 2[x]
e) lı́m .
x→3/2 x3
Resp.: 8/9.
Sugerencia: Calcular los lı́mites laterales.

sen x
f) lı́m .
x→0 |x|
Resp.: -1 (por la izq); 1 (por la dcha).

3.- Determinar los valores de a para los que existen los lı́mites late-
rales lı́m f (x), lı́m f (x), y los casos en que dichos lı́mites son
x→a+ x→a−
iguales:
p
(a) f (x) = x − [x].
Resp.: Existen ∀a ∈ R. Son iguales ∀a ∈ R \ Z.

p
(b) f (x) = [x] + x − [x].
Resp.: Existen y son iguales ∀a ∈ R.

1
(c) f (x) = .
[1/x]
Resp.: Existen cuando a 6= 0 y a ≤ 1 (por la izquierda); cuando a 6= 0
y a < 1 (por la derecha). Son iguales cuando a 6= 1/n, n = −1, −2, . . .

4.- Encontrar las ası́ntotas de las siguientes funciones:


x2 − 3
a) f (x) = .
2x − 4
Resp.: (V) x = 2; (O) y = 1 + x/2.

2x
b) f (x) = √ .
4x2 − 4
Resp.: (V) x = 1, x = −1; (H) y = 1, y = −1.

|x + 1|
c) f (x) = √ .
x2 + 1
Resp.: (H) y = 1.

117

x2 + 5 − 3
d) f (x) = .
x2 − 4
Resp.: (H) y = 0.

3x + a
5.- Determinar la función f (x) = para que y = 2 sea ası́ntota
px + 2
horizontal y (−2, 0) sea el punto de corte con el eje X . Dibujar
su gráfica.
Resp.: a = 6, p = 3/2.

6.- (a) Si no existen lı́m f (x) ni lı́m g(x), ¿pueden existir lı́m [f (x) +
x→a x→a x→a
g(x)] ó lı́m f (x)g(x)?
x→a

Resp.: Sı́. Ejemplos: f (x) = 1/x, g(x) = −1/x con a = 0; f (x) = |x|/x,
g(x) = x/|x| con a = 0.

(b) Si existen lı́m f (x) y lı́m [f (x) + g(x)], ¿debe existir lı́m g(x)?
x→a x→a x→a

Resp.: Sı́ porque g(x) = [f (x) + g(x)] − f (x).

(c) Si existe lı́m f (x) y no existe lı́m g(x), ¿puede existir lı́m [f (x)+
x→a x→a x→a
g(x)]?
Resp.: No, porque si existiera, no se cumplirı́a el apartado anterior.

(d) Si existen lı́m f (x) y lı́m f (x)g(x), ¿existe lı́m g(x)?


x→a x→a x→a

Resp.: Sólo en el caso en que lı́m f (x) 6= 0.


x→a

7.- Decidir si los siguientes planteamientos son verdaderos o falsos:


(a) Si lı́m f (x) = L, entonces lı́m f (a + h) = L.
x→a h→0

Resp.: Verdadero: basta hacer el cambio x − a = h.

(b) Si lı́m f (x) = L, entonces lı́m f (x − a) = L.


x→a x→0

Resp.: Falso: ejemplo f (x) = x.

(c) Si lı́m f (x) = L, entonces lı́m f (x3 ) = L.


x→0 x→0

Resp.: Verdadero: basta hacer el cambio x3 = t.

118
(d) Si lı́m f (x) = L, lı́m f (|x|) = L.
x→0+ x→0

Resp.: Verdadero.

(e) Si lı́m f (x) = L, entonces lı́m |f |(x) = |L|.


x→a x→a

Resp.: Verdadero, pues |f (x)| − |L| ≤ |f (x) − L|.

(f) Si lı́m f (x) = L, entonces lı́m f (1/x) = L.


x→∞ x→0+

Resp.: Verdadero.

(g) lı́m f (x) = L ⇐⇒ lı́m (f (x) − L) = 0.


x→a x→a

Resp.: Verdadero.

(h) Si f (x) < g(x), ∀x, entonces lı́m f (x) < lı́m g(x).
x→a x→a
(
2x2 si x 6= 0
Resp.: Falso. Ejemplo: f (x) = x2 , g(x) = en a = 0.
1 si x = 0

(i) Si |f (x)−L| < ε cuando 0 < |x−a| < δ , entonces |f (x)−L| < ε/2
cuando 0 < |x − a| < δ/2.
Resp.: Falso. Ejemplo: f (x) = x2 con a = 1.

(j) Si lı́m f (x) = 0, entonces lı́m f (x) sen(1/x) = 0.


x→0 x→0

Resp.: Verdadero, pues −1 ≤ sen(1/x) ≤ 1.

119
CAPÍTULO IV.
CONTINUIDAD DE
FUNCIONES

SECCIONES
A. Definición de función continua.
B. Propiedades de las funciones continuas.
C. Ejercicios propuestos.

121
A. DEFINICIÓN DE FUNCIÓN CONTINUA.

Una función y = f (x) se dice continua en un punto x = c cuando existe el


lı́mite de la función en el punto x = c y dicho lı́mite es f (c).
Esta definición da lugar a tres condiciones que debe cumplir la función para
ser continua en c:
a) c está en el dominio de la función.
b) existe lı́m f (x) (es decir, los lı́mites laterales son finitos e iguales).
x→c

c) lı́m f (x) = f (c).


x→c

Esto quiere decir que para que una función sea continua no basta que tenga
lı́mite, sino que además dicho lı́mite tiene que coincidir con el valor de la
función en el punto correspondiente.
Las funciones que no son continuas se llaman discontinuas. Hay varios tipos
de discontinuidad dependiendo de la condición que no se cumple.
A) Discontinuidad evitable: Corresponde al caso en que la función tiene
lı́mite pero no coincide con el valor f (c). Se llama evitable porque
basta definir f (c) como el lı́mite de la función en c para que la función
sea ahora continua.
B) Discontinuidad de primera especie: Puede ser de salto finito cuando
existen los dos lı́mites laterales pero son distintos, o de salto infinito
cuando alguno de los lı́mites laterales es infinito.
C) Discontinuidad esencial o de segunda especie: Si alguno de los dos lı́mites
laterales no existe.
Las operaciones algebraicas con funciones continuas dan como resultado
nuevas funciones continuas, salvo en la división por cero y las raı́ces de
ı́ndice par de funciones que toman valores negativos.

PROBLEMA 4.1.

x + |x|
Estudiar la continuidad de la función f (x) = .
2

122
Solución

Esta es una función algebraica sólo que el valor absoluto hace que cambie
la forma de la función en el punto x = 0. Esto quiere decir que si x 6= 0, la
función es continua.
Para estudiar el comportamiento de la función en x = 0, debemos calcular
los lı́mites laterales.
x + |x| x−x
lı́m = lı́m = 0,
x→0− 2 x→0− 2
x + |x| x+x
lı́m = lı́m = 0,
x→0+ 2 x→0 + 2

lo que indica que la función también es continua en x = 0.


Podemos comprobar este resultado dibujando la gráfica de la función. Esta
es de la forma:
Y

y=x

y=0 X

PROBLEMA 4.2.

Estudiar la continuidad de las siguientes funciones indicando los


puntos de discontinuidad:
a) f (x) = [x2 ].

b) f (x) = [ x].
c) f (x) = [2x].
p
d) f (x) = [x].
p
e) f (x) = x − [x].
f) f (x) = [x] + [−x].

123
Solución

Sabiendo que la parte entera sólo es discontinua en los enteros, los puntos
de discontinuidad son, respectivamente:

a) x2 = n ⇐⇒ x = ± n con n ∈ N. (En x = 0 la función es continua.)

b) x = n ⇐⇒ x = n2 con n = 0, 1, . . .

c) 2x = n ⇐⇒ x = n/2 con n ∈ Z.

d) Como el dominio de la función es [0, ∞), los puntos de discontinuidad


son los enteros positivos.

e) Como x − [x] ≥ 0 para todo x, los puntos de discontinuidad son x ∈ Z.

f) Si n es cualquier número entero, los lı́mites laterales son


lı́m [x] + [−x] = n − 1 + (−n) = −1; lı́m [x] + [−x] = n + (−n − 1) = −1.
x→n− x→n+

Como f (n) = 0 6= −1, la discontinuidad es evitable en todo Z.

PROBLEMA 4.3.

Estudiar la continuidad de las funciones:


1
a) f (x) = .
x2
+1
1
b) f (x) = 2 .
x −1

Solución

a) Como 1 + x2 = 0 no tiene raı́ces reales, la función es continua en todo


R.

b) Como la ecuación x2 − 1 = 0 tiene raı́ces x = 1 y x = −1, la función es


1 1
continua en R\{−1, 1}. Además, como lı́m 2 = lı́m 2 = ∞,
x→1 x − 1 x→−1 x − 1
la función presenta discontinuidades de primera especie infinitas en los
puntos x = −1 y x = 1.

124
PROBLEMA 4.4.

Estudiar la continuidad de las funciones


1
a) f (x) = .
x
1
b) f (x) = .
x−1
c) f (x) = x2/3 .
2
d) f (x) = x−1/3 .
3
1
e) f (x) = x .
e

Solución

a) La función es continua en todo x 6= 0, porque es racional y la única raı́z


del denominador es x = 0.
b) La función es continua en todo x 6= 1 por la misma razón del apartado
anterior.
c) La función es continua en todo el campo real pues el ı́ndice de la raı́z es
impar.
2 2
d) Como f (x) = x−1/3 = √ , la función es continua en R \ {0}.
3 33x
e) Como el denominador no se anula en ningún valor real, la función es
continua en todo R.

PROBLEMA 4.5.

Indicar la naturaleza de la discontinuidad en x = 0 de las siguientes


funciones:
a) f (x) = cosec x.
p
b) f (x) = 1/x.
p
c) f (x) = 3 1/x.
d) f (x) = cos(1/x).

125
Solución

a) Como lı́m cosec x = ∞, la función presenta una discontinuidad infinita


x→0
de primera especie.
p
b) Como lı́m 1/x no existe, la discontinuidad es de segunda especie.
x→0−
p
c) Tenemos que lı́m 3 1/x = ∞, por lo que la discontinuidad es infinita de
x→0
primera especie.
d) En este caso, lı́m cos(1/x) no existe, por lo que la discontinuidad es de
x→0
segunda especie.

PROBLEMA 4.6.

x4 − 5x3 + 5x2 + 5x − 6
Dada la función f (x) = , determinar la cla-
x2 − 5x + 6
se de discontinuidad que posee en los puntos x = 2 y x = 3.

Solución

x4 − 5x3 + 5x2 + 5x − 6 (x − 2)(x − 3)(x2 − 1)


Como f (x) = = , resulta que
x2 − 5x + 6 (x − 2)(x − 3)
lı́m f (x) = 3 y lı́m f (x) = 8, pero los puntos x = 2 y x = 3 no están en
x→2 x→3
el dominio, por lo que las discontinuidades en dichos puntos son evitables.

PROBLEMA 4.7.

Encontrar los puntos de discontinuidad de las siguientes funcio-


nes:
x
a) f (x) = .
(x − 2)(x − 4)
p
b) f (x) = (x − 3)(6 − x), 3 ≤ x ≤ 6.
1
c) f (x) = .
1 + 2 sen x

126
Solución

a) La función es discontinua en los puntos donde se anula el denominador,


es decir x = 2, x = 4. La discontinuidad en ambos puntos es infinita
de primera especie porque el lı́mite es infinito en ambos casos.
b) No hay puntos de discontinuidad en el dominio de la función.
c) El denominador se anula cuando 1 + 2 sen x = 0, es decir cuando sen x =
7π 11π
−1/2, lo cual ocurre si x = + 2kπ, + 2kπ, k ∈ Z. En estos
6 6
puntos la discontinuidad es infinita de primera especie.

PROBLEMA 4.8.

1 − cos x
Estudiar la continuidad de la función f (x) = .
x

Solución

El dominio de la función es R \ {0}. En el punto x = 0 tenemos:

√ p
1 − cos x x2 /2 x 1
lı́m = lı́m = lı́m √ = √ ;
x→0+ x x→0+ x x→0+ x 2 2
√ p
2
1 − cos x x /2 −x −1
lı́m = lı́m = lı́m √ = √ .
x→0− x x→0− x x→0− x 2 2
La función presenta en x = 0 una discontinuidad de salto finito.

PROBLEMA 4.9.

Estudiar la continuidad de las funciones


a) f (x) = tg x.
b) f (x) = cotg x.
x2 − 4
c) f (x) = .
x2 − 5x + 6
¿Tiene discontinuidad evitable alguna de ellas?

127
Solución

sen x
a) Como tg x = , la función es continua donde cos x 6= 0, es decir
cos x
en el conjunto {x ∈ R | x 6= π/2 + kπ, k ∈ Z}. Como en los puntos
de discontinuidad el lı́mite es infinito, la discontinuidad es de primera
especie.
b) Análogamente al anterior, la función es continua en el conjunto {x ∈ R |
sen x 6= 0} = {x | x 6= kπ, k ∈ Z} y en el resto la discontinuidad es de
primera especie.
c) Como el denominador se anula cuando x = 2 y x = 3, la función es conti-
x2 − 4 x+2
nua en R\{2, 3}. Como lı́m 2 = lı́m = −4, la disconti-
x→2 x − 5x + 6 x→2 x − 3
x2 − 4
nuidad en x = 2 es evitable. Sin embargo, como lı́m 2 = ∞,
x→3 x − 5x + 6
la discontinuidad en x = 3 es de primera especie.

PROBLEMA 4.10.

¿Es evitable la discontinuidad en el origen de la función


2
f (x) = e−1/x ? ¿Y la discontinuidad de la función f (x) = sen(π/x)?

Solución

2
Debido a que lı́m e−1/x = 0, la discontinuidad es evitable.
x→0

Sin embargo, el lı́mite lı́m sen(π/x) no existe, por lo que la discontinuidad


x→0
es esencial.

PROBLEMA 4.11.

x2 sen x cos x
Estudiar la continuidad de la función F (x) = en el
1 − cos x
origen, siendo F (0) = 0.

128
Solución

Si aplicamos la equivalencia de infinitésimos 1−cos x ∼ x2 /2, tenemos:

x2 sen x cos x x2 sen x cos x sen x cos x


lı́m = lı́m 2
= lı́m = 0.
x→0 1 − cos x x→0 x /2 x→0 1/2
Como lı́m F (x) = F (0) = 0, la función es continua en x = 0.
x→0

PROBLEMA 4.12.

Estudiar la continuidad de la función

2x6 + x5 − x4
F (x) =
cos4 x sen3 2x
en el origen, sabiendo que F (0) = 0.

Solución

Aplicando la equivalencia sen 2x ∼ 2x, resulta:

x3 (2x3 + x2 − x)
lı́m F (x) = lı́m
x→0 x→0 cos4 x sen3 2x
x3 (2x3 + x2 − x) 2x3 + x2 − x
= lı́m = lı́m = 0.
x→0 cos4 x · (2x)3 x→0 8 cos4 x
La función F es continua en x = 0 porque lı́m F (x) = F (0) = 0.
x→0

PROBLEMA 4.13.

Estudiar la continuidad en el origen de la función

x7 + 2x5 + x3
F (x) = ,
sen2 (x/2) tg(x/4)

siendo F (0) = 0.

129
Solución

Debido a las equivalencias sen x/2 ∼ x/2 y tg x/4 ∼ x/4,


x3 (x4 + 2x2 + 1) x3 (x4 + 2x2 + 1)
lı́m F (x) = lı́m = lı́m
x→0 x→0 sen2 (x/2) tg(x/4) x→0 (x/2)2 · (x/4)

x4 + 2x2 + 1
= lı́m = 16.
x→0 1/16

Como existe lı́m F (x) pero es distinto a F (0), existe una discontinuidad
x→0
evitable en el origen.

PROBLEMA 4.14.

3
Estudiar la continuidad de la función f (x) = en el punto
4 + 4tg x
x = π/2.

Solución

3 3 3 3 3
Como lı́m tg x
= = 0 y lı́m tg x
= = , la función
x→π/2 4 + 4
− ∞ x→π/2 4 + 4
+ 4+0 4
posee una discontinuidad finita de primera especie en x = π/2.

PROBLEMA 4.15.

π
Estudiar la continuidad de la función f (x) = 2 sen en el origen.
x

Solución

La función no tiene lı́mite cuando x → 0, por lo que la discontinuidad es


esencial o de segunda especie.

PROBLEMA 4.16.

¿Qué clase de discontinuidad posee la función f (x) = 31/x en el


origen?

130
Solución

Los lı́mites laterales son lı́m 31/x = 3−∞ = 0 y lı́m 31/x = 3∞ = ∞,


x→0− x→0+
respectivamente.
Como uno de ellos es infinito, tenemos una discontinuidad infinita de primera
especie.

PROBLEMA 4.17.

1
Estudiar la continuidad de la función f (x) = en el origen.
2 + e1/x

Solución

Si x → 0+ , 1/x → +∞ y e1/x → +∞, por lo que lı́m f (x) = 0.


x→0+

Si x → 0− , 1/x → −∞ y e1/x → 0, por lo que lı́m f (x) = 1/2.


x→0−

Como existen pero son distintos los lı́mites laterales, la función presenta una
discontinuidad de primera especie finita.

PROBLEMA 4.18.

1 + e1/x
Estudiar la continuidad de la función f (x) = .
1 − e1/x

Solución

El denominador 1 − e1/x nunca se anula por lo que la función está definida


en R \ {0}. Además,

1 + e1/x
 
2
lı́m = lı́m −1 + = −1, pues lı́m e1/x = ∞;
x→0+ 1 − e1/x x→0+ 1 − e1/x x→0+

1 + e1/x
 
2
lı́m = lı́m −1 + = 1, pues lı́m e1/x = 0.
x→0− 1 − e1/x x→0− 1 − e1/x x→0−

131
Se deduce que la función presenta una discontinuidad de salto finito en x = 0.

PROBLEMA 4.19.

sen(1/x)
Estudiar la continuidad de la función f (x) = .
1 + e1/x

Solución

El único punto donde no está definida la función es x = 0.


Cuando x → 0+ , el numerador se mantiene acotado mientras el denominador
sen 1/x
tiende a infinito. Luego lı́m = 0.
x→0+ 1 + e1/x

Cuando x → 0− , el denominador tiende a uno, pero como lı́m sen(1/x) no


x→0
existe, tampoco existe el lı́mite del cociente.
Tenemos entonces una discontinuidad esencial o de segunda especie.

PROBLEMA 4.20.

etg x + 2
Estudiar la continuidad de la función f (x) = en el punto
etg x − 2
x = π/2.

Solución

Teniendo en cuenta que lı́m etg x = e−∞ = 0, resulta


x→(π/2)+

etg x + 2
 
4
lı́m = lı́m 1 + = −1.
x→(π/2)+ etg x − 2 x→(π/2)+ etg x − 2

Por otra parte, como lı́m etg x = e∞ = ∞, resulta


x→(π/2)−

etg x + 2
 
4
lı́m = lı́m 1 + tg x = +1.
x→(π/2)− etg x − 2 x→(π/2)− e −2
Se presenta entonces una discontinuidad finita de primera especie o de salto.

132
PROBLEMA 4.21.

1
Estudiar la continuidad de la función f (x) = ln .
1 + ex

Solución

1
Como para todo x, 1 + ex 6= 0 y > 0, la función está definida en todo
1 + ex
R, y es continua por ser el logaritmo de una función continua.

PROBLEMA 4.22.

Estudiar la continuidad de las funciones


a) f (x) = ln(x + 2)(x + 3).
b) f (x) = ln sen x.
c) f (x) = ln tg x.

Solución

a) La función es continua en su dominio, es decir, donde (x + 2)(x + 3) > 0.


La solución de esta inecuación es (−∞, −3) ∪ (−2, ∞).
b) Como en el caso a), la función f (x) = ln sen x es continua donde sen x >
0, es decir, en todos los intervalos de la forma (2kπ, (2k + 1)π) con
k ∈ Z.
c) También en este caso, el dominio es el conjunto de puntos para los que
tg x > 0, lo cual ocurre en los intervalos de la forma (kπ, (2k + 1)π/2),
con k ∈ Z, y en el dominio la función es continua.

PROBLEMA 4.23.

Estudiar las discontinuidades de la función


etg x − 1
f (x) = .
etg x + 1

133
Solución

Como el denominador nunca se anula, los únicos puntos donde puede haber
π
discontinuidad son de la forma x = + kπ, pues en ellos la tangente no
2
está definida.

Por una parte, como tg x → −∞ y etg x → 0 cuando x → (π/2 + kπ)+ , se


tiene:
etg x − 1
lı́m = −1.
x→(π/2+kπ)+ etg x + 1

Por otra parte, debido a que tg x → ∞ y etg x → ∞, cuando x → (π/2+kπ)− ,


se tiene:
etg x − 1 1 − etg1 x
lı́m = lı́m 1 = 1.
x→(π/2+kπ)− etg x + 1 x→(π/2+kπ)− 1 + tg
e x

En dichos puntos la función presenta pues discontinuidad finita de primera


especie.

PROBLEMA 4.24.

1 1
a) Demostrar que la función f (x) = sen presenta una discontinui-
x x
dad de segunda especie en x = 0.
1
b) Demostrar que la función f (x) = x sen presenta una discontinui-
x
dad evitable en x = 0.

Solución

a) Basta observar que en todo entorno de x = 0 la función toma infinitas


veces cualquier valor, porque sen 1/x oscila entre −1 y 1 y 1/x tiende
a infinito.
1
b) Como | sen(1/x)| ≤ 1 y lı́m x = 0, se tendrá que lı́m x sen = 0 y la
x→0 x→0 x
discontinuidad es evitable. Si definimos f (0) = 0, la función queda
continua.

134
PROBLEMA 4.25.

Estudiar las discontinuidades de las funciones:


 
1
a) f (x) = x sen .
sen(1/x)
sen ln x
b) f (x) = , (x > 0, x 6= 1).
ln x

Solución

a) La función no está definida cuando x = 0 y sen(1/x) = 0, es decir en los


1
puntos x = 0 y x = , (k ∈ Z, k 6= 0).

1
Como lı́m = ∞, la función presenta discontinuidades de se-
x→1/(kπ) sen 1
x
1
gunda especie en los puntos x = , (k ∈ Z, k 6= 0).

En x = 0 se tiene que f (x) → 0 por ser el producto de una fun-
ción acotada por una función con lı́mite cero. En este caso tenemos
discontinuidad evitable.
sen ln x
b) Debido a la equivalencia sen ln x ∼ ln x, lı́m = 1. Esto quiere de-
x→1 ln x
cir que la discontinuidad en x = 1 es evitable. En el resto del dominio,
la función es continua.

PROBLEMA 4.26.

Estudiar la continuidad de la función



−3x
 si x < 0,
2
f (x) = −x + 2x si 0 ≤ x ≤ 2,

1 si x > 2.

Solución

El dominio de la función está formado por todos los números reales pues la
unión de los intervalos donde está definida da todo R y en cada intervalo la

135
función está definida. Estudiaremos la continuidad por separado en cada in-
tervalo de definición de la función, considerando aparte los puntos extremos
de cada intervalo.
- Si x < 0, la función es una recta, y por lo tanto, continua.
- Si x = 0:
lı́m f (x) = lı́m −3x = 0;
x→0− x→0−
lı́m f (x) = lı́m (−x2 + 2x) = 0.
x→0+ x→0+

Como además f (0) = 0, la función es continua en x = 0.


- Si 0 < x < 2, la función es un polinomio de grado dos, y por lo tanto,
continua.
- Si x = 2:
lı́m f (x) = lı́m (−x2 + 2x) = 0;
x→2− x→2−
lı́m f (x) = lı́m 1 = 1.
x→2+ x→2+

Por tanto, la función tiene una discontinuidad de salto en x = 2.


- Si x > 2, la función es constante con lo que también es continua.
En definitiva, la función es continua en todo R salvo en x = 2, donde presenta
una discontinuidad de salto finito. Este salto es de una unidad.
Gráficamente la situación es la siguiente:

0 1 2

PROBLEMA 4.27.
(
x2 sen(1/x) si x 6= 0,
¿Es continua la función f (x) = en el pun-
0 si x = 0,
to x = 0?

136
Solución

Debido a que f (x) en x 6= 0 es el producto de una función sen(1/x) acotada


y una función x2 con lı́mite cero, entonces lı́m f (x) = f (0) = 0, con lo que
x→0
la función es continua en x = 0.

PROBLEMA 4.28.
(
x sen(ln x2 ) si x 6= 0,
Dada la función f (x) = probar que es con-
0 si x = 0,
tinua en x = 0.

Solución

Debido a que | sen(ln x2 )| ≤ 1, se tiene que lı́m x sen(ln x2 ) = 0. Como


x→0
f (0) = lı́m f (x), la función es continua en x = 0.
x→0

PROBLEMA 4.29.

Estudiar la continuidad de las funciones siguientes en los puntos


que se indican:
(
sen x
x si x 6= 0,
a) f (x) = en x = 0.
0 si x = 0,
( 3
x −8
2 si x 6= 2
b) f (x) = x −4 en x = 2.
3 si x = 2,
(
sen πx si 0 < x < 1,
c) f (x) = en x = 1.
ln x si 1 < x < 2

Solución

sen x
a) Como lı́m = 1 6= f (0) = 0, se tiene una discontinuidad evitable en
x→0 x
x = 0.

137
b) La función es continua en x = 2 porque

x3 − 8 (x − 2)(x2 + 2x + 4) 4+4+4
lı́m 2
= lı́m = = 3 = f (2).
x→2 x − 4 x→2 (x − 2)(x + 2) 4

c) En x = 1, los lı́mites laterales son

lı́m f (x) = lı́m sen πx = 0; lı́m f (x) = lı́m ln x = 0.


x→1− x→1− x→1+ x→1+

Como la función no está definida en x = 1, tiene una discontinuidad


evitable.

PROBLEMA 4.30.

x si x > 1,
e

Estudiar la continuidad de f (x) = 0 si x = 1,

e−x+1 si x < 1.

Solución

Los lı́mites laterales son

lı́m f (x) = lı́m (e − x + 1) = e; lı́m f (x) = lı́m ex = e.


x→1− x→1− x→1+ x→1+

La discontinuidad es evitable, porque existe lı́m f (x) pero es distinto de


x→1
f (1).

PROBLEMA 4.31.
(
x3 − x si x ≤ 1,
Dada la función f (x) = 2
encontrar los valores
k x−4 si x > 1,
que debe tomar k para que sea continua.

Solución

El dominio de la función está formado por todos los números reales. Pa-
ra ver si es continua debemos tomar en cuenta la forma de la función en

138
cada intervalo de definición y en los puntos donde cambia la forma de las
ecuaciones.

- Si x < 1, la función es un polinomio de grado 3, y por lo tanto, conti-


nua.

- Si x > 1, se trata de una recta que es también continua.

- Si x = 1:

lı́m f (x) = lı́m (x3 − x) = 0;


x→1− x→1−
lı́m f (x) = lı́m (k 2 x − 4) = k 2 − 4.
x→1+ x→1+

Para que la función sea continua en x = 1, deben ser los lı́mites laterales
iguales, es decir, k 2 − 4 = 0 de donde se obtienen dos posibles valores para
k, k = 2 y k = −2. Para cualesquiera de ellos, la función será continua en
todo R.

PROBLEMA 4.32.

Estudiar la continuidad de la función compuesta f ◦g , donde f (x) =


signo (x), g(x) = x(1 − x2 ).

Solución

 
1
 si x > 0 1
 si g(x) > 0
Como f (x) = 0 si x = 0 , (f ◦ g)(x) = 0 si g(x) = 0 .
 
−1 si x < 0 −1 si g(x) < 0
 

Basta pues estudiar el signo de g. Para ello hacemos la descomposición


g(x) = x(1 − x)(1 + x), y realizamos el correspondiente estudio de signos
(ver capı́tulo 1); resulta que g(x) > 0 en (−∞, −1) ∪ (0, 1), g(x) = 0 en
{−1, 0, 1} y g(x) < 0 en (−1, 0) ∪ (1, ∞).

1
 si (−∞, −1) ∪ (0, 1)
Resulta en definitiva que (f ◦ g)(x) = 0 si x ∈ {−1, 0, 1} . Por lo

−1 si (−∞, −1) ∪ (0, 1)

tanto, es continua en R \ {−1, 0, 1} y en los puntos x = −1, 0, 1 tiene discon-
tinuidades finitas de primera especie.

139
PROBLEMA 4.33.

Dadas las funciones


( (
x si 0 < x ≤ 1, x si x ∈ Q,
f (x) = g(x) =
2−x si 1 < x < 2, 2−x si x ∈
6 Q,
estudiar la continuidad de f ◦ g en (0, 2).

Solución

(
f (x) si x es racional,
Por definición, (f ◦ g)(x) = .
f (2 − x) si x es irracional
Tenemos pues:
- Si x ∈ (0, 1] y es racional, (f ◦ g)(x) = f (x) = x.
- Si x ∈ (0, 1] y es irracional, (f ◦ g)(x) = f (2 − x) = 2 − (2 − x) = x.
- Si x ∈ (1, 2) y es racional, (f ◦ g)(x) = f (x) = 2 − x.
- Si x ∈ (1, 2) y es irracional, (f ◦ g)(x) = f (2 − x) = 2 − x.
En definitiva, f ◦g = f en el intervalo (0, 2) y la función es continua en dicho
intervalo, porque lı́m (f ◦ g)(x) = lı́m (f ◦ g)(x) = (f ◦ g)(1) = 1.
x→1− x→1+

PROBLEMA 4.34.

Sea f una función continua en x = 0 y que satisface f (x + y) =


f (x) + f (y), ∀x, y . Probar que f es continua en todo x.

Solución

Probaremos en primer lugar que f (0) = 0. Para ello basta aplicar la hipótesis
a los puntos x y 0; resulta: f (x) = f (x+0) = f (x)+f (0) =⇒ f (0) = 0.
A continuación, teniendo en cuenta que lı́m f (y) = f (0) por la continui-
y→0
dad de f en el origen, veremos que lı́m f (x + y) = f (x) lo que garantiza la
y→0
continuidad de f en cualquier x:
lı́m f (x + y) = lı́m [f (x) + f (y)] = f (x) + lı́m f (y) = f (x) + f (0) = f (x).
y→0 y→0 y→0

140
B. PROPIEDADES DE LAS FUNCIONES CONTINUAS.

Tres propiedades importantes de las funciones continuas que serán útiles en


las aplicaciones son:

(a) Teorema de Weierstrass (existencia de máximos y mı́nimos).

Toda función continua en un intervalo cerrado alcanza sus valo-


res máximo y mı́nimo, es decir, existen por lo menos dos puntos
x0 y x1 en el intervalo cerrado [a, b] tales que f (x0 ) ≥ f (x) ≥ f (x1 )
para cualquier x ∈ [a, b].

En este caso se dice que la función alcanza en el intervalo [a, b] el


máximo cuando x = x0 y el mı́nimo cuando x = x1 .

(b) Teorema de Bolzano (existencia de raı́ces).

Sea f una función continua en un intervalo cerrado [a, b] y tal que


f (a) y f (b) son de distinto signo. Entonces existe algún c ∈ (a, b)
tal que f (c) = 0, es decir, su gráfica corta al eje X en algún punto
del intervalo (a, b).

Este resultado es evidente gráficamente pero la demostración debe


hacerse de forma rigurosa. Además debe tenerse en cuenta que pueden
existir varios valores donde la gráfica de la función corte al eje de
abscisas.

(c) Teorema del valor intermedio o propiedad de Darboux.

Una función continua en un intervalo cerrado [a, b] alcanza todos


los valores comprendidos entre f (a) y f (b) al menos una vez.

Esto quiere decir que f toma todos los valores intermedios entre f (a)
y f (b) aunque pueda tomar en algunos casos otros valores aparte de
ellos.

Los siguientes problemas ilustran la aplicación de estos teoremas.

141
PROBLEMA 4.35.

Demostrar que existe algún número que sea igual a su cubo menos
uno.

Solución

Sea x dicho número. Debe verificar la ecuación x = x3 −1, o bien x3 −x−1 =


0.
Si construimos la función f (x) = x3 − x − 1, el problema se puede plantear
ahora ası́:
Probar que la función f (x) toma el valor cero en algún punto.
Utilizando el teorema de Bolzano, eso se podrá asegurar si la función f (x)
es continua en algún intervalo cerrado [a, b] en el cual f (a) y f (b) tengan
diferente signo.
Evidentemente la función es continua en todo R pues se trata de un polino-
mio de grado 3.
Comprobando .a manoçon algunos puntos, obtenemos que f (1) = −1, f (2) =
5. De aquı́ se deduce que en el intervalo [1, 2] la función pasa de valores ne-
gativos a valores positivos. Como el cero es un número comprendido entre
-1 y 5, en algún punto del intervalo [1, 2] la función tomará ese valor. Gráfi-
camente la situación es la siguiente:

Nota: Hemos comprobado la existencia de tal número pero no lo hemos


encontrado porque no es un número “común”. Sin embargo, el saber un
intervalo que lo contiene permite obtener buenas aproximaciones de dicho
número.

142
PROBLEMA 4.36.

Encontrar, en caso de que existan, los máximos y mı́nimos de la


función f (x) = |x| en los intervalos que se indican.
(a) [1, 3]; (b) [−1, 1]; (c) (−1, 3).

Solución

La función dada es continua en todo R. Por lo tanto en cualquier intervalo


cerrado se cumplirán las condiciones del teorema de Weierstrass.
(a) Como [1, 3] es un intervalo cerrado, la función alcanza los valores máxi-
mo y mı́nimo. Además en este intervalo f (x) = x es la bisectriz del
primer cuadrante. Como toma valores cada vez mayores (la función es
creciente), el mı́nimo será f (1) y el máximo f (3).
(b) En este caso también se cumplen las condiciones del teorema de Weiers-
trass. Sin embargo la función decrece desde -1 hasta 0 y crece desde
0 hasta 1. Además f (−1) = f (1) = 1. En ambos puntos se alcanza el
máximo. El mı́nimo corresponde a x = 0 y vale f (0) = 0.
(c) Como el intervalo (−1, 3) no es cerrado, no se puede asegurar que se
alcancen los valores máximo y mı́nimo. En este caso no hay máximo
(corresponderı́a a x = 3 pero no pertenece al intervalo) pero sı́ hay
mı́nimo en x = 0.

PROBLEMA 4.37.

Supongamos que f y g son dos funciones continuas tales que f (0) <
g(0) < g(1) < f (1). Probar que existe algún c ∈ (0, 1) para el cual
f (c) = g(c).

Solución

Construimos la función h(x) = f (x) − g(x). Por ser diferencia de dos fun-
ciones continuas, h también será continua. Además h(0) = f (0) − g(0) < 0
y h(1) = f (1) − g(1) > 0.
La función h cumple todas las condiciones del teorema de Bolzano. Se con-
cluye que para algún valor c del intervalo (a, b) se verifica que h(c) = 0, es
decir f (c) − g(c) = 0. Por lo tanto, f (c) = g(c).

143
PROBLEMA 4.38.

Sea f una función continua en [a, b] tal que f (x) ∈ Q, ∀x. Probar
que f es constante.

Solución

Haremos la demostración por reducción al absurdo. Si f no fuera constan-


te, existirı́an x1 , x2 ∈ [a, b] tales que f (x1 ) 6= f (x2 ). Por la propiedad de
Darboux, en el intervalo que determinan x1 y x2 , f debe alcanzar todos los
valores comprendidos entre f (x1 ) y f (x2 ). Pero entre dos números reales hay
siempre algún número irracional, lo que contradice el hecho de que f sólo
toma valores racionales. Esta contradicción permite asegurar que f debe ser
constante.

144
C. EJERCICIOS PROPUESTOS.

1.- Estudiar la continuidad de las siguientes funciones:


x
a) f (x) = x + .
|x|
Resp.: Discontinuidad de salto finito en x = 0. Continua en el resto.
b) f (x) = x[x].

Resp.: Discontinuidad de salto finito en Z \ {0}. Continua en el resto.

c) f (x) = [3x].
Resp.: Discontinuidad de salto finito en el conjunto {k/3 : k ∈ Z}.

d) f (x) = x[1/x].
Resp.: Discontinuidad evitable en x = 0; discontinuidad de salto finito
en x = 1/n, n ∈ Z \ {0}; continua en el resto.

e) f (x) = [x] sen x.


Resp.: Discontinuidad de salto finito en Z \ {0}; continua en el resto.

f) f (x) = sen 1/x.


Resp.: Discontinuidad esencial en x = 0 (no existe el lı́mite de la
función cuando x → 0). Continua en R \ {0}.

5 · 21/x − 7
g) f (x) = .
25 · 21/x + 8
Resp.: Discontinuidad de salto finito en x = 0; continua en el resto.

1
h) f (x) = , a > 0, b > 1.
a + btg x
Resp.: La función es continua en todo x 6= π/2 + kπ. En el resto, la
discontinuidad es infinita de primera especie.

sen(1/x)
i) f (x) = .
e1/x + 1

145
Resp.: En el punto x = 0 se presenta una discontinuidad de segunda
especie.

(
x
|x| si x ≤ π/2,
j) f (x) =
tg x si x > π/2.
Resp.: Salto finito en x = 0; salto infinito en x = π/2 + kπ, k ∈ N.


0
√ si x < 1,
k) f (x) = x−1 si 1 ≤ x ≤ 2,

(x − 1)2 + 2 si x > 2.

Resp.: Discontinuidad de salto finito en x = 2. Continua en el resto. l)





x−1 si x < 0,

−2 si x = 0,
f (x) = 2
3x + 1

 si 0 < x ≤ 1,

| − 6x + 2| si x > 1.
Resp.: Discontinuidad de salto finito en x = 0; continua en el resto.

(
1
1+2tg x si x 6= π/2,
m) f (x) =
1/2 si x = π/2.
Resp.: Discontinuidad de salto finito en x = (π/2) + kπ, k ∈ Z.

(
0 si x es irracional,
n) f (x) =
sen πx si x es racional.
Resp.: Continua en Z; discontinuidad de segunda especie en el resto.

2.- Encontrar los valores de las constantes para que las siguientes
funciones sean continuas en su dominio.
(
x2 − 1 si x < 3,
a) f (x) =
2ax si x ≥ 3.
Resp.: a = 4/3.


bx
 si x < −1,
b) f (x) = bx2 − a si − 1 ≤ x ≤ 1,

−x si x > 1.

146
Resp.: a = 2, b = 1.

√ √
x+2− 3
 x−1
 si x > 1,
3.- Dada la función f (x) = 4 si x = 1, encontrar el valor

p x
2 +p si x < 1,
de p para que exista lı́m f (x). ¿Dónde es f (x) continua?
x→1

Resp.: p = −5/12. La función es continua en su dominio excepto en


x = 1 (discontinuidad evitable). D(f ) = [5/6, ∞).


4.- Sean f (x) = x3 + 3x − 2, g(x) = x. Estudiar la continuidad de
f ◦ g.
Resp.: Continua en su dominio [0, ∞).

(
f (x) si x 6= 0,
5.- Demostrar que si existe lı́m f (x)/x, entonces g(x) =
x→0 0 si x = 0
es continua en x = 0.
Resp.: lı́m f (x) = 0 porque, en caso contrario, no existirı́a el lı́mite del
x→0
cociente. Esto asegura la continuidad de g.

6.- Encontrar, en caso de que existan, los máximos y mı́nimos de las


siguientes funciones en los intervalos que se indican:
(
x2 si x ≤ a,
a) f (x) = en (−a − 1, a + 1).
a+2 si x > a,
Resp.: Si −1 < a ≤ −1/2, f es constante y máx f = mı́n f = a + 2.
Si −1/2 < a ≤ 0, máx f = a + 2 y se alcanza en x = 0; mı́n f = a2
y se alcanza en x = a.

Si 0 < a ≤ (1 + 5)/2, máx f = a + 2 y se alcanza en el intervalo
(a, a + 1); mı́n f = 0 y se alcanza en x = 0.

Si (1 + 5)/2 < a, f ( no alcanza el máximo; mı́n f = 0 y se alcanza
x si x es racional,
en x = 0. b) f (x) = en [0, a].
0 si x es irracional,

Resp.: Si a es racional, máx f = a y mı́n f = 0. Si a es irracional, f no


alcanza el máximo pero mı́n f = 0.

147
7.- Probar que la ecuación x5 − 2x4 + x − 2 = 0 tiene una raı́z real en
el intervalo (1, 3).
Sugerencia: Comprobar el teorema de Bolzano.

8.- Sea f : [0, 1] → [0, 1] una función continua. Probar que existe algún
punto c en [0, 1] para el cual f (c) = c.
Sugerencia: Comprobar que la función g(x) = f (x) − x verifica el
teorema de Bolzano en el intervalo [0, 1].

163
9.- Probar que la ecuación x179 + = 119 tiene solución.
1 + + sen2 x
x2
Sugerencia: Utilizar el teorema del valor intermedio para una función
adecuada en el intervalo conveniente.

10.- Sea f (x) = tg x. Comprueba que f (π/4) = 1 y f (3π/4) = −1 y que


en el intervalo [π/4, 3π/4] no hay ningún valor x tal que f (x) = 0.
¿Contradice esto el teorema de Bolzano? ¿Por qué?
Resp.: La función no es continua en x = π/2.

11.- Sean f y g dos funciones continuas en el intervalo [a, b] con


f (a) < g(a) y f (b) > g(b). Demostrar que f (x) = g(x) para algún
x ∈ (a, b).
Sugerencia: Estudiar la función h(x) = f (x) − g(x).

12.- Probar que existe un número real x tal que x5 − 4x + 1 = 7, 21.

13.- Dada la función f (x) = x5 + 5x4 + 2x + 1, encontrar un entero n


tal que f (x) = 0 para algún valor x del intervalo (n, n + 1).

14.- Demostrar que existe algún x tal que sen x = x − 1.

15.- Decidir si son verdaderos o falsos los siguientes planteamientos:


(a) Si f es continua en x = a y g es discontinua en x = a,
entonces f + g es discontinua en x = a.
Resp.: Verdadero.

(b) Si f y g son discontinuas en x = a, entonces f · g es discon-


tinua en x = a.

148
Resp.: Falso. Ejemplo: f (x) = 0 si x 6= 0 y f (0) = 1; g(x) = 1 si x 6= 0
y g(0) = 0.

(c) Si una función f está definida en el intervalo cerrado [a, b] y


toma todos los valores comprendidos entre f (a) y f (b), entonces
es continua en [a, b].
Resp.: Falso. Ejemplo: f (x) = x − [x] en [0, 2].

(d) Si f es una función continua que verifica [f (x)]2 = x2 , enton-


ces f (x) = x.
Resp.: Falso. Ejemplo: f (x) = −x.

(e) Si |f | es continua en x = a, entonces f es continua en x = a.


Resp.: Falso. Ejemplo: f (x) = x + 1 si x ≥ 0 y x − 1 si x < 0.

(f) Si f y g son continuas en x = a, entonces máx{f, g} es conti-


nua en x = a.
Resp.: Verdadero.

149
CAPÍTULO V.
DERIVABILIDAD DE
FUNCIONES

SECCIONES
A. Definición de derivada.
B. Reglas de derivación.
C. Derivadas sucesivas.
D. Funciones implı́citas. Derivación logarı́tmica.
E. Ecuaciones paramétricas.
F. Recta tangente y normal.
G. Ejercicios propuestos.

151
A. DEFINICIÓN DE DERIVADA.

Una función y = f (x) se dice que es derivable en un punto c del dominio


cuando existe el lı́mite del cociente incremental siguiente:

f (x) − f (c)
(1a) lı́m .
x→c x−c
Cada uno de los lı́mites laterales de la expresión anterior se llama derivada
lateral de f en el punto x = c. Cuando las dos derivadas laterales existen
(son finitas) y son iguales, la función es derivable en x = c y el resultado se
llama derivada de la función en x = c. Otra forma de expresar la derivada
de una función f en el punto c es:

f (c + h) − f (c)
(1b) lı́m .
h→0 h
La fórmula (1a) la aplicaremos para calcular derivadas de funciones en pun-
tos particulares. Sin embargo es más conveniente utilizar (1b) para calcular
derivadas de funciones en puntos genéricos.
Observa que para calcular estos lı́mites se debe resolver la forma indeter-
minada 0/0, para lo cual utilizaremos las técnicas mostradas en el capı́tulo
3.
De la definición se deduce que toda función derivable en un punto es nece-
sariamente continua en dicho punto.
La notación que utilizaremos para expresar la derivada de una función es
alguna de las siguientes:

df (x) dy
f 0 (x) = Df (x) = o bien y 0 = Dy = .
dx dx
Para las derivadas laterales se usará la notación análoga f 0 (x+ ) o bien
f 0 (x− ), según sea el caso.

PROBLEMA 5.1.

1 1
Calcular la derivada de la función f (x) = + en el punto x =
x x2
−1/2.

152
Solución

Utilizando la fórmula (1a) y teniendo en cuenta que f (−1/2) = 2, resul-


ta:

f (x) − f (−1/2) 1/x + 1/x2 − 2


f 0 (−1/2) = lı́m = lı́m
x→−1/2 x + 1/2 x→−1/2 x + 1/2
x + 1 − 2x2 −(2x + 1)(x − 1)
= lı́m 2
= lı́m
x→−1/2 x (2x + 1)/2 x→−1/2 x2 (2x + 1)/2
−(x − 1) 3/2
= lı́m 2
= = 12.
x→−1/2 x /2 1/8

PROBLEMA 5.2.


Calcular la derivada de la función f (x) = 3x − 2.

Solución

En este caso utilizaremos la fórmula (1b) para calcular la derivada en un


punto cualquiera:

p √
0 f (x + h) − f (x) 3(x + h) − 2 − 3x − 2
f (x) = lı́m = lı́m
h→0 h h→0 h
p √ p √
( 3(x + h) − 2 − 3x − 2)( 3(x + h) − 2 + 3x − 2)
= lı́m p √
h→0 h( 3(x + h) − 2 + 3x − 2)
3(x + h) − 2 − (3x − 2)
= lı́m p √
h→0 h( 3(x + h) − 2 + 3x − 2)

3h 3
= lı́m p √ = √ .
h→0 h( 3(x + h) − 2 + 3x − 2) 2 3x − 2

153
PROBLEMA 5.3.

Demostrar:
d
a) (c) = 0 siendo c una constante arbitraria.
dx
d
b) (x) = 1.
dx
d
c) (cx) = c.
dx
d n
d) (x ) = nxn−1 .
dx

Solución

d c−c
a) (c) = lı́m = lı́m 0 = 0.
dx h→0 h h→0

d x+h−x h
b) (x) = lı́m = lı́m = 1.
dx h→0 h h→0 h

d c(x + h) − cx ch
c) (cx) = lı́m = lı́m = c.
dx h→0 h h→0 h

d) Aplicando la fórmula del binomio de Newton,

xn + nxn−1 h + n2 xn−2 h2 + · · · + hn − xn

d n (x + h)n − xn
(x ) = lı́m = lı́m
dx h→0 h h→0 h
nxn−1 h + n2 xn−2 h2 + · · · + hn

= lı́m
  h
h→0
n n−2
= lı́m (nxn−1 + x h + · · · + hn−1 ) = nxn−1 .
h→0 2

PROBLEMA 5.4.

Utilizando la definición, calcular las derivadas de las siguientes


funciones en los puntos que se indican:
3+x
a) f (x) = en x = 2.
3−x

b) f (x) = 2x − 1 en x = 5.

154
Solución

a) Como f (2) = 5, tenemos


 
0 f (2 + h) − f (2) 1 5+h
f (2) = lı́m = lı́m −5
h→0 h h→0 h 1−h
1 6h 6
= lı́m · = lı́m = 6.
h→0 h 1 − h h→0 1 − h

b) En este caso f (5) = 3, con lo que



0 f (5 + h) − f (5) 9 + 2h − 3
f (5) = lı́m = lı́m
h→0 h h→0 h
√ √
9 + 2h − 3 9 + 2h + 3
= lı́m ·√
h→0 h 9 + 2h + 3
9 + 2h − 9 2 1
= lı́m √ = lı́m √ = .
h→0 h( 9 + 2h + 3) h→0 9 + 2h + 3 3

PROBLEMA 5.5.

1
Hallar la derivada de y = en los puntos x = 1 y x = 3.
x−2
Demostrar que la función no es derivable en el punto x = 2 en el
que presenta una discontinuidad.

Solución

Para x 6= 2,
1 1
f (x + h) − f (x) −
f 0 (x) = lı́m = lı́m x+h−2 x−2
h→0 h h→0 h
−1 −1
= lı́m = .
h→0 (x − 2)(x + h − 2) (x − 2)2

−1 −1
Ası́, f 0 (1) = = −1 y f 0 (3) = = −1.
(1 − 2)2 (3 − 2)2
Sin embargo, como la función no está definida en x = 2, no es continua, con
lo que tampoco será derivable en dicho punto.

155
PROBLEMA 5.6.

2x − 3
Hallar la derivada de f (x) = y demostrar que la función no
3x + 4
es derivable en el punto x = −4/3 en el que presenta una disconti-
nuidad.

Solución

Por la definición de derivada,


2(x+h)−3 2x−3
0 f (x + h) − f (x) 3(x+h)+4 − 3x+4
f (x) = lı́m = lı́m
h→0 h h→0 h
17h 17 17
= lı́m = lı́m = .
h→0 h(3x + 4)(3x + 3h + 4) h→0 (3x + 4)(3x + 3h + 4) (3x + 4)2

Para x = −4/3 la función no es derivable porque se anula el denominador.

PROBLEMA 5.7.


Hallar la derivada de f (x) = 2x + 1.

Solución

La función está definida en el intervalo [−1/2, ∞) por lo que es derivable en


el intervalo abierto (−1/2, ∞). Tenemos:
p √
0 f (x + h) − f (x) 2(x + h) + 1 − 2x + 1
f (x) = lı́m = lı́m
h→0 h h→0 h
p √ p √
( 2(x + h) + 1 − 2x + 1)( 2(x + h) + 1 + 2x + 1)
= lı́m p √
h→0 h( 2(x + h) + 1 + 2x + 1)
[2(x + h) + 1] − (2x + 1)
= lı́m p √
h→0 h( 2(x + h) + 1 + 2x + 1)

2 2 1
= lı́m p √ = √ =√ .
h→0 2(x + h) + 1 + 2x + 1 2 2x + 1 2x + 1

156
PROBLEMA 5.8.

Calcular la derivada de f (x) = x1/3 y como aplicación estudiar


f 0 (0).

Solución

De acuerdo a la definición, tenemos:

f (x + h) − f (x) (x + h)1/3 − x1/3


f 0 (x) = lı́m = lı́m
h→0 h h→0 h
[(x + h)1/3 − x1/3 ][(x + h)2/3 + (x + h)1/3 x1/3 + x2/3 ]
= lı́m
h→0 h[(x + h)2/3 + (x + h)1/3 x1/3 + x2/3 ]
(x + h) − x
= lı́m 2/3
h→0 h[(x + h) + (x + h)1/3 x1/3 + x2/3 ]
1 1
= lı́m 2/3 1/3 1/3 2/3
= 2/3 .
h→0 (x + h) + (x + h) x + x 3x

La función no es derivable en x = 0 porque el denominador es cero. Obsérvese


sin embargo que la función sı́ es continua en el punto x = 0. Geométrica-
mente, esto significa que la gráfica de f tiene una tangente vertical en el
punto de abscisa x = 0.

157
PROBLEMA 5.9.

Utilizando la definición, calcular las derivadas de las siguientes


funciones:
a) f (x) = sen x.
b) f (x) = arc sen x.

Solución

a) Según la definición:
f (x + h) − f (x) sen(x + h) − sen x
f 0 (x) = lı́m = lı́m
h→0 h h→0 h
2 cos x+h+x
2 sen x+h−x
2 cos 2x+h
2 sen 2
h
= lı́m = lı́m = cos x,
h→0 h h→0 h/2
debido a la equivalencia sen(h/2) ∼ h/2.
arc sen(x + h) − arc sen x
b) Como f 0 (x) = lı́m , si hacemos el cambio de
h→0 h
variable k = arc sen(x + h) − arc sen x y llamamos t = arc sen x, enton-
ces x = sen t, x + h = sen(t + k), de modo que h = sen(t + k) − sen t
y cuando h → 0, también k → 0. Tenemos pues:
k 1 1
f 0 (x) = lı́m = lı́m sen(t+k)−sen t
= ,
k→0 sen(t + k) − sen t k→0 cos t
k
pues el denominador corresponde a la derivada de sen t. Deshaciendo
el cambio de variable,
1 1 1
f 0 (x) = =p =√ .
cos(arc sen x) 2
1 − sen (arc sen x) 1 − x2

PROBLEMA 5.10.
(
x sen(1/x) si x 6= 0,
Sea f (x) =
0 si x = 0.
a) ¿Es f continua en x = 0?
b) ¿Es derivable en x = 0?

158
Solución

a) Como f es producto de sen 1/x, que es acotada, y x, que tiene lı́mite


0 cuando x → 0, lı́m f (x) = 0 = f (0). Esto indica que la función es
x→0
continua en x = 0.
b) Por otra parte,

f (0 + h) − f (0) f (h) − f (0)


f 0 (0) = lı́m = lı́m
h→0 h h→0 h
h sen 1/h − 0
= lı́m = lı́m sen 1/h.
h→0 h h→0

Como este lı́mite no existe, la función, aun siendo continua, no tiene


derivada en el punto x = 0.

PROBLEMA 5.11.

Sea f (x) = |x|. Probar que f no es derivable en x = 0 calculando


las derivadas laterales.

Solución

(
x si x ≥ 0
Debido a que |x| = , tenemos
−x si x < 0

f (h) − f (0) |h| − 0 h


f+0 (0) = lı́m = lı́m = lı́m = 1;
h→0+ h h→0 + h h→0 h+

f (h) − f (0) |h| − 0 −h


f−0 (0) = lı́m = lı́m = lı́m = −1.
h→0− h h→0− h h→0− h

Al ser distintas las derivadas laterales de la función en el punto x = 0, la


función no es derivable en dicho punto.

PROBLEMA 5.12.

Comprobar si la función f (x) = x2 + 1 + |2x − 1| es o no derivable


en x = 1/2.

159
Solución

Debido a la presencia del valor absoluto, debemos calcular las derivadas


laterales y ver si son iguales:
f (x) − f (1/2) x2 + 1 − (2x − 1) − 5/4
f 0 (1/2− ) = lı́m = lı́m
x→1/2− x − 1/2 x→1/2− x − 1/2
(x − 1/2)(x − 3/2)
= lı́m = lı́m (x − 3/2) = −1.
x→1/2− x − 1/2 x→1/2−

f (x) − f (1/2) x2 + 1 + (2x − 1) − 5/4


f 0 (1/2+ ) = lı́m = lı́m
x→1/2+ x − 1/2 x→1/2+ x − 1/2
(x − 1/2)(x + 5/2)
= lı́m = lı́m (x + 5/2) = 3.
x→1/2+ x − 1/2 x→1/2+

La función no es derivable en x = 1/2 pues las derivadas laterales, aunque


existen, son distintas. Como se observa en la gráfica anterior, aun siendo la
función continua en el punto, presenta un ”pico”. Esto es caracterı́stico de
las funciones continuas no derivables.

PROBLEMA 5.13.
(
x2 si x ≤ 0
Demostrar que la función f (x) = no es derivable
x si x > 0
en x = 0.

160
Solución

Las derivadas laterales son


f (h) − f (0) h
f+0 (0) = lı́m = lı́m = 1;
h→0+ h h→0 h+

f (h) − f (0) h2
f−0 (0) = lı́m = lı́m = lı́m h = 0.
h→0− h h→0− h h→0−

Como f−0 (0) 6= f+0 (0), se deduce que f no es derivable en x = 0.


La gráfica muestra el comportamiento de la función en un entorno de x =
0.

PROBLEMA 5.14.

p
Dada la función f (x) = |x|, comprobar que no es derivable en
x = 0.

Solución

Teniendo en cuenta que |h| = h si h > 0 y |h| = −h si h < 0, resulta:



0 f (h) − f (0) h 1
f+ (0) = lı́m = lı́m = lı́m √ = ∞;
h→0+ h h→0+ h h→0+ h

f (h) − f (0) −h −1
f−0 (0) = lı́m = lı́m = lı́m √ = −∞.
h→0 − h h→0 − h h→0 − −h
Lo anterior prueba que la función no es derivable en x = 0. La situación que
se presenta corresponde a la figura siguiente:

161
PROBLEMA 5.15.

Sea f una función que verifica |f (x)| ≤ x2 , ∀x. Demostrar que f


es derivable en x = 0.

Solución

Aplicando la hipótesis a x = 0 resulta que |f (0)| ≤ 0, es decir f (0) = 0.


f (x) |x|2

2 2
Además, de |f (x)| ≤ x = |x| tenemos que 0 ≤ ≤ = |x|, de don-
x |x|
de lı́m |f (x)/x| = 0. De aquı́ se deduce que también lı́m f (x)/x = 0.
x→0 x→0

f (h) − f (0) f (h)


Por tanto, f 0 (0) = lı́m = lı́m = 0, lo que prueba que f es
h→0 h h→0 h
derivable en x = 0.

PROBLEMA 5.16.

Sabiendo que la función f es derivable en x = 0, calcular


f (x0 + h) − f (x0 − h)
lı́m .
h→0 2h

Solución

Sumando y restando f (x0 ) al numerador tenemos:


f (x0 + h) − f (x0 − h) f (x0 + h) − f (x0 ) f (x0 − h) − f (x0 )
lı́m = lı́m − lı́m
h→0 2h h→0 2h h→0 2h
1 f (x0 + h) − f (x0 ) 1 f (x0 − h) − f (x0 )
= lı́m + lı́m
2 h→0 h 2 h→0 −h
1 0 1 0
= f (x0 ) + f (x0 ) = f 0 (x0 ).
2 2

162
B. REGLAS DE DERIVACIÓN.

Algunas reglas útiles que se deducen de la definición de derivada para las


operaciones algebraicas con funciones son:
(a) (f ± g)0 (x) = f 0 (x) ± g 0 (x). Derivada de la suma y la resta.
(b) (f · g)0 (x) = f 0 (x)g(x) + f (x)g 0 (x). Derivada del producto.
 0
f f 0 (x)g(x) − f (x)g 0 (x)
(c) (x) = . Derivada del cociente.
g [g(x)]2
(d) (f n )0 (x) = nf n−1 (x)f 0 (x). Derivada de la potencia.
(e) (f ◦ g)0 (x) = f 0 [g(x)] · g 0 (x). Derivada de la función compuesta (regla
de la cadena).
1
(f) (f −1 )0 (x) = . Derivada de la función inversa.
f 0 [f −1 (x)]
Algunas derivadas de funciones más comunes mediante las cuales se pueden
obtener muchas más son (u representa cualquier función de x, c y n son
constantes arbitrarias):
1.- f (x) = c =⇒ f 0 (x) = 0.
2.- f (x) = un =⇒ f 0 (x) = nun−1 · u0 .
3.- f (x) = sen u =⇒ f 0 (x) = cos u · u0 .
4.- f (x) = cos u =⇒ f 0 (x) = − sen u · u0 .
5.- f (x) = tg u =⇒ f 0 (x) = sec2 u · u0 .
6.- f (x) = sec u =⇒ f 0 (x) = sec u · tg u · u0 .
7.- f (x) = cosec u =⇒ f 0 (x) = − cosec u · cotg u · u0 .
8.- f (x) = cotg u =⇒ f 0 (x) = − cosec2 u · u0 .
u0
9.- f (x) = arc sen u =⇒ f 0 (x) = √ .
1 − u2
u0
10.- f (x) = arc cos u =⇒ f 0 (x) = − √ .
1 − u2
u0
11.- f (x) = arc tg u =⇒ f 0 (x) = .
1 + u2
12.- f (x) = au =⇒ f 0 (x) = au · ln a · u0 .
u0
13.- f (x) = loga u =⇒ f 0 (x) = .
u · ln a

163
14.- f (x) = sh u =⇒ f 0 (x) = ch u · u0 .
15.- f (x) = ch u =⇒ f 0 (x) = sh u · u0 .
16.- f (x) = th u =⇒ f 0 (x) = sech2 u · u0 .
17.- f (x) = sech u =⇒ f 0 (x) = − sech u · th u · u0 .
18.- f (x) = csch u =⇒ f 0 (x) = − csch u · coth u · u0 .
19.- f (x) = coth u =⇒ f 0 (x) = − csch2 u · u0 .
u0
20.- f (x) = argsh u =⇒ f 0 (x) = √ .
u2 + 1
u0
21.- f (x) = argch u =⇒ f 0 (x) = √ .
u2 − 1
u0
22.- f (x) = argth u =⇒ f 0 (x) = .
1 − u2

PROBLEMA 5.17.

Sea f una función derivable en x = a y g(x) = c · f (x). Probar que


g es derivable en x = a y g 0 (a) = c · f 0 (a).

Solución

Como g es el producto de dos funciones derivables, ella misma es derivable


en x = a.
Si aplicamos la regla de derivación del producto, tenemos:

g 0 (a) = (c)0 · f (a) + c · f 0 (a) = c · f 0 (a)

porque la derivada de la función constante es cero.

PROBLEMA 5.18.

Si f (x) = xn para algún n ∈ N, probar que f 0 (a) = nan−1 , ∀a.

164
Solución

Vamos a utilizar el método de inducción sobre n.


Para n = 1, es evidente que si f (x) = x, f 0 (x) = 1 y f 0 (a) = 1.
Suponemos ahora que la propiedad es cierta para n, de modo que si f (x) =
xn , entonces f 0 (a) = nan−1 , ∀a. Probaremos la propiedad para n + 1.
Sea g(x) = xn+1 . Si escribimos g(x) = x · xn y aplicamos la regla de deriva-
ción del producto, tenemos:

g 0 (x) = (x)0 · (xn ) + x · (xn )0 = xn + x · nxn−1 = xn + nxn = (n + 1)xn ,

por lo que g 0 (a) = (n + 1)an , ∀a.

PROBLEMA 5.19.

Calcular la derivada de la función f (x) = x10 + 7x5 − x3 + 1.

Solución

En este caso se debe aplicar la regla de la suma repetidas veces. Cada su-
mando es producto de una constante por una potencia de la variable x. El
resultado quedará entonces ası́:

f 0 (x) = 10x9 + 7 · 5x4 − 3x2 = 10x9 + 35x4 − 3x2 .

En general, la derivada de un polinomio es otro polinomio de grado una


unidad menor al anterior.

PROBLEMA 5.20.

Encontrar los puntos de discontinuidad de f 0 (x) si se define


   
1 1 1 1
f (x) = (x + |x|) + 2 x − + x − + 4 x − + x − .
2 2 4 4

165
Solución

Para poder derivar f (x) la estudiamos por separado en los intervalos si-
guientes:
- Si x < 0, f (x) = (x − x) + 2 x − 12 − x − 12 + 4 x − 14 − x − 14 =
 

0.
- Si 0 ≤ x < 1/4, f (x) = (x+x)+2 x − 21 − x − 12 +4 x − 14 − x − 14 =
 

2x.
- Si 1/4 ≤ x < 1/2, f (x) = (x+x)+2 x − 21 − x − 12 +4 x − 14 + x − 14 =
 

10x − 2.
- Si 1/2 ≤ x, f (x) = (x + x) + 2 x − 21 + x − 21 + 4 x − 14 + x − 14 =
 

14x − 4.
Por tanto, los puntos de discontinuidad de f 0 son x = 0, x = 1/4, x = 1/2,
porque
f 0 (0− ) = 0; f 0 (1/4− ) = 2; f 0 (1/2− ) = 10.
0 +
f (0 ) = 2; 0 +
f (1/4 ) = 10; f 0 (1/2+ ) = 14.

PROBLEMA 5.21.


3
Calcular la derivada de la función f (x) = x2 + x − 1.

Solución

Para utilizar la regla de la potencia, podemos escribir la función como f (x) =


(x2 + x − 1)1/3 .
De este modo, podemos aplicar la regla de la cadena y la derivada será el pro-
ducto de las derivadas de la potencia 1/3 por la del polinomio que está dentro
de la raı́z; resulta de la forma:

1 2
f 0 (x) = (x + x − 1)1/3−1 · (2x + 1)
3
1 2 2x + 1
= (x + x − 1)−2/3 · (2x + 1) = p .
3 3 (x2 + x − 1)2
3

166
PROBLEMA 5.22.

x
Calcular la derivada de la función f (x) = √ .
x2 − 4

Solución

Tenemos que aplicar en este caso la regla del cociente, la cual da como
resultado:
√ √ √
0 (x)0 x2 − 4 − x( x2 − 4)0 x2 − 4 − x · (1/2) · (x2 − 4)−1/2 · 2x
f (x) = √ =
( x2 − 4)2 x2 − 4
2 −4−x2
x√
x2 −4 −4 −4
= 2
= √ = 2 .
x −4 (x2 − 4) x2 − 4 (x − 4)3/2

PROBLEMA 5.23.

Derivar las siguientes funciones:


a) y = 4 + 2x − 3x2 − 5x3 − 8x4 + 9x5 .
1 3 2
b) y = + 2 + 3.
x x x
c) y = 2x1/2 + 6x1/3 − 2x3/2 .
2 6 2 4
d) y = + − − .
x1/2 x1/3 x3/2 x3/4
e) s = (t2 − 3)4 .

f) y = x2 + 6x + 3.
g) y = (x2 − 4)2 (2x3 − 1)3 .
3 − 2x
h) y = .
3 + 2x
x2
i) y = √ .
4 − x2

167
Solución

a) Por la regla de derivación de una suma, tenemos:


y 0 = 0+2·1−3·2x−5·3x2 −8·4x3 +9·5x4 = 2−6x−15x2 −32x3 +45x4 .

b) Escribimos y = x−1 + 3x−2 + 2x−3 y tenemos:


y 0 = −x−2 + 3 · (−2x−3 ) + 2 · (−3x−4 )
1 6 6
= −x−2 − 6x−3 − 6x−4 = − 2 − 3 − 4 .
x x x

c) Análogamente al caso anterior,


y 0 = 2·(1/2)x−1/2 +6·(1/3)x−2/3 −2·(3/2)x1/2 = x−1/2 +2x−2/3 −3x1/2 .

d) Si escribimos y = 2x−1/2 + 6x−1/3 − 2x−3/2 − 4x−3/4 , resulta:


y 0 = 2 · (−1/2)x−3/2 + 6 · (−1/3)x−4/3 − 2 · (−3/2)x−5/2 − 4 · (−3/4)x−7/4
1 2 3 3
= −x−3/2 − 2x−4/3 + 3x−5/2 + 3x−7/4 = − 3/2 − 4/3 + 5/2 + 7/4 .
x x x x

e) s0 = 4(t2 − 3)3 · (2t) = 8t(t2 − 3)3 .



f) Si y = x2 + 6x + 3 = (x2 + 6x + 3)1/2 , tenemos
y 0 = (1/2) · (x2 + 6x + 3)−1/2 · (x2 + 6x + 3)0
x+3
= (1/2) · (x2 + 6x + 3)−1/2 · (2x + 6) = √ .
x2 + 6x + 3

g) Según la regla de derivación del producto,


d d 2
y 0 = (x2 − 4)2 · (2x3 − 1)3 + (2x3 − 1)3 · (x − 4)2
dx dx
d d 2
= (x2 − 4)2 · 3(2x3 − 1)2 · (2x3 − 1) + (2x3 − 1)3 · 2(x2 − 4) · (x − 4)
dx dx
= (x2 − 4)2 · 3(2x3 − 1)2 · 6x2 + (2x3 − 1)3 · 2(x2 − 4) · 2x
= 2x(x2 − 4)(2x3 − 1)2 [(x2 − 4) · 3 · 3x + (2x3 − 1) · 2]
= 2x(x2 − 4)(2x3 − 1)2 (13x3 − 36x − 2).

h) Aplicamos la regla de derivación del cociente:


d d
(3 + 2x) dx (3 − 2x) − (3 − 2x) dx (3 + 2x)
y0 = 2
(3 + 2x)
(3 + 2x)(−2) − (3 − 2x)(2) −12
= 2
= .
(3 + 2x) (3 + 2x)2

168
i) Procediendo como en el caso anterior tenemos:
d d
(4 − x2 )1/2 dx (x2 ) − x2 dx (4 − x2 )1/2
y0 =
4 − x2
(4 − x2 )1/2 · 2x − x2 (1/2)(4 − x2 )−1/2 (−2x)
=
4 − x2
(4 − x ) · 2x + x3 (4 − x2 )−1/2 (4 − x2 )1/2
2 1/2
= ·
4 − x2 (4 − x2 )1/2
2x(4 − x2 ) + x3 8x − x3
= = .
(4 − x2 )3/2 (4 − x2 )3/2

PROBLEMA 5.24.

Calcular la derivada de la función

f (x) = sen4 x + 2 sen2 x cos2 x + cos4 x.

Solución

Aplicaremos la regla de derivación de la suma y en cada sumando la regla


de la potencia de las funciones trigonométricas que aparecen. Obtenemos
la siguiente secuencia de igualdades (trata de justificar por tı́ mismo cada
identidad y ası́ entender el proceso):
f 0 (x) = 4 sen3 x cos x + 2[2 sen x cos x cos2 x
+ sen2 x · 2 cos x(− sen x)] + 4 cos3 x(− sen x)
= 4 sen3 x cos x + 4 sen x cos3 x − 4 sen3 x cos x − 4 cos3 x sen x = 0.
El resultado podı́a haberse obtenido también si escribimos originalmente la
función como f (x) = (sen2 x + cos2 x)2 = 12 = 1 y tenemos en cuenta que
la derivada de una constante es cero.

PROBLEMA 5.25.
(
x2 sen(1/x) si x 6= 0
Sea f (x) =
0 si x = 0.
a) ¿Es f derivable en x = 0?
b) ¿Es f 0 continua en x = 0?

169
Solución

a) Aplicando la definición de derivada,


f (0 + h) − f (0) h2 sen 1/h − 0
f 0 (0) = lı́m = lı́m = lı́m h sen 1/h = 0,
h→0 h h→0 h h→0

pues | sen(1/x)| ≤ 1 y lı́m h = 0.


h→0

Ası́ pues la función es derivable, y en consecuencia continua, en x = 0.


b) Utilizando las reglas elementales de derivación,
d d
f 0 (x) = x2 (sen 1/x) + (sen 1/x) (x2 )
dx dx
= x2 (cos 1/x)(−1/x2 ) + (sen 1/x)(2x) = − cos 1/x + 2x sen 1/x.
Como lı́m f 0 (x) = lı́m (− cos 1/x+2x sen 1/x) no existe pues lı́m cos 1/x
x→0 x→0 x→0
no existe, entonces f 0 (x) no puede ser continua en x = 0, pese a existir
f 0 (0).
Esto muestra que no se puede calcular f 0 (0) en este caso simplemente
calculando f 0 (x) y haciendo luego x = 0. Solamente cuando la derivada
es continua en un punto el procedimiento es correcto.

PROBLEMA 5.26.

Hallar y 0 en los siguientes casos:


p
a) y = x a2 − x2 + a2 arc sen(x/a).
1 p
b) y = x arccosec + 1 − x2 .
x
 
1 b
c) y = arc tg tg x .
ab a

Solución

a) Aplicando las reglas correspondientes,


1 1 1
y 0 = x · (a2 − x2 )−1/2 (−2x) + (a2 − x2 )1/2 + a2 p ·
2 1 − (x/a)2 a
−x2 p a 2 p
= √ + a2 − x2 + √ = 2 a2 − x2 .
2
a −x 2 a − x2
2

170
−(−1/x2 ) 1 1 1
b) y 0 = x · p + arccosec + (1 − x2 )−1/2 (−2x) = arccosec .
(1/x) (1/x)2 − 1 x 2 x
c) Aplicamos sucesivas veces la regla de la cadena:

1 1 b 1 a2 b
y0 = · ·
2 a · sec 2
x = · 2 2 2 · · sec2 x
ab 1 + b ab a + b tg x a

a tg x
sec2 x 1
= = 2 .
a2 + b2 tg2 x a cos2 x + b2 sen2 x

PROBLEMA 5.27.

Hallar la derivada de la función


√ √
y = arc tg x + 1 + arc sen(cos x + 1).

Solución

1 1 1 √ 1
y0 = · √ +p √ · (− sen x + 1) · √
1 + (x + 1) 2 x + 1 1 − cos2 x + 1 2 1+x
 
1 1 1 1 1
= · √ − √ = √ −1
x+2 2 x+1 2 1+x 2 1+x x+2

−(x + 1) − x+1
= √ = .
2 x + 1(x + 2) 2(x + 2)

PROBLEMA 5.28.

Hallar la derivada, simplificando el resultado, de la función


√ √
2 sen x 2 3 3(1 − cos x)
y= − arc tg .
2 + cos x 3 3 sen x

171
Solución

2 cos x(2 + cos x) + 2 sen2 x


y0 =
(2 + cos x)2
√ √ √
2 3 1 3 3 sen2 x − 3 3 cos x(1 − cos x)
− · x)2
·
3 1 + (1−cos 2
9 sen2 x
3 sen x
√ √ √
2 + 4 cos x 2 3 3 sen2 x 3 3 − 3 3 cos x
= − · ·
(2 + cos x)2 3 3 sen2 x + (1 − cos x)2 9 sen2 x
√ √
2 + 4 cos x 2 3 3 sen2 x 3 3(1 − cos x)
= − · ·
(2 + cos x)2 3 3(1 − cos x)(1 + cos x) + (1 − cos x)2 9 sen2 x
√ √
2 + 4 cos x 2 3 3 sen2 x 3
= 2
− · ·
(2 + cos x) 3 3(1 + cos x) + (1 − cos x) 3 sen2 x
2 + 4 cos x 2 2 + 4 cos x − (2 + cos x) 3 cos x
= − = = .
(2 + cos x)2 2(2 + cos x) (2 + cos x)2 (2 + cos x)2

PROBLEMA 5.29.
r
1 − x2
Dada la función y = arc cos(sen 3x) + arc tg , calcular (sim-
1 + x2
plificando lo más posible) el valor de z = (y 0 + 3)2 .

Solución

r
1 − x2
Si llamamos y1 = arc cos(sen 3x) e y2 = arc tg , tenemos:
1 + x2
−3 cos 3x −3 cos 3x
y10 = √ = = −3.
2
1 − sen 3x cos 3x

1 1 −2x(1 + x2 ) − 2x(1 − x2 )
y20 = 1−x2
· q ·
1+ 2 1−x
2 (1 + x2 )2
1+x2 1+x2
−x −x −x
= q =q = √ .
2 (1+x2 )2 (1−x2 ) 1 − x4
(1 + x2 ) 1−x
1+x2 1+x2

x x2
Como y = y1 + y2 , entonces y 0 = −3 − √ . Luego z = .
1 − x4 1 − x4

172
PROBLEMA 5.30.

x2 + 2x
Hallar el valor de la derivada de la función y = ln .
x2 − 2x

Solución

dy x2 − 2x (2x + 2x ln 2)(x2 − 2x ) − (2x − 2x ln 2)(x2 + 2x )


= ·
dx x2 + 2x (x2 − 2x )2
1 x x+1
−4x · 2 + 2 x ln 22 2x+1 (x ln 2 − 2)x
= · = .
x + 2x
2 x2 − 2x x4 − 22x

PROBLEMA 5.31.
s
1 − tg(x/2)
Hallar la derivada de la función y = ln .
1 + tg(x/2)

Solución

1
De las propiedades de los logaritmos: y = 2 [ln (1 − tg(x/2)) − ln (1 + tg(x/2))].

Entonces:
" #
1 − 21 sec2 (x/2) 1
sec 2 (x/2)
y0 = − 2
2 1 − tg(x/2) 1 + tg(x/2)
sec2 (x/2) 1 + tg(x/2) + (1 − tg(x/2))
= − ·
4 1 − tg2 (x/2)
− sec2 (x/2) 1 −1
= 2 =− 2 2
= .
2(1 − tg (x/2)) 2[cos (x/2) − sen (x/2)] 2 cos x

173
PROBLEMA 5.32.

Hallar dy/dx para las funciones siguientes:


a) y = sh 3x.
b) y = ch x/2.
c) y = th(1 + x2 ).
d) y = coth 1/x.
e) y = x sech x2 .
f) y = csch2 (x2 + 1).
1 1
g) y = sh 2x − x.
4 2
h) y = ln th 2x.

Solución

a) y 0 = 3 ch 3x.

b) y 0 = (1/2) sh x/2.

c) y 0 = 2x sech2 (1 + x2 ).

d) y 0 = (1/x2 ) csch2 1/x.

e) y 0 = x · 2x(− sech x2 th x2 ) + sech x2 = −2x2 sech x2 th x2 + sech x2 .

f) y 0 = 2 csch(x2 + 1)(− csch(x2 + 1) coth(x2 + 1)) · 2x = −4x csch2 (x2 +


1) coth(x2 + 1).

1 1
g) y 0 = ch 2x − = sh2 x.
2 2

1 2
h) y 0 = · 2 sech2 2x = = 4 csch 4x.
th 2x sh 2x ch 2x

174
PROBLEMA 5.33.

Hallar dy/dx en las siguientes funciones:


a) y = argsh 3x.
b) y = argch ex .
c) y = 2 argth(tg x/2).

Solución

3 3
a) y 0 = p =√ .
(3x)2 +1 9x2 + 1
ex
b) y 0 = √ .
e2x − 1
1 sec2 (x/2)
c) y 0 = 2 · · sec 2
(x/2) · (1/2) = = sec x.
1 − tg2 x/2 1 − tg2 x/2

PROBLEMA 5.34.


Calcular la derivada dr/dt en t = 0 sabiendo que r = s + 1, s =
16t2 − 20t.

Solución

Aplicando la regla de derivación de la función compuesta, resulta:


dr dr ds
= · = (1/2)(s + 1)−1/2 (32t − 20).
dt ds dt
Ahora bien, cuando t = 0, al sutituir en la segunda función resulta s = 0.
Por lo tanto:
dr
(0) = (1/2)(1)−1/2 (−20) = −10.
dt

PROBLEMA 5.35.

Sea f (x) = x3 + 2x + 1 una función definida para x > 0 y llamemos


g(x) = f −1 (x). Calcular g 0 (x) en el punto donde g(x) = 1.

175
Solución

1
Por la fórmula de derivación de la función inversa, g 0 (x) = 0 , tene-
f [g(x)]
mos:
f 0 (x) = 3x2 + 2 =⇒ f 0 [g(x)] = 3[g(x)]2 + 2.
1
Cuando g(x) = 1, f 0 (1) = 5. En ese punto, g 0 (x) = = 1/5.
f 0 (1)

PROBLEMA 5.36.

p
Hallar dy/dx en la función x = y 1 − y2.

Solución

dx 1 − 2y 2
Como = (1 − y 2 )1/2 + (1/2)y(1 − y 2 )−1/2 (−2y) = p se deduce que
dy 1 − y2
p
dy 1 1 − y2
= = .
dx dx/dy 1 − 2y 2

PROBLEMA 5.37.

Dada una función derivable


( g , encontrar los valores de a y b para
g(x) si x < x0 ,
que la función f (x) = sea derivable en x0 .
ax + b si x ≥ x0 ,

Solución

En primer lugar f debe ser continua en x0 . Como

lı́m f (x) = g(x0 ), lı́m f (x) = ax0 + b,


x→x−
0 x→x+
0

debe cumplirse que g(x0 ) = ax0 + b.

176
Por otra parte las derivadas laterales en x0 deben ser iguales. Tenemos:

f 0 (x− 0 0 +
0 ) = g (x0 ), f (x0 ) = a,

con lo que a = g 0 (x0 ).


Sustituyendo en la ecuación anterior, obtenemos que b = g(x0 ) − x0 · g 0 (x0 ).

PROBLEMA 5.38.

Encontrar un polinomio F (x) para el que la derivada de la función


φ(x) = e2x F (x) sea igual a e2x (x2 − 1).

Solución

Calculamos en primer lugar la derivada de φ(x):


d d 2x
φ(x) = [e F (x)] = 2e2x F (x) + e2x F 0 (x) = e2x [2F (x) + F 0 (x)].
dx dx
Como debe verificarse que e2x [2F (x) + F 0 (x)] = e2x (x2 − 1), resulta 2F (x) +
F 0 (x) = x2 − 1. De aquı́ se deduce que F (x) debe ser un polinomio de
segundo grado.
Si escribimos F (x) = ax2 + bx + c, F 0 (x) = 2ax + b. Entonces:
2ax2 + 2bx + 2c + 2ax + b = x2 − 1 o bien 2ax2 + (2a + 2b)x + (b + 2c) =
x2 − 1.
Identificando coeficientes tenemos:

2a = 1, 2a + 2b = 0, b + 2c = −1 =⇒ a = 1/2, b = −1/2, c = −1/4.

El polinomio buscado es pues F (x) = x2 /2 − x/2 − 1/4.

C. DERIVADAS SUCESIVAS.

Al calcular la derivada de una función en cualquier punto donde esta exista,


se obtiene como resultado otra función, llamada función derivada o función

177
primera derivada. Si derivamos esta nueva función, se obtiene la llamada
derivada segunda de la función original. Reiterando el proceso n veces, po-
demos llegar a la llamada derivada de orden n o derivada n-ésima de la
función original. Ası́ pues, son válidas las mismas fórmulas y reglas de deri-
vación con las derivadas sucesivas.
La notación utilizada para indicar derivadas de orden n es:
dn f (x) dn y
f (n) (x) = D(n) f (x) = o bien y (n)
= D (n)
y = .
dxn dxn
En algunos casos puede utilizarse un proceso de recurrencia para obtener
las derivadas de cualquier orden de una función.

PROBLEMA 5.39.

Sea y = f (x) un polinomio cualquiera de orden n, es decir, el que


tiene por ecuación y = an xn + an−1 xn−1 + an−2 xn−2 + · · · + a1 x + a0 .
Calcular todas sus derivadas.

Solución

Las derivadas sucesivas son:

y 0 = nan xn−1 + (n − 1)an−1 xn−2 + (n − 2)an−2 xn−3 + · · · + a1 ;


y 00 = n(n − 1)an xn−2 + (n − 1)(n − 2)an−1 xn−3 + · · · + 2a2 ;
...
(n)
y = n(n − 1)(n − 2)..,2 · 1 · an = n! an ;
(n+1)
y = 0;
(n+k)
y = 0, ∀k > 0.

Como se observa, el grado del polinomio va disminuyendo con cada derivada,


hasta que se anula. Las derivadas de orden mayor a n son todas cero.

PROBLEMA 5.40.

Hallar y 00 en la función y = e−x ln x.

178
Solución

Las dos primeras derivadas son, respectivamente:

1 e−x
y 0 = e−x · − e−x ln x = − y.
x x
−xe−x − e−x
y 00 = − y0
x2
−xe−x − e−x e−x
= − + e−x ln x = −e−x (2/x + 1/x2 − ln x).
x2 x

PROBLEMA 5.41.

p
3 1
Si la función s(t) = (4 + 2t)2 + expresa la ecuación del
(t + 2)2
movimiento, calcular la velocidad inicial y la aceleración inicial.

Solución

Como sabemos, la velocidad es la derivada respecto al tiempo de la distancia,


y la aceleración es la derivada de la velocidad respecto al tiempo, o lo que es
lo mismo, la derivada segunda de la distancia respecto al tiempo. Debemos
calcular las dos primeras derivadas de la función s(t) y sustituir t = 0
en ellas, lo que corresponde al inicio del movimiento (escribimos s(t) =
(4 + 2t)2/3 + (t + 2)−2 para calcular más cómodamente las derivadas).

2
v(t) = s0 (t) = (4 + 2t)−1/3 · 2 − 2(t + 2)−3 .
3
2 −1
a(t) = s00 (t) = · (4 + 2t)−4/3 · 2 · 2 − 2 · (−3)(t + 2)−4 .
3 3
4 1 −2 3
Sustituyendo el valor t = 0, se obtiene que v(0) = √ 3
− ; a(0) = √ 3
+ .
3 4 4 9 4 8

PROBLEMA 5.42.

Mostrar que la función y = x3 cos ln x + 2x3 sen ln x verifica la ecua-


ción x2 y 00 − 5xy 0 + 10y = 0.

179
Solución

Las dos primeras derivadas de la función son:

y 0 = 3x2 cos ln x − x3 (1/x) sen ln x + 6x2 sen ln x + 2x3 (1/x) cos ln x


= 5x2 cos ln x + 5x2 sen ln x.
y 00 = 10x cos ln x − 5x2 (1/x) sen ln x + 10x sen ln x + 5x2 (1/x) cos ln x
= 15x cos ln x + 5x sen ln x.

Sustituyendo entonces en la ecuación,

x2 y 00 −5xy 0 +10y = (15x3 −25x3 +10x3 ) cos ln x+(5x3 −25x3 +20x3 ) sen ln x = 0.

PROBLEMA 5.43.

Hallar las derivadas sucesivas de f (x) = x4/3 en x = 0.

Solución

f 0 (x) = (4/3)x1/3 y f 0 (0) = 0.


f 00 (x) = (4/3) · (1/3)x−2/3 y f 00 (0) no existe, por lo que no existen las
siguientes derivadas en x = 0.
Por tanto, para x = 0 sólo existe la primera derivada.

PROBLEMA 5.44.

2
Dada la función f (x) = , calcular f (n) (x).
1−x

Solución

Escribimos f (x) = 2(1 − x)−1 y las derivadas sucesivas son:

f 0 (x) = 2(−1)(1 − x)−2 (−1) = 2(1 − x)−2 ;


f 00 (x) = 2(−2)(1 − x)−3 (−1) = 2 · 2!(1 − x)−3 ;
f 000 (x) = 2 · 2!(−3)(1 − x)−4 (−1) = 2 · 3!(1 − x)−4 ;

180
lo que sugiere la fórmula general f (n) (x) = 2 · n!(1 − x)−(n+1) .
Esta fórmula se puede probar por el método de inducción:
Suponiendo que f (k) (x) = 2 · k!(1 − x)−(k+1) , se verifica que
d (k)
f (k+1) (x) = f (x) = −2 · k!(k + 1)(1 − x)−(k+2) (−1)
dx
= 2 · (k + 1)!(1 − x)−(k+2) .

PROBLEMA 5.45.

Hallar las derivadas de orden n de las funciones y1 = sen x e y2 =


cos x.

Solución

Teniendo en cuenta que cos x = sen(x + π/2) y que sen x = − cos(x + π/2),
resulta que:
y10 = cos x = sen(x + π/2);
y100 = cos(x + π/2) = sen(x + 2π/2);
y1000 = cos(x + 2π/2) = sen(x + 3π/2);
...
(n)
y1 = sen(x + nπ/2).
Análogamente,
y20 = − sen x = cos(x + π/2);
y200 = − sen(x + π/2) = cos(x + 2π/2);
y2000 = − sen(x + 2π/2) = cos(x + 3π/2);
...
(n)
y2 = cos(x + nπ/2).

PROBLEMA 5.46.

Hallar la derivada n-ésima de las funciones siguientes:


a) y = eax sen bx.
b) y = sen2 x.

181
Solución

a) y 0 = aeax sen bx + beax cos bx = eax (a sen bx + b cos bx).


Haciendo b/a = tg φ, es b2 /a2 = tg2 φ y 1 + b2 /a2 = 1 + tg2 φ =
1/ cos2 φ, con lo que, operando, tenemos a = (a2 + b2 )1/2 cos φ y b =
(a2 + b2 )1/2 sen φ.
Sustituimos en la derivada y nos queda:

y 0 = eax (a2 +b2 )1/2 (sen bx cos φ+cos bx sen φ) = eax (a2 +b2 )1/2 sen(bx+φ).

Al derivar sucesivas veces llegamos a

y (n) = eax (a2 + b2 )n/2 sen(bx + nφ).

1 − cos 2x 1 1
b) Teniendo en cuenta que sen2 x = = − cos 2x, resulta del
2 2 2
problema anterior que:

dn dn 1 1 1 dn
 
2
(sen x) = − cos 2x = − · (cos 2x)
dxn dxn 2 2 2 dxn
1  nπ   nπ 
= − · 2n cos 2x + = −2n−1 cos 2x + .
2 2 2

D. FUNCIONES IMPLÍCITAS. DERIVACIÓN LOGARÍTMICA.

Cuando en la ecuación que define una función, la variable y no aparece despe-


jada en función de x, se dice que la función está definida en forma implı́cita,
y la ecuación que la define tiene la forma general F (x, y) = 0.
Aunque generalmente una expresión de la forma F (x, y) = 0 no da lugar a
una sino a varias funciones, es posible calcular sus derivadas sin tener que
despejar la variable y. Para ello basta aplicar las reglas usuales de derivación
teniendo en cuenta que y es una función desconocida de x, y por lo tanto,
su derivada debe dejarse indicada para después despejarla si es posible. Por
ejemplo, ası́ como la derivada de x2 es 2x, la derivada de y 2 es 2y · y 0 (debido
a la regla de la cadena).

182
El uso de las funciones implı́citas da lugar a otro método de derivación,
llamado derivación logarı́tmica. Este consiste en tomar logaritmos en la fun-
ción (esto permite pasar de potencias a productos y de productos a sumas,
cada uno más fácil de derivar que el anterior), para después derivar térmi-
no a término la función implı́cita que resulta. Ası́, funciones de la forma
y = f (x)g(x) se transforman en ln y = g(x) ln f (x) y, aunque las funciones
tengan ahora forma implı́cita, para derivarlas basta utilizar la regla del pro-
ducto.
Los problemas siguientes permiten aclarar estos conceptos.

PROBLEMA 5.47.

Hallar dy/dx en las funciones definidas en forma implı́cita por


a) x3 − xy + y 3 = 1.
b) (x + y)3 + (x − y)3 = x4 + y 4 .

Solución

a) Derivando formalmente ambos miembros de la ecuación obtenemos:

(x3 )0 − (xy)0 + (y 3 )0 = (1)0 ;

3x2 − (x · y 0 + 1 · y) + 3y 2 · y 0 = 0;
y 0 (3y 2 − x) = y − 3x2 ;
y − 3x2
y0 = .
3y 2 − x

b) En ambos miembros se debe aplicar la regla de derivación de la suma,


donde cada sumando es una potencia:

3(x + y)2 (x + y)0 + 3(x − y)2 (x − y)0 = 4x3 + 4y 3 y 0 ;

3(x + y)2 (1 + y 0 ) + 3(x − y)2 (1 − y 0 ) = 4x3 + 4y 3 y 0 ;


3(x + y)2 + 3(x + y)2 y 0 + 3(x − y)2 − 3(x − y)2 y 0 = 4x3 + 4y 3 y 0 ;
y 0 [3(x + y)2 − 3(x − y)2 − 4y 3 ] = −3(x + y)2 − 3(x − y)2 + 4x3 .
Despejando y simplificando, queda en definitiva:
−3(x2 + y 2 ) + 2x3
y0 = .
2y(3x − y 2 )

183
PROBLEMA 5.48.

Hallar las derivadas de las funciones definidas en forma implı́cita


por las ecuaciones:
a) x2 y − xy 2 + x2 + y 2 = 0.
b) y 2 sen x + y = arc tg x.

Solución

a) Derivando término a término tenemos:


d 2 d d 2 d 2
(x y) − (xy 2 ) + (x ) + (y ) = 0;
dx dx dx dx
d d d d d 2 d 2
x2 (y) + y (x2 ) − x (y 2 ) − y 2 (x) + (x ) + (y ) = 0;
dx dx dx dx dx dx
x2 y 0 + 2xy − 2xyy 0 − y 2 + 2x + 2yy 0 = 0;
y despejando resulta:
y 2 − 2x − 2xy
y0 = .
x2 + 2y − 2xy

b) Si y 2 sen x + y = arc tg x,
1
2yy 0 sen x + y 2 cos x + y 0 = ;
1 + x2
1
y 0 (2y sen x + 1) = − y 2 cos x;
1 + x2
1 − (1 + x2 )y 2 cos x
y0 = .
(1 + x2 )(2y sen x + 1)

PROBLEMA 5.49.

Hallar la primera derivada de las siguientes funciones aplicando


la derivación logarı́tmica:
a) y = (x2 + 2)3 (1 − x3 )4 .
x(1 − x2 )2
b) y = .
(1 + x2 )1/2

184
Solución

a) Como ln y = 3 ln(x2 + 2) + 4 ln(1 − x3 ), derivando tenemos:

y0 2x −3x2 6x(1 − x3 ) − 12x2 (x2 + 2) 6x(1 − 4x − 3x3 )


= 3· 2 +4· 3
= 2 3
= ;
y x +2 1−x (x + 2)(1 − x ) (x2 + 2)(1 − x3 )

=⇒ y 0 = 6x(x2 + 2)2 (1 − x3 )3 (1 − 4x − 3x3 ).

b) Análogamente al anterior, como ln y = ln x+2 ln(1−x2 )−(1/2) ln(1+x2 ),


resulta:
y0 1 4x x (1 − x2 )(1 + x2 ) − 4x2 (1 + x2 ) − x2 (1 − x2 )
= − − =
y x 1 − x2 1 + x2 x(1 − x2 )(1 + x2 )
2
1 − 5x − 4x 4
= ;
x(1 − x2 )(1 + x2 )

x(1 − x2 )2 1 − 5x2 − 4x4 (1 − 5x2 − 4x4 )(1 − x2 )


=⇒ y 0 = · = .
(1 + x2 )1/2 x(1 − x2 )(1 + x2 ) (1 + x2 )3/2

PROBLEMA 5.50.

Calcular la derivada segunda de la función y = f (x) definida en


forma implı́cita por xy + y 2 = 1.

Solución

Empezaremos calculando la derivada primera:


y
y + xy 0 + 2yy 0 = 0 =⇒ y 0 (x + 2y) = −y =⇒ y 0 = − .
x + 2y
Aplicamos la regla del cociente al resultado para calcular la derivada segun-
da:
−y 0 (x + 2y) + y(1 + 2y 0 ) −xy 0 − 2yy 0 + y + 2yy 0 −xy 0 + y
y 00 = = = .
(x + 2y)2 (x + 2y)2 (x + 2y)2

Sustituyendo y 0 por el valor conseguido anteriormente, resulta:


xy
x+2y +y xy + (x + 2y)y 2xy + 2y 2
y 00 = = = .
(x + 2y)2 (x + 2y)3 (x + 2y)3

185
El resultado final viene expresado únicamente en función de x e y.

PROBLEMA 5.51.

Hallar y 0 e y 00 en la ecuación x2 − xy + y 2 = 3.

Solución

d 2 d d 2 d
(x ) − (xy) + (y ) = (3) =⇒ 2x − xy 0 − y + 2yy 0 = 0;
dx dx dx dx
2x − y
y0 = .
x − 2y
d d
(x − 2y) dx (2x − y) − (2x − y) dx (x − 2y)
y 00 = 2
(x − 2y)
(x − 2y)(2 − y ) − (2x − y)(1 − 2y 0 )
0 3xy 0 − 3y
= =
(x − 2y)2 (x − 2y)2
2x−y
3x x−2y − 3y 6(x2 − xy + y 2 ) 18
= = = .
(x − 2y)2 (x − 2y)3 (x − 2y)3

PROBLEMA 5.52.

Calcular la derivada de y = xx (sen x)tg x .

Solución

Llamamos y1 = xx e y2 = (sen x)tg x . Entonces, por derivación logarı́tmi-


ca:

y10
ln y1 = x ln x =⇒ = ln x + x · (1/x) = 1 + ln x =⇒ y10 = xx (1 + ln x);
y1

y20 1 cos x ln sen x


ln y2 = tg x ln sen x =⇒ = 2
ln sen x + tg x = +1
y2 cos x sen x cos2 x
 
ln sen x
=⇒ y20 = (sen x)tg x +1 .
cos2 x

186
Para calcular y 0 aplicamos la regla de derivación del producto:
 
0 ln sen x
y = (sen x)tg x y10 + xx y20 tg x x
= (sen x) x
x (1 + ln x) + x (sen x) tg x
+1
cos2 x
 
ln sen x
= xx (sen x)tg x 2 + ln x + .
cos2 x

PROBLEMA 5.53.

tg x
Derivar la función y = (sen x)(cos x) .

Solución

Si llamamos y1 = (cos x)tg x , la función se escribe como y = (sen x)y1 . To-


y0 cos x
mando logaritmos, ln y = y1 ln sen x, con lo que = y1 · + y10 · ln sen x.
y sen x
Calculamos aparte la derivada de y1 por derivación logarı́tmica:

y10 − sen x
ln y1 = tg x ln cos x =⇒ = tg x · + sec2 x ln cos x
y1 cos x

= − tg2 x + sec2 x ln cos x

=⇒ y10 = (cos x)tg x (− tg2 x + sec2 x ln cos x).

Si sustituimos este resultado obtenemos:


h cos x i
y 0 = (cos x)tg x · + (cos x)tg x (− tg2 x + sec2 x ln cos x) ln sen x
sen x
tg x
×(sen x)(cos x)
tg x
h cos x i
= (sen x)(cos x) · (cos x)tg x + (− tg2 x + sec2 x ln cos x) ln sen x .
sen x

187
E. ECUACIONES PARAMÉTRICAS.

Frecuentemente en las aplicaciones fı́sicas se escriben las ecuaciones de las


curvas no expresando una variable en función de la otra sino que ambas
coordenadas se expresan como funciones de una nueva variable, llamada
parámetro, dando lugar a las llamadas ecuaciones paramétricas de la curva.
La forma general es:
x = f (t), y = g(t).
En los casos donde se pueda despejar t en la primera ecuación y sustituirlo
en la segunda, se obtiene la ecuación cartesiana común. Sin embargo, escrita
la ecuación en su forma paramétrica, es posible resolver problemas analı́ticos
y geométricos.
La derivada de y respecto a x se calcula ası́:

dy g 0 (t)
(2) = 0 , siempre que f 0 (t) 6= 0.
dx f (t)

PROBLEMA 5.54.

t
Dada la curva en forma paramétrica x = , y = t2 , escribir la
1+t
curva en la forma y = f (x) y calcular de dos maneras la derivada
dy/dx comprobando que son iguales.

Solución

En primer lugar debemos despejar t de la primera ecuación. Ası́:


x
x(1 + t) = t =⇒ x = t − xt =⇒ t = .
1−x
Sustituyendo el valor en la segunda ecuación resulta la ecuación en su forma
explı́cita:
x2
y= .
(1 − x)2
Si derivamos esta ecuación obtenemos:

2x(1 − x)2 − 2(1 − x)(−1)x2 2x(1 − x) + 2x2 2x


y0 = 4
= 3
= .
(1 − x) (1 − x) (1 − x)3

188
Derivando la ecuación original mediante la fórmula (2) tenemos:

dy/dt 2t
y0 = = 1+t−t = 2t(1 + t)2 .
dx/dt (1+t)2

t
Para comprobar la igualdad de los dos resultados, basta sustituir x por
1+t
x
en el primero, o sustituir t por en el segundo.
1−x

PROBLEMA 5.55.

Dibujar la curva representada por la ecuación x = 2 cos t, y =


3 sen t.

Solución

Para escribir la ecuación en su forma cartesiana, es decir, sin que intervenga


el parámetro t, elevaremos al cuadrado las dos ecuaciones después de des-
pejar cos t y sen t en ellas (en este caso es preferible a despejar t en una de
las ecuaciones). Tenemos ası́:

x2 y2
= (cos t)2 , = (sen t)2 .
4 9

Si sumamos ambas ecuaciones se elimina la variable t y queda:

x2 y 2
+ = 1,
4 9

que corresponde a una elipse de centro el origen de coordenadas y ejes los


de coordenadas.

Para dibujar la gráfica, calculamos los vértices o puntos de corte con los
ejes.

Cuando x = 0, y = ±3. Cuando y = 0, x = ±2. La gráfica queda enton-


ces:

189
F. RECTA TANGENTE Y NORMAL.

Se llama recta secante a una curva de ecuación y = f (x) en los puntos


P0 (x0 , f (x0 )) y P1 (x1 , f (x1 )) a la recta que pasa por dichos puntos. La pen-
diente de esta recta es entonces

f (x1 ) − f (x0 )
.
x1 − x0

Si ahora hacemos que el punto P1 se mueva a lo largo de la curva y = f (x)


aproximándose cada vez más al punto P0 , la posición lı́mite de las rectas
secantes se llamará recta tangente a la curva y = f (x) por el punto P0 .
En este caso la pendiente se obtendrá también como un lı́mite y tendrá la
forma:

f (x1 ) − f (x0 )
(3) m = lı́m .
x1 →x0 x1 − x0

Esta fórmula corresponde precisamente a la derivada de la función y = f (x)


en el punto P0 . De este modo, la recta tangente a una curva en un punto
tiene por pendiente la derivada de dicha función en el punto.

La ecuación de la recta tangente será entonces:

(4) y − f (x0 ) = f 0 (x0 )(x − x0 ).

190
P1

P0 m = f 0 (x0 )

x0 x1

Se llama recta normal a la curva y = f (x) en el punto P0 (x0 , f (x0 )) a la


recta que, pasando por P0 , es perpendicular a la recta tangente a f en ese
punto. Su ecuación debe ser, por lo tanto:
−1
(5) y − f (x0 ) = (x − x0 ).
f 0 (x0 )

PROBLEMA 5.56.

Hallar las ecuaciones de la recta tangente y normal a la función


f (x) = tg 2x en el origen de coordenadas.

Solución

La pendiente de la recta tangente será f 0 (0). Derivemos en primer lugar la


función para después sustituir en x = 0:

f 0 (x) = sec2 (2x) · 2 =⇒ f 0 (0) = 2.

La ecuación de la recta tangente es: y − 0 = 2(x − 0), o bien, y = 2x.


La recta normal tiene por ecuación: y = −x/2.

PROBLEMA 5.57.

Encontrar las ecuaciones de las rectas tangentes a la función y =


x2 que pasen por el punto (0, −1).

191
Solución

Como el punto (0, −1) no pertenece a la curva, no podemos seguir el pro-


cedimiento anterior. Hay que averiguar en primer lugar el punto o puntos
de tangencia. Supongamos que dicho punto es (a, a2 ) (recuerda que debe
pertenecer a la curva).
La pendiente de la recta tangente se puede calcular de dos formas:
- Sabiendo que es la derivada de la función en el punto de tangencia, y como
y 0 = 2x, obtenemos:
m = f 0 (a) = 2a.
- Sabiendo que pasa por los puntos (0, −1) y (a, a2 ), resulta:

a2 + 1
m= .
a
a2 + 1
Igualando ambos resultados, tenemos la ecuación 2a = . Resolviendo
a
esta ecuación:
2a2 = a2 + 1 =⇒ a2 = 1 =⇒ a = ±1.
Tenemos entonces que hay dos rectas tangentes que verifican la condición,
una con pendiente 2 y la otra con pendiente −2. Sus ecuaciones son, respec-
tivamente:
y + 1 = 2x; y + 1 = −2x.
Si queremos encontrar también las rectas normales, ya no tienen que pasar
por el punto (−1, 0) pero sı́ por los puntos de tangencia (1, 1) y (−1, 1). La
primera tendrá pendiente −1/2 y la segunda, 1/2. Las ecuaciones son por
tanto:
1 1
y − 1 = − (x − 1); y − 1 = (x + 1)
2 2

PROBLEMA 5.58.

Hallar las rectas tangente y normal a la curva de ecuación x2 y 2 = 9


en el punto P (−1, 3).

Solución

Como sabemos, la pendiente de la recta tangente es el valor de la deriva-


da de la función en el punto de tangencia. Derivando la función implı́cita

192
tenemos:
2xy 2 y
2xy 2 + x2 · 2yy 0 = 0 =⇒ y 0 = − 2
=− ,
2x y x
siempre que x 6= 0 e y 6= 0.
Como el punto P pertenece a la curva, tenemos directamente que m =
−3/ − 1 = 3.
La ecuación de la recta tangente es: y −3 = 3(x+1) o bien, y = 3x+6.
1
La recta normal tiene por ecuación: y − 3 = − (x + 1), es decir, 3y +x−8 =
3
0.

PROBLEMA 5.59.

Escribir las ecuaciones de las rectas tangentes a la curva

x = t cos t, y = t sen t

en (0, 0) y en el punto correspondiente a t = π/4.

Solución

En primer lugar debemos calcular la derivada de la función. Como:

x0 (t) = cos t − t sen t, y 0 (t) = sen t + t cos t,

dy y 0 (t) sen t + t cos t


resulta que = 0 = .
dx x (t) cos t − t sen t
La pendiente de la recta tangente se obtiene sustituyendo en la derivada el
valor de t correspondiente al punto de tangencia. Para obtener el valor de
t correspondiente al origen de coordenadas, sustituimos los valores x = 0 e
y = 0 en la ecuación de la curva; debemos resolver las ecuaciones

0 = t cos t, 0 = t sen t.
π 3π 5π
De la primera se obtiene que t = 0, , , , . . .
2 2 2
Las soluciones de la segunda ecuación son: t = 0, π, 2π, 3π, . . .
El único valor común para las dos es el de t = 0.
Haciendo t = 0 en la fórmula de la derivada, resulta: m = 0.

193
Ası́, la ecuación de la recta tangente en el punto (0, 0) es y − 0 = 0(x − 0),
es decir, y = 0.

2
π (1 + π/4) 1 + π/4
Si t = , la pendiente vale: m = √2 = .
4 2
− π/4) 1 − π/4
2 (1
El punto de tangencia se obtiene sustituyendo en la ecuación de la curva el
valor t = π/4. Tenemos:
√ √
π 2 π 2
x(π/4) = , y(π/4) = .
8 8
√ √ !
π 2 1 + π/4 π 2
La ecuación de la recta tangente es y − = x− .
8 1 − π/4 8

PROBLEMA 5.60.

Calcular la pendiente de la curva x = y 2 − 4y en los puntos de


intersección con el eje Y .

Solución

Los puntos de corte con el eje Y tienen como ordenada las soluciones de
0 = y 2 − 4y, es decir son los puntos (0, 0) y (0, 4).
Además la pendiente de la recta tangente se obtiene sustituyendo en la
derivada de la función los puntos anteriores. Ası́ tenemos:
dx dy 1 1
= 2y − 4 =⇒ = = .
dy dx dx/dy 2y − 4

En (0, 0) la pendiente es −1/4 y en (0, 4) la pendiente es 1/4.

194
G. EJERCICIOS PROPUESTOS.

1.- Encontrar, usando la definición, las derivadas de las siguientes


1
funciones: a) f (x) = x2 + x + en el punto x = 1.
x

Resp.: f 0 (1) = 2. b) f (x) = (x2 + 2x)3 .

2x − 3
Resp.: f 0 (x) = 6(x + 1)(x2 + 2x)2 . c) f (x) = .
3x + 2

13
Resp.: f 0 (x) = . d) f (x) = cos x.
(3x + 2)2

Sugerencia: Recuerda la identidad cos(a + b) = cos a cos b − sen a sen b


y aplı́cala a la expresión cos(x + h).
Resp.: f 0 (x) = − sen x.

2.- Sea f (x) = x3/2 . Probar que f tiene derivada lateral en x = 0 por
la derecha y hallar su valor.
Sugerencia: Calcula directamente f 0 (0+ ) mediante la definición.
Resp.: f 0 (0+ ) = 0.

1/x
 si x < 2,
3.- Dada la función f (x) = 9 si 2 ≤ x ≤ 8,
 x2 −1

x−1 si x > 8,
hallar, en caso de que existan, f (0), f 0 (2), f 0 (8).
Resp.: No existen.
(
x3 + 2x + 2 si x ≤ 0,
4.- Dada la función f (x) =
x2 − 3x + 2 si x > 0.
(a) Estudiar la continuidad y derivabilidad.
(b) Probar que f (x) = 0 tiene al menos una raı́z en el intervalo
(−1, −1/2).
Resp.: (a) Continua en todo R pero derivable en R \ {0}.
(b) Aplicar el teorema de Bolzano.

195
5.- Calcular las derivadas de las siguientes funciones.


a) f (x) = x3 − x2 .
3x2 − 2x
Resp.: f 0 (x) = √ .
2 x3 − x2
r
1−x
b) f (x) = .
1 + x2
1 − x −1/2 x2 − 2x − 1
 
0
Resp.: f (x) = · .
1 + x2 2(1 + x2 )2

c) f (x) = (x2 − 1)2 (2x2 + 3x + 2)2 .


Resp.: f 0 (x) = 2(x2 − 1)(2x2 + 3x + 2)(8x3 + 9x2 − 3).


2 x(x − 1)
d) f (x) = .
x2 + 2
−x3 + 3x2 + 6x − 2
Resp.: f 0 (x) = √ .
x(2 + x2 )2

e) f (x) = sen2 x − 2 sen x cos x.


Resp.: f 0 (x) = sen 2x − 2 cos 2x.

f) f (x) = sen(sen(sen x)).


Resp.: f 0 (x) = cos(sen(sen x)) · cos(sen x) · cos x.

p
g) f (x) = 3
arc tg2 (x3 ).
2x2
Resp.: f 0 (x) = p .
(1 + x6 ) 3 arc tg(x3 )


h) f (x) = cos(x3 + 1) − sec(2 x).
1 √ √
Resp.: f 0 (x) = −3x2 sen(x3 + 1) − √ sec(2 x) tg(2 x).
x

i) f (x) = tg3 x + cos(x2 ) + sen4 (x2 + x).


Resp.: f 0 (x) = 3 tg2 x sec2 x−2x sen(x2 )+4(2x+1) sen3 (x2 +x) cos(x2 +
x).

196
j) f (x) = cos(x2 + 1) − arc tg(sec x).
sec x tg x
Resp.: f 0 (x) = −2x sen(x2 + 1) − .
1 + sec2 x

x
k) f (x) = ex .
x
Resp.: f 0 (x) = ex · xx · (1 + ln x).

6.- Sean f y g dos funciones derivables. Calcular la derivada de la


función y = f [g(xn )] · g[f (x)n ].
Resp.: y 0 = f 0 [g(xn )]·g 0 (xn )nxn−1 g[f (x)n ]+f [g(xn )]·g 0 [f (x)n ]nf (x)n−1 f 0 (x).

7.- Sean f y g dos funciones derivables tales que f (2) = 3, f 0 (2) =


−1, g(2) = −5, g 0 (2) = 2. Calcular
(a) (g − f )0 (2).
(b) (f · f )0 (2).
(c) (g/f )0 (2).
Resp.: (a) 3; (b) - 6; (c) 1/9.

8.- Sabiendo que f (0) = 1, g(0) = 2, h(0) = 1, f 0 (0) = 2, g 0 (0) = 1, h0 (0) =


2, h0 (1) = 1, h(1) = 2, f 0 (2) = 1, g(1) = 1, g 0 (1) = 3, calcular:
(a) (f ◦ g)0 (0).
(b) (f ◦ h ◦ g)0 (1).
Resp.: (a) f 0 [g(0)] · g 0 (0) = 1;
(b) f 0 [(h ◦ g)(1)] · h0 [g(1)] · g 0 (1) = 1 · 1 · 3 = 3.


9.- Calcular dy/dx si y = u2 + 1, u = sec x tg x.
Resp.: dy/dx = u(u2 + 1)−1/2 (sec3 x + sec x tg2 x).

x3 x2
10.- Sea f (x) = + − 2x. Encontrar todos los valores de x para
3 2
los que
(a) f 0 (x) = 0.
(b) f 0 (x) = −2.
(c) f 0 (x) = 10.

197
Resp.: (a) 1, -2; (b) 0, -1; (c) 3, -4.

11.- Probar que si f (x) es un polinomio cuyas n raı́ces x1 , x2 , . . . , xn


n
f 0 (x) X 1
son todas distintas, entonces = .
f (x) x − xi
i=1

Sugerencia: Escribir f (x) = a(x − x1 )(x − x2 ) . . . (x − xn ) y aplicar


derivación logarı́tmica.

12.- Dada la función f (x) = ax3 + bx2 + cx + d, hallar los valores de


las constantes a, b, c y d para que f (0) = 1, f 0 (−1) = 8, f 00 (0) =
−4, f 000 (x) = 2.
Resp.: a = 1/3, b = −2, c = 3, d = 1.

13.- Una bola se deja caer de una altura inicial de 122,5 m. y su


posición viene dada por s(t) = 122, 5 − 4, 9t2 .
(a) ¿Cuál es la velocidad instantánea en t = 1/2?
(b) ¿Cuánto tiempo tarda la bola en llegar al suelo?
(c) ¿Cuál es la velocidad de impacto?
Resp.: v(1/2) = −4, 9 m/sg; t = 5 sg; v(5) = −49 m/sg.

14.- Calcular la derivada de orden 10 de las siguientes funciones: a)

f (x) = 1/x.
Resp.: f (10) (x) = 10! x−11 . b) f (x) = sen x.

Resp.: f (10) (x) = − sen x.

15.- Calcular la derivada de orden 100 de las siguientes funciones:


1+x
a) f (x) = √ .
1−x

1 · 3 · 5 . . . 197 · (−x + 399)


Resp.: f (100) (x) = .
2100 · (1 − x)201/2

b) f (x) = x ln x.
Resp.: f (100) (x) = 98!x−99 .

198
16.- Mostrar que la función que se indica verifica la ecuación respec-
tiva.

a) y = x + sen 2x; y 00 + 4y = 4x.

b) y = x3 + x; y (v) + y (iv) + y 000 + y 00 + y 0 + y = x3 + 3x2 + 7x + 7.

17.- Dada la curva (x2 + y 2 )3 − 3(x2 + y 2 ) + 1 = 0, calcular y 0 e y 00 .


x x2 + y 2
Resp.: y 0 = − si x2 + y 2 6= 1 y además y 6= 0; y 00 = − .
y y3

18.- Probar que las curvas x2 + y 2 = 2, y 2 = 2x − 1 son ortogonales.


(Dos curvas son ortogonales cuando sus rectas tangentes en los
puntos de intersección de ambas curvas son perpendiculares.)
x 1
Resp.: Puntos de intersección (1, 1) y (1, −1). Derivadas y 0 = − , y 0 = .
y y

19.- Dadas las curvas x2 −y 2 = 5, 4x2 +9y 2 = 72, encontrar sus puntos
de intersección. Verificar que en tales puntos las curvas se cortan
en ángulo recto.
Resp.: (3, 2), (−3, 2), (3, −2), (−3, −2).
x 0 4x
Las derivadas de las curvas son y 0 = , y = − , respectivamente.
y 9y

20.- Dibujar la curva que tiene por ecuación:


a) x = 1 − t, y = 1 + t.
b) x = t + 1, y = 1t .

c) x = t, y = t.
Resp.: a) Recta x + y = 2.
1
b) Hipérbola y = .
x−1
c) Parábola y = x2 con x ≥ 0.


21.- Dadas las curvas 2x2 + 3y 3 = 5, {x = t, y = t para t > 0}.
(a) Encontrar el punto de intersección.

199
(b) Encontrar el ángulo de intersección de ambas curvas en dicho
punto.
Resp.: (a) (1, 1).
(b) tg β = 17/14.

22.- Encontrar laspecuaciones


√ de las rectas tangente y normal a la
curva f (x) = 3 1 + 1 + x en el punto de abscisa x = 0.
√ √ √ √
Resp.: y − 3 2 = x/(6 3 4); y − 3 2 = −6 3 4x.

23.- Escribir las ecuaciones de las rectas tangente y normal a la


función f (x) = arc cos 3x en el punto de intersección con el eje
Y.
Resp.: y − π/2 = −3x; y − π/2 = x/3.

24.- Hallar las ecuaciones de las tangentes a y = 4x2 que pasen por
(2, 0).
Resp.: y = 0; y = 32x − 64.

25.- Encontrar los puntos de la curva f (x) = 2x3 − 3x2 − 12x + 20


donde la tangente es paralela al eje X.
Resp.: (2, 0), (−1, 27).

26.- Encontrar los puntos de la curva f (x) = x3 + 2x2 − 4x + 5 cuya


recta tangente es paralela a la recta 8x + 2y − 5 = 0.
Resp.: (0, 5); (−4/3, 311/27).

27.- Probar que no existe ninguna recta que pase por (1, 2) y que sea
tangente a la curva y = 4 − x2 .
4 − x2 − 2
Resp.: La ecuación = −2x no tiene ninguna solución real.
x−1

x2 + a
28.- Hallar la ecuación de la tangente a la curva y = en x = 1.
x2 + b
Encontrar a y b de forma que la tangente hallada pase por el
punto (2, 1) y sea perpendicular a la recta 3x + 3y − 7 = 0.
1+a 2(b − a)
Resp.: y − = (x − 1); a = −1, b = 1.
1+b (1 + b)2

200
12
29.- Demostrar que las tangentes a la curva y = −4x + en sus
x
puntos de corte con la recta 2x − y = 0 son paralelas entre sı́.
Resp.: m1 = m2 = −10.

30.- Hallar las ecuaciones de las rectas que pasan por (6, 5) y son
tangentes a la curva y = x2 − 3x − 4.
Resp.: y = 15x − 85; y = 3x − 13.

31.- Encontrar los puntos de la curva y = x3 cuya recta tangente en


ese punto corta al eje X en el punto 4.
Resp.: (0, 0), (6, 216).

32.- Encontrar a, b, c, d de forma que la curva y = ax3 +bx2 +cx+d sea


tangente a la recta 3x−y−1 = 0 en (1, 2) y a la recta 10x−y−12 = 0
en (2, 8).
Resp.: a = 1, b = −1, c = 2, d = 0.

33.- Escribir la ecuación de la recta tangente a la curva (x+y)3 +x2 y =



y en (0, 1).
Resp.: 6x + 5y − 5 = 0.

34.- Probar que la recta tangente a la curva xy = k en el punto (a, b)


es bx + ay = 2k.
Resp.: Tener en cuenta que a · b = k para que el punto esté en la curva.

35.- ¿En qué puntos de la gráfica de x2 − 2xy + y 2 − 4 = 0 la recta


tangente es horizontal o vertical?
Resp.: Como y 0 = 1, no hay rectas tangentes horizontales ni verticales.

36.- Dada la curva yx2 + 2y 2 − x − 11 = 0, hallar las rectas tangentes


en los puntos de abscisa x = −1.
5 5 4
Resp.: y − 2 = (x + 1); y + = (x + 1).
9 2 9

201
37.- Dada la curva en coordenadas paramétricas

x(t) = 5 cos t + 1, y(t) = 4 sen t + 1.

(a) Hallar la ecuación cartesiana e identificar la curva.



(b) Hallar la ecuación de la recta tangente en (7/2, 2 3 + 1).
(c) ¿Para qué valores de t la recta tangente es paralela al eje X ?
(x − 1)2 (y − 1)2
Resp.: (a) + = 1; elipse centrada en (1, 1) y ejes x =
25 16
1, y = 1.
√ √
(b) 4x + 5 3y − 44 − 5 3 = 0.
(c) t = π/2, t = 3π/2.

38.- Encontrar las ecuaciones de las rectas tangentes a la curva



x(t) = sen t cos t, y(t) = t + 1.

que sean horizontales o verticales.


Resp.: No tiene tangentes horizontales.
sen(−2) 1 1
Tangentes verticales: x = ; x= ; x=− .
2 2 2

39.- Dada la curva


1 1
x = t + , y = t − para t > 0.
t t
(a) Encontrar la ecuación cartesiana.
(b) Encontrar los valores de t que hacen verticales a las tangentes
a la curva.
Resp: (a) x2 − y 2 = 4 (hipérbola).
(b) t = 1.

40.- Sea L1 la recta tangente a la curva y = f (x) definida implı́cita-



mente por la ecuación x2 y + y − 3x + 1 = 0 en el punto (1, 1).
Sea L2 la recta normal a la curva representada por la ecuación
x = t − 1, y = t2 , en el punto correspondiente a t = 0.
Calcular el punto de intersección de L1 y L2 .
Resp.: Rectas y − 1 = 23 (x − 1); x = −1. Intersección (−1, −1/3).

202
41.- Decidir si las proposiciones siguientes son verdaderas o falsas:
(a) Si y = f (x) es derivable en el punto x = a pero y = g(x) no
es derivable en x = a, entonces la función y = (f + g)(x) no es
derivable en x = a.
Resp.: Verdadero.

(b) Si f y g no son derivables en x = a, entonces f + g no es


derivable en x = a.
p p
Resp.: Falso. Ejemplo: f (x) = |x|, g(x) = −1/ |x| en x = 0.

(c) Si f y g no son derivables en x = a, entonces f · g no es


derivable en x = a.
√ √
3
Resp.: Falso. Ejemplo: f (x) = 3 x, g(x) = x2 en x = 0.

(d) Si y = f (x) es derivable en el intervalo (a, ∞) y existe lı́m f (x),


x→∞
entonces existe lı́m f 0 (x). El recı́proco es falso.
x→∞

sen x2
Resp.: La primera parte es falsa como muestra el ejemplo f (x) = .
x
La segunda parte es cierta pues basta comprobar el hecho con la fun-
ción f (x) = cos(ln x).

203
CAPÍTULO VI.
APLICACIONES DE LA
DERIVADA

SECCIONES
A. Crecimiento y decrecimiento. Máximos y mı́nimos locales.
B. Concavidad. Puntos de inflexión.
C. Representación gráfica de funciones.
D. Problemas de máximos y mı́nimos.
E. Teoremas del valor medio. Regla de L’Höpital.
F. Ejercicios propuestos.

205
A. CRECIMIENTO Y DECRECIMIENTO. MÁXIMOS Y MÍNI-
MOS LOCALES.

Los intervalos de crecimiento y decrecimiento de una función y = f (x) se


obtienen a partir de la primera derivada de la función por la siguiente re-
gla:
(a) f crece en un intervalo (a, b) si f 0 (x) > 0 para todo x en (a, b).
(b) f decrece en un intervalo (a, b) si f 0 (x) < 0 para todo x en (a, b).
Los puntos extremos de intervalos en donde cambia el signo de la derivada
son los máximos o mı́nimos, según la derivada cambie de positiva a negativa
o de negativa a positiva, respectivamente. En resumen:
(a) Un punto x0 del dominio de la función corresponde a un máximo local o
relativo si existe un intervalo (x0 − δ, x0 ) en donde f crece y otro intervalo
(x0 , x0 + δ) en donde f decrece.
(b) Un punto x0 del dominio de la función corresponde a un mı́nimo local o
relativo si existe un intervalo (x0 − δ, x0 ) en donde f decrece y otro intervalo
(x0 , x0 + δ) en donde f crece.
Los máximos y mı́nimos locales se encuentran entre los llamados puntos
singulares o crı́ticos, es decir, puntos del dominio de la función en donde la
derivada se anula o no existe.

PROBLEMA 6.1.

Estudiar el crecimiento y decrecimiento de la función



f (x) = x( x + 1).

Solución

Calculamos la derivada de la función:


√ √
0 √ x 2x + 2 x + x 3x + 2 x
f (x) = x + 1 + √ = √ = √ .
2 x 2 x 2 x
√ √
La derivada se anula cuando 3x + 2 x = 0 y no existe cuando 2 x = 0.
Despejamos x en ambas ecuaciones:
√ √ 4
3x + 2 x = 0 =⇒ 3x = −2 x =⇒ 9x2 = 4x =⇒ x = 0 ó x = .
9

206
Como el valor x = 4/9 no verifica la primera ecuación, el único valor que
anula f 0 (x) es x = 0. Por otra parte,

2 x = 0 ⇐⇒ x = 0.

El único punto crı́tico es x = 0. Como el dominio de la función es el intervalo


[0, ∞) y f 0 (x) ≥ 0 en todo el dominio, la función es siempre creciente. Por
tanto, el punto (0, 0) es el mı́nimo de la función.

PROBLEMA 6.2.

Estudiar el crecimiento y decrecimiento de la función


x3 + 4
f (x) = .
x2

Solución

De nuevo calculamos la derivada:


3x2 · x2 − (x3 + 4) · 2x x4 − 8x x3 − 8
f 0 (x) = 4
= 4
= .
x x x3
La derivada se anula cuando x3 − 8 = 0 y no existe cuando x3 = 0. Despe-
jaremos x en ambas ecuaciones:

x3 − 8 = 0 ⇐⇒ x3 = 8 ⇐⇒ x = 2.

x3 = 0 ⇐⇒ x = 0.
Como el dominio de la función es R \ {0}, el único punto crı́tico es x = 2.
Estudiamos el crecimiento en los intervalos (−∞, 0), (0, 2) y (2, ∞). Para
ello, sustituimos la derivada de la función en cualquier punto interior a los
intervalos. El signo de la derivada indicará si la función original crece o
decrece. Ası́:
f 0 (−1) = −9/ − 1 > 0 =⇒ la función crece en (−∞, 0).
f 0 (1) = −7/1 < 0 =⇒ la función decrece en (0, 2).
f 0 (3) = 19/27 > 0 =⇒ la función crece en (2, ∞).
Un método más cómodo por su claridad visual consiste en representar el
dominio de la función sobre la recta real. A continuación, colocar en la
misma recta los puntos crı́ticos. De esta manera quedan ya delimitados los
intervalos que se van a estudiar. Después de sustituir en la derivada de la

207
función algún punto intermedio de cada intervalo, colocar el signo + ó −
según si dicha derivada es positiva o negativa. Ası́ quedan completamente
determinados los intervalos y el comportamiento de la función en cada uno
de ellos. En este ejemplo hubiera quedado ası́:

(−∞, 0) (0, 2) (2, ∞)


f 0 (x) ++ –– ++

PROBLEMA 6.3.

Encontrar los máximos y mı́nimos locales de la función


f (x) = x5 − 5x + 6.

Solución

Busquemos los puntos crı́ticos de la función:

f 0 (x) = 5x4 − 5; f 0 (x) = 0 ⇐⇒ 5x4 = 5 ⇐⇒ x4 = 1 ⇐⇒ x = ±1.

Como los puntos crı́ticos son x = 1 y x = −1, estudiamos el signo de la


derivada en los intervalos siguientes:

(−∞, −1) (−1, 1) (1, ∞)


f 0 (x) ++ –– ++

Como a la izquierda de x = −1 la función es creciente y a la derecha es


decreciente, el punto corresponde a un máximo. Sustituyendo en la función
se obtiene que el punto es (−1, 10).
Análogamente, a la izquierda de x = 1 la función decrece y a la derecha crece.
El punto corresponde a un mı́nimo y sus coordenadas son (1, 2).

PROBLEMA 6.4.

Encontrar los máximos y mı́nimos locales de la función



f (x) = x 1 − x2 .

208
Solución

Los puntos crı́ticos se obtienen de la siguiente forma:


p −x 1 − x2 − x2 1 − 2x2
f 0 (x) = 1 − x2 + x · √ = √ =√ .
1 − x2 1 − x2 1 − x2
1
f 0 (x) = 0 ⇐⇒ 1 = 2x2 ⇐⇒ x2 = 1/2 ⇐⇒ x = ± √ .
2

Además f 0 (x) no existe cuando 1 − x2 = 0, es decir, cuando x = 1 ó x =
−1.
Como el dominio de la función es el intervalo [−1, 1], estudiamos el signo de
la derivada en los intervalos que se ilustran en la tabla:

√ √ √ √
(−1, −1/ 2) (−1/ 2, 1/ 2) (1/ 2, 1)
f 0 (x) –– ++ ––

1
Lo anterior quiere decir que el punto x = − √ da lugar a un mı́nimo local
2
1
y x = √ a un máximo local. Los puntos correspondientes de la función son
√ 2 √
(−1/ 2, −1/2) y (1/ 2, 1/2).

PROBLEMA 6.5.

¿La función f (x) = (x−1)3 (x+2)2 alcanza un máximo o un mı́nimo


en el punto A(1, 0)?

Solución

Como

f 0 (x) = 3(x − 1)2 (x + 2)2 + 2(x − 1)3 (x + 2) = (x − 1)2 (x + 2)(5x + 4),

se obtiene que f 0 (1) = 0, con lo que el punto (1, 0) es singular. Para ver
si corresponde a un máximo o un mı́nimo, estudiamos el crecimiento de la
función en un entorno de x = 1:
Si x ∈ (1 − δ, 1), f 0 (x) > 0 y f es creciente;

209
Si x ∈ (1, 1 + δ), f 0 (x) > 0 y f es también creciente.
Como no cambia el signo de la derivada, la función en el punto (1, 0) no
alcanza ni un máximo ni un mı́nimo relativo.

PROBLEMA 6.6.

Dada la función y = (x + 1)2 e−x , hallar los máximos y mı́nimos


locales.

Solución

Calculamos la primera derivada para determinar los puntos crı́ticos:


f 0 (x) = 2(x + 1) · e−x − e−x (x + 1)2 = e−x (x + 1)(1 − x);
f 0 (x) = 0 ⇐⇒ e−x (x + 1)(1 − x) = 0 ⇐⇒ x = 1 ó x = −1.
Los únicos posibles máximos y mı́nimos se alcanzan en los puntos P1 (1, 4/e)
y P2 (−1, 0). Estudiamos a continuación el crecimiento de la función:

(−∞, −1) (−1, 1) (1, ∞)


f 0 (x) –– ++ ––

De lo anterior se deduce que el punto P1 (1, 4/e) es un máximo local y que


el punto P2 (−1, 0) es un mı́nimo local de la función.

PROBLEMA 6.7.

Hallar, en caso de que existan, las abscisas de los máximos y mı́ni-


mos de la función
1 5
y = x3 + x2 + 14x + 10 ln(x + 3) + 90 ln(x − 5).
3 2

Solución

La primera derivada es
10 90
y 0 = x2 + 5x + 14 + +
x+3 x−5
(x2 + 5x + 14)(x + 3)(x − 5) + 10(x − 5) + 90(x + 3)
= .
(x + 3)(x − 5)

210
Al anular la derivada, obtenemos:

y 0 = 0 ⇐⇒ (x2 + 5x + 14)(x + 3)(x − 5) + 10(x − 5) + 90(x + 3) = 0


⇐⇒ x4 + 3x3 − 11x2 − 3x + 10 = 0.

Si aplicamos la regla de Ruffini, resultan las raı́ces 1, −1, 2 y −5.


Teniendo en cuenta que el dominio de la función es el conjunto D(f ) = {x |
x > 5}, ninguno de los puntos anteriores pertenece al dominio por lo que la
función carece de máximos y mı́nimos. Además, como f 0 (x) > 0, ∀x ∈ D(f ),
f es siempre creciente.

PROBLEMA 6.8.

Hallar un polinomio de tercer grado sabiendo que para x = 3 al-


canza un mı́nimo local, para x = 2 alcanza un máximo local y para
x = 0 y x = 1 toma los valores 1 y 29/6, respectivamente.

Solución

Escribimos la forma general de un polinomio de grado 3 como f (x) = ax3 +


bx2 + cx + d.
Para x = 0, f (0) = 1 = d.
Para x = 1, 29/6 = a + b + c + d.
Como f 0 (x) = 3ax2 + 2bx + c, tenemos:
Para x = 3, f 0 (3) = 0 = 27a + 6b + c.
Para x = 2, f 0 (2) = 0 = 12a + 4b + c.
Resulta ası́ el sistema de ecuaciones:

a + b + c + d = 29/6,
27a + 6b + c = 0,
12a + 4b + c = 0,
d = 1.

Al resolver el sistema llegamos a la solución a = 1/3, b = −5/2, c = 6, d =


1 5
1, y la función solución es f (x) = x3 − x2 + 6x + 1.
3 2

211
PROBLEMA 6.9.

Sea y = f (x) una función cuya derivada tiene una gráfica como la
que se muestra en la figura.
a) ¿Qué se puede decir de f en x = b?
b) ¿Tiene f algún máximo?
c) ¿Dónde f decrece?

y = f 0 (x)

a b c

Solución

a) De la gráfica se deduce que f 0 (x) no es continua en x = b, porque


los lı́mites laterales son diferentes. Esto indica que la función f no es
derivable en x = b.
b) En todo el intervalo (a, b) la derivada de f es positiva. Por lo tanto, la
función crece.
De la misma manera, en el intervalo (b, c) la función f también crece.
El máximo se encuentra en c, que es el extremo derecho del intervalo
donde está definida la función.
c) De lo anterior se deduce que en ningún momento la función decrece,
porque la derivada nunca es negativa.

B. CONCAVIDAD. PUNTOS DE INFLEXIÓN.

Una función se dice cóncava hacia arriba o convexa en un intervalo (a, b)


cuando al unir dos puntos de la curva en ese intervalo, el segmento que se

212
forma queda por encima de la curva. De la misma forma, será cóncava hacia
abajo o cóncava cuando cualquiera de dichos segmentos queda por debajo
de la curva.

Ilustramos lo anterior con las siguientes figuras:

a b a b a b
cóncava hacia arriba cóncava hacia abajo ambas concavidades

Para determinar los intervalos de concavidad de una función utilizamos el


siguiente criterio:

La gráfica de una función y = f (x) es

a) cóncava hacia arriba en todos los intervalos para los que f 00 (x) > 0.

b) cóncava hacia abajo en todos los intervalos para los que f 00 (x) < 0.

Reuniendo este criterio con el de determinación de intervalos de crecimiento


de f , podemos también establecer la siguiente regla:

a) En los intervalos donde f 0 (x) sea creciente, f (x) será cóncava hacia
arriba.

b) En los intervalos donde f 0 (x) sea decreciente, f (x) será cóncava hacia
abajo.

Se llaman puntos de inflexión los puntos en donde cambia la concavidad de


una función, ya sea de arriba hacia abajo, o viceversa. Para ello, si la función
posee derivadas de segundo orden, un punto x0 del dominio de f será punto
de inflexión si f 00 (x0 ) = 0 y ocurre alguna de las siguientes situaciones:

a) existe un intervalo (x0 − δ, x0 ) en donde f 00 (x) < 0 y otro intervalo


(x0 , x0 + δ) en donde f 00 (x) > 0.

b) existe un intervalo (x0 − δ, x0 ) en donde f 00 (x) > 0 y otro intervalo


(x0 , x0 + δ) en donde f 00 (x) < 0.

213
PROBLEMA 6.10.

x2
Estudiar la concavidad de la función f (x) = y localizar sus
1+x
puntos de inflexión.

Solución

En primer lugar calcularemos las derivadas de primer y segundo orden de la


función:
2x(1 + x) − x2 x2 + 2x
f 0 (x) = = .
(1 + x)2 (1 + x)2

(2x + 2)(1 + x)2 − (x2 + 2x) · 2(1 + x)


f 00 (x) =
(1 + x)4
(2x + 2)(1 + x) − (x2 + 2x) · 2 2
= 3
= .
(1 + x) (1 + x)3
Para estudiar el signo de la segunda derivada, observamos que el numerador
nunca se anula. En cambio, el denominador se anula en el punto de abscisa
x = −1, que es precisamente el punto que no está en el dominio. Para
estudiar el signo en los intervalos que este punto determina, construimos el
siguiente diagrama de signos:

(−∞, −1) (−1, ∞)


f 00 (x) –– ++

Al calcular f 00 (x) en un punto del intervalo (−∞, −1) resultó un valor nega-
tivo y al sustituir en la función un punto del intervalo (−1, ∞) dió un valor
positivo. Eso quiere decir que:
f es cóncava hacia arriba en el intervalo (−1, ∞);
f es cóncava hacia abajo en el intervalo (−∞, −1).
Sin embargo no hay punto de inflexión pues el punto x = −1, en donde
cambia la concavidad, no está en el dominio.

PROBLEMA 6.11.

Estudiar la concavidad de la función f (x) = 2 − |x5 − 1| y localizar


sus puntos de inflexión.

214
Solución

Debemos descomponer en primer lugar la función para eliminar el valor


absoluto.
Como x5 − 1 = (x − 1)(x4 + x3 + x2 + x + 1) y el segundo factor es siempre
positivo, resulta que
( (
2 − (x5 − 1) si x ≥ 1 3 − x5 si x ≥ 1,
f (x) = =
2 + (x5 − 1) si x < 1 1 + x5 si x < 1.

Debemos separar el estudio de la concavidad para cada uno de los intervalos


(−∞, 1) y (1, ∞).
* En (−∞, 1): f 0 (x) = 5x4 (siempre positiva; la función es creciente en ese
intervalo).
f 00 (x) = 20x3 . Realizamos el estudio de los signos como en los ejemplos
anteriores:

(−∞, 0) (0, 1)
f 00 (x) –– ++

* En (1, ∞): f 0 (x) = −5x4 (siempre negativa; la función es decreciente en


ese intervalo).
f 00 (x) = −20x3 . En este intervalo x es siempre positiva; de este modo f 00 (x)
es negativa.
En definitiva llegamos al siguiente resultado:
f es cóncava hacia arriba en (0, 1) y cóncava hacia abajo en (−∞, 0) y en
(1, ∞).
Los puntos de abscisa x = 0 y x = 1 son de inflexión pues cambia la
concavidad. Al sustituir dichos valores en la función se obtienen los puntos
(0, 1) y (1, 2).

PROBLEMA 6.12.

Estudiar la concavidad y convexidad de la función


1
f (x) = .
1 + x2

215
Solución

Para esta función,

−2x (1 + x2 )2 (−2) + 2x · [2(1 + x2 ) · 2x] 2(3x2 − 1)


f 0 (x) = ; f 00
(x) = = .
(1 + x2 )2 (1 + x2 )4 (1 + x2 )3

Observamos que el denominador


p de f 00 es siempre positivo y el numerador se
anula cuando x = ± 1/3, con lo que tenemos la siguiente tabla de signos:

p p p p
x < − 1/3 − 1/3 < x < 1/3 x> 1/3
3x2 − 1 ++ –– ++
f 00 (x) ++ –– ++
p p
De aquı́ deducimos que f pes convexa
p en (−∞, − 1/3) y ( 1/3, ∞), mien-
tras que es cóncava en (− 1/3, 1/3).

C. REPRESENTACIÓN GRÁFICA DE FUNCIONES.

En este apartado se resumen todas las técnicas estudiadas anteriormente y


se agrupan para dibujar con relativa precisión las curvas correspondientes
a funciones definidas en forma explı́cita. Toda la información que se pueda
obtener de la función será útil para conseguir una gráfica más exacta.
En la primera parte se darán algunas informaciones de cierta función des-
conocida y se tratará de dibujar una función que verifique dichos datos. El
método más adecuado consiste en seguir los siguientes pasos:

1) Construir una tabla donde se expresen los intervalos conocidos de creci-


miento y concavidad de la función, tal como se hizo en los apartados ante-
riores.

2) Dibujar en un sistema de coordenadas los puntos por donde se sepa que


pasa la función, especificando si son máximos, mı́nimos o puntos de inflexión,
u otro tipo de puntos. Después se deben trazar las ası́ntotas conocidas.

216
3) Por último, trazar ”pedazos”de curva que tengan la forma indicada por el
crecimiento y la concavidad correspondientes a cada intervalo, y de acuerdo
a la tabla construida, de modo que pase por los puntos dibujados en el paso
2 y tenga como ası́ntotas las ya dibujadas.

En la segunda parte se pretende hacer el estudio completo de funciones


definidas en forma explı́cita y dibujar su gráfica.

Para dibujar en forma precisa la gráfica de una curva definida en forma


explı́cita y = f (x), es aconsejable realizar los siguientes pasos:

a) Calcular el dominio de la función.

b) Determinar las posibles simetrı́as de la función.

- Será simétrica respecto al eje Y si f (x) = f (−x), es decir, si es par.

- Será simétrica respecto al origen si f (x) = −f (−x), es decir, si es impar.

c) Encontrar los puntos de intersección de la curva con los ejes de coorde-


nadas:

- Haciendo x = 0 en la función se obtienen los puntos de corte con el eje


Y.

- Haciendo y = 0, se obtienen los puntos de corte con el eje X.

d) Determinar los intervalos de crecimiento y concavidad de la función, me-


diante el estudio del signo de las derivadas de primero y segundo orden.

e) Encontrar los puntos de la gráfica donde esta toma valores máximos y


mı́nimos locales y puntos de inflexión. Estos serán los puntos donde cambie
de signo la derivada primera y segunda, respectivamente.

f) Encontrar las posibles ası́ntotas de la función.

Después de esto, podemos dibujar la gráfica de la función siguiendo los tres


pasos que aplicamos en la primera parte. En este caso la información sobre
la función es completa y sólo puede haber una función cuya gráfica sea la
conseguida de este modo.

217
PROBLEMA 6.13.

Trazar una curva que verifique las siguientes condiciones:

f (−2) = 8; f (0) = 4; f (2) = 0;

f 0 (x) > 0 si |x| > 2; f 0 (2) = f 0 (−2) = 0; f 0 (x) < 0 si |x| < 2;
f 00 (x) < 0 si x < 0; f 00 (x) > 0 si x > 0.

Solución

Como conocemos el signo de las derivadas primera y segunda, podemos ma-


nejar a la vez el crecimiento y la concavidad de la función. Será conveniente
por lo tanto conocer la forma de la gráfica para las distintas combinacio-
nes de crecimiento y concavidad. Las cuatro posibilidades se ilustran en el
siguiente diagrama.

Cualquier gráfica de curva se puede realizar componiendo ”trozos”de cur-


va como los anteriores sin más que conocer los intervalos de crecimiento y
concavidad.

En nuestro caso la siguiente tabla permitirá ver con facilidad cuáles son esos
intervalos, debido a que se conocen los signos de las derivadas primera y
segunda:

218
(−∞, −2) (−2, 0) (0, 2) (2, ∞)
f 0 (x) ++ –– –– ++
f 00 (x) –– –– ++ ++

Añadiendo a esta información los datos f (−2) = 8, f (0) = 4, f (2) = 0,


podemos dibujar algo como:

PROBLEMA 6.14.

Dibujar la gráfica de una función y = f (x) que cumpla las condi-


ciones siguientes:
(a) f (1) = f (3) = 0.
(b) f 0 (x) < 0 si x ∈ (−∞, 2) ∪ (3, ∞); f 0 (x) > 0 si x ∈ (2, 3); f 0 (2) no
existe; f 0 (3) = 0.
(c) f 00 (x) < 0 excepto en x = 2.
(d) La recta y = 3 es ası́ntota.

Solución

De la información suministrada podemos extraer la siguiente tabla de sig-


nos:

219
(−∞, 2) (2, 3) (3, ∞)
f 0 (x) –– ++ ––
f 00 (x) –– –– ––

Además sabemos que la función corta al eje X en los puntos (1, 0) y (3, 0).
El punto (3, 0) debe ser un mı́nimo relativo porque a la izquierda de x = 3
la función es creciente, y a la derecha es decreciente. No hay puntos de
inflexión porque en ningún momento cambia de signo la derivada segunda.
Si dibujamos en primer lugar los puntos conocidos y la ası́ntota horizontal
y = 3, la información anterior produce una gráfica como la siguiente:

Observa que la ası́ntota y = 3 sólo se acerca a la función cuando x → −∞


pero no cuando x → ∞. Esto es válido porque no necesariamente una función
tiene que tener la misma ası́ntota en ambos extremos.

Se debe tener claro que esta no es la única función que verifica los datos
proporcionados. Lo que tratamos de hacer es construir una de las funciones
que verifique dichos datos.

220
PROBLEMA 6.15.

Dibujar la gráfica de una función y = f (x) que cumpla las condi-


ciones siguientes:
(a) D(f ) = (−∞, 0) ∪ (0, ∞).
(b) Decrece en (−∞, −3) ∪ (0, 2) ∪ (2, 4).
(c) Tiene un mı́nimo local en x = 4 y un punto de inflexión en
x = 1.
(d) La recta y = (3x − 6)/2 es ası́ntota y corta a la curva en el
punto de abscisa x = 3/2.
(e) lı́m f (x) = 3, lı́m f (x) = −2, lı́m f (x) = −4,
x→−∞ x→−3− x→−3+
lı́m f (x) = 0, lı́m f (x) = ∞,
x→0− x→0+
lı́m f (x) = −∞, lı́m f (x) = ∞.
x→2− x→2+

Solución

Los datos que aquı́ se presentan dan lugar a la tabla siguiente:

(−∞, −3) (−3, 0) (0, 2) (2, 4) (4, ∞)


f 0 (x) –– ++ –– –– ++

Como no se da información sobre la derivada segunda, tenemos libertad


de graficar la concavidad como nos parezca más adecuado. El criterio que
seguiremos aquı́ es el de dibujar una gráfica lo más suave posible. De todas
formas en x = 1 debe haber un punto de inflexión.
La parte (e) indica que las ası́ntotas de la función son las rectas y = 3
(cuando x → −∞), x = 0 (sólo para x > 0), x = 2 (a ambos lados),
y = (3x − 6)/2 (cuando x → ∞).
Después de dibujar las ası́ntotas y de señalar en la oblicua el punto x = 3/2
(por donde debe pasar la función), la información de la tabla da lugar a una
gráfica como la que se muestra.

221
Observa que x = 0 no está en el dominio tal como se pide. Además debemos
definir de cualquier manera f (2) para que esté en el dominio (en la gráfica
se hizo f (2) = 0). También debe estar en el dominio x = −3, por lo que se
puede definir arbitrariamente.

En todos los ejercicios siguientes se debe hacer el estudio completo y dibujar


la gráfica de la función que se indique.

PROBLEMA 6.16.

Dibujar la gráfica de la función f (x) = x2/3 (8 − x).

Solución

(a) D(f ) = R porque la raı́z cúbica existe para cualquier número real.
(b) f (−x) = (−x)2/3 (8 − (−x)) = x2/3 (8 + x).
Como f (−x) 6= f (x) y f (−x) 6= −f (x), la función no es simétrica.
(c) Si x = 0, f (0) = 0. Por tanto, la curva corta al eje Y en el punto
(0, 0).
Si y = 0, 0 = x2/3 (8 − x). Entonces x = 0 ó x = 8. La curva corta al eje X
en los puntos (0, 0) y (8, 0).

222
(d) Al calcular las dos primeras derivadas de la función resulta:

16 − 5x 16
y 0 = x−1/3 · ; puntos crı́ticos: x = 0, x = .
3 5
−16
y 00 = x−4/3 · ; punto crı́tico: x = 0.
9
Estudiando el signo de estas derivadas, podemos escribir la siguiente ta-
bla:

(−∞, 0) (0, 16/5) (16/5, ∞)


f 0 (x) –– ++ ––
f 00 (x) –– –– ––

Por tanto, f crece en (0, 16/5) y decrece en (−∞, 0) ∪ (16/5, ∞).


Además, f es cóncava hacia abajo en todo su dominio.
(e) Observando la tabla anterior se pueden ver los puntos donde cambia de
signo alguna derivada. De esto se deduce la siguiente información:
Cuando x = 0 la función alcanza un mı́nimo local, y cuando x = 16/5
alcanza un máximo local.
Además no hay puntos de inflexión.
Los puntos de la curva correspondientes a estos valores de x son (0, 0) (mı́ni-
√ !
16 96 3 20
mo local) y , (máximo local).
5 25

(f) - Como D(f ) = R, no hay ası́ntotas verticales.


- Para buscar las ası́ntotas horizontales, debemos calcular los siguientes lı́mi-
tes:
lı́m f (x) = lı́m x2/3 (8 − x) = −∞.
x→∞ x→∞

lı́m f (x) = lı́m x2/3 (8 − x) = ∞.


x→−∞ x→−∞

No hay ası́ntotas horizontales porque ambos lı́mites son infinitos.


- Veamos si hay ası́ntotas oblicuas:

f (x) 8−x
m = lı́m = lı́m x2/3 · = lı́m x2/3 (8/x − 1) = −∞.
x→∞ x x→∞ x x→∞

f (x) 8−x
m = lı́m = lı́m x2/3 · = lı́m x2/3 (8/x − 1) = −∞.
x→−∞ x x→−∞ x x→−∞

223
No hay ası́ntotas oblicuas.
(g) Reuniendo toda la información anterior, debemos dibujar en un sistema
de coordenadas los puntos ya obtenidos para después unirlos con secciones
de curva cuya forma obedezca a la conseguida por el crecimiento y la con-
cavidad. Intenta por tı́ mismo dibujar la curva y verás que te sale como la
siguiente:

PROBLEMA 6.17.

x
Representar gráficamente la función f (x) = √
3
.
x−4

Solución

(a) D(f ) = R \ {4} por ser x = 4 el único punto donde se anula el denomi-
nador (la raı́z cúbica se puede calcular para cualquier número real).
−x
(b) f (−x) = √
3
.
−x − 4
Como f (−x) 6= f (x) y f (−x) 6= −f (x), la función no es simétrica.
(c) Si x = 0, f (0) = 0. Por tanto, la curva corta al eje Y en el punto
(0, 0).
x
Si y = 0, 0 = √
3
. Entonces x = 0. La curva corta al eje X en el mismo
x−4
punto (0, 0).

224
(d) Al calcular las dos primeras derivadas de la función resulta:

2(x − 6)
f 0 (x) = ; punto crı́tico: x = 6.
3(x − 4)4/3

(El punto x = 4, aunque anula el denominador, no está en el dominio.)

2(−x + 12) 00
f 00 (x) = ; f (x) = 0 ⇐⇒ x = 12.
9(x − 4)7/3

Estudiando el signo de estas derivadas, podemos escribir la siguiente ta-


bla:

(−∞, 4) (4, 6) (6, 12) (12, ∞)


f 0 (x) –– –– ++ ++
f 00 (x) –– ++ ++ ––

Por tanto, f crece en (6, ∞) y decrece en (−∞, 4) ∪ (4, 6).


Además, f es cóncava hacia arriba en (4, 12) y hacia abajo en (−∞, 4) ∪
(12, ∞).
(e) Observando la tabla anterior se pueden ver los puntos donde cambia de
signo alguna derivada. De esto se deduce la siguiente información:
Cuando x = 6, la función alcanza un mı́nimo local.
Además hay un punto de inflexión cuando x = 12 (aunque en x = 4 cambia la
concavidad, el punto no es de inflexión porque no está en el dominio).
Los puntos de la curva correspondientes a estos valores de x son:

3
(6, 6/ 2), (12, 6).

(f) - La recta x = 4 puede ser ası́ntota vertical, porque dicho punto no


está en el dominio.
En efecto, lı́m f (x) = ∞.
x→4

- Veamos si hay ası́ntotas horizontales:


x
lı́m f (x) = lı́m √
3
= ∞.
x→∞ x→∞ x−4

(Se trata de un lı́mite indeterminado de la forma ∞/∞ cuyo resultado es


∞ porque el grado del numerador es mayor que el grado del denomina-
dor).
x
lı́m f (x) = lı́m √
3
= ∞.
x→−∞ x→−∞ x−4

225
No hay ası́ntotas horizontales porque ambos lı́mites son infinitos.
- Veamos si hay ası́ntotas oblicuas:

f (x) 1
m = lı́m = lı́m √
3
= 0.
x→∞ x x→∞ x−4

f (x) 1
m = lı́m = lı́m √
3
= 0.
x→−∞ x x→−∞ x−4
Tampoco hay ası́ntotas oblicuas (la pendiente no puede ser 0).
(g) La gráfica que resulta del estudio realizado es la siguiente:

PROBLEMA 6.18.


3
Representar la gráfica de la función f (x) = 6x2 + x3 .

Solución

(a) D(f ) = R (la raı́z cúbica se puede calcular para cualquier número
real).
p √
(b) f (−x) = 3 6(−x)2 + (−x)3 = 3 6x2 − x3 .
Como f (−x) 6= f (x) y f (−x) 6= −f (x), la función no es simétrica.
(c) Si x = 0, f (0) = 0. Por tanto, la curva corta al eje Y en el punto
(0, 0).

Si y = 0, 0 = 3 6x2 + x3 . Entonces x = 0 ó x = −6.

226
La curva corta al eje X en los puntos (0, 0) y (−6, 0).
(d) Al calcular las dos primeras derivadas de la función resulta:

x(4 + x)
f 0 (x) = ; puntos crı́ticos: x = 0, x = −4, x = −6.
(6x2 + x3 )2/3

−8x2
f 00 (x) = ; puntos crı́ticos: x = 0, x = −6.
(6x2 + x3 )5/3
Estudiando el signo de estas derivadas, podemos escribir la siguiente ta-
bla:

(−∞, −6) (−6, −4) (−4, 0) (0, ∞)


f 0 (x) ++ ++ –– ++
f 00 (x) ++ –– –– ––

Por tanto, f crece en (−∞, −4) ∪ (0, ∞) y decrece en (−4, 0).


Además, f es cóncava hacia arriba en (−∞, −6) y hacia abajo en (−6, ∞).
(e) Observando la tabla anterior se pueden ver los puntos donde cambia de
signo alguna derivada. De esto se deduce la siguiente información:
Cuando x = −4, la función alcanza un máximo local y cuando x = 0, un
mı́nimo local.
Además cuando x = −6 hay un punto de inflexión (aquı́ la derivada segunda
no existe).
Los puntos de la curva correspondientes a estos valores de x son:

3
(−6, 0), (−4, 2 4), (0, 0).

(f) - No hay ası́ntotas verticales porque D(f ) = R.


- Veamos si hay ası́ntotas horizontales:
p
3
lı́m f (x) = lı́m 6x2 + x3 = ∞.
x→∞ x→∞
r
p
3 6 3
lı́m f (x) = lı́m 6x2 + x3+ 1 = −∞.
= lı́m x
x→−∞ x→−∞ x→−∞ x
No hay ası́ntotas horizontales porque ambos lı́mites son infinitos.
- Veamos si hay ası́ntotas oblicuas:

q
x 3 x6 + 1
3
r
f (x) 6x2 + x3 3 6
m1 = lı́m = lı́m = lı́m = lı́m + 1 = 1.
x→∞ x x→∞ x x→∞ x x→∞ x

227
r
f (x) 3 6
m2 = lı́m = lı́m + 1 = 1.
x→−∞ x x→−∞ x
p
3
b1 = lı́m [f (x) − m1 x] = lı́m ( 6x2 + x3 − x)
x→∞ x→∞
√ p √
( 3 6x2 + x3 − x)( 3 (6x2 + x3 )2 + x 3 6x2 + x3 + x2 )
= lı́m p √
x→∞ 3
(6x2 + x3 )2 + x 3 6x2 + x3 + x2
6x2 + x3 − x3 6
= lı́m p √3
= = 2,
x→∞ 3 (6x2 + x3 )2 + x 6x2 + x3 + x2 1+1+1
(el grado del numerador es igual al grado del denominador; sólo se necesita
dividir los coeficientes de los términos de mayor grado).
Análogamente se calcula b2 = lı́m [f (x) − m2 x] y se obtiene también que
x→−∞
b2 = 2.
Se deduce que la recta y = x + 2 es ası́ntota oblicua.
(g) La gráfica que resulta del estudio realizado es la siguiente:

PROBLEMA 6.19.

x
Representar gráficamente la función f (x) = .
x2 − 4x + 3

Solución

(a) D(f ) = R \ {1, 3}.

228
−x −x
(b) f (−x) = = 2 .
(−x)2 − 4(−x) + 3 x + 4x + 3
Como f (−x) 6= f (x) y f (−x) 6= −f (x), la función no es simétrica.
(c) Si x = 0, f (0) = 0. Por tanto, la curva corta al eje Y en el punto
(0, 0).
Si y = 0, entonces x = 0. La curva corta al eje X en el mismo punto
(0, 0).
(d) Al calcular las dos primeras derivadas de la función resulta:

−x2 + 3
f 0 (x) = .
(x2 − 4x + 3)2
√ √
Puntos crı́ticos: x = 3, x = − 3.

2(x3 − 9x + 12)
f 00 (x) = .
(x2 − 4x + 3)3
La segunda derivada se hará cero en algún valor ”a” comprendido entre −4 y
−3, porque el numerador toma un valor negativo cuando x = −4 y un valor
positivo cuando x = −3. Por el teorema de Bolzano, debe tener una raı́z en
algún punto del intervalo (−4, −3). Esa raı́z es única porque el polinomio
x3 − 9x + 12 es positivo cuando x > −3.
Estudiando el signo de estas derivadas, podemos construir la siguiente ta-
bla:

√ √ √ √
(−∞, a) (a, − 3) (− 3, 1) (1, 3) ( 3, 3) (3, ∞)
f 0 (x) –– –– ++ ++ –– ––
f 00 (x) –– ++ ++ –– –– ++

(e) Observando la tabla anterior se pueden ver los puntos donde cambia de
signo alguna derivada. De esto se deduce la siguiente información:
√ √
Cuando x = − 3, la función alcanza un mı́nimo local. Cuando x = 3,
alcanza un máximo. Los puntos correspondientes de la función son:
√ √ √ √ √ √
(− 3, − 3/(6 + 4 3)), ( 3, 3/(6 − 4 3)).

Además hay un punto de inflexión cuando x = a.


(f) - Las rectas x = 1 y x = 3 pueden ser ası́ntotas verticales, porque los
puntos no están en el dominio.
En efecto, es evidente que lı́m f (x) = ∞ y además lı́m f (x) = ∞.
x→1 x→3

229
- Veamos si hay ası́ntotas horizontales:
x
lı́m f (x) = lı́m = 0.
x→∞ x→∞ x2 − 4x + 3
(Se trata de un lı́mite indeterminado de la forma ∞/∞ cuyo resultado
es 0 porque el grado del numerador es menor que el grado del denomi-
nador.)
x
lı́m f (x) = lı́m 2 = 0.
x→−∞ x→−∞ x − 4x + 3
La recta y = 0 es ası́ntota horizontal en ambos extremos del dominio.
- No puede haber ası́ntotas oblicuas al haber ya horizontales.
(g) La gráfica que resulta del estudio realizado es la siguiente:

PROBLEMA 6.20.

Representar la curva f (x) = ln(x2 − 3x + 2).

Solución

(a) El dominio será el conjunto de puntos solución de la inecuación x2 −


3x + 2 > 0. Como las raı́ces del primer miembro son 2 y 1, la inecuación se
escribe como (x − 1)(x − 2) > 0 y la solución es (−∞, 1) ∪ (2, ∞).

230
(b) Como f (−x) = ln[(−x)2 − 3(−x) + 2] = ln(x2 + 3x + 2), la curva no
es simétrica (además, como el dominio no es simétrico, la curva tampoco
puede serlo).
(c) Si x = 0, f (0) = ln 2. Por tanto, la curva corta al eje Y en el punto
(0, ln 2).
Si y = 0,

2 2 2 3± 5
0 = ln(x − 3x + 2) =⇒ 1 = x − 3x + 2 =⇒ x − 3x + 1 = 0 =⇒ x = .
2
√ √
3+ 5 3− 5
La curva corta al eje X en los puntos ( , 0) y ( , 0).
2 2
(d) Al calcular las dos primeras derivadas de la función resulta:

2x − 3
f 0 (x) = ; la derivada se anula cuando x = 3/2.
x2 − 3x + 2

2(x2 − 3x + 2) − (2x − 3)2 −2x2 + 6x − 5


f 00 (x) = = .
(x2 − 3x + 2)2 (x2 − 3x + 2)2
Como el numerador no tiene raı́ces reales, f 00 (x) 6= 0, ∀x ∈ D(f ).
Estudiando el signo de estas derivadas, podemos escribir la siguiente ta-
bla:

(−∞, 1) (2, ∞)
f 0 (x) –– ++
f 00 (x) –– ––

Por tanto, f crece en (2, ∞) y decrece en (−∞, 1) (recordemos que x = 3/2


no está en el dominio).
Además, f es siempre cóncava hacia abajo.
(e) De la tabla anterior se deduce que no hay máximos ni mı́nimos locales
ası́ como tampoco puntos de inflexión.
(f) - Para obtener las ası́ntotas verticales debemos calcular:

lı́m f (x) = lı́m ln(x2 − 3x + 2) = −∞;


x→1− x→1−

lı́m f (x) = lı́m ln(x2 − 3x + 2) = −∞.


x→2+ x→2+

Por tanto, las rectas x = 1 y x = 2 son ası́ntotas verticales.

231
- Veamos si hay ası́ntotas horizontales:

lı́m f (x) = lı́m ln(x2 − 3x + 2) = ∞.


x→∞ x→∞

lı́m f (x) = lı́m ln(x2 − 3x + 2) = ∞.


x→−∞ x→−∞
No hay ası́ntotas horizontales porque ambos lı́mites son infinitos.
- Veamos si hay ası́ntotas oblicuas:
f (x) ln(x2 − 3x + 2)
m1 = lı́m = lı́m
x→∞ x x→∞ x
ln[x (1 − 3/x + 2/x2 )]
2 2 ln x + ln(1 − 3/x + 2/x2 )
= lı́m = lı́m = 0.
x→∞ x x→∞ x
f (x) 2 ln(−x) + ln(1 − 3/x + 2/x2 )
m2 = lı́m = lı́m = 0.
x→−∞ x x→−∞ x
Esto quiere decir que tampoco hay ası́ntotas oblicuas.
(g) La gráfica que resulta del estudio realizado es la siguiente:

Observa la simetrı́a de la gráfica con respecto a la recta x = 3/2, es decir


que se verifica la identidad f (x + 3/2) = f (−x + 3/2).

D. PROBLEMAS DE MÁXIMOS Y MÍNIMOS.

Para resolver problemas donde se pretendan encontrar valores máximos o


mı́nimos de ciertas cantidades, se procede de la siguiente manera:

232
1) Determinar la cantidad (que llamaremos y) que se quiere hacer máxima
o mı́nima.

2) Construir mediante una fórmula adecuada una función cuya variable de-
pendiente sea la cantidad y y que dependa de otra u otras cantidades (por
ejemplo y = f (x1 , x2 , ...)).

3) Encontrar una relación entre las variables de la que depende la cantidad


original a partir de los datos del problema.

4) Sustituir en la función construida las variables en función de otra para


que resulte una función de una sola variable y determinar los posibles valores
(dominio natural) que puede tomar.

5) Aplicar las técnicas de cálculo (derivadas) para encontrar los valores que
hacen máxima o mı́nima (según el caso) la función resultante.

6) Interpretar los resultados de acuerdo al enunciado original.

PROBLEMA 6.21.

Se construye un canal de desagüe de modo que su sección trans-


versal es un trapecio con lados de igual pendiente y longitud 5 m
(lados y fondo). ¿Cómo elegir el ángulo de inclinación β para que
el área del corte sea máxima? (se supone 0 ≤ β ≤ π/2).

Solución

1) Se trata de maximizar el área de un trapecio.


(B + b)h
2) Escribimos la fórmula del área: A = , la cual depende de tres
2
variables, B, b y h.

233
3) Si definimos las variables que se indican en la figura,

obtenemos las relaciones b = 5; B = 5 + 2x; cos β = x/5; sen β = h/5.


4) Sustituyendo en la fórmula del área, nos queda:
(10 + 2x)h (10 + 10 cos β) · 5 sen β
A= = = 25 sen β(1 + cos β)
2 2
De este modo ya tenemos una función que depende de una sola variable, que
puede tomar valores entre 0 y π/2 (pues se trata de un ángulo agudo).
5) El problema de encontrar el máximo de dicha función se reduce a utilizar
métodos de cálculo y determinar los valores de β que anulen la derivada
primera. Si derivamos la función, tenemos:
A0 = 25 cos β(1 + cos β) + 25 sen β(− sen β)
= 25 cos β + 25 cos2 β − 25(1 − cos2 β) = 25(2 cos2 β + cos β − 1).
Al hacer A0 = 0 y despejar la variable, tenemos:

−1 ± 1 + 8 1
cos β = =⇒ cos β = ó cos β = −1.
4 2
Como 0 ≤ β ≤ π/2, sólo es válido el valor cos β = 1/2, por lo que β =
π/3.
Para comprobar que el área es máxima para este valor, debemos calcular la
derivada segunda y comprobar que A00 (π/3) < 0. En efecto:
√ √ √
A00 = 25[4 cos β(− sen β)−sen β] =⇒ A00 (π/3) = 25(2 3/2− 3/2) = 25 3/2.

6) Como el enunciado del problema pide que se calcule el ángulo, la respuesta


debe ser β = π/3.

PROBLEMA 6.22.

Una ventana tiene forma rectangular con semicı́rculo en la parte


superior. El rectángulo es de vidrio claro y el semicı́rculo de vi-
drio coloreado. El coloreado sólo transmite la mitad de luz por m2
que el claro. Si el perı́metro de la ventana es fijo, determinar las
proporciones de la ventana que admitirá más luz.

234
Solución

En la figura siguiente se especifican las variables que se van a utilizar.

1) El problema trata de maximizar la cantidad de luz.


2) La cantidad de luz será el producto del área de cada sección por la luz
que transmite por unidad de área. Denotaremos por c a la cantidad de luz
por m2 que transmite el vidrio coloreado. De este modo, la cantidad total
de luz viene dada por:
L = 2c · 2ab + cπa2 /2.
3) En este caso, c es una constante pero a y b son variables. Para poder
escribir L como función de una sola variable utilizamos el dato de que el
perı́metro, que llamaremos P , es fijo. Como
P = 2a + 2b + πa =⇒ 2b = P − 2a − πa.
4) Sustituyendo 2b en la ecuación de L, obtenemos:
L = 2ca(P − 2a − πa) + cπa2 /2.
5) Ya podemos manejar las herramientas del cálculo para encontrar el máxi-
mo de la función L con variable a. Derivando en primer lugar resulta:
L0 = 2c(P − 2a − πa) + 2ca(−2 − π) + cπa = 2cP − 8ca − 3πca.
−2cP 2P
Si L0 = 0, entonces a = = .
−8c − 3πc 8 + 3π
Para comprobar que este valor de a da lugar al máximo de la función cal-
cularemos L00 :
L00 = c(−8 − 3π) < 0.
6) De este modo, las dimensiones de la ventana que maximizan la cantidad
de luz son
2P
a = ;
8+ 3π 
1 4P 2P π 8P + 3P π − 4P − 2P π (4 + π)P
b = P− − = = .
2 8 + 3π 8 + 3π 2(8 + 3π) 2(8 + 3π)

235
PROBLEMA 6.23.

La rigidez de una viga rectangular es proporcional al producto de


su anchura por el cubo de su profundidad, pero no está relacionada
con su longitud. Hallar las proporciones de la viga más rı́gida que
puede cortarse de un tronco de diámetro dado.

Solución

1) La sección de la viga cortada de un tronco circular, cuya rigidez queremos


calcular, tiene la forma de la figura:

2) De acuerdo al enunciado, la rigidez de la viga viene expresada por:

R = kap3 donde a (anchura) y p (profundidad) son variables.

3) Sin embargo dichas variables están relacionadas por la fórmula


p
a2 + p2 = (2r)2 =⇒ a = d2 − p2 siendo d el diámetro del tronco.
p
4) Ası́ pues, R = kp3 d2 − p2 y resulta una función con una sola variable
(k y d son constantes).
5) Buscamos los puntos crı́ticos:
p −2p 3kp2 (d2 − p2 ) − kp4 kp2 (3d2 − 4p2 )
R0 = 3kp2 d2 − p2 +kp3 p = p = p .
2 d 2 − p2 d 2 − p2 d 2 − p2

Para que R0 = 0, debe ser p = 0 ó 3d2 − 4p2 =√0. El primer caso no es


posible y el segundo da como resultado que p = d 3/2.
Volviendo a derivar se comprueba que dicho valor de p produce un máxi-
mo.
p
6) Para obtener la anchura, basta sustituir p en la fórmula a = d2 − p2 .
Resulta que a = d/2 = r.

236
PROBLEMA 6.24.

Hallar el perı́metro y área del rectángulo de área máxima que puede


inscribirse entre la parábola y = 9 − x2 y el eje X .

Solución

La situación que se presenta viene representada en la siguiente gráfica:

De este modo, la base del rectángulo mide 2x (debido a la simetrı́a de la


curva dada) y la altura 9 − x2 (ya que los puntos superiores del rectángulo
deben estar sobre la parábola). El área será entonces:

A = 2x(9 − x2 )

la cual ya viene expresada como función de una sola variable.


Derivando e igualando a cero la derivada, tenemos que:

A0 = 18 − 6x2 y A0 = 0 ⇐⇒ x = ± 3.
√ √
Como A00 = −12x y A00 ( 3) < 0, el punto x = 3 corresponde
√ a un máximo.
Sustituyendo en la fórmula del área, resulta A = 12 3.
Como también se pide el perı́metro del rectángulo
√ y este vale P = 4x+2(9−
x2 ), al sustituir nos queda que P = 12 + 4 3.

PROBLEMA 6.25.

Calcular las dimensiones del cono inscrito en una esfera de radio


R para que el volumen sea máximo.

237
Solución

1
La fórmula del volumen del cono es V = πr2 h, donde r es el radio de la
3
base y h la altura del cono.

Ahora bien, observando en la figura el triángulo rectángulo OM B de di-


mensiones OM = h − R, OB = R, M B = r, por el teorema de Pitágoras
R2 = (h − R)2 + r2 , de donde r2 = R2 − (h − R)2 .
Sustituyendo en la fórmula del volumen tenemos:
1 1
V = π[R2 − (h − R)2 ]h = π(−h3 + 2Rh2 ).
3 3
Derivando,
1 1
V 0 = π(−3h2 + 4Rh) = πh(−3h + 4R).
3 3
Para encontrar los extremos hacemos V 0 = 0 lo que ocurre cuando h = 0
ó h = 4R/3. El primer valor no tiene sentido en este contexto; el segundo lo
sustituimos en la derivada segunda. Tenemos:
1 1
V 00 = π(−6h + 4R) =⇒ V 00 (4R/3) = π(−8R + 4R) < 0
3 3
con lo que el valor h = 4R/3 corresponde a un máximo.
p √
El radio del cono será r = R2 − (h − R)2 = 2 2R/3.

PROBLEMA 6.26.

Las caras laterales de una pirámide triangular regular se abaten


sobre el plano de la base y hacia el exterior. La circunferencia que
pasa por los tres abatimientos del vértice superior tiene de radio
10 cm. Calcular la longitud del lado de la base para que el volumen
de la pirámide sea máximo.

238
Solución

Se trata de maximizar la función volumen de la pirámide cuya fórmula es


1
V = SB h, donde SB es el área de la base y h es la altura de la pirámi-
3
de.

Sea r el radio de la circunferencia inscrita en el triángulo de√la base y l


el lado del mismo. Como el √ triángulo
√ es equilátero, SB = l2 3/4. Como
√ (2 3r)2 3 √
además l = 2 3r, SB = = 3 3r2 .
4
La altura de la pirámide es un cateto de un triángulo rectángulo, cuyo otro
cateto es r y la hipotenusa 10 − r. Tenemos ası́:
p √
h= (10 − r)2 − r2 = 100 − 20r.

Sustituyendo en la fórmula del volumen, resulta:

1 √ 2√ √ √ p
V = · 3 3r 100 − 20r = 3 · 100 − 20r · r2 = 300r4 − 60r5
3

Derivamos para calcular los extremos:

1200r3 − 300r4
V0 = √ .
2 300r4 − 60r5

Ası́ V 0 = 0 ⇐⇒ 1200r3 = 300r2 ⇐⇒ r = 4 ó r = 0.

Como r = 4 hace que V 00 < 0, corresponde a un máximo.



Las dimensiones buscadas son pues r = 4, l = 8 3.

239
PROBLEMA 6.27.

Hallar la posición más favorable para meter gol desde un punto de


la banda lateral de un campo de fútbol, siendo a y b las distancias
respectivas desde el córner a los postes de la porterı́a.

Solución

La posición más favorable será aquella desde la cual se vea la porterı́a bajo
mayor ángulo.
Sea x la distancia del córner C1 a la posición P que suponemos más favorable.
De acuerdo a la gráfica adjunta, tenemos que tg α = a/x y tg β = b/x. Como
b/x − a/x (b − a)x
γ = β − α, tendremos tg γ = tg(β − α) = 2
= .
1 + ab/x ab + x2
(b − a)x
Debemos maximizar la función y = tg γ = . Derivando:
ab + x2
(b − a)(ab + x2 ) − 2x2 (b − a) (b − a)(ab − x2 )
y0 = 2 2
= .
(x + ab) (x2 + ab)2

Al igualar a cero resulta x2 = ab =⇒ x = ± ab.

Además, la derivada segunda es y 00 ( ab)
√ < 0 por lo que la distancia al córner
sobre la banda más favorable es x = ab.

240
PROBLEMA 6.28.

Hallar la altura del cilindro circular recto, de volumen máximo que


se puede inscribir en una esfera de radio R.

Solución

La función a maximizar es el volumen del cilindro, de ecuación V = πr2 h.


La relación entre el radio y la altura viene dada por la fórmula

r2 + (h/2)2 = R2 =⇒ r2 = R2 − h2 /4,

que al sustituir en la fórmula del volumen resulta V = πh(R2 − h2 /4).


Derivando con respecto a h,

V 0 = π[(R2 − h2 /4) + h(−h/2)] = π[R2 − 3h2 /4].



V 0 = 0 ⇐⇒ R2 = 3h2 /4 ⇐⇒ h = ± 4R2 /3 = ±2R/ 3.
p


Nótese que V 00 = −(3/4)π·2h y V 00 (2R/
√ 3) < 0 lo que indica que el volumen
máximo se alcanza cuando h = 2R/ 3.

E. TEOREMAS DEL VALOR MEDIO. REGLA DE L’HÔPITAL.

Tres propiedades fundamentales de las funciones derivables corresponden a


los siguientes teoremas del valor medio:

a) Teorema de Rolle.
Si una función y = f (x) verifica las siguientes hipótesis:
a) f es continua en un intervalo cerrado [a, b];
b) f es derivable en el intervalo abierto (a, b);
c) f (a) = f (b);

241
entonces se puede asegurar la existencia de otro punto c comprendido
entre a y b para el cual f 0 (c) = 0.

El teorema de Rolle se puede interpretar diciendo que entre dos puntos


donde una función continua y derivable toma el mismo valor, debe haber
otro punto donde la derivada se anule. Similar al teorema anterior aunque
más general es el siguiente:

b) Teorema del valor medio de Lagrange.

Si una función y = f (x) verifica las siguientes hipótesis:

a) f es continua en un intervalo cerrado [a, b];

b) f es derivable en el intervalo abierto (a, b);

entonces se puede asegurar la existencia de otro punto c comprendido


f (b) − f (a)
entre a y b para el cual f 0 (c) = .
b−a
La última igualdad indica que las pendientes de la recta tangente a y = f (x)
por el punto x = c y de la recta secante que pasa por (a, f (a)) y (b, f (b))
son iguales, es decir, que tales rectas son paralelas.

c) Teorema del valor medio de Cauchy.

Dadas dos funciones y = f (x), y = g(x) continuas en un intervalo


cerrado [a, b] y derivables en el intervalo abierto (a, b), entonces existe
f 0 (c) f (b) − f (a)
un punto c ∈ (a, b) tal que 0 = .
g (c) g(b) − g(a)

Consecuencia de este teorema es la regla de L’Hôpital para resolver lı́mites


indeterminados utilizando derivadas. Los casos en que se puede aplicar son
los siguientes:

REGLA 1 ( CASO 0/0):


f 0 (x) f (x)
Si lı́m f (x) = lı́m g(x) = 0 y lı́m 0
= L, entonces lı́m = L.
x→a x→a x→a g (x) x→a g(x)

REGLA 2 (CASO ∞/∞):


f 0 (x) f (x)
Si lı́m f (x) = lı́m g(x) = ∞ y lı́m 0
= L, entonces lı́m = L.
x→a x→a x→a g (x) x→a g(x)

OBSERVACIONES:

1) En ambos casos a puede tomar cualquier valor infinito.

242
2) Las indeterminaciones 0/0 e ∞/∞ son los únicos casos en que es posi-
ble aplicar la regla de L’Hôpital. Cualquier otra indeterminación se debe
previamente transformar en una de estas dos.
3) Esta regla es apropiada cuando el lı́mite del cociente de las derivadas es
más sencillo que el original. En caso contrario, se debe aplicar alguna otra
técnica como las ya estudiadas.
4) La regla se puede aplicar sucesivas veces mientras se mantenga la indeter-
minación y se puedan derivar las funciones, es decir, podemos derivar tantas
veces f y g como sea posible y hasta que alguna de estas derivadas tenga
lı́mite distinto de cero o de infinito.

PROBLEMA 6.29.

¿Se puede aplicar el teorema de Rolle a la función f (x) = −3 +


3(x − 1)2/3 en el intervalo [0, 2]?

Solución

Veamos si se cumplen todas las hipótesis del teorema:


a) f está definida para cualquier valor real; es continua en todo R.
b) Como f 0 (x) = 2(x − 1)−1/3 , no está definida para x = 1; no es derivable
en el intervalo (0, 2).
No se cumplen las condiciones del teorema de Rolle. No se puede asegurar
la existencia de puntos en el intervalo (0, 2) que anulen la derivada de la
función (aunque puede haberlos).

PROBLEMA 6.30.

Probar que f (x) = 6x4 − 7x + 1 no tiene más de dos raı́ces reales.

Solución

Supongamos que la función dada sı́ tiene más de dos raı́ces reales (lo cual
es posible pues se trata de un polinomio de grado 4). Es decir, que existen

243
por lo menos tres puntos a, b, c para los que se cumple que f (a) = f (b) =
f (c) = 0.

Veamos si se cumplen las condiciones del teorema de Rolle en los intervalos


[a, b] y [b, c]:

Como es un polinomio, se trata de una función continua en todo R.

Además es derivable y su derivada es f 0 (x) = 24x3 − 7.

Como se cumplen todas las condiciones del teorema, deben existir dos va-
lores, r y s, uno en cada intervalo donde se verifique que f 0 (r) = f 0 (s) =
0.

Sin embargo,
p para que f 0 (x) = 0 debe ocurrir que x3 = 7/24, es decir,
x = 3 7/24 y este valor es único.

La suposición inicial de que puede haber más de dos raı́ces reales es falsa y
queda probado lo que se pedı́a en el ejercicio.

PROBLEMA 6.31.


Averiguar si la función f (x) = 3 x − 4x cumple las hipótesis del
teorema del valor medio en el intervalo [1, 4]. Caso de que sea ası́,
encontrar los valores de c que verifican el teorema.

Solución

f es continua en el intervalo (está definida para todos los valores x ≥


0).

Como f 0 (x) = −4 + 3/2 x, f es derivable en el intervalo (1, 4).

Se verifican todas las condiciones del teorema. Para encontrar los valores de√
c, plantearemos la ecuación correspondiente. Por un lado, f 0 (c) = −4 + 3/2 c,
f (4) − f (1) −10 + 1
y por otro, = = −3.
4−1 3
√ √
Al resolver la ecuación −4 + 3/2 c = −3, resulta 3/2 c = 1 =⇒ c = 9/4.

244
PROBLEMA 6.32.

Resolver el mismo problema anterior para la función


x
f (x) = en [−1, 1].
x−1

Solución

f no está definida en el punto x = 1 pues se anula el denominador. Esto


quiere decir que no es continua en todo el intervalo [−1, 1]. No se puede
aplicar el teorema del valor medio.

PROBLEMA 6.33.

Un automovilista recorre 80 Km en una hora. Probar que la velo-


cidad del automóvil fue de 80 Km/h por lo menos una vez durante
el trayecto.

Solución

Sea f (t) la función que indica la distancia recorrida por el móvil al cabo del
tiempo t (medido en horas). Suponemos que f es una función continua y
derivable (lo cual quiere decir que el movimiento no ha sufrido frenadas brus-
cas ni arrancadas instantáneas). Por el teorema del valor medio, debe existir
algún momento t0 comprendido entre 0 y 1 para el cual se verifique:
f (1) − f (0)
f 0 (t0 ) = = 80.
1−0
Como la derivada de la distancia es la velocidad, hemos comprobado que
esta es de 80 Km/h en el momento t0 , por lo menos.

PROBLEMA 6.34.

Comprobar el teorema del valor medio para la función f (x) = 2x2 −


7x + 10, en el intervalo [2, 5].

245
Solución

Según el teorema del valor medio, si f es una función continua en [a, b] y


f (b) − f (a)
derivable en (a, b), entonces existe c ∈ (a, b) para el que f 0 (c) = .
b−a
Como en nuestro caso se trata de una función polinómica, es claro que es
continua y derivable en todo R y como f 0 (x) = 4x−7, f (2) = 4, f (5) = 25, el
25 − 4
teorema del valor medio dice que existe c ∈ (2, 5) tal que 4c − 7 = =7
5−2
de modo que c = 3, 5 que evidentemente pertenece al intervalo (2, 5).

PROBLEMA 6.35.

xn − 1
Resolver lı́m .
x→1 x − 1

Solución

Tenemos una indeterminación del tipo 0/0. Si derivamos numerador y de-


nominador, el lı́mite queda de la forma:

xn − 1 nxn−1
lı́m = lı́m = n.
x→1 x − 1 x→1 1

PROBLEMA 6.36.

sen x − x cos x
Calcular lı́m .
x→0 x(1 − cos x)

Solución

Utilizamos la equivalencia 1 − cos x ∼ x2 /2 y la regla de L’Hôpital:


sen x − x cos x sen x − x cos x
L = lı́m 2
= lı́m
x→0 x · x /2 x→0 x3 /2
cos x − (cos x − x sen x) x sen x sen x
= 2 lı́m = 2 lı́m = (2/3) lı́m = 2/3.
x→0 3x2 x→0 3x2 x→0 x

246
PROBLEMA 6.37.
 
1 1
Resolver lı́m − .
x→0 sen x x

Solución

La indeterminación es ahora de la forma ∞ − ∞. Podemos hacer denomina-


dor común y convertir la expresión en un cociente:
 
1 1 x − sen x
L = lı́m − = lı́m .
x→0 sen x x x→0 x sen x

Ahora el lı́mite es de la forma 0/0 y podemos aplicar la regla de L’Hôpital:

x − sen x 1 − cos x
L = lı́m = lı́m .
x→0 x sen x x→0 sen x + x cos x

Nuevamente el lı́mite presenta la indeterminación 0/0. Volvemos a aplicar


la regla:
sen x 0
L = lı́m = = 0.
x→0 cos x − x sen x + cos x 2

PROBLEMA 6.38.
  
2 1
Calcular lı́m x − x ln 1 + .
x→∞ x

Solución

Si hacemos el cambio x = 1/z y aplicamos la regla de L’Hôpital, tene-


mos:
1
1 − 1+z
 
1 1 z − ln(1 + z)
L = lı́m − 2 ln(1 + z) = lı́m = lı́m
z→0+ z z z→0+ z2 z→0+ 2z
z 1 1
= lı́m = lı́m = .
z→0+ 2z(1 + z) z→0+ 2(1 + z) 2

247
PROBLEMA 6.39.


x
Calcular lı́m ch x.
x→0

Solución

Como el lı́mite es de la forma 1∞ , tomamos logaritmos y aplicamos la regla


de L’Hôpital:
1 ln ch x sh x/ ch x
ln L = lı́m ln ch x = lı́m = lı́m = 0.
x→0 x x→0 x x→0 1
En definitiva, L = e0 = 1.

PROBLEMA 6.40.

Calcular lı́m sen x · e1/(1−cos x) .


x→0

Solución

Utilizando la equivalencia de los infinitésimos sen x ∼ x y aplicando la regla


de L’Hôpital, tenemos:
− sen x
e1/(1−cos x) (1−cos x)2
· e1/(1−cos x)
L = lı́m = lı́m
x→0 1/x x→0 −1/x2
x2 sen x
= lı́m e1/(1−cos x) · .
x→0 (1 − cos x)2
Aplicando ahora las equivalencias sen x ∼ x y 1 − cos x ∼ x2 /2, tene-
mos
4e1/(1−cos x)
L = lı́m = ∞.
x→0 x

PROBLEMA 6.41.
 
sen x 1 ch x
Calcular lı́m x − .
x→0+ x2 x sh x

248
Solución

Calcularemos por separado los lı́mites de cada uno de los factores.


Si llamamos A = lı́m xsen x ,
x→0+

ln x 1/x
ln A = lı́m sen x ln x = lı́m = lı́m
x→0+ x→0+ 1/ sen x x→0+ − cos x/ sen2 x
− sen2 x −x2 −x
= lı́m = lı́m = lı́m = 0.
x→0 + x cos x x→0 x cos x
+ x→0 cos x
+

 
1 ch x sh x − x ch x ch x − (ch x + x sh x)
B = lı́m 2
− = lı́m 2
= lı́m
x→0+ x x sh x x→0+ x sh x x→0+ 2x sh x + x2 ch x
−x sh x − sh x − ch x
= lı́m = lı́m = lı́m
x→0+ x(2 sh x + x ch x) x→0+ 2 sh x + x ch x x→0+ 2 ch x + ch x + x sh x
− ch x 1
= lı́m =− .
x→0 3 ch x + x sh x
+ 3
En definitiva, L = A · B = −1/3.

PROBLEMA 6.42.

ap − 1
Calcular lı́m .
p→0 p

Solución

Si aplicamos la regla de L’Hôpital, tenemos:


ap ln a
L = lı́m = ln a.
p→0 1

PROBLEMA 6.43.

Calcular lı́m xx .
x→0

249
Solución

Tomando logaritmos, resulta:

ln x 1/x −x2
ln L = lı́m x ln x = lı́m = lı́m 2
= lı́m = 0.
x→0 x→0 1/x x→0 −1/x x→0 x

Se obtiene entonces que L = e0 = 1.

PROBLEMA 6.44.

x
Calcular lı́m xx .
x→0

Solución

Teniendo en cuenta el resultado del problema anterior, tenemos:


lı́m xx ln x
L = ex→0 = e1·(−∞) = 0.

PROBLEMA 6.45.

 tg x
1
Calcular lı́m .
x→0+ x

Solución

Procederemos como en casos anteriores:


ln(1/x) (1/x)−1 · (−1/x2 )
ln L = lı́m tg x ln(1/x) = lı́m = lı́m
x→0+ x→0+ cotg x x→0+ −1/ sen2 x
−1/x sen x
= lı́m 2
= lı́m · sen x = 0,
x→0+ −1/ sen x x→0+ x

de donde L = 1.

250
F. EJERCICIOS PROPUESTOS.

1.- Encontrar los intervalos de crecimiento y decrecimiento de las


funciones:

(a) f (x) = x3 /3 + 3x2 /2 − 10x.

Resp.: Crece en (−∞, −5) y en (2, ∞); decrece en (−5, 2).

(b) f (x) = x3 + x + 4/x.

Resp.: Crece en (−∞, −1) ∪ (1, ∞); decrece en (−1, 0) ∪ (0, 1).

x
(c) f (x) = √ .
x2+1
Resp.: Crece en (−∞, ∞).

2.- Encontrar los máximos y mı́nimos locales de las funciones:

(a) f (x) = x5 − 5x.

Resp.: Máximo local: (−1, 4); mı́nimo local: (1, −4).

(b) f (x) = x + 1/x.

Resp.: Máximo local: (−1, −2); mı́nimo local: (1, 2).

[Observa que el máximo está por debajo del mı́nimo. Esto no es extraño
porque la función es discontinua en x = 0 y en las proximidades de
este punto la función tiende a ±∞ ].

(c) f (x) = |4 − x2 |.

Resp.: Máximo local: (0, 4); mı́nimos locales: (−2, 0) y (2, 0).

3.- Sea y = f (x) una función definida en [a, h] cuya derivada tiene la
siguiente gráfica:

251
y = f 0 (x)

(a) ¿Dónde es f creciente?


(b) ¿Dónde tiene f un máximo local?
(c) ¿Cuáles son los puntos de inflexión?
(d) Representar aproximadamente la función f .
Resp.: f es creciente en (b, d).
f tiene un máximo local en x = d.
f tiene puntos de inflexión en x = c y en x = e.

4.- Estudiar la concavidad de las funciones:

x3
(a) f (x) = .
x2 − 4
Resp.: Cóncava hacia arriba: (−2, 0), (2, ∞); cóncava hacia abajo:
(−∞, −2), (0, 2).

(b) f (x) = (5 − x)5/3 + 2.


Resp.: Cóncava hacia arriba: (−∞, 5); cóncava hacia abajo: (5, ∞).

252
5.- Estudiar el crecimiento y la concavidad de las siguientes funcio-
nes:


x
(a) f (x) = .
1 − x2

Resp.: Crece en (−∞, −1) ∪ (0, 1) y es siempre cóncava hacia arriba.


x− x2 + 1
(b) f (x) = .
x
Resp.: Es siempre creciente y es cóncava hacia arriba en (−∞, 0).

(c) f (x) = 2 cos x + cos2 x en el intervalo [0, 2π].

Resp.: Es creciente en (π, 2π) y cóncava hacia arriba en (π/3, 5π/3).

6.- Dada la curva y = ax3 + bx2 + cx, encontrar el valor de las cons-
tantes a, b y c para que f tenga un punto de inflexión en (1, 2)
y además la pendiente de la recta tangente en ese mismo punto
sea −2.

Resp.: Resolver el sistema a + b + c = 2 6a + 2b = 0 3a + 2b + c = −2

Se obtienen los valores a = 4, b = −12, c = 10.

7.- Representar aproximadamente una función cuya derivada es la


que se tiene a continuación:

Resp.: Tener en cuenta que f crece en (−3, 4/5) y decrece en (4/5, 7).

También se observa en la gráfica que f 0 (x) = 0 cuando x = −3, x = 4/5


y x = 7.

253
8.- Dibujar la gráfica de una función y = f (x) que cumpla las condi-
ciones siguientes:
(a) f es continua y derivable en (−∞, 0) ∪ (0, ∞).
(b) lı́m f (x) = 0.
x→−∞

(c) Si −∞ < x < 0 : f (x) < 0, f 0 (x) < 0, f 00 (x) < 0.


(d) lı́m f (x) = −∞.
x→0

(e) Si 0 < x < 1 : f (x) < 0, f 0 (x) > 0, f 00 (x) < 0.


(f) f (1) = 0.
(g) Si 1 < x < 2 : f (x) > 0, f 0 (x) > 0, f 00 (x) < 0.
(h) f (2) = 1.
(i) Si 2 < x < 3 : f (x) > 0, f 0 (x) < 0, f 00 (x) < 0.
(j) f (3) = 1/2.
(k) Si 3 < x < ∞ : f (x) > 0, f 0 (x) < 0, f 00 (x) > 0.
(l) lı́m f (x) = 0.
x→∞

Resp. Tabla de signos:

(−∞, 0) (0, 1) (1, 2) (2, 3) (3, ∞)


f 0 (x) –– ++ ++ –– ––
f 00 (x) –– –– –– –– ++

254
Gráfica:

9.- Dibujar la gráfica de una función y = f (x) que cumpla las condi-
ciones siguientes:
(a) D(f ) = [−2, 4) ∪ (4, ∞).
(b) f (−2) = −3; f es continua en su dominio.
(c) f 0 (x) < 0 si x ∈ (2, 3) ∪ (4, 6); f 0 (x) > 0 si x ∈ [−2, 2) ∪ (3, 4) ∪
(6, ∞); f 0 (2) = f 0 (6) = 0; f 0 (3) no existe.
(d) f 00 (x) < 0 si x ∈ (0, 3); f 00 (x) > 0 si x ∈ (−2, 0) ∪ (3, 4) ∪ (4, ∞);
f 00 (0) = 0.
(e) La recta x = 4 es ası́ntota.
Resp. La tabla de signos es la siguiente:

(−2, 0) (0, 2) (2, 3) (3, 4) (4, 6) (6, ∞)


f 0 (x) ++ ++ –– ++ –– ++
f 00 (x) ++ –– –– ++ ++ ++

Un posible resultado es el indicado en la siguiente gráfica:

255
10.- Dibujar la gráfica de una función y = f (x) que cumpla las con-
diciones siguientes:
(a) Es continua en (−∞, 3) ∪ (3, ∞).
(b) f (−2) = 8, f (0) = 4, f (2) = −1.
(c) f 0 (x) > 0 si |x| > 2; f 0 (x) < 0 si |x| < 2; f 0 (2) = 0; f 0 (−2) no
existe.
(d) f 00 (x) < 0 si −2 < x < 0; f 00 (x) > 0 si x > 0.
(e) lı́m f (x) = −4; x = 3 es ası́ntota vertical.
x→−∞

Resp. Tabla de signos:

(−∞, −2) (−2, 0) (0, 2) (2, 3) (3, ∞)


f 0 (x) ++ –– –– ++ ++
f 00 (x) –– ++ ++ ++

Gráfica:

11.- Dibujar la gráfica de una función y = f (x) que cumpla las con-
diciones siguientes:
(a) D(f ) = (−∞, −1) ∪ (−1, ∞).
(b) Es continua y derivable en D(f ).
(c) El único punto de intersección con los ejes es (0, 0).
(d) No hay simetrı́a con el eje Y .
(e) f 0 (x) > 0 en (−∞, −2) ∪ (0, ∞).

256
(f) Tiene un mı́nimo local en (0, 0) y un máximo local en (−2, −4).

(g) La recta y = x − 1 es ası́ntota.

(h) lı́m f (x) = +∞, lı́m f (x) = −∞.


x→−1+ x→−1−

Resp. Algunos datos se resumen en la tabla:

(−∞, −2) (−2, −1) (−1, 0) (0, ∞)


f 0 (x) ++ –– –– ++

La gráfica buscada puede ser como la siguiente:

12.- Representar la gráfica de las funciones siguientes:

(a) f (x) = (x + 1)3 (x − 1).

Resp.: La tabla de signos es la siguiente:

(−∞, −1) (−1, 0) (0, 1/2) (1/2, ∞)


f 0 (x) –– –– –– ++
f 00 (x) ++ –– ++ ++

y la gráfica es la que se ilustra a continuación:

257
√ √
(b) f (x) = 8+x− 8 − x.

Resp.: Como el dominio es [−8, 8], no tiene sentido calcular ası́ntotas.


La tabla de signos es la siguiente:

(−4, 0) (0, 4)
f 0 (x) ++ ++
f 00 (x) –– ++

La gráfica es la siguiente:

r
x
(c) f (x) = .
4−x

Resp.: Tabla de signos:

258
(0, 1) (1, 4)
f 0 (x) ++ ++
f 00 (x) –– ++

Gráfica:

(d) f (x) = x3 + 3/x.

Resp.: Tabla de signos:

(−∞, −1) (−1, 0) (0, 1) (1, ∞)


f 0 (x) ++ –– –– ++
f 00 (x) –– –– ++ ++

Gráfica:

2x
(e) f (x) = √ .
4x2 − 4
Sugerencia: La función es impar; basta estudiarla en el intervalo (0, ∞)
y dibujarla de acuerdo a la simetrı́a.

259
Resp. Tabla de signos:

(−∞, −1) (1, ∞)


f 0 (x) –– ––
f 00 (x) –– ++

Gráfica:

(x − 2)3
(f) f (x) = .
x2
Resp. Tabla de signos:

(−∞, −4) (−4, 0) (0, 2) (2, ∞)


f 0 (x) ++ –– ++ ++
f 00 (x) –– –– –– ++

Gráfica:

13.- Se construye un depósito como el de la figura con una pieza de


metal de 3 m. de ancho y 20 m. de longitud. Expresar el volumen
en términos de β y hallar el máximo volumen posible.

260
Sugerencia: La figura es un prisma recto cuya base es un trapecio. El
volumen es igual al área de la base por la altura.

Resp.: V = 20(1 + cos β) sen β; Vmáx = 15 3.

14.- Se construye un tanque de gas formado por un cilindro y dos


semiesferas. El costo por m2 de las semiesferas es doble a la parte
cilı́ndrica. Si la capacidad es 10π , ¿cuáles son las dimensiones
que minimizan el costo?


3

Resp.: r = 15/2, h = 30/ 3 225.

15.- Dada una esfera de radio R, encontrar las dimensiones del cono
circular recto de volumen mı́nimo que contiene a la esfera.
Sugerencia: La generatriz del cono es tangente a la esfera, por lo tanto
perpendicular al radio en el punto de corte. Establecer semejanza de
triángulos rectángulos.

Resp.: h = 4R, r = R 2.

16.- Un pescador se encuentra en un bote de remos a 2 Km de la


costa. Desea llegar a un punto 6 Km más allá del primero y
se sabe que puede remar a 3 Km/h y caminar a 5 Km/h. ¿A
qué punto de la playa debe llegar si el tiempo de la trayectoria
debe ser mı́nimo?
Sugerencia: El tiempo de trayectoria es la suma del tiempo que pa-
sa remando y el tiempo que tarda caminando. En ambos casos, t =
dist/veloc. (se supone movimiento uniforme).

261
Resp.: Debe remar hasta un punto situado 1,5 Km del punto más
próximo en la costa.

17.- Una recta variable pasa por (1, 2) y corta al eje X en A(a, 0) y
al eje Y en B(0, b). Hallar el área del triángulo AOB de menor
área si a, b > 0.
Resp.: Area(mı́n) = 4.


18.- Hallar las coordenadas del punto sobre y = x más cercano al
(4, 0).
Sugerencia: En vez de minimizar la distancia, es más cómodo minimi-
zar el cuadrado de la distancia. La variable independiente no cambia.
p
Resp.: (7/2, 7/2).

19.- Con una pieza de 48 cm de ancho se quiere construir un ca-


nal para lo cual se doblan los extremos formando un triángulo
isósceles. ¿Qué ángulo deben formar sus lados para que el canal
contenga la mayor cantidad de lı́quido?
Resp.: El ángulo debe ser de 90 grados.

20.- Hallar las dimensiones de un triángulo rectángulo de hipotenusa


dada que engendre un cono de volumen máximo al girar alrededor
de uno de sus catetos.

Resp.: Si la√hipotenusa mide a, la altura del cono es h = a/ 3 y el
radio r = a 2/3.

21.- La trayectoria de un proyectil lanzado desde Tierra tiene por


ecuación

x = 30 cos(π/4) · t; y = 30 sen(π/4) · t − gt2 /2.

(a) Encontrar la ecuación cartesiana.


(b) Calcular la altura máxima.
(c) Calcular el tiempo que tarda en alcanzar la altura máxima.
Resp.: (a) y = x−gx2 /900. Corresponde a la ecuación de una parábola.
(b) ymáx = 225/g.

(c) t = 30/g 2.

262
22.- Si f (x) = |2x − 1| − 3, al calcular su derivada se observa que esta
nunca se anula. Además f (2) = f (−1) = 0. ¿Contradice esto el
teorema de Rolle?
Resp.: No, porque en el punto x = 1/2 que pertenece al intervalo
(−1, 2) la función no es derivable. Como falla una de las hipótesis, no
se puede aplicar el teorema.

23.- Si la gráfica de un polinomio tiene tres intersecciones con el eje


X , probar:
a) que hay al menos dos puntos donde la tangente es horizontal.
b) que al menos en un punto se anula f 00 .
Resp.: Basta aplicar el teorema de Rolle en los intervalos correspon-
dientes a puntos consecutivos donde las funciones f y f 0 se anulan.

24.- Probar que la función f (x) = 2x3 − 3x2 − 12x − 6 sólo tiene una
raı́z en el intervalo [−1, 0].
Resp.: La función cambia de signo en el intervalo y la derivada no se
anula en ningún punto del mismo.

25.- Probar que la ecuación x2 = x sen x + cos x tiene exactamente dos


soluciones.
Sugerencia: Estudiar el crecimiento de la función f (x) = x2 − x sen x −
cos x y encontrar sus máximos y mı́nimos.
Otro método: Suponer que tiene más de dos raı́ces y utilizar el teorema
de Rolle en los dos intervalos que dichas raı́ces determinan. ¿Qué pa-
sarı́a si sólo tuviera una raı́z?
(
x3 + 2x + 2 si x < 0,
26.- Sea f (x) = 2
Probar que f (x) = 0 sólo
x − 3x + 2 si x ≥ 0.
tiene una raı́z en el intervalo (−1, 1).
Sugerencia: Comprobar que la función es creciente y cambia de signo
en los extremos del intervalo (−1, 0) y que no tiene raı́ces en el intervalo
(0, 1).

27.- Averiguar si alguna de las siguientes funciones cumple las hipóte-


sis del teorema del valor medio. En caso afirmativo, encontrar
los valores de c que verifican el teorema:

263

a) f (x) = x − 1 en [1, 3].
Resp.: Sı́; c = 3/2.

b) f (x) = −3 + 3(x − 1)2/3 en [0, 2].


Resp.: No.

(
3−x2
2 si x ≤ 1,
c) f (x) = en [0, 2].
1/x si x > 1

Resp.: Sı́; c1 = 1/2, c2 = 2.

28.- Una persona que recorrió 202 Km en dos horas aseguró que nun-
ca excedió el lı́mite de velocidad de 100 Km/h. Usar el teorema
del valor medio para demostrar que mintió.

29.- Demostrar que si f es continua en c y existe lı́m f 0 (x), entonces


x→c
existe f 0 (c) y es igual a lı́m f 0 (x).
x→c

Sugerencia: Aplicar el teorema del valor medio a f en los intervalos


[c, c + h] y [c − h, c].

30.- Probar que sen x ≤ x para todo x en [0, ∞).


Sugerencia: Aplicar el teorema del valor medio a f (x) = sen x − x en
el intervalo [0, x].

31.- Demostrar que | sen x − sen y| ≤ |x − y|.


Sugerencia: Aplicar el teorema del valor medio a f (x) = sen x en cual-
quier intervalo [x, y].

32.- Sea f una función continua en [a, b] con derivada segunda f 00 en


(a, b) y tal que el segmento que une los puntos (a, f (a)) y (b, f (b))
corta a la curva en (c, f (c)) donde a < c < b. Probar que
f 00 (t) = 0 para algún t en (a, b).
Sugerencia: Probar que existen c1 , c2 ∈ (a, b) tales que f 0 (c1 ) = f 0 (c2 )
aplicando elteorema del valor medio a f en los intervalos [a, c] y [c, b].

264
x + sen πx
33.- Resolver lı́m .
x→0 x − sen πx
1+π
Resp.: .
1−π

 
x π
34.- Resolver lı́m − .
x→π/2 cotg x 2 cos x
Resp.: −1.

sen 4x · sen 3x
35.- Resolver lı́m .
x→π/2 x sen 2x
Resp.: 4/π.

tg πx
36.- Resolver lı́m .
x→−2 x+2
Resp.: π.

x − sen x
37.- Resolver lı́m .
x→0 x − tg x

Resp.: −1/2.

2x
38.- Resolver lı́m √ .
x→0+ x + 1/ x
Resp.: 0.

39.- Resolver lı́m (sec x − tg x).


x→π/2

Resp.: 0.

ex − 1 − x
40.- Resolver lı́m .
x→0 sen2 x
Resp.: 1/2.

 
x+a
41.- Resolver lı́m x ln .
x→∞ x−a
Resp.: 2a.

265
xn − an
42.- Resolver lı́m .
x→a ln xn − ln an

Resp.: an .

x2
43.- Resolver lı́m .
x→∞ x − sen x

Resp.: ∞.

 
1 1
44.- Resolver lı́m − .
x→1 ln x x − 1
Resp.: 1/2.

 x
2
45.- Resolver lı́m arc tg x .
x→∞ π
Resp.: e−2/π .

xln x
46.- Resolver lı́m .
x→∞ (ln x)x

Resp.: 0.
Sugerencia: Calcular ln L y hacer el cambio de variable ln x = t.

47.- Resolver lı́m (tg x)tg 2x .


x→π/4

Resp.: -1.

266
CAPÍTULO VII.
INTEGRACIÓN
INDEFINIDA

SECCIONES
A. Integrales inmediatas.
B. Integración por sustitución.
C. Integración por partes.
D. Integración por fracciones simples.
E. Aplicaciones de la integral indefinida.
F. Ejercicios propuestos.

267
A. INTEGRALES INMEDIATAS.

Se dice que una función y = F (x) es integral indefinida (también llamada


primitiva o antiderivada) de otra función y = f (x) cuando F 0 (x) = f (x). La
notación usual para representar este hecho es la siguiente:
Z
F (x) = f (x)dx.

El término ”dx”indica que la variable respecto a la cual se está integrando


es ”x”.
Para calcular integrales se deben encontrar funciones cuya derivada sea la
función original. Se tratará entonces de aplicar las reglas de derivación en
sentido inverso, donde conocidas las derivadas de las funciones, se encuentren
las propias funciones.
Una diferencia fundamental consiste en que mientras cada función sólo tiene
una derivada, tiene infinitas integrales, porque si F 0 (x) = f (x), entonces
[F (x) + C]0 = f (x) para cualquier constante C.
R
Esto se indicará escribiendo f (x)dx = F (x) + C. De este modo, todas las
primitivas de una función se obtienen sumando una constante arbitraria a
una primitiva particular. Las siguientes propiedades permitirán descompo-

ner integrales en otras más sencillas:


i) f 0 (x)dx = f (x) + C.
R
R R R
ii) [f (x) ± g(x)]dx = f (x)dx ± g(x)dx.
R R
iii) kf (x)dx = k f (x)dx, k ∈ R.

De las fórmulas de derivación se obtiene la siguiente tabla de integrales


inmediatas, sin más que cambiar el orden de las fórmulas.
xn+1
xn dx =
R
1) + C si n 6= −1.
n+1
R
2) sen xdx = − cos x + C.
R
3) cos xdx = sen x + C.
sec2 xdx = tg x + C.
R
4)
R
5) sec x tg xdx = sec x + C.
R
6) cosec x cotg xdx = − cosec x + C.
cosec2 xdx = − cotg x + C.
R
7)

268
Z
1
8) √ dx = arc sen x + C.
1 − x2
Z
1
9) dx = arc tg x + C.
1 + x2
Z
1
10) √ dx = arcsec x + C.
x x2 − 1
Z
1
11) dx = ln |x| + C.
x
ax
ax dx =
R
12) + C.
ln a
Veremos a continuación algunos casos de aplicación de las fórmulas anterio-
res.

PROBLEMA 7.1.

(4x3 − 5x2 + 7)dx.


R
Resolver la integral

Solución

Aplicaremos las propiedades (ii) y (iii) para descomponer la integral en otras


integrales más simples.
Z Z Z
I = 4 x3 dx − 5 x2 dx + 7 dx.

Aplicando la regla (1) se pueden resolver las integrales que resultan:

4x4 5x3 5x3



I= + 7x + C = x4 − + 7x + C.
4 3 3
Ten en cuenta que dx = x0 dx = x1 /1 + C = x + C.
R R

Aunque se deberı́a sumar una constante a cada integral, como esa constante
es arbitraria, se añade al resultado final una constante, que serı́a la suma de
cada una de las restantes.

PROBLEMA 7.2.
Z
1
Resolver dx.
x2

269
Solución

Escribimos 1/x2 como x−2 y tenemos:

x−1
Z
1
I = x−2 dx = + C = − + C.
−1 x

PROBLEMA 7.3.

R √
3
Resolver zdz .

Solución

Si escribimos el integrando en forma de potencia:

z 4/3
Z
3
I = z 1/3 dz = + C = z 4/3 + C.
4/3 4

PROBLEMA 7.4.

R √
Resolver (1 − x) xdx.

Solución

Si separamos en dos integrales, resulta:


√ √
Z Z Z Z
2 2
I= xdx − x xdx = x dx − x3/2 dx = x3/2 − x5/2 + C.
1/2
3 5

PROBLEMA 7.5.


Z  
x 2
Resolver x− + √ dx.
2 x

270
Solución

Si escribimos el integrando en forma de potencia, tenemos:


Z Z Z
1 2 1
1/2
I = x dx − xdx + 2 x−1/2 dx = x3/2 − x2 + 4x1/2 + C.
2 3 4

PROBLEMA 7.6.

(3s + 4)2 ds.


R
Resolver

Solución

Desarrollando la potencia,
Z Z Z Z
2 2
I = (9s + 24s + 16)ds = 9s ds + 24sds + 16ds

s3 s2
= 9 + 24 + 16s + C = 3s3 + 12s2 + 16s + C.
3 2

PROBLEMA 7.7.

4x3 − 5x2 + 7
Z
Resolver dx.
x2

Solución

Si dividimos cada sumando por el denominador común, podemos obtener


una suma de términos y descomponer en suma de integrales:
Z   Z Z Z
7
I = 4x − 5 + 2 dx = 4 xdx − 5 dx + 7 x−2 dx
x
x 2 x−1 7
= 4 − 5x + 7 + C = 2x2 − 5x − + C.
2 −1 x

271
PROBLEMA 7.8.

(4x2 + 7)2 x2 dx.


R
Resolver

Solución

Al desarrollar el cuadrado del binomio 4x2 + 7, multiplicar por x2 y separar


la integral en suma de varias, tendremos:

16x7 56x5 49x3


Z Z
I = (16x4 +56x2 +49)x2 dx = (16x6 +56x4 +49x2 )dx = + + +C
7 5 3

PROBLEMA 7.9.

(1 + x)2
Z
Resolver √ dx.
x

Solución

Descomponiendo en sumandos, tenemos:

1 + 2x + x2
Z Z
4 2
I= √ dx = (x−1/2 +2x1/2 +x3/2 )dx = 2x1/2 + x3/2 + x5/2 +C.
x 3 5

PROBLEMA 7.10.

x2 + 2x
Z
Resolver dx.
(x + 1)2

Solución

Completando cuadrados en el numerador e integrando por separado, tene-


mos:
(x + 1)2 − 1
Z Z  
1 1
I= 2
dx = 1− 2
dx = x + + C.
(x + 1) (x + 1) x+1

272
PROBLEMA 7.11.

3x3 − 4x2 + 3x
Z
Resolver dx.
x2 + 1

Solución

Si descomponemos la fracción en dos, resulta:


3x2
Z  
4
I= 3x − 4 + 2 dx = − 4x + 4 arc tg x + C.
x +1 2

PROBLEMA 7.12.
Z
sen x + tg x
Resolver dx.
tg x

Solución

La integral anterior se puede descomponer en suma de dos integrales en la


forma siguiente:
Z Z Z Z
sen x tg x
I= dx + dx = cos xdx + dx = sen x + x + C.
tg x tg x

PROBLEMA 7.13.
Z
sen y
Resolver dy .
cos2 y

Solución

Esta integral es inmediata debido a que


Z
I = tg y sec ydy = sec y + C.

273
PROBLEMA 7.14.

(tg 2x + sec 2x)2 dx.


R
Resolver

Solución

Desarrollando el integrando, tenemos:


Z
I = (tg2 2x + 2 tg 2x sec 2x + sec2 2x)dx
Z
= (2 sec2 2x + 2 tg 2x sec 2x − 1)dx = tg 2x + sec 2x − x + C.

PROBLEMA 7.15.
Z
1
Resolver dx.
1 + cos x

Solución

Multiplicando numerador y denominador por 1 − cos x:


1 − cos x 1 − cos x
Z Z
I = 2
dx = dx
1 − cos x sen2 x
Z
= (cosec2 x − cotg x cosec x)dx = − cotg x + cosec x + C.

PROBLEMA 7.16.
Z
1
Resolver dx.
sen2 x cos2 x

274
Solución

Aplicando la identidad trigonométrica sen2 x + cos2 x = 1, resulta:

sen2 x + cos2 x
Z Z Z
dx dx
I= dx = + = tg x − cotg x + C.
sen2 x cos2 x cos2 x sen2 x

B. INTEGRACIÓN POR SUSTITUCIÓN.

Cuando el integrando no es la derivada de una función conocida, todavı́a


es posible que lo sea de una función compuesta. A partir de la regla de la
cadena
D[f (g(x))] = f 0 (g(x)) · g 0 (x),
se deduce la correspondiente regla de integración
Z
f 0 (g(x)) · g 0 (x) · dx = f (g(x)) + C.

Las fórmulas siguientes se deducen de la aplicación de la regla de la cadena


en las fórmulas simples escritas en el apartado A.
f (x)n+1
1) [f (x)]n · f 0 (x)dx =
R
+ C si n 6= −1.
n+1
2) f 0 (x) · sen f (x)dx = − cos f (x) + C.
R

3) f 0 (x) · cos f (x)dx = sen f (x) + C.


R

4) f 0 (x) · sec2 f (x)dx = tg f (x) + C.


R

5) f 0 (x) · sec f (x) · tg f (x)dx = sec f (x) + C.


R

6) f 0 (x) · cosec f (x) · cotg f (x)dx = − cosec f (x) + C.


R

7) f 0 (x) · cosec2 f (x)dx = − cotg f (x) + C.


R

f 0 (x)
Z
8) p dx = arc sen f (x) + C.
1 − f (x)2
f 0 (x)
Z
9) dx = arc tg f (x) + C.
1 + f (x)2

275
f 0 (x)
Z
10) p dx = arcsec f (x) + C.
f (x) f (x)2 − 1
f 0 (x)
Z
11) dx = ln |f (x)| + C.
f (x)
af (x)
Z
12) af (x) · f 0 (x)dx = + C.
ln a
En la práctica, como no es fácil determinar si el integrando puede expresarse
como la derivada de una función compuesta, se hace un cambio de variable
paraRintentar expresar la integral en forma más sencilla. Ası́, en la expresión
I = f 0 (g(x)) 0 0
R 0 · g (x) · dx, si hacemos g(x) = t, entonces g (x)dx = dt, con
lo que I = f (t) · dt = f (t) + C = f (g(x)) + C.
Hay algunas sustituciones especiales para casos concretos que iremos ilus-
trando en la resolución de los problemas que siguen.

PROBLEMA 7.17.

R √
Resolver 4x 2x2 − 1dx.

Solución

Si f (x) = 2x2 −1, tenemos que f 0 (x) = 4x; se trata de calcular f (x)1/2 f 0 (x)dx.
R

La regla (1) indica que el resultado es:

(2x2 − 1)3/2 2
I= + C = (2x2 − 1)3/2 + C.
3/2 3

PROBLEMA 7.18.

(x3 + 2)2 3x2 dx.


R
Resolver

Solución

Haciendo el cambio x3 + 2 = u tenemos du = 3x2 dx, con lo que:


Z
1 1
I = u2 du = u3 + C = (x3 + 2)3 + C.
3 3

276
Otra forma es escribir directamente I = (x3 +2)2 d(x3 +2) = 13 (x3 +2)3 +C.
R

PROBLEMA 7.19.

R √
3
Resolver 2x − 6dx.

Solución

En este caso llamamos f (x) = 2x − 6. Sin embargo, f 0 (x) = 2 no aparece ex-


plı́citamente en la integral. Como las constantes se pueden multiplicar tanto
dentro como fuera de la integral (propiedad iii), podemos escribir:

1 √ √
Z Z
1
I= · 2 3 2x − 6dx = 2 3 2x − 6dx.
2 2

Ahora la integral tiene la forma en que se puede aplicar la regla (1). Ası́:

1 (2x − 6)(1/3)+1 3
I= · + C = (2x − 6)4/3 + C.
2 4/3 8

PROBLEMA 7.20.

(x3 + 2)1/2 x2 dx.


R
Resolver

Solución

Hacemos el cambio de variable u = x3 +2, con lo que du = 3x2 dx. Ası́:

1 u3/2
Z Z
1 3 1/2 2 1 2
I= (x + 2) · 3x dx = u1/2 du = · = (x3 + 2)3/2 + C.
3 3 3 3/2 9

También otra forma es la siguiente:


Z Z
1 3 1
I = (x + 2)1/2 · 3x2 dx = (x3 + 2)1/2 d(x3 + 2)
3 3
1 (x3 + 2)3/2 2
= · + C = (x3 + 2)3/2 + C.
3 3/2 9

277
PROBLEMA 7.21.

8x2
Z
Resolver dx.
(x3 + 2)3

Solución

Haciendo u = x3 + 2 tenemos du = 3x2 dx; por tanto:

8 u−2 −4
Z Z
du/3 8 −3
I=8 3
= u du = · +C = + C.
u 3 3 −2 3(x + 2)2
3

PROBLEMA 7.22.

x2
Z
Resolver √
4
dx.
x3 + 2

Solución

Haciendo u = x3 + 2, du = 3x2 dx y tenemos:


Z Z
du/3 1 1 4 4
I= √
4
= u−1/4 du = · u3/4 + C = (x3 + 2)3/4 + C.
u 3 3 3 9

PROBLEMA 7.23.

R √
Resolver 3x 1 − 2x2 dx.

Solución

Haciendo el cambio 1 − 2x2 = u, du = −4xdx, resulta:


√ du
Z Z
3 3 2 1
I=3 u· =− u1/2 du = − · u3/2 + C = − (1 − 2x2 )3/2 + C.
−4 4 4 3 2

278
PROBLEMA 7.24.
Z
x+3
Resolver dx.
(x2 + 6x)1/3

Solución

Hacemos el cambio x2 + 6x = u, con lo que (2x + 6)dx = du y resulta:


Z Z
du/2 1 1 3 3
I= 1/3
= u−1/3 du = · u2/3 + C = (x2 + 6x)2/3 + C.
u 2 2 2 4

PROBLEMA 7.25.

R √
Resolver x 3 1 − x2 dx.

Solución

Procediendo directamente, tenemos


Z Z
1 1
I = − (1 − x2 )1/3 (−2x)dx = − (1 − x2 )1/3 d(1 − x2 )
2 2
1 3 3
= − · (1 − x2 )4/3 + C = − (1 − x2 )4/3 + C.
2 4 8

PROBLEMA 7.26.

R√
Resolver x2 − 2x4 dx.

Solución

Sacando x2 factor común en la raı́z, podemos escribir:


Z Z
2 1/2 1
I = (1 − 2x ) · xdx = − (1 − 2x2 )1/2 (−4xdx)
4
1 2 1
= − · (1 − 2x2 )3/2 + C = − (1 − 2x2 )3/2 + C.
4 3 6

279
PROBLEMA 7.27.

(ex + 1)3 ex dx.


R
Resolver

Solución

Hacemos el cambio u = ex + 1, con lo que du = ex dx. Ası́:

u4 (ex + 1)4
Z
I = u3 du = +C = + C.
4 4

PROBLEMA 7.28.
Z
x
Resolver dx.
(x2 + a2 )n

Solución

Sabiendo que la derivada de x2 + a2 es 2x, tenemos:


Z Z
1 2xdx 1
I = = (x2 + a2 )−n d(x2 + a2 )
2 (x2 + a2 )n 2
1 (x2 + a2 )−n+1 −1
= · +C = + C.
2 −n + 1 2(n − 1)(x2 + a2 )n−1

PROBLEMA 7.29.
Z
1
Resolver √ dx.
(1 − x2 ) 1 − x2

280
Solución

Utilizamos en este caso el siguiente artificio:


x−3 dx
Z Z
dx
I = p = √
(1 − x2 )3 [x−1 1 − x2 ]3
x−3 −2x−3
Z Z
1
= √ dx = − √ dx
( x−2 − 1)3 2 ( x−2 − 1)3
Z
1 x
= − (x−2 − 1)−3/2 d(x−2 − 1) = (x−2 − 1)−1/2 + C = √ + C.
2 1 − x2

PROBLEMA 7.30.

R
Resolver sen(x/2)dx.

Solución

Aplicando la fórmula (2), tenemos:


Z
I = 2 sen(x/2)(dx/2) = −2 cos(x/2) + C.

PROBLEMA 7.31.

R
Resolver cos 3xdx.

Solución

Aplicamos en este caso la fórmula (3) y obtenemos:


Z
1 1
I= cos 3x(3dx) = sen 3x + C.
3 3

PROBLEMA 7.32.

sen2 x cos xdx.


R
Resolver

281
Solución

Tenemos que:
sen3 x
Z
I= sen2 xd(sen x) = + C.
3

PROBLEMA 7.33.

sen3 xdx.
R
Resolver

Solución

Si descomponemos sen3 x = sen2 x · sen x, debemos expresar sen2 x en fun-


ción de cos x debido a que en el integrando aparece (cos x)0 = − sen x. En-
tonces:
Z Z Z Z
I = sen x sen xdx = (1−cos x) sen xdx = sen xdx+ − cos2 x sen xdx.
2 2

La
R 2primera integral es inmediata. Como la segunda integral es de la forma
0
f (x)f (x)dx, con f (x) = cos x, resulta que:

cos3 x
I = − cos x + + C.
3

PROBLEMA 7.34.

sec2 (2ax)dx.
R
Resolver

Solución

Aplicando la fórmula (4), tenemos:


Z
1 1
I= sec2 (2ax)(2adx) = tg(2ax) + C.
2a 2a

282
PROBLEMA 7.35.

ex cos ex dx.
R
Resolver

Solución

Procediendo de forma directa,


Z
I = cos ex (ex dx) = sen ex + C.

PROBLEMA 7.36.
Z
1
Resolver dx.
x+2

Solución

Aplicamos la fórmula (11):


Z
d(x + 2)
I= = ln |x + 2| + C.
x+2

PROBLEMA 7.37.
Z
1
Resolver dx.
2x − 3

Solución

Análogamente al anterior,

d(2x − 3)
Z
1 1
I= = ln |2x − 3| + C.
2 2x − 3 2

283
PROBLEMA 7.38.
Z
x
Resolver dx.
x2 −1

Solución

Si escribimos en el numerador la derivada del denominador, resulta:

d(x2 − 1)
Z Z
1 2x 1 1
I= 2
dx = 2
= ln |x2 − 1| + C.
2 x −1 2 x −1 2

PROBLEMA 7.39.

x2
Z
Resolver dx.
1 − 2x3

Solución

Multiplicando y dividiendo por -6,

−6x2
Z
1 1
I=− 3
dx = − ln |1 − 2x3 | + C.
6 1 − 2x 6

PROBLEMA 7.40.
Z
x+2
Resolver dx.
x+1

Solución

Separamos en dos fracciones y tenemos:


Z   Z Z
1 1
I= 1+ dx = dx + dx = x + ln |x + 1| + C.
x+1 x+1

284
PROBLEMA 7.41.
Z
1
Resolver dx.
ex +1

Solución

Usaremos aquı́ el siguiente artificio:

e−x −e−x
Z Z
I= dx = − dx = − ln(1 + e−x ) + C.
e−x (ex + 1) 1 + e−x

No es necesario en este caso el valor absoluto porque 1 + e−x > 0, para todo
x.

PROBLEMA 7.42.

R
Resolver tg xdx.

Solución

Procederemos ası́:
− sen x
Z Z
sen x
I= dx = − dx = − ln | cos x| + C = ln | sec x| + C.
cos x cos x

PROBLEMA 7.43.

R
Resolver tg 2xdx.

Solución

Análogamente al anterior:
Z
1 1
I= tg 2x(2dx) = ln | sec 2x| + C.
2 2

285
PROBLEMA 7.44.
Z
sen x + cos x
Resolver dx.
cos x

Solución

Separando la fracción en dos, tenemos:


Z
I = (tg x + 1)dx = ln | sec x| + x + C.

PROBLEMA 7.45.

(1 + tg x)2 dx.
R
Resolver

Solución

Desarrollando el cuadrado, tenemos:


Z Z
I = (1 + 2 tg x + tg x)dx = (sec2 x + 2 tg x)dx = tg x + 2 ln | sec x| + C.
2

PROBLEMA 7.46.

x cotg x2 dx.
R
Resolver

Solución
Z Z
cos x
Sabiendo que cotg xdx = dx = ln | sen x| + C, resulta:
sen x
Z
1 1
I= cotg x2 · (2xdx) = ln | sen x2 | + C.
2 2

286
PROBLEMA 7.47.

R
Resolver sec xdx.

Solución

Es útil en este caso utilizar el siguiente artificio:


sec x tg x + sec2 x
Z Z
sec x(sec x + tg x)
I= dx = dx = ln | sec x+tg x|+C.
sec x + tg x sec x + tg x

PROBLEMA 7.48.
Z √
sec x
Resolver √ dx.
x

Solución

Teniendo en cuenta el problema anterior, resulta:


√ dx √ √
Z
I = 2 sec x √ = 2 ln | sec x + tg x| + C.
2 x

PROBLEMA 7.49.

R
Resolver cosec udu.

Solución

Un artificio similar al realizado anteriormente permite escribir:


cosec u(cosec u − cotg u)
Z
I = du
cosec u − cotg u
cosec2 u − cosec u cotg u
Z
= du = ln | cosec u − cotg u| + C.
cosec u − cotg u

287
PROBLEMA 7.50.
Z
sec x tg x
Resolver dx.
a + b sec x

Solución

Si hacemos que el numerador sea la derivada del denominador, tenemos


Z
1 b sec x tg xdx 1
I= = ln |a + b sec x| + C.
b a + b sec x b

PROBLEMA 7.51.
Z
1
Resolver dx.
cosec 2x − cotg 2x

Solución

Aplicando las identidades cosec 2x = 1/ sen 2x y cotg 2x = cos 2x/ sen 2x:
Z Z
sen 2x 1 2 sen 2xdx 1
I = dx = = ln(1 − cos 2x) + C
1 − cos 2x 2 1 − cos 2x 2
1 1
= ln(2 sen2 x) + C = (ln 2 + 2 ln | sen x|) + C = ln | sen x| + C 0 .
2 2

PROBLEMA 7.52.

e−x dx.
R
Resolver

288
Solución

La fórmula (12) da casi directamente:


Z
I = − e−x (−dx) = −e−x + C.

PROBLEMA 7.53.

a2x dx.
R
Resolver

Solución

De nuevo por la fórmula (12),

1 a2x
Z
1
I= a2x (2dx) = · + C.
2 2 ln a

PROBLEMA 7.54.

e1/x
Z
Resolver dx.
x2

Solución

Análogamente a los anteriores,


Z  
1/x dx
I=− e − 2 = −e1/x + C.
x

PROBLEMA 7.55.

e3 cos 2x sen 2xdx.


R
Resolver

289
Solución

Podemos hacer el cambio 3 cos 2x = u o bien proceder directamente:


Z
1 1
I=− e3 cos 2x (−6 sen 2xdx) = − e3 cos 2x + C.
6 6

PROBLEMA 7.56.
Z
1
Resolver √ dx.
a2 − x2

Solución

De acuerdo a la fórmula (8) tenemos:


Z Z
dx d(x/a)
I= p = p = arc sen x/a + C.
a 1 − (x/a) 2 1 − (x/a)2

PROBLEMA 7.57.
Z
1
Resolver √ dx.
25 − 16x2

Solución

Teniendo en cuenta el problema anterior, resulta:


Z Z
1 4dx 1 4dx/5 1 4x
I= p = p = arc sen + C.
4 52 − (4x)2 4 1 − (4x/5)2 4 5

PROBLEMA 7.58.

x2
Z
Resolver √ dx.
1 − x6

290
Solución

Análogamente a los anteriores:

3x2 dx
Z
1 1
I= p = arc sen x3 + C.
3 1− (x3 )2 3

PROBLEMA 7.59.
Z
x+3
Resolver √ dx.
1 − x2

Solución

Si separamos en dos integrales:


Z Z
xdx dx
I = √ +3 √
2 2
Z 1−x Z1 − x
−2xdx dx p
= − √ +3 √ = − 1 − x2 + 3 arc sen x + C.
2 1 − x2 1 − x2

PROBLEMA 7.60.
Z
1
Resolver √ dx.
20 + 8x − x2

Solución

Completando cuadrados en la raı́z y teniendo en cuenta los problemas ante-


riores, obtenemos:

x−4
Z Z
dx dx
I= p = p = arc sen + C.
36 − (x2 − 8x + 16) 62 − (x − 4)2 6

291
PROBLEMA 7.61.
Z
1
Resolver √ dx.
28 − 12x − x2

Solución

Análogamente al anterior resulta:


Z Z
dx dx x+6
I= p = p = arc sen + C.
2
64 − (x + 12x + 36) 2
8 − (x + 6)2 8

PROBLEMA 7.62.
Z
x+3
Resolver √ dx.
5 − 4x − x2

Solución

Procedemos de la siguiente manera:

(−2x − 6)dx (−2x − 4) − 2 (−2x − 4)


Z Z Z
1 1 1
I = − √ =− √ dx = − √ dx
2 5 − 4x − x 2 2 5 − 4x − x 2 2 5 − 4x − x2
(−2x − 4)
Z Z Z
dx 1 dx
+ √ =− √ dx + p dx
5 − 4x − x 2 2 5 − 4x − x 2 9 − (x + 2)2
p x+2
= − 5 − 4x − x2 + arc sen + C.
3

PROBLEMA 7.63.
Z
1
Resolver dx.
9 + x2

292
Solución

Aplicamos la fórmula (9) y tenemos:


Z Z
dx 1 dx/3 1
I= = = arc tg(x/3) + C.
9[1 + (x/3)2 ] 3 1 + (x/3)2 3

PROBLEMA 7.64.
Z
1
Resolver dx.
4x2 +9

Solución

Procediendo análogamente al problema anterior:


Z Z
1 2dx 1 2dx 1 2x
I= = = arc tg + C.
2 (2x)2 + 32 2 9[(2x/3)2 + 1] 6 3

PROBLEMA 7.65.
Z
x
Resolver dx.
x4 +3

Solución

Razonando análogamente a los problemas anteriores, tendremos:


√ √
x2 x2 3
Z
1 2xdx 1 1 3
I= = · √ arc tg √ + C = arc tg + C.
2 (x2 )2 + 3 2 3 3 6 3

PROBLEMA 7.66.
Z
1
Resolver dy .
y2 + 10y + 30

293
Solución

Al completar cuadrados en el denominador, podemos reducirlo a los casos


anteriores:
Z Z
dy dy 1 y+5
I= 2
= 2
= √ arc tg √ + C.
(y + 10y + 25) + 5 (y + 5) + 5 5 5

Observación: Este método sólo es posible porque el denominador no tiene


raı́ces reales. En caso contrario, deberemos aplicar el método de integración
por fracciones simples (ver apartado D.)

PROBLEMA 7.67.
Z
1
Resolver dx.
ex + e−x

Solución

Multiplicando numerador y denominador por ex obtenemos directamen-


te:
ex dx
Z
I= = arc tg ex + C.
e2x + 1

PROBLEMA 7.68.
Z
sec x tg x
Resolver dx.
9 + 4 sec2 x

Solución

Teniendo en cuenta los problemas anteriores, resulta también:


Z
1 2 sec x tg xdx 1 2 sec x
I= 2 2
= arc tg + C.
2 3 + (2 sec x) 6 3

294
PROBLEMA 7.69.

2x − 7
Z
Resolver dx.
x2 + 9

Solución

Si separamos en dos integrales, resulta:


Z Z
2xdx dx 7
I= −7 = ln(x2 + 9) − arc tg(x/3) + C.
x2 + 9 x2 + 9 3

PROBLEMA 7.70.
Z
x+1
Resolver dx.
x2 − 4x + 8

Solución

Si intentamos escribir en el numerador la derivada del denominador, obte-


nemos las siguientes integrales:
(2x − 4)dx (2x − 4)dx
Z Z Z Z
1 dx 1 dx
I = 2
+3 2
= 2
+3
2 x − 4x + 8 x − 4x + 8 2 x − 4x + 8 (x − 2)2 + 4
1 3 x−2
= ln(x2 − 4x + 8) + arc tg + C.
2 2 2

PROBLEMA 7.71.
Z
1
Resolver √ dx.
x 4x2 − 9

Solución

Aplicando adecuadamente la fórmula (10) resulta:


Z Z
dx dx
I = p = p
2
x (2x) − 3 2 3x (2x/3)2 − 1
Z
1 2dx/3 1 2x
= p = arcsec + C.
3 2
(2x/3) (2x/3) − 1 3 3

295
PROBLEMA 7.72.
Z
1
Resolver √ dx.
x x4 − 1

Solución

Análogamente al problema anterior podemos escribir:


Z
1 2xdx 1 1
I= p = arcsec x2 + C = arc cos(1/x2 ) + C.
2 2 2 2
x (x ) − 1 2 2

PROBLEMA 7.73.

sen2 xdx.
R
Resolver

Solución

Aplicando la fórmula sen2 x = (1 − cos 2x)/2 resulta:


Z
1 1 1
I= (1 − cos 2x)dx = x − sen 2x + C.
2 2 4

PROBLEMA 7.74.

cos2 λxdx.
R
Resolver

Solución

Aplicamos en este caso la identidad cos2 λx = (1 + cos 2λx)/2:


Z
1 x sen 2λx
I= (1 + cos 2λx)dx = + + C.
2 2 4λ

296
PROBLEMA 7.75.

cos5 xdx.
R
Resolver

Solución

Realizamos la siguiente descomposición:


Z Z Z Z
I = cos x cos xdx = (1 − sen x) cos xdx = cos xdx − 2 sen2 x cos xdx
4 2 2

Z
2 1
+ sen4 x cos xdx = sen x − sen3 x + sen5 x + C.
3 5

PROBLEMA 7.76.

sen2 x cos3 xdx.


R
Resolver

Solución

De la identidad fundamental sen2 x + cos2 x = 1 resulta:


Z Z
I = sen x cos x cos xdx = sen2 x(1 − sen2 x) cos xdx
2 2

Z Z
1 1
= sen x cos xdx − sen4 x cos xdx = sen3 x − sen5 x + C.
2
3 5

PROBLEMA 7.77.

cos4 2x sen3 2xdx.


R
Resolver

Solución

Como el exponente de sen 2x es impar, procedemos ası́:


Z Z
I = cos4 2x sen2 2x sen 2xdx = cos4 2x(1 − cos2 2x) sen 2xdx
Z Z
1 1
= cos4 2x sen 2xdx − cos6 2x sen 2xdx = − cos5 2x + cos7 2x + C.
10 14

297
PROBLEMA 7.78.

sen3 3x cos5 3xdx.


R
Resolver

Solución

Análogamente al ejercicio anterior tenemos:


Z Z
I = (1 − cos 3x) cos 3x sen 3xdx = cos5 3x sen 3xdx
2 5

Z
1 1
− cos7 3x sen 3xdx = − cos6 3x + cos8 3x + C.
18 24
Otra forma similar es usar el hecho de que el exponente de cos 3x también
es impar:
Z
I = sen3 3x(1 − sen2 3x)2 cos 3xdx
Z Z Z
= sen 3x cos 3xdx − 2 sen 3x cos 3xdx + sen7 3x cos 3xdx
3 5

1 1 1
= sen4 3x − sen6 3x + sen8 3x + C.
12 9 24
Obsérvese que ambos resultados difieren en una constante aunque no lo
parezca a simple vista.

PROBLEMA 7.79.

sen4 xdx.
R
Resolver

Solución

Como el exponente de sen x es par, utilizamos la identidad sen2 x = (1 −


cos 2x)/2 y posteriormente cos2 2x = (1 + cos 4x)/2:
Z Z Z Z
2 2 1 2 1 1
I = (sen x) dx = (1 − cos 2x) dx = dx − cos 2xdx
4 4 2
Z Z Z Z
1 1 1 1
+ cos2 2xdx = dx − cos 2xdx + (1 + cos 4x)dx
4 4 2 8
1 1 1 1 3 1 1
= x − sen 2x + x + sen 4x + C = x − sen 2x + sen 4x + C.
4 4 8 32 8 4 32

298
PROBLEMA 7.80.

sen2 x cos2 xdx.


R
Resolver

Solución

Por la fórmula sen 2x = 2 sen x cos x tenemos:


Z Z Z
1 1 1
I = (2 sen x cos x)2 dx = sen2 2xdx = (1 − cos 4x)dx
4 4 8
Z Z
1 1 1 1
= dx − cos 4xdx = x − sen 4x + C.
8 8 8 32

PROBLEMA 7.81.

sen4 3x cos2 3xdx.


R
Resolver

Solución

Por ser ambos exponentes pares tenemos como antes:


Z Z
2 2 2 1
I = (sen 3x cos 3x) sen 3xdx = sen2 6x(1 − cos 6x)dx
8
Z Z Z
1 2 1 2 1
= sen 6xdx − sen 6x cos 6xdx = (1 − cos 12x)dx
8 8 16
Z
1 1 1 1
− sen2 6x cos 6xdx = x − sen 12x − sen3 6x + C.
8 16 192 144

PROBLEMA 7.82.

tg4 xdx.
R
Resolver

299
Solución

Aplicamos la fórmula sec2 x = 1 + tg2 x y tenemos:


Z Z Z Z
I = tg x tg xdx = tg x(sec x − 1)dx = tg x sec xdx − tg2 xdx
2 2 2 2 2 2

Z Z
1
= tg xd(tg x) − (sec2 x − 1)dx = tg3 x − tg x + x + C.
2
3

PROBLEMA 7.83.

tg5 xdx.
R
Resolver

Solución

De forma análoga al problema anterior podemos escribir:


Z Z Z Z
I = tg x tg xdx = tg x(sec x − 1)dx = tg x sec xdx − tg3 xdx
3 2 3 2 3 2

Z Z
1 1
= tg xd(tg x) − tg x(sec2 x − 1)dx = tg4 x − tg2 x + ln | sec x| + C.
3
4 2

PROBLEMA 7.84.

sec4 2xdx.
R
Resolver

Solución

También en este caso tenemos:


Z Z
I = sec2 2x sec2 2xdx = sec2 2x(1 + tg2 2x)dx
Z Z
1 1
= sec2 2xdx + tg2 2x sec2 2xdx = tg 2x + tg3 2x + C.
2 6

300
PROBLEMA 7.85.

tg3 3x sec4 3xdx.


R
Resolver

Solución

Nuevamente, de la identidad sec2 3x = 1 + tg2 3x, resulta:


Z Z Z
I = tg 3x(1 + tg 3x) sec 3xdx = tg 3x sec 3xdx + tg5 3x sec2 3xdx
3 2 2 3 2

1 4 1 6
= tg 3x + tg 3x + C.
12 18

PROBLEMA 7.86.

tg3 2x sec3 2xdx.


R
Resolver

Solución

En este caso integramos con respecto a d(sec 2x) como sigue:


Z Z
I = tg2 2x sec2 2x sec 2x tg 2xdx = (sec2 2x − 1) sec2 2x sec 2x tg 2xdx
Z Z
1 1
= sec4 2xd(sec 2x) − sec2 2xd(sec 2x)
2 2
1 1
= sec5 2x − sec3 2x + C.
10 6

PROBLEMA 7.87.

cotg3 2xdx.
R
Resolver

301
Solución

De la fórmula cosec2 2x−1 = cotg2 2x y teniendo en cuenta que d(cotg 2x) =


−2 cosec2 2xdx, resulta:
Z
1 1
I = cotg 2x(cosec2 2x − 1)dx = − cotg2 2x + ln | cosec 2x| + C.
4 2

PROBLEMA 7.88.

cotg4 3xdx.
R
Resolver

Solución

Procediendo de forma análoga al anterior tenemos:


Z Z Z
I = cotg 3x(cosec 3x − 1)dx = cotg 3x cosec 3xdx − cotg2 3xdx
2 2 2 2

Z Z
1
= − cotg2 3xd(cotg 3x) − (cosec2 3x − 1)dx
3
1 1
= − cotg3 3x + cotg 3x + x + C.
9 3

PROBLEMA 7.89.

cosec6 xdx.
R
Resolver

Solución

De forma similar a los anteriores,


Z Z Z
I = cosec2 x(1 + cotg2 x)2 dx = cosec2 xdx + 2 cotg2 x cosec2 xdx
Z
2 1
+ cotg4 x cosec2 xdx = − cotg x − cotg3 x − cotg5 x + C.
3 5

302
PROBLEMA 7.90.

cotg3 x cosec5 xdx.


R
Resolver

Solución

Sabiendo que d(cosec x) = − cosec x cotg xdx, tenemos:


Z
I = cotg2 x cosec4 x cosec x cotg xdx
Z
= (cosec2 x − 1) cosec4 x cosec x cotg xdx
Z Z
= − cosec xd(cosec x) + cosec4 xd(cosec x)
6

1 1
= − cosec7 x + cosec5 x + C.
7 5

PROBLEMA 7.91.

R√
Resolver 1 − cos xdx.

Solución

Debido a la fórmula 1 − cos x = 2 sen2 (x/2), tenemos:


√ Z √
I = 2 sen(x/2)dx = −2 2 cos(x/2) + C.

PROBLEMA 7.92.
Z
1
Resolver √ dx.
1 − sen 2x

303
Solución

Aplicamos la identidad sen x = cos(π/2 − x) y procedemos como en el pro-


blema anterior:
Z Z
1 1
I = p dx = √ dx
1 − cos(π/2 − 2x) 2 sen(π/4 − x)
√ Z √
2 2
= cosec(π/4 − x)dx = − ln | cosec(π/4 − x) − cotg(π/4 − x)| + C.
2 2

PROBLEMA 7.93.

(1 + cos 3x)3/2 dx.


R
Resolver

Solución

Utilizamos la fórmula 1 + cos 3x = 2 cos2 (3x/2):


√ Z √ Z
I = 2 2 cos (3x/2)dx = 2 2 [1 − sen2 (3x/2)] cos(3x/2)dx
3


 
2 2 3
= 2 2 sen(3x/2) − sen (3x/2) + C.
3 9

PROBLEMA 7.94.
Z
1
Resolver dx.
sen3 x cos3 x

304
Solución

De la identidad 1 = sen2 x+cos2 x, tenemos la siguiente descomposición:


(sen2 x + cos2 x)2 sen4 x + cos4 x + 2 sen2 x cos2 x
Z Z
I = dx = dx
sen3 x cos3 x sen3 x cos3 x
Z Z Z
sen x cos x 1
= 3
dx + 3
dx + 2 dx
cos x sen x sen x cos x
sec2 x
Z Z Z
−3 −3
= − cos xd(cos x) + sen xd(sen x) + 2 dx
tg x
1 1
= − + 2 ln | tg x| + C.
2 cos x 2 sen2 x
2

PROBLEMA 7.95.

sen λx cos µxdx, donde λ2 − µ2 6= 0.


R
Resolver

Solución

Utilizamos la fórmula sen(a + b) + sen(a − b) = 2 sen a cos b:


Z
1
I = [sen(λx + µx) + sen(λx − µx)]dx
2
 
1 cos(λ + µ)x cos(λ − µ)x
Z
1
= [sen(λ + µ)x + sen(λ − µ)x]dx = − + + C.
2 2 λ+µ λ−µ

PROBLEMA 7.96.

sen λx sen µxdx, donde λ2 − µ2 6= 0.


R
Resolver

Solución

Similar al anterior con la fórmula cos(a−b)−cos(a+b) = 2 sen a sen b:


Z
1
I = [cos(λx − µx) − cos(λx + µx)]dx
2
 
1 sen(λ − µ)x sen(λ + µ)x
Z
1
= [cos(λ − µ)x − cos(λ + µ)x]dx = − + C.
2 2 λ−µ λ+µ

305
PROBLEMA 7.97.

cos λx cos µxdx, donde λ2 − µ2 6= 0.


R
Resolver

Solución

Aplicamos en este caso la fórmula cos(a+b)+cos(a−b) = 2 cos a cos b:


 
1 sen(λ + µ)x sen(λ − µ)x
Z
1
I= [cos(λx+µx)+cos(λx−µx)]dx = + +C.
2 2 λ+µ λ−µ

PROBLEMA 7.98.
Z
1
Resolver √ dx.
x2 +1

Solución

Si hacemos el cambio de variable x = tg u, entonces dx = sec2 udu. Ası́:


Z Z
1 2
p
I= sec udu = sec udu = ln | sec u+tg u|+C = ln(x+ x2 + 1)+C.
sec u

PROBLEMA 7.99.
Z
1
Resolver √ dx.
4x2 + 9

Solución

Escribimos la función como en el problema anterior y aplicamos el re-


sultado obtenido o bien hacemos el cambio 2x/3 = tg u, de modo que

306
dx = (3/2) sec2 udu:

(3/2) sec2 udu


Z Z Z
1 1
I = p dx = p dx =
(2x)2 + 32 3 (2x/3)2 + 1 3 sec u

1 1 2x
4x2 + 9

= ln | sec u + tg u| + C = ln + + C.
2 2 3 3

PROBLEMA 7.100.
Z
x+2
Resolver √ dx.
x2 + 9

Solución

Separamos la integral en dos y aplicamos en la segunda la sustitución x/3 =


tg u:
Z Z Z
1 2x + 4 1 2x 1
I = √ dx = √ dx + 2 √ dx
2 2
x +9 2 2
x +9 x2 + 9
x √x2 + 9
Z
p p
= x2 + 9 + 2 sec udu = x2 + 9 + 2 ln + + C.

3 3

PROBLEMA 7.101.
Z
1
Resolver √ dx.
x2 − 1

Solución

Si hacemos x = sec u, entonces dx = sec u tg udu. Por tanto:


Z
1 p
I= sec u tg udu = ln | sec u + tg u| + C = ln |x + x2 − 1| + C.
tg u

307
PROBLEMA 7.102.
Z
1
Resolver √ dz .
9z 2− 25

Solución

Si hacemos el cambio 3z/5 = sec u, 3dz/5 = sec u tg udu, entonces:


Z Z
1 3/5 1 sec u tg u
I = p dz = du
3 2
(3z/5) − 1 3 tg u

1 1 3z 9z 2 − 25
= ln | sec u + tg u| + C = ln + + C.
3 3 5 5

PROBLEMA 7.103.
Z
1
Resolver √ ds.
4s + s2

Solución

Completando cuadrados en el denominador y haciendo el cambio (s+2)/2 =


sec u, resulta:

Z Z
1 1
I = p ds = p ds
(s + 2)2 −4 [(s + 2)/2]2 − 1
2
s + 2 √4s + s2
Z
= sec udu = ln | sec u + tg u| + C = ln + + C.

2 2

PROBLEMA 7.104.
Z
x+2
Resolver √ dx.
x2 + 2x − 3

308
Solución

Separamos en dos integrales de modo que la primera sea la derivada de una


raı́z y en la segunda hacemos el cambio (x + 1)/2 = sec u:
Z Z Z
2x + 4 2x + 2 1/2
I = √ dx = √ dx + p dx
2
2 x + 2x − 3 2
2 x + 2x − 3 [(x + 1)/2]2 − 1
p Z
= x2 + 2x − 3 + sec udu
x + 1 √x2 + 2x − 3

p
= x2 + 2x − 3 + ln + + C.

2 2

PROBLEMA 7.105.

R√
Resolver 3 − 4x2 dx.

Solución

√ √
Haremos el cambio 2x/ 3 = sen u, 2dx/ 3 = cos udu después de escribir la
función de forma más conveniente:

√ Z q √ √ Z 3
I = 3 2
1 − (2x/ 3) dx = 3 cos2 udu
2
Z  
3 1 + cos 2u 3 sen 2u
= du = u+ +C
2 2 4 2
r
3 √ 3 2x 3 − 4x2
= arc sen(2x/ 3) + · 2 · √ · +C
4 8 3 3
3 √ xp
= arc sen(2x/ 3) + 3 − 4x2 + C.
4 2

PROBLEMA 7.106.

R√
Resolver 3 − 2x − x2 dx.

309
Solución

Completando cuadrados y procediendo como en el problema anterior con el


cambio (x + 1)/2 = sen u, resulta:
Z p Z p Z
I = 4 − (x + 1) dx = 2 1 − [(x + 1)/2] dx = 4 cos2 udu
2 2

 
u sen 2u x + 1 x + 1p
= 4 + + C = 2 arc sen + 3 − 2x − x2 + C.
2 4 2 2

PROBLEMA 7.107.
Z
1
Resolver √ dx.
x2 4 + x2

Solución


Si hacemos el cambio tg z = x/2, tendremos dx = 2 sec2 zdz y 4 + x2 =
2 sec z. Entonces:
2 sec2 z
Z Z
1 sec z
I = 2 dz = dz
(4 tg z)(2 sec z) 4 tg2 z

4 + x2
Z
1 −2 1
= sen z cos zdz = − +C =− + C.
4 4 sen z 4x

PROBLEMA 7.108.

x2
Z
Resolver √ dx.
x2 − 4

Solución


Hacemos el cambio x/2 = sec z, con lo que dx = 2 sec z tg zdz y x2 − 4 =
2 tg z:

4 sec2 z
Z Z
I = (2 sec z tg zdz) = 4 sec3 zdz
2 tg z
1 p p
= 2 sec z tg z + 2 ln | sec z + tg z| + C = x x2 − 4 + 2 ln |x + x2 − 4| + C 0 .
2

310
(Ver problema 7.124 para la resolución de la última integral.)

PROBLEMA 7.109.

Z √
9 − 4x2
Resolver dx.
x

Solución


Haciendo 2x/3 = sen z tendremos 2dx/3 = cos zdz y 9 − 4x2 = 3 cos z.
Ası́:
cos2 z
Z Z
3 cos z
I = (3/2) cos zdz = 3 dz
(3/2) sen z sen z
1 − sen2 z
Z Z Z
= 3 dz = 3 cosec zdz − 3 sen zdz
sen z
3 − √9 − 4x2 p

= 3 ln | cosec z − cotg z| + 3 cos z + C = 3 ln + 9 − 4x2 + C 0 .

2x

PROBLEMA 7.110.

(16 − 9x2 )3/2


Z
Resolver dx.
x6

Solución


Haciendo 3x/4 = sen z tendremos dx = (4/3) cos zdz y 16 − 9x2 = 4 cos z.
Ası́:
64 cos3 z · (4/3) cos z cos4 z
Z Z
243
I = dz = dz
(4096/729) sen6 z 16 sen6 z
Z
243 243
= cotg4 z cosec2 zdz = − cotg5 z + C
16 80
243 (16 − 9x2 )5/2 1 (16 − 9x2 )5/2
= − · + C = − · + C.
80 243x5 80 x5

311
PROBLEMA 7.111.

x2
Z
Resolver √ dx.
2x − x2

Solución

x2
Z
Volvemos a escribir la integral como p dx y hacemos el cam-
1)2
1 − (x −√
bio x − 1 = sen z. Tendremos ası́ dx = cos zdz y 2x − x2 = cos z. Resul-
ta:
(1 + sen z)2
Z Z
I = cos zdz = (1 + 2 sen z + sen2 z)dz
cos z
Z  
1 − cos 2z 3 1
= 1 + 2 sen z + dz = z − 2 cos z − sen 2z + C
2 2 4
3 p 1 p
= arc sen(x − 1) − 2 2x − x2 − (x − 1) 2x − x2 + C
2 2
3 1 p
= arc sen(x − 1) − (x + 3) 2x − x2 + C.
2 2

PROBLEMA 7.112.
Z
1
Resolver dx.
(4x2 − 24x + 27)3/2

Solución

Z
1
Completando cuadrados tenemos que I = dx.
[4(x − 3)2 − 9]3/2

Haciendo 2(x−3)/3 = sec z, resulta dx = (3/2) sec z tg zdz y 4x2 − 24x + 27 =
3 tg z, con lo que:
Z Z
(3/2) sec z tg zdz 1
I = = sen−2 z cos zdz
27 tg3 z 18
1 1 x−3
= − sen−1 z + C = − · √ + C.
18 9 4x2 − 24x + 27

312
PROBLEMA 7.113.
Z
1
Resolver √ dx.
(1 + x2 ) 1 + x2

Solución


Haciendo x2 = t, 2xdx = dt, es decir, dx = dt/(2 t). Por tanto,
Z √ Z
dt/2 t 1 dt
I= √ = p .
(1 + t) 1 + t 2 (1 + t) (1 + t)t
1 1 1
Haciendo ahora = z =⇒ − 1 = t =⇒ dt = − 2 dz, tenemos:
1+t z z
− z12 dz · z −dz/z
Z Z
1 1
I = p = p
2 (1/z)[(1/z) − 1] 2 (1/z 2 ) − (1/z)
−dz/z
Z Z
1 1 dz
= √ =− √
2 (1/z) 1 − z 2 1−z
1 (1 − z)1/2
Z
1
= − (1 − z)−1/2 dz = · +C
2 2 1/2

r r
1 1+t−1
= 1−z+C = 1− +C = +C
1+t 1+t

t x
= √ +C = √ + C.
1+t 1 + x2

C. INTEGRACIÓN POR PARTES.

Este método se basa en la fórmula de derivación de un producto de dos


funciones: integrandoRla fórmula (f · g)0R(x) = f 0 (x) · g(x) + f (x) · g 0 (x), se
obtiene f (x) · g(x) = f 0 (x) · g(x)dx + f (x) · g 0 (x)dx, de donde,
Z Z
f (x) · g 0 (x) · dx = f (x) · g(x) − f 0 (x) · g(x) · dx.

313
Otra forma de escribir esta fórmula es llamar u = f (x) y v = g(x), con lo
que resulta: Z Z
u · dv = u · v − v · du.

Este método suele aplicarse cuando el integrando es producto de dos fun-


ciones de distinta clase, como por ejemplo, polinómica por exponencial, tri-
gonométrica por exponencial, polinómica por logarı́tmica, etc.
Una gran variedad de integrales que se pueden resolver por este método se
ofrece en los problemas que siguen.

PROBLEMA 7.114.

xex dx.
R
Resolver

Solución

Hacemos u = x, dv = ex dx. Entonces du = dx, v = ex y tenemos:


Z
I = xe − ex dx = xex − ex + C.
x

PROBLEMA 7.115.

x3 e2x dx.
R
Resolver

Solución

Haciendo u = x3 , dv = e2x dx tendremos du = 3x2 dx, v = (1/2)e2x , con lo


que: Z
1 3
I = x3 e2x − x2 e2x dx.
2 2
Haciendo en la integral resultante u = x2 y dv = e2x dx tendremos du =
2xdx, v = (1/2)e2x , de modo que:
 Z  Z
1 3 2x 3 1 2 2x 1 3 2 2x 3
I= x e − x e − xe dx = x3 e2x −
2x
x e + xe2x dx.
2 2 2 2 4 2

314
Haciendo en la integral resultante u = x y dv = e2x dx tendremos du = dx y
v = (1/2)e2x y nuevamente,
 Z 
1 3 2x 3 2 2x 3 1 2x 1 2x
I = x e − x e + xe − e dx
2 4 2 2 2
1 3 2x 3 2 2x 3 2x 3 2x
= x e − x e + xe − e + C.
2 4 4 8

PROBLEMA 7.116.

2
x3 ex dx.
R
Resolver

Solución

2 2
Hacemos u = x2 y dv = xex dx, de donde du = 2xdx y v = (1/2)ex .
Aplicando la fórmula de integración por partes tenemos:
Z
1 2 x2 2 1 2 1 2
I = x e − xex dx = x2 ex − ex + C.
2 2 2

PROBLEMA 7.117.

R √
Resolver x 1 + xdx.

Solución


Haciendo u = x, dv = 1 + xdx tenemos que du = dx, v = (2/3)(1 +
x)3/2 :
Z
2 3/2 2 2 4
I = x(1 + x) − (1 + x)3/2 dx = x(1 + x)3/2 − (1 + x)5/2 + C.
3 3 3 15

315
PROBLEMA 7.118.

ln(x2 + 2)dx.
R
Resolver

Solución

2xdx
Hacemos u = ln(x2 + 2) y dv = dx, de donde du = 2 y v = x. Por
x +2
tanto:
2x2 dx
Z Z  
2 2 4
I = x ln(x + 2) − = x ln(x + 2) − 2− 2 dx
x2 + 2 x +2
√ √
= x ln(x2 + 2) − 2x + 2 2 arc tg(x/ 2) + C.

(Ver por ejemplo el problema 7.63 para la resolución de la última integral.)

PROBLEMA 7.119.

R
Resolver ln(x + 1/x)dx.

Solución

x2 − 1
Integrando por partes con u = ln(x + 1/x) y dv = dx, du = dx,
x(x2 + 1)
v = x, tenemos:
  Z 2
1 x −1
I = x ln x + − dx
x x2 + 1
  Z  
1 2
= x ln x + − 1− dx = x ln(x + 1/x) − x + 2 arc tg x + C.
x 1 + x2

PROBLEMA 7.120.

R √
Resolver ln(x + x2 − 1)dx.

316
Solución

√ dx
Hacemos u = ln(x + x2 − 1) y dv = dx, con lo que du = √ , v = x.
x2 − 1
Entonces:
Z
p x p p
I = x ln(x + x2 − 1) − √ dx = x ln(x + x2 − 1) − x2 − 1 + C.
x2 − 1

PROBLEMA 7.121.

R
Resolver x ln(x + 1)dx.

Solución

Hacemos u = ln(x + 1) y dv = xdx; du = dx/(x + 1), v = x2 /2. Enton-


ces:

x2 x2 x2
Z Z  
1 1 1
I = ln(x + 1) − dx = ln(x + 1) − x−1+ dx
2 2 x+1 2 2 x+1
x2 1 x2
 
= ln(x + 1) − − x + ln(x + 1) + C
2 2 2
x2 − 1 x2 x
= ln(x + 1) − + + C.
2 4 2

PROBLEMA 7.122.

R
Resolver x sen xdx.

Solución

Para utilizar el método de integración por partes podemos seguir los siguien-
tes caminos:
a) u = x sen x, dv = dx. Entonces du = (sen x+x cos x)dx, v = x. Ası́:
Z
I = x · x sen x − x(sen x + x cos x)dx.

317
La integral que resulta es menos sencilla que la original por lo cual se descarta
este camino.
b) u = sen x, dv = xdx. Por tanto, du = cos xdx, v = x2 /2 y resulta:
Z
1 2 1 2
I = x sen x − x cos xdx.
2 2
La integral que resulta es menos sencilla que la original y también descarta-
mos este camino.
c) u = x, dv = sen xdx. Por tanto du = dx, v = − cos x y resulta:
Z
I = −x cos x − − cos xdx = −x cos x + sen x + C.

PROBLEMA 7.123.

x2 sen xdx.
R
Resolver

Solución

Haciendo u = x2 , dv = sen xdx tendremos du = 2xdx, v = − cos x.


Ası́: Z
2
I = −x cos x + 2 x cos xdx.

Hacemos en la integral resultante u = x, dv = cos xdx, du = dx, v = sen x


y tenemos:
 Z 
I = −x cos x + 2 x sen x − sen xdx = −x2 cos x + 2x sen x + 2 cos x + C.
2

PROBLEMA 7.124.

sec3 xdx.
R
Resolver

318
Solución

Haciendo u = sec x, dv = sec2 xdx tendremos du = sec x tg xdx, v = tg x.


Ası́:
Z Z
I = sec x tg x − sec x tg xdx = sec x tg x − sec x(sec2 x − 1)dx
2

Z Z
3
= sec x tg x − sec xdx + sec xdx.

Por tanto,
Z
2I = sec x tg x + sec xdx = sec x tg x + ln | sec x + tg x| + C;

1
I= (sec x tg x + ln | sec x + tg x|) + C 0 .
2

PROBLEMA 7.125.

R√
Resolver x2 − 36dx.

Solución

Si hacemos el cambio x = 6 sec t, entonces dx = 6 sec t tg tdt y resulta:

Z Z Z Z
2 2 3
I = 36 tg t sec tdt = 36 (sec t − 1) sec tdt = 36 sec tdt − sec tdt.

De acuerdo al problema anterior, resulta:

I = 18(sec t tg t + ln | sec t + tg t|) − 36 ln | sec t + tg t| + C


x √x2 − 36

xp 2
= x − 36 − 18 ln + + C.

2 6 6

PROBLEMA 7.126.

R√
Resolver 3x2 + 5dx.

319
Solución

p √ q √ √
Escribimos el integrando como 5[(3/5)x2 + 1] = 5 (x 3/ 5)2 + 1 y
√ √
hacemos el cambio x 3/ 5 = tg u:
Z
5 5
I = √ sec3 udu = √ (sec u tg u + ln | sec u + tg u|) + C
3 2 3
x√3 √3x2 + 5

1 p 2 5
= x 3x + 5 + √ ln √ + √ + C.

2 2 3 5 5

PROBLEMA 7.127.

R√
Resolver 4x2 − 4x + 5dx.

Solución

Completamos cuadrados en el radicando y procedemos como en los proble-


mas anteriores, haciendo el cambio 2x − 1 = 2 tg u:
Z p Z
I = (2x − 1)2 + 4dx = 2 sec3 udu = sec u tg u + ln | sec u + tg u| + C
2x − 1 √4x2 − 4x + 5

2x − 1 p 2
= 4x − 4x + 5 + ln + + C.

4 2 2

PROBLEMA 7.128.
Z
x
Resolver dx.
cos2 x

Solución

De nuevo integramos por partes con u = x, dv = dx/ cos2 x:


Z Z
I = xd(tg x) = x tg x − tg xdx = x tg x + ln | cos x| + C.

320
PROBLEMA 7.129.

x2
Z
Resolver dx.
(x cos x − sen x)2

Solución

 
1 x sen x
Como d = dx, podemos integrar por par-
x cos x − sen x (x cos x − sen x)2
x sen x
tes con u = x/ sen x, dv = dx:
(x cos x − sen x)2
Z Z
x x sen x 1 x dx
I = · 2
dx = · +
sen x (x cos x − sen x) x cos x − sen x sen x sen2 x
x cos x x sen x + cos x
= − +C = + C.
(x cos x − sen x) sen x sen x x cos x − sen x

PROBLEMA 7.130.

R
Resolver cos x ln(1 + cos x)dx.

Solución

Integramos por partes con u = ln(1 + cos x), dv = cos xdx.


− sen x
Entonces du = , v = sen x. Por tanto:
1 + cos x
sen2 x
Z
I = sen x ln(1 + cos x) + dx
1 + cos x
sen2 x(1 − cos x)
Z
= sen x ln(1 + cos x) + dx
(1 + cos x)(1 − cos x)
Z
= sen x ln(1 + cos x) + (1 − cos x)dx = sen x ln(1 + cos x) + x − sen x + C.

PROBLEMA 7.131.

R
Resolver arc sen xdx.

321
Solución

dx
Haciendo u = arc sen x, dv = dx tenemos que du = √ , v = x.
1 − x2
Ası́: Z
xdx p
I = x arc sen x − √ = x arc sen x + 1 − x2 + C.
1 − x2

PROBLEMA 7.132.

x−1
Z
Resolver arc tg dx.
x+1

Solución

x−1
Hacemos u = arc tg y dv = dx; entonces du = dx/(1 + x2 ) y v = x.
x+1
Ası́:
x−1 x−1 1
Z
x
I = x arc tg − 2
dx = x arc tg − ln(1 + x2 ) + C.
x+1 1+x x+1 2

PROBLEMA 7.133.
Z
x arc sen x
Resolver √ dx.
1 − x2

Solución

xdx
Hacemos u = arc sen x y dv = √ e integramos por partes:
1 − x2
Z p p Z
I = 2 2
arc sen xd(− 1 − x ) = − 1 − x arc sen x + dx
p
= − 1 − x2 arc sen x + x + C.

322
PROBLEMA 7.134.

x3 arc tg xdx.
R
Resolver

Solución

Hacemos en este caso u = arc tg x, dv = x3 dx, con lo que du = dx/(1 + x2 ),


v = x4 /4:

x4 x4 x4
Z Z  
1 1 2 1
I = arc tg x − dx = arc tg x − x −1+ dx
4 4 1 + x2 4 4 1 + x2
x4 − 1 x3 x
= arc tg x − + + C.
4 12 4

PROBLEMA 7.135.

x2
Z
Resolver arc tg xdx.
1 + x2

Solución

Separamos en dos integrales ası́:


Z   Z Z
1 1
I= 1− arc tg xdx = arc tg xdx − arc tg xdx.
1 + x2 1 + x2

En la primera integral hacemos u = arc tg x, dv = dx. Entonces du =


dx/(1 + x2 ), v = x, con lo que
Z
x 1
I1 = x arc tg x − dx = x arc tg x − ln(1 + x2 ) + C1 .
1 + x2 2

Como la segunda integral es inmediata, resulta en definitiva:


p 1
I = x arc tg x − ln 1 + x2 − (arc tg x)2 + C.
2

323
PROBLEMA 7.136.
Z
arc sen x
Resolver √ dx.
(1 − x2 ) 1 − x2

Solución

dx
Si integramos por partes haciendo u = arc sen x, dv = √ , se
(1 − x2 ) 1 − x2
dx x
tiene du = √ ,v=√ . Ası́:
1−x 2 1 − x2
Z
x x dx
I = √ arc sen x − √ ·√
1−x 2 2 1 − x2
Z 1−x
x 1 −2x x 1
= √ arc sen x + 2
dx = √ arc sen x + ln |1 − x2 | + C.
1−x 2 2 1−x 1−x2 2

PROBLEMA 7.137.

R
Resolver x arc sen xdx.

Solución

Integramos por partes con u = arc sen x y dv = xdx:

x2 x2
Z
1
I= arc sen x − √ dx.
2 2 1 − x2
x2
Z Z Z p
1
Ahora bien, √ dx = √ dx − 1 − x2 dx.
1 − x2 1 − x2

Haciendo en esta última integral u = 1 − x2 y dv = dx, tenemos:

x2
Z p p Z
2
1 − x dx = x 1 − x + √ 2 dx.
1 − x2
x2
Z
1 p
Resulta entonces que √ dx = [arc sen x − x 1 − x2 ] + C. Por tan-
1 − x2 2
to:
2x2 − 1 xp
I= arc sen x + 1 − x2 + C.
4 4

324
PROBLEMA 7.138.
Z
1 + sen x x
Resolver e dx.
1 + cos x

Solución

1 + sen x 1
Recordamos la fórmula = [1 + tg(x/2)]2 y tenemos:
1 + cos x 2
Z Z Z
1 2 x 1
I= [1+tg (x/2)+2 tg(x/2)]e dx = sec (x/2)e dx+ ex tg(x/2)dx.
2 x
2 2
En la primera integral hacemos u = ex , dv = sec2 (x/2)dx, con lo que du =
ex dx, v = 2 tg(x/2):
Z Z
I = e tg(x/2) − e tg(x/2)dx + ex tg(x/2)dx = ex tg(x/2) + C.
x x

PROBLEMA 7.139.

eax sen bxdx, J = eax cos bxdx.


R R
Resolver las integrales I =

Solución

Integrando por partes cada una de ellas, resulta:


Z
1 ax b 1 b
I = e sen bx − eax cos bxdx = eax sen bx − J.
a a a a
Z
1 ax b 1 b
J = e cos bx + eax sen bxdx = eax cos bx + I.
a a a a
Basta pues resolver el sistema aI + bJ = e sen bx, bI − aJ = −eax cos bx,
ax

para obtener los valores de I y J. En definitiva,

eax
I = (a sen bx − b cos bx) + C;
a2 + b2
eax
J = (b sen bx + a cos bx) + C 0 .
a2 + b2

325
PROBLEMA 7.140.

xearc sen x
Z Z
Resolver I = √ dx y J = earc sen x dx.
1 − x2

Solución

Vamos a integrar I por partes siguiendo dos caminos distintos:


xdx
En primer lugar hacemos u = earc sen x , dv = √ y tenemos:
1 − x2
p Z p
arc sen x
2
I =− 1−x e + earc sen x dx = − 1 − x2 earc sen x + J.

earc sen x
En segundo lugar hacemos u = x, dv = √ dx y tenemos:
1 − x2
Z
arc sen x
I = xe − earc sen x dx = xearc sen x − J.

Sumando y restando ordenadamente las dos fórmulas obtenidas llegamos


a:
1 p
I = (x − 1 − x2 )earc sen x + C;
2
1 p
J = (x + 1 − x2 )earc sen x + C.
2

PROBLEMA 7.141.

earc tg x
Z
Resolver √ dx.
(1 + x2 ) 1 + x2

Solución

dx
Integramos por partes haciendo u = earc tg x , dv = √ . Ası́ re-
(1 + x2 ) 1 + x2
sulta:

326
xearc tg x xearc tg x
Z
I=√ − √ dx.
1 + x2 (1 + x2 ) 1 + x2
En esta última integral, que también resolvemos por partes, hacemos u = earc tg x ,
xdx
dv = √ y se tiene
(1 + x ) 1 + x2
2

xearc tg x earc tg x
Z Z
1
√ dx = − √ earc tg x + √ dx.
(1 + x2 ) 1 + x2 1 + x2 (1 + x2 ) 1 + x2

De aquı́ se deduce inmediatamente:


x + 1 arc tg x
I= √ e + C.
2 1 + x2
Z
xdx
Observación: La integral √ es inmediata pues es igual
(1 + x ) 1 + x2
2
a

1 (1 + x2 )−3/2+1
Z
1 1
(1+x2 )−3/2 d(1+x2 ) = · = −(1+x2 )−1/2 = − √ .
2 2 −3/2 + 1 1 + x2

PROBLEMA 7.142.
Z
1
Resolver In = dx.
(1 + x2 )n+1

Solución

Utilizamos el siguiente artificio:

1 + x2 − x2 1 + x2 x2
Z Z Z
In = dx = dx − dx
(1 + x2 )n+1 (1 + x2 )n+1 (1 + x2 )n+1
x2 dx x2 dx
Z Z Z
dx
= − = In−1 − .
(1 + x2 )n (1 + x2 )n+1 (1 + x2 )n+1

Para la última integral utilizamos el método de integración por partes. Para


xdx −1
ello hacemos u = x, dv = 2 n+1
, con lo que du = dx, v = .
(1 + x ) 2n(1 + x2 )n
Ası́:
x2 dx −x
Z
1
2 n+1
= 2 n
+ In−1 .
(1 + x ) 2n(1 + x ) 2n

327
En definitiva,
x 1 2n − 1 x
In = In−1 + 2 n
− In−1 = In−1 + .
2n(1 + x ) 2n 2n 2n(1 + x2 )n

D. INTEGRACIÓN POR FRACCIONES SIMPLES.

Este método es exclusivo para integrar funciones racionales. El procedimien-


to general es el siguiente:
Z
p(x)
Para calcular dx donde p y q son polinomios, realizaremos los si-
q(x)
guientes pasos:
p(x) r(x)
1) Se realiza la división = c(x) + donde c es el cociente y r el
q(x) q(x)
resto, con grado r < grado q. Entonces:
Z Z Z
p(x) r(x)
dx = c(x)dx + dx.
q(x) q(x)
La primera integral es inmediata y a continuación estudiaremos la segun-
da.
Observación: Si el grado de p ya es menor que el grado de q, este paso se
omite, pues p(x) = r(x).
2) Se factoriza el denominador a partir de sus raı́ces (ya sean reales o com-
plejas). Tenemos:
q(x) = a(x − r1 )m1 · · · · · (x − rn )mn (x2 + a1 x + b1 )q1 · · · · · (x2 + ap x + bp )qp .
Nota: En lo que sigue supondremos que a = 1 pues, en caso contrario, puede
salir de la integral como una constante.
3) Se descompone el integrando en fracciones simples:
r(x) A1 Am1 K1 Kmn
= + ··· + m
+ · · · + + · · · +
q(x) x − r1 (x − r1 ) 1 x − rn (x − rn )mn
α1 x + β1 αq x + βq1
+ 2 + ··· + 2 1 + ...
x + a1 x + b1 (x + a1 x + b1 )q1
σ 1 x + τ1 σq x + βqp
+ 2 + ··· + 2 p .
x + ap x + bp (x + ap x + bp )qp

328
4) Se calculan las constantes A1 , . . . , Am1 , . . . , K1 , . . . , Kmn , α1 , β1 , . . . , αq1 , βq1 ,
. . . , σ1 , τ1 , . . . , σqp , τqp igualando los numeradores de ambos miembros.
5) Se integra por separado cada fracción simple.
Los siguientes problemas ilustran la forma de integrar según sea la descom-
posición de la fracción.

PROBLEMA 7.143.
Z
1
Resolver dx.
x2 −4

Solución

Como x2 − 4 = (x − 2)(x + 2), podemos descomponer la fracción como

1 A B
= + ,
x2 −4 x−2 x+2
de donde 1 = A(x + 2) + B(x − 2).
Para x = 2, 1 = 4A =⇒ A = 1/4. Para x = −2, 1 = −4B =⇒ B =
−1/4.
Tenemos entonces:

1 x − 2
Z Z
1 dx 1 dx 1 1
I= − = ln |x−2|− ln |x+2|+C = ln +C.
4 x−2 4 x+2 4 4 4 x + 2

PROBLEMA 7.144.
Z
1
Resolver dx.
9x2 − 16

Solución

Procediendo como el anterior, tenemos:


1 A B
= + ,
9x2 − 16 3x − 4 3x + 4

329
de donde 1 = A(3x+4)+B(3x−4) =⇒ A = 1/8, B = −1/8. Entonces:
Z Z
1 1 1 1
I = dx − dx
8 3x − 4 8 3x + 4

1 1 1 3x − 4
= ln |3x − 4| − ln |3x + 4| + C = ln + C.
24 24 24 3x + 4

PROBLEMA 7.145.
Z
1
Resolver dx.
x2 + 6x + 8

Solución

Como x2 + 6x + 8 = (x + 2)(x + 4), tenemos:

1 A B
= + ,
x2
+ 6x + 8 x+2 x+4
de donde 1 = A(x + 4) + B(x + 2) =⇒ A = 1/2 y B = −1/2. Resulta
entonces:

1 1 x + 2
I = (ln |x + 2| − ln |x + 4|) + C = ln + C.
2 2 x + 4

PROBLEMA 7.146.
Z
1
Resolver dx.
9 − x2

Solución

Como 9 − x2 = (3 − x)(3 + x), resulta:


1 A B
2
= + =⇒ 1 = A(3 + x) + B(3 − x).
9−x 3−x 3+x
Obtenemos los valores A = B = 1/6, con lo que:

1 1 3 + x
I = (− ln |3 − x| + ln |3 + x|) + C = ln + C.
6 6 3 − x

330
PROBLEMA 7.147.
Z
1
Resolver dx.
4x − x2

Solución

Como 4x − x2 = x(4 − x), tenemos:


1 A B
= + =⇒ 1 = A(4 − x) + Bx.
4x − x2 x 4−x
De aquı́, A = B = 1/4; por tanto:

1 1 x
I = (ln |x| − ln |4 − x|) + C = ln + C.
4 4 4 − x

PROBLEMA 7.148.

2−x
Z
Resolver dx.
4x2 + 4x − 3

Solución

Al factorizar el denominador tenemos 4x2 +4x−3 = (2x−1)(2x+3) y:


2−x A B
= + =⇒ 2 − x = A(2x + 3) + B(2x − 1).
4x2 + 4x − 3 2x − 1 2x + 3
De aquı́ se obtiene que A = 3/8, B = −7/8. Entonces:
(2x − 1)3

3 7 1
I= ln |2x − 1| − ln |2x + 3| + C = ln + C.
16 16 16 (2x + 3)7

PROBLEMA 7.149.
Z
x+1
Resolver dx.
x3 + x2 − 6x

331
Solución

Factorizamos en primer lugar el denominador: x3 +x2 −6x = x(x−2)(x+3).


Por tanto:
x+1 A B C
= + + ⇒ x+1 = A(x−2)(x+3)+Bx(x+3)+Cx(x−2)
x3 + x2 − 6x x x−2 x+3
Para x = 0, 1 = −6A =⇒ A = −1/6.
Para x = 2, 3 = 10B =⇒ B = 3/10.
Para x = −3, −2 = 15C =⇒ C = −2/15.
La integral queda entonces:
Z Z Z
1 dx 3 dx 2 dx
I = − + −
6 x 10 x − 2 15 x+3
1 3 2 |x − 2|3/10
= − ln |x| + ln |x − 2| − ln |x + 3| + C = ln 1/6 + C.
6 10 15 |x| |x + 3|2/15

PROBLEMA 7.150.
Z
3x + 5
Resolver dx.
x3 − x2 − x + 1

Solución

Al factorizar el denominador tenemos x3 − x2 − x + 1 = (x + 1)(x − 1)2 .


Ası́:
3x + 5 A B C
= + +
x3 − x2 − x + 1 x + 1 x − 1 (x − 1)2
con lo que 3x + 5 = A(x − 1)2 + B(x − 1)(x + 1) + C(x + 1).
Para x = −1, 2 = 4A =⇒ A = 1/2.
Para x = 1, 8 = 2C =⇒ C = 4.
Para determinar la constante B se sustituye otro valor de x, por ejemplo
x = 0. Resulta 5 = A − B + C =⇒ B = −1/2. Por tanto:
Z Z Z
1 dx 1 dx dx
I = − +4
2 x+1 2 x−1 (x − 1)2

1 1 4 4 1 x + 1
= ln |x + 1| − ln |x − 1| − +C =− + ln + C.
2 2 x−1 x − 1 2 x − 1

332
PROBLEMA 7.151.

x4 − x3 − x − 1
Z
Resolver dx.
x3 − x2

Solución

En primer lugar se realiza la división y resulta

x4 − x3 − x − 1 x+1 x+1
3 2
=x− 3 2
=x− 2 .
x −x x −x x (x − 1)
Después se descompone la fracción resultante en fracciones simples:
x+1 A B C
= + 2+ =⇒ x + 1 = Ax(x − 1) + B(x − 1) + Cx2 .
x2 (x
− 1) x x x−1
Para x = 0, 1 = −B =⇒ B = −1.
Para x = 1, 2 = C.
Para x = 2, 3 = 2A + B + 4C =⇒ A = −2.
Por tanto:
Z Z Z Z
dx dx dx
I = xdx + 2 + 2
−2
x x x−1

1 2 1 1 2 1 x
= x + 2 ln |x| − − 2 ln |x − 1| + C = x − + 2 ln + C.
2 x 2 x x − 1

PROBLEMA 7.152.

x2
Z
Resolver dx.
a4 − x4

Solución

Descomponemos el integrando en fracciones simples:

x2 A B Cx + D
= + + 2 .
a4 − x4 a−x a+x a + x2

333
Por tanto, x2 = A(a + x)(a2 + x2 ) + B(a − x)(a2 + x2 ) + (Cx + D)(a − x)(a +
x).
Para x = a, a2 = 4Aa3 y A = 1/4a.
Para x = −a, a2 = 4Ba3 y B = 1/4a.
Para x = 0, 0 = Aa3 + Ba3 + Da2 = a2 /2 + Da2 y D = −1/2.
Para x = 2a, 4a2 = 15Aa3 − 5Ba3 − 6Ca3 − 3Da2 y C = 0. Ası́ pues:
Z Z Z
1 dx 1 dx 1 dx
I = + −
4a a − x 4a a+x 2 a + x2
2

1 1 1
= − ln |a − x| + ln |a + x| − arc tg(x/a) + C.
4a 4a 2a

PROBLEMA 7.153.

2x2 + 3
Z
Resolver dx.
(x2 + 1)2

Solución

Si descomponemos el integrando, tenemos

2x2 + 3 Ax + B Cx + D
2 2
= 2 + 2 .
(x + 1) x +1 (x + 1)2
Por tanto:

2x2 + 3 = (Ax + B)(x2 + 1) + Cx + D = Ax3 + Bx2 + (A + C)x + (B + D).

Igualando términos del mismo grado resulta A = 0, B = 2, A + C = 0,


B + D = 3, lo que al resolver queda A = 0, B = 2, C = 0, D = 1.
Entonces, Z Z
2dx dx
I= + .
x2 + 1 (x2 + 1)2
Para la segunda integral hacemos el cambio x = tg z con lo cual

sec2 z
Z Z Z
dx
= dz = cos2 zdz = z/2 + (1/4) sen 2z + C,
(x2 + 1)2 sec4 z
de donde
x/2 x/2
I = 2 arc tg x + (1/2) arc tg x + + C = (5/2) arc tg x + 2 + C.
x2 + 1 x +1

334
PROBLEMA 7.154.

x3 + x2 + x + 2
Z
Resolver dx.
x4 + 3x2 + 2

Solución

Debido a que x4 + 3x2 + 2 = (x2 + 1)(x2 + 2) tenemos

x3 + x2 + x + 2 Ax + B Cx + D
= 2 + 2 ,
x4 + 3x2 + 2 x +1 x +2
de donde:

x3 + x2 + x + 2 = (Ax + B)(x2 + 2) + (Cx + D)(x2 + 1)


= (A + C)x3 + (B + D)x2 + (2A + C)x + (2B + D),

luego A + C = 1, B + D = 1, 2A + C = 1, 2B + D = 2.
Resolviendo el sistema resulta A = 0, B = 1, C = 1, D = 0. Por tanto:
Z Z
dx xdx 1
I= 2
+ 2
= arc tg x + ln(x2 + 2) + C.
x +1 x +2 2

PROBLEMA 7.155.

x5 − x4 + 4x3 − 4x2 + 8x − 4
Z
Resolver dx.
(x2 + 2)3

Solución

Tenemos
x5 − x4 + 4x3 − 4x2 + 8x − 4 Ax + B Cx + D Ex + F
2 3
= 2 + 2 2
+ 2 ,
(x + 2) x +2 (x + 2) (x + 2)3

de donde: x5 − x4 + 4x3 − 4x2 + 8x − 4 = (Ax + B)(x2 + 2)2 + (Cx + D)(x2 +


2) + Ex + F = Ax5 + Bx4 + (4A + C)x3 + (4B + D)x2 + (4A + 2C + E)x +
(4B + 2D + F )

335
y se obtiene A = 1, B = −1, C = 0, D = 0, E = 4, F = 0.
La integral queda entonces:

x−1
Z Z
x 1 2 2 x 1
I= dx+4 dx = ln(x +2)− arc tg √ − 2 +C.
x2 + 2 2
(x + 2)3 2 2 2 (x + 2)2

E. APLICACIONES DE LA INTEGRAL INDEFINIDA.

Es común tener que resolver problemas en donde se trata de encontrar una


función conocida una expresión que involucra a alguna de sus derivadas.
Estos problemas reciben el nombre de ecuaciones diferenciales y se resuelven
mediante el proceso de integración.
Ahora bien, como hay muchas funciones que tienen la misma derivada, para
encontrar una de ellas se necesita una condición adicional. Generalmente,
esa condición consiste en proporcionar un punto por donde pasa la función.

PROBLEMA 7.156.

dy
Encontrar una función cuya derivada es = 3x2 y que pase por
dx
el punto (1, 0).

Solución

La función que buscamos debe ser primitiva de la función dada. De este


modo Z
y = 3x2 dx = x3 + C.

Si sustituimos el punto dado, resulta que 0 = 13 + C. Por tanto, C =


−1.
La función que cumple las condiciones del problema es f (x) = x2 − 1.

336
PROBLEMA 7.157.

x4 + 1
Encontrar una función f sabiendo que f 00 (x) = .
x3

Solución

En este caso debemos realizar dos integraciones: la primera para determinar


f 0 y la segunda para obtener f. Deben aparecer por lo tanto dos constan-
tes arbitrarias, una por cada integral. Dichas constantes no tienen que ser
necesariamente las mismas.

x2 x−2 x2 x−2
Z 4 Z
0 x +1 1
f (x) = dx = (x + )dx = + = − + C1 .
x3 x3 2 −2 2 2

x2 x−2 x3 x−1
Z  
f (x) = − + C1 dx = + + C1 x + C2 .
2 2 6 2
Un caso común de esta situación consiste en encontrar la posición en cada

instante de un punto que se mueve en lı́nea recta conocida la velocidad o la


aceleración del mismo.

PROBLEMA 7.158.

La aceleración de una partı́cula que se mueve a lo largo de una


recta es
a(t) = π 2 cos πt m2 /sg.
Si en el instante inicial (t = 0), la posición de la partı́cula es s = 0
y la velocidad es v = 8, hallar s cuando t = 1.

Solución

Mediante integración directa de la aceleración con respecto al tiempo, se


obtiene la velocidad en cualquier instante t:
Z Z
v(t) = π 2 cos πtdt = π π cos πtdt = π sen πt + C.

337
Para determinar la constante C, evaluamos la función en el punto t =
0:
v(0) = 8 = π sen π0 + C = C; ası́ pues, v(t) = π sen πt + 8.
Integrando de nuevo respecto a t, de la velocidad se obtiene la posición
s(t) : Z
s(t) = (π sen πt + 8)dt = − cos πt + 8t + K.

La constante K se obtiene conocida la posición en el instante t = 0:

s(0) = 0 = − cos π0 + 8 · 0 + K = −1 + K, de donde K = 1.

En definitiva, s(t) = − cos πt + 8t + 1. Como la pregunta que se plantea es


calcular s(1), al sustituir resulta s(1) = − cos π + 8 + 1 = 10 metros. Existen

ecuaciones diferenciales más generales en donde no es posible despejar la


derivada de la función con respecto a la variable independiente. Un caso
simple de resolver y el único que estudiaremos aquı́ es aquél en donde se
pueden separar en ambos miembros de la ecuación las variables x e y. El
siguiente ejemplo ilustra el procedimiento a seguir.

PROBLEMA 7.159.

dy √
Resolver la ecuación = y sen 2x sabiendo que y(3π/4) = 1
dx

Solución

En primer lugar intentamos que las variables x e y estén separadas cada una
en un miembro de la ecuación. Para ello tratamos a la derivada dy/dx como
un cociente de diferenciales y podemos escribir
dy
√ = sen 2xdx.
y

A continuación se integran ambos miembros de la ecuación. El primero res-


pecto de la variable y y el segundo respecto de la variable x. Es decir escri-
bimos: Z Z
dy
√ = sen 2xdx,
y
de modo que
y 1/2
= −(1/2) cos 2x + C.
1/2

338
(La constante se añade a uno de los miembros de la ecuación, pues representa
la diferencia entre las funciones que tienen la misma derivada). El valor de
C se calcula mediante la condición inicial y(3π/4) = 1.

11/2
= −(1/2) cos(2 · 3π/4) + C =⇒ 2 = C.
1/2

Despejando el valor de y llegamos a:

y 1/2 = (1/2)(−(1/2) cos 2x+2) = −(1/4) cos 2x+1 =⇒ y = [−(1/4) cos 2x+1]2 .

En general no se puede obtener la ecuación en su forma explı́cita. La ecuación


implı́cita será suficiente para definir la función.

339
F. EJERCICIOS PROPUESTOS.

Resolver las siguientes integrales indefinidas:

4x2 + 7
Z
1.- dx.
x2
Resp.: 4x − (7/x) + C.

(4x − 1)43 dx.


R
2.-
Resp.: (4x − 1)44 /176 + C.

x(ax3 + b)2 dx.


R
3.-
Resp.: a2 x8 /8 + 2abx5 /5 + b2 x2 /2 + C.

Z
4x
4.- √ dx.
1 − 2x2

Resp.: −2 1 − 2x2 + C.

R
5.- sen t cos t(sen t + cos t)dt.
Resp.: 31 (sen3 t − cos3 t) + C.

Z
cos x
6.- dx.
sen2 x
Resp.: − cosec x + C.

Z
4
7.- √ dx.
1 − 4x2
Resp.: 2 arc sen 2x + C.

Z
x
8.- √ dx.
x+1
Resp.: 32 (x + 1)3/2 − 2(x + 1)1/2 + C.
Sug.: Sumar y restar 1 al numerador. Después separar en dos integrales.

340
R √
3
9.- 8x7 dx.
Resp.: 3x10/3 /5 + C.

Z
1
10.- √ √ dx.
x(1 + x)2
−2
Resp.: √ + C.
1+ x

sen t + cos2 t
Z
11.- √ dt.
cos2 t t + sec t

Resp.: 2 t + sec t + C.

(2x2 − 5x + 3)dx.
R
12.-
Resp.: 2x3 /3 − 5x2 /2 + 3x + C.

Z
1
13.- dx.
1 − x2

1 1 + x
Resp.: ln + C.
2 1 − x

Z
1
14.- dx.
x2 − 4

1 x − 2
Resp.: ln + C.
4 x + 2

Z
1
15.- dy .
25 − 16y 2

1 5 + 4y
Resp.: ln + C.
40 5 − 4y

(4x3 + 3x2 + 2x + 5)dx.


R
16.-
Resp.: x4 + x3 + x2 + 5x + C.

(3 − 2x − x4 )dx.
R
17.-
Resp.: 3x − x2 − x5 /5 + C.

341
x3 + 5x2 − 4
Z
18.- dx.
x2
Resp.: x2 /2 + 5x + 4/x + C.

(x2 − 1)2 dx.


R
19.-
Resp.: x5 /5 + x − 2x3 /3 + C.
Z
1
20.- √ dx.
4 − x2
Resp.: arc sen(x/2) + C.
Z
1
21.- p dx.
4 − (x + 2)2
x+2
Resp.: arc sen + C.
2
Z
x+2
22.- √ dx.
4x − x2
p x−2
Resp.: − 4x − x2 + 4 arc sen + C.
2
Z
2x + 3
23.- dx.
9x2 − 12x + 8
1 13 3x − 2
Resp.: ln(9x2 − 12x + 8) + arc tg + C.
9 18 2

cos3 xdx.
R
24.-
Resp.: sen x − (1/3) sen3 x + C.

sen3 x cos5 xdx.


R
25.-
Resp.: (1/8) cos8 x − (1/6) cos6 x + C.

tg2 x sec3 xdx.


R
26.-
Resp.: (1/4) sec3 x tg x − (1/8) sec x tg x − (1/8) ln | sec x + tg x| + C.

cotg 3x cosec4 3xdx.


R
27.-
Resp.: −(1/6) cotg2 3x − (1/12) cotg4 3x + C.

342
sen2 λxdx.
R
28.-
Resp.: (x/2) − (1/4λ) sen 2λx + C.

cos2 3xdx.
R
29.-
Resp.: x/2 + (1/12) sen 6x + C.

cos3 (x/3)dx.
R
30.-
Resp.: 3 sen(x/3) − sen3 (x/3) + C.

R
31.- sen 3x sen 2xdx.
Resp.: (1/2) sen x − (1/10) sen 5x + C.

R
32.- sen 3x cos 5xdx.
Resp.: (1/4) cos 2x − (1/16) cos 8x + C.

R
33.- cos 4x cos 2xdx.
Resp.: (1/4) sen 2x + (1/12) sen 6x + C.

R√
34.- 25 − x2 dx.

Resp.: (1/2)x 25 − x2 + (25/2) arc sen(x/5) + C.
Z
1
35.- √ dx.
x 9 + 4x2

1 9 + 4x2 − 3
Resp.: ln + C.
3 x

x2 ln xdx.
R
36.-
Resp.: (1/3)x3 ln x − (1/9)x3 + C.

x tg2 xdx.
R
37.-
Resp.: x tg x + ln | cos x| − x2 /2 + C.

x2 arc tg xdx.
R
38.-
Resp.: (1/3)x3 arc tg x − x2 /6 + (1/6) ln(1 + x2 ) + C.

343
Z
1
39.- dx.
x2−1

1 x − 1
Resp.: ln + C.
2 x + 1

2x − 3
Z
40.- dx.
4x2 − 11
√ √
1 2 3 11 2x − 11
Resp.: ln |4x − 11| − ln √ + C.
4 44 2x + 11

1
41.- Sea y = f (x) una función cuya derivada es f 0 (x) = . Cal-
(x − 1)2
cular f (4) si f (2) = −1.
Resp.: f (x) = −1/(x − 1) + C; f (4) = −1/3.

42.- Definir y representar gráficamente una función y = f (x) que


verifique
f 00 (x) = 2, f 0 (1) = 2, f (3) = 5.
Resp.: f (x) = x2 − 4.

43.- Encontrar la velocidad y la posición de una partı́cula en cual-


quier instante t si ésta se mueve en lı́nea recta con una acelera-
ción dada por a(t) = 3t − t2 si además la velocidad y la posición
en el instante t = 1 sg. son v = 7/6 m/sg. y s = 1 m.
Resp.: v(t) = 3t2 /2 − t3 /3 m/sg.; s(t) = t3 /2 − t4 /12 + 7/12 m.

44.- Responder a las mismas preguntas del ejercicio anterior si


a(t) = 18 sen 3t; v = −6 y s = 4 cuando t = 0.
Resp.: v(t) = −6 cos 3t; s(t) = 4 − 2 sen 3t.

p dy
45.- Resolver la ecuación y 4 − x2 = 3x sabiendo que y(0) = −2.
dx
p
Resp.: y = − −6(4 − x2 )1/2 + 16.

46.- Determinar
√ la curva cuya pendiente en cada punto (x, f (x)) es
x 1 + x2 y que pase por el punto (0, −3).
(1 + x2 )3/2 − 10
Resp.: f (x) = .
3

344
47.- Hallar la ecuación de la curva que pasa por el punto (4, 2) y cuya
pendiente en cada punto es x/y .
Resp.: x2 − y 2 = 12.

48.- Sea y = f (x) una función cuya derivada está dibujada a conti-
nuación.

Si f (x) es continua en −2 y f (−2) = 0, definir f y representarla.


( 2
x
+x si − 3 ≤ x ≤ −2,
Resp.: f (x) = 2
−x − 2 si − 2 < x ≤ 1.

345
CAPÍTULO VIII.
CONVERGENCIA DE
SUCESIONES

SECCIONES
A. Criterios de convergencia.
B. Ejercicios propuestos.

347
A. CRITERIOS DE CONVERGENCIA.

Una función cuyo dominio es el conjunto de los números naturales se dice


sucesión.

Si f : N → R es una sucesión y f (n) = an , n ∈ N, representamos la sucesión


por {an }n∈N o simplemente {an } y an se llama término general (o n-ésimo)
de la sucesión.

Una sucesión {an } es convergente cuando existe y es finito lı́m an . Si dicho


n→∞
lı́mite es infinito, la sucesión es divergente, y si no existe, la sucesión es
oscilante.

Las propiedades de los lı́mites de funciones se aplican a sucesiones en forma


directa. Por tanto, para estudiar la convergencia de una sucesión son válidos
los mismos métodos utilizados en el cálculo de lı́mites de funciones. También
se pueden aplicar las equivalencias entre infinitésimos nombradas allı́ (ver
capı́tulo 3). También es válida aquı́ la fórmula lı́m (1 + un )1/un = e, cuando
n→∞
un → 0.

Sin embargo existen otros criterios especı́ficos para las sucesiones que enun-
ciamos a continuación:

1) Media aritmética: Si lı́m an = a, entonces


n→∞

a1 + · · · + an
lı́m = a.
n→∞ n
2) Media geométrica: Si lı́m an = a y an > 0, ∀n, entonces
n→∞

lı́m n a1 · · · · · an = a.
n→∞
an √
3) Cociente-Raı́z: Si an > 0, ∀n y lı́m = L, entonces lı́m n an = L.
n→∞ an−1 n→∞

4) Stolz: Si {an } es una sucesión arbitraria, {bn } es una sucesión creciente


an − an−1 an
tal que lı́m bn = +∞ y lı́m = L, entonces lı́m = L.
n→∞ n→∞ bn − bn−1 n→∞ bn

Aparte de estos criterios, es útil la fórmula de Stirling:


n! √
lı́m √ = 1, es decir, n! y nn e−n 2πn son infinitos equivalentes.
n→∞ nn e−n 2πn

En los problemas que siguen se desarrollan distintos métodos para los dife-
rentes casos de indeterminación en el cálculo de lı́mites de sucesiones.

348
PROBLEMA 8.1.

Calcular el lı́mite de la sucesión de término general


√ √
an = n2 + 4n − n2 − n.

Solución

Multiplicamos y dividimos por el conjugado y se obtiene:


√ √ √ √
( n2 + 4n − n2 − n)( n2 + 4n + n2 − n)
L = lı́m √ √
n→∞ n2 + 4n + n2 − n
5n 5 5
= lı́m √ √ = lı́m p p = .
n→∞ n2 + 4n + n2 − n n→∞ 1 + 4/n + 1 − 1/n 2

PROBLEMA 8.2.

Calcular el lı́mite de la sucesión de término general



an = n2 + n + 1 − n.

Solución

Como tenemos una indeterminación ∞ − ∞, multiplicamos y dividimos por


el conjugado:
p √
L = lı́m ( n2 + n + 1 − n2 )
n→∞
√ √ √ √
( n2 + n + 1 − n2 )( n2 + n + 1 + n2 )
= lı́m √ √
n→∞ n2 + n + 1 + n2
n+1 1 + 1/n 1
= lı́m √ √ = lı́m p √ = .
n→∞ 2
n +n+1+ n 2 n→∞ 2
1 + 1/n + 1/n + 1 2

PROBLEMA 8.3.

Calcular el lı́mite de la sucesión de término general


p √ √
an = n2 + n4 + 1 − 2n.

349
Solución

Multiplicamos y dividimos dos veces por el conjugado y obtenemos:


q p √ 
L = lı́m n2 + n4 + 1 − 2n2
n→∞
p √ √  p √ √ 
n2 + n4 + 1 − 2n2 n2 + n4 + 1 + 2n2
= lı́m p √ √
n→∞
n2 + n4 + 1 + 2n2
√ √
n4 + 1 − n4
= lı́m p √ √
n→∞
n2 + n4 + 1 + 2n2
n4 + 1 − n4
= lı́m p √ √  √ √  = 0.
n→∞
n2 + n4 + 1 + 2n2 n4 + 1 + n4

PROBLEMA 8.4.

Calcular el lı́mite de la sucesión de término general


√ √
an = n2 + n + 1 − 3n2 − 1 − 3n.

Solución

Debido a la indeterminación ∞ − ∞, procedemos ası́:


p p n2 + n + 1 − 3n2 + 1
L = lı́m ( n2 + n + 1 − 3n2 − 1) − lı́m 3n = lı́m √ √
n→∞ n→∞ n→∞ n2 + n + 1 + 3n2 − 1
−2n2 + . . .
− lı́m 3n = lı́m √ √ − ∞ = −∞.
n→∞ n→∞ n2 + . . . + 3n2 + . . .

PROBLEMA 8.5.

Calcular el lı́mite de la sucesión de término general


√ √
an = 3 n3 + 2n2 − 3 n3 − n.

350
Solución

Teniendo en cuenta la factorización a3 − b3 = (a − b)(a2 + ab + b2 ), si


llamamos
√ √
a = 3 n3 + 2n2 y b = 3 n3 − n, resulta:
a3 − b3
L = lı́m (a − b) = lı́m
n→∞ n→∞ a2 + ab + b2
n3 + 2n2 − n3 + n
= lı́m
n→∞ (n3 + 2n2 )2/3 + (n3 + 2n2 )1/3 (n3 − n)1/3 + (n3 − n)2/3

2n2 + n
= lı́m √ √ √
n→∞ 3 n6 + . . . + 3 n6 + . . . + 3 n6 + . . .
2 + 1/n 2
= lı́m √
3

3

3
= .
n→∞ 1 + ... + 1 + ... + 1 + ... 3

PROBLEMA 8.6.

Calcular el lı́mite de la sucesión de término general


√ √
an = 3 n9 + 2n − n6 − 7n3 .

Solución

p p
Teniendo en cuenta que L = lı́m 6 (n9 + 2n)2 − 6 (n6 − 7n3 )3 , si llamamos
p p n→∞
a = 6 (n9 + 2n)2 , b = 6 (n6 − 7n3 )3 , aplicamos la fórmula
a6 − b6
a−b=
a5 + a4 b + a3 b2 + a2 b3 + ab4 + b5
y obtenemos
(n9 + 2n)2 − (n6 − 7n3 )3
L = lı́m (a − b) = lı́m p p
n→∞ n→∞ 6 (n9 + 2n)10 + · · · + 6 (n6 − 7n3 )15

4n10 + 4n2 − (−21n15 + 147n12 − 73 n9 ) 21 7


= lı́m √
6

6
= = .
n→∞ 90 90
n + ... + ··· + n + ... 6 2

PROBLEMA 8.7.

Calcular el lı́mite de la sucesión de término general


p
3
p3
an = n3 + 5 + 8n3 + 4n2 − 3n.

351
Solución

En primer lugar, transformamos la indeterminación ∞ − ∞ en 0 · ∞:


hp p i
L = lı́m 3 n3 (1 + 5/n3 ) + 3 8n3 (1 + 4n2 /8n3 ) − 3n
n→∞
h i
= lı́m n(1 + 5/n3 )1/3 + 2n(1 + 1/2n)1/3 − 3n
n→∞
h i
= lı́m n (1 + 5/n3 )1/3 + 2(1 + 1/2n)1/3 − 3 .
n→∞

Aplicamos ahora la fórmula de Newton:


1/3
1/3 52
    
5 1/3 5
1+ = 1+ + + ...
n3 1 n3 2 n6
1 5 1/3(1/3 − 1) 52 5 25
= 1+ · 3 + · 6 + ··· = 1 + 3 − 6 + ...
3 n 2! n 3n 9n
 1/3    
1 1/3 1 1/3 1
1+ = 1+ + + ...
2n 1 2n 2 2 n2
2

1 1 1/3(1/3 − 1) 1 1 1
= 1+ · + · 2 2 + ··· = 1 + − + ...
3 2n 2! 2 n 6n 36n2
Sustituyendo en la última expresión de L:
 
5 25 1 1
L = lı́m n 1 + 3 − 6 + · · · + 2 + − + · · · − 3
n→∞ 3n 9n 3n 18n2
   
1 1 1 1 1
= lı́m n − 2
+ . . . = limn→∞ − + ... = .
n→∞ 3n 18n 3 18n 3

PROBLEMA 8.8.

Calcular el lı́mite de la sucesión de término general


p
a
p
a
an = na + na−1 − na − na−1 , a ∈ N.

Solución

En primer lugar sacamos n factor común en cada sumando:


s  s 
a 1 a 1
L = lı́m n 1+ −n 1− .
n→∞ n n

352
Desarrollamos ahora cada término en serie de potencias:

1 1/a
       
1/a 1 1/a 1 1/a 1
1+ = 1+ + + + ...
n 1 n 2 n2 3 n3
1 1 1/a(1/a − 1) 1 1/a(1/a − 1)(1/a − 2) 1
= 1+ · + · 2+ · 3 + ...
a n 2! n 3! n
1 1−a (1 − a)(1 − 2a)
= 1+ + + + ...
an 2a2 n2 6a3 n3
 1/a      
1 1/a 1 1/a 1 1/a 1
1− = 1− + 2
− + ...
n 1 n 2 n 3 n3
1 1−a (1 − a)(1 − 2a)
= 1− + 2 2− + ...
an 2a n 6a3 n3
Sustituyendo ahora en la última expresión del lı́mite, resulta:
 
2 (1 − a)(1 − 2a)
L = lı́m n + + ...
n→∞ an 3a3 n3
 
2 (1 − a)(1 − 2a) 2
= lı́m + 3 2
+ ... = .
n→∞ a 3a n a

PROBLEMA 8.9.

Calcular el lı́mite de la sucesión de término general



n( n + 2n + 1)
an = .
n2 + 3

Solución

Comparando los grados del numerador y denominador, resulta:



n n + 2n2 + n
L = lı́m = 2.
n→∞ n2 + 3

PROBLEMA 8.10.

Calcular el lı́mite de la sucesión de término general


(3n − 2)(n + 3)(2n − 5)2
an = .
n2 (2n + 6)(3n − 5)

353
Solución

Comparando de nuevo los grados del numerador y del denominador, obte-


nemos:

(3n2 + 9n − 2n − 6)(4n2 + 25 − 20n) 12n4 + . . .


L = lı́m = lı́m = 2.
n→∞ n2 (6n2 − 10n + 18n − 30) n→∞ 6n4 + . . .

PROBLEMA 8.11.

Calcular el lı́mite de la sucesión de término general


1 2 n
an = 2 + 2 + · · · + 2 .
n n n

Solución

Aplicamos la fórmula 1 + 2 + · · · + n = n(n + 1)/2. Resulta:

1 + 2 + ··· + n n(n + 1) n+1 1


L = lı́m 2
= lı́m 2
= lı́m = .
n→∞ n n→∞ 2n n→∞ 2n 2

PROBLEMA 8.12.

Calcular el lı́mite de la sucesión de término general


 
1 n
an = a , a ∈ N.
n a

Solución

n

Al desarrollar a resulta directamente:

1 n(n − 1)(n − 2) . . . (n − a + 1) 1 na + . . . 1
L = lı́m a
· = lı́m a
= .
n→∞ n a! a! n→∞ n a!

354
PROBLEMA 8.13.

Calcular el lı́mite de la sucesión de término general



n
an = q p √
.
n+ n+ n

Solución


Si dividimos numerador y denominador por n, tenemos:

1 1
L = lı́m q √ √ = lı́m r
n→∞ n+ n+ n n→∞ q √
n+ n
n 1+ n2
1
= lı́m r = 1.
n→∞ q p
1+ 1/n + n/n4

PROBLEMA 8.14.

Calcular el lı́mite de la sucesión de término general


√ √5
3 3 4 − 4 n2

an = 3 √ .
n − 3(4 − 5 n)

Solución

Para comparar los grados del numerador y denominador escribimos:



15

15

15

15
3 45 − 4 n 6 3 45 − 4 n 6
L = lı́m 15 p √
15
= lı́m 15 √ √
15
n→∞ (n − 3)5 (4 − n3 ) n→∞ 4 n5 − 5n4 · 3 + . . . − n8 + . . .

15 √
15
3 45√
−4 n6
n
15 8 0
= lı́m √ √ = = 0.
n→∞ 4 15 n5 −5n4 ·3+...−

15
n8 +... 0−1
15 8
n

355
PROBLEMA 8.15.

Calcular el lı́mite de la sucesión de término general


p √ p √
n2 + n − n2 − n
an =  p √ p √ .
n 3 n3 + n − 3 n3 − n

Solución

Si racionalizamos numerador y denominador, tenemos:


√ √ p √
(n2 + n − n2 + n)( 3 (n3 + n)2 + . . . )
L = lı́m √ √ p √ p √
n→∞ n(n3 + n − n3 + n)( n2 + n + n2 − n)
√ p √ √ p √ 
2 n 3 (n3 + n)2 + 3 n6 − n + 3 (n3 − n)2 3
= lı́m √ p 2 √ p √ = ,
n→∞ 2n n( n + n + n − n) 2 2
debido a que los grados del numerador y denominador son iguales.

PROBLEMA 8.16.

Calcular el lı́mite de la sucesión de término general


3n + 7
an = .
5n − 4

Solución

Debido a la indeterminación ∞/∞, dividimos numerador y denominador


por 5n :
(3/5)n + (7/5n ) 0
L = lı́m n
= = 0,
n→∞ 1 − (4/5 ) 1
debido a que (3/5)n → 0.

PROBLEMA 8.17.

Calcular el lı́mite de la sucesión de término general


√4

n + h − 4 n + k 12p
an = √3

3
· n + j.
n+h− n+k

356
Solución

En primer lugar tenemos:


p
4
p4
n(1 + h/n) −
n(1 + k/n) 12p
L = lı́m p p · n(1 + j/n)
n→∞ 3 n(1 + h/n) − 3 n(1 + k/n)

n1/4 (1 + h/n)1/4 − (1 + k/n)1/4


 
= lı́m 1/3   · n1/12 · (1 + j/n)1/12
n→∞ n (1 + h/n)1/3 − (1 + k/n)1/3
(1 + h/n)1/4 − (1 + k/n)1/4
= lı́m · (1 + j/n)1/12 .
n→∞ (1 + h/n)1/3 − (1 + k/n)1/3

Desarrollamos ahora las potencias de los binomios:


1/4
1/4 h2
    
h 1/4 h
1+ = 1+ + + ...
n 1 n 2 n2
1 h 1/4(1/4 − 1) h2 h 3h2
= 1+ · + · 2 + ··· = 1 + − + ...
4 n 2! n 4n 32n2
1/3
1/3 h2
    
h 1/3 h
1+ = 1+ + + ...
n 1 n 2 n2
1 h 1/3(1/3 − 1) h2 h h2
= 1+ · + · 2 + ··· = 1 + − 2 + ...
3 n 2! n 3n 9n
 1/12     2
j 1/12 j 1/12 j
1+ = 1+ + + ...
n 1 n 2 n2
1 j 1/12(1/12 − 1) j 2 j 11j 2
= 1+ · + · 2 + ··· = 1 + − + ...
12 n 2! n 12n 288n2

Sustituimos en la fórmula del lı́mite y tenemos:


 
h 3h2 k 3k2
1+ 4n −
+ · · · − 1 + 4n
32n2
− 32n 2 + ...
L = lı́m  
n→∞ h h2 k k2
1 + 3n − 9n 2 + · · · − 1 + 3n − 9n2
+ . . .
2
 
j 11j
× 1+ − + ...
12n 288n2
h−k 3(h2 −k2 )
4n − + ... 11j 2
 
32n2 j
= lı́m h−k h2 −k2 1+ − + ...
n→∞ 12n 288n2
3n − 9n2 + . . .
1
4 (h − k) 3
= 1 ·1= .
3 (h − k) 4

357
PROBLEMA 8.18.

Calcular el lı́mite de la sucesión de término general


1/(2n2 ) + 1 − cos 1/n
an = .
n4

Solución

Operando directamente resulta L = 0/∞ = 0.

PROBLEMA 8.19.

Calcular el lı́mite de la sucesión de término general


ln(5n4 − 4n3 + 6n2 + 3n − 2)
an = .
ln(6n3 + 4n2 − 5n + 7)

Solución

Por las propiedades de los logaritmos, tenemos:


ln n4 (5 − 4/n + 6/n2 + 3/n3 − 2/n4 )
L = lı́m
n→∞ ln n3 (6 + 4/n − 5/n2 + 7/n3 )
ln n4 + ln(5 − 4/n + 6/n2 + 3/n3 − 2/n4 )
= lı́m
n→∞ ln n3 + ln(6 + 4/n − 5/n2 + 7/n3 )
4 ln n + ln(5 − 4/n + 6/n2 + 3/n3 − 2/n4 )
= lı́m
n→∞ 3 ln n + ln(6 + 4/n − 5/n2 + 7/n3 )
4 + ln(5 − 4/n + 6/n2 + 3/n3 − 2/n4 )/ ln n 4
= lı́m 2 3
= .
n→∞ 3 + ln(6 + 4/n − 5/n + 7/n )/ ln n 3

PROBLEMA 8.20.

Calcular el lı́mite de la sucesión de término general


πn
sen 2n−1
an = −π
√ .
sen 4n−2 · 4 n4 + 1

358
Solución

πn
Debido a que sen → sen π/2 = 1, y teniendo en cuenta la equivalencia
2n − 1
−π −π
de infinitésimos sen ∼ , resulta:
4n − 2 4n − 2
πn
sen 2n−1 4n − 2 4
L = lı́m −π √ 4
= lı́m √
4
=− .
n→∞
4n−2 ·
4
n +1 n→∞ 4
−π n + 1 π

PROBLEMA 8.21.

Sabiendo que 2 sen A sen B = − cos(A + B) + cos(A − B), calcular el


lı́mite de la sucesión de término general
1 2 3 n
an = sen 2 + sen 2 + sen 2 + · · · + sen 2 .
n n n n

Solución

1
Multiplicando y dividiendo el término general de la sucesión por 2 sen :
2n2
1 2·1 1 2·2 1 2n 1
 
an = 2 sen(1/2n2 )
2 sen 2n2 sen 2n2 + 2 sen 2n2 sen 2n2 + · · · + 2 sen 2n2 sen 2n2 .

Si aplicamos ahora la fórmula dada, tenemos:


   
3 1 2n + 1 2n − 1
an = − cos 2 + cos 2 + · · · + − cos + cos
2n 2n 2n2 2n2
 
1 1 1 2n + 1
× 2
= 2
cos 2 − cos
2 sen(1/2n ) 2 sen(1/2n ) 2n 2n2
1 n+1 n
= 2
· 2 sen 2
sen 2 .
2 sen(1/2n ) 2n 2n

Entonces, debido a la equivalencia sen un ∼ un cuando un → 0:

1 n+1 n n+1 1
L = lı́m 2
·2· 2
· 2 = lı́m = .
n→∞ 2 · (1/2n ) 2n 2n n→∞ 2n 2

359
PROBLEMA 8.22.

Calcular el lı́mite de la sucesión de término general


πn
tg 2n+1
an = √
3
.
n3 + 2n − 1

Solución

Por una parte,


 
π π
tg 2 − 4n+2
π
cotg 4n+2 π
1/ tg 4n+2
L = lı́m √3
= lı́m √
3
= lı́m √ .
n→∞ n3 + 2n − 1 n→∞ n3 + 2n − 1 n→∞ 3 n3 + 2n − 1
π π
Teniendo en cuenta la equivalencia tg ∼ , podemos escribir
4n + 2 4n + 2
4n+2
4n + 2
π
L = lı́m √
3
= lı́m √
3
n→∞ n3
+ 2n − 1 n→∞ π n3 + 2n − 1
4 + 2/n 4
= lı́m p = .
n→∞ π 3 1 + 2/n2 − 1/n3 π

PROBLEMA 8.23.

Calcular el lı́mite de la sucesión de término general


r
n+a
an = n ln .
n−a

Solución

n+a n+a
Debido a la equivalencia ln ∼ − 1, tenemos:
n−a n−a
 
n n+a n n+a
L = lı́m · ln = lı́m −1
n→∞ 2 n − a n→∞ 2 n − a
n n+a−n+a 2an
= lı́m · = lı́m = a.
n→∞ 2 n−a n→∞ 2n − 2a

360
PROBLEMA 8.24.

Calcular el lı́mite de la sucesión de término general


n+1 m
 
an = (4n + 3)m ln donde m ∈ R.
n−2

Solución

n+1 n+1
Aplicamos la equivalencia ln ∼ − 1 y resulta:
n−2 n−2
n+1 m
    m
n+1
L = lı́m (4n + 3) ln = lı́m (4n + 3) −1
n→∞ n−2 n→∞ n−2
 m
3
= lı́m (4n + 3) = 12m .
n→∞ n−2

PROBLEMA 8.25.

Calcular el lı́mite de la sucesión de término general



an = n n a − n, siendo a > 0.

Solución

Aplicaremos la equivalencia a1/n − 1 ∼ (1/n) ln a:


1
L = lı́m (na1/n − n) = lı́m n(a1/n − 1) = lı́m n · ln a = ln a.
n→∞ n→∞ n→∞ n

PROBLEMA 8.26.

Calcular el lı́mite de la sucesión de término general


(a + n)(n − 1)n−1
an = .
nn

361
Solución

Si tomamos logaritmos y utilizamos la equivalencia ln(1 + un ) ∼ un , cuando


un → 0, resulta:

a + n (n − 1)n−1
   
a+n n−1
ln L = lı́m ln · = lı́m ln + lı́m (n − 1) ln
n→∞ n nn−1 n→∞ n n→∞ n
 
a+n n−1
= ln lı́m + lı́m (n − 1) − 1 = ln 1 + (−1) = −1.
n→∞ n n→∞ n

Por tanto, L = e−1 .

PROBLEMA 8.27.

Calcular el lı́mite de la sucesión de término general


an = (n/3) ln(n + a)(n + b)(n + c) − ln nn .

Solución

Tenemos una indeterminación ∞−∞ que operamos del siguiente modo:


h i
L = lı́m (n/3) ln(n + a)(n + b)(n + c) − ln(nn/3 nn/3 nn/3 )
n→∞
= lı́m [(n/3) ln(n + a) + (n/3) ln(n + b) + (n/3) ln(n + c)
n→∞
−(n/3) ln n − (n/3) ln n − (n/3) ln n]
n+a n+b n+b
= lı́m (n/3) ln + lı́m (n/3) ln + lı́m (n/3) ln
n→∞ n n→∞ n n→∞ n
n a n b n c
= lı́m · + lı́m · + lı́m ·
n→∞ 3 n n→∞ 3 n n→∞ 3 n
a b c a+b+c
= + + = .
3 3 3 3

PROBLEMA 8.28.

Calcular el lı́mite de la sucesión de término general


3n4 sen2 (1/n) ln(1 + 1/n)
an = .
(n + 5) cos πn+5
4n+1

362
Solución

Debido a las equivalencias sen 1/n ∼ 1/n y ln(1+1/n) ∼ 1/n, tenemos:


3n4 1/n2 · 1/n
L = lı́m · √
n→∞ n + 5 2/2
4
3n · 2 6 √
= lı́m √ = √ = 3 2.
n→∞ 2n3 (n + 5) 2

PROBLEMA 8.29.

Calcular el lı́mite de la sucesión de término general


an = n2 (ln sen(π/2 + 2/n) − ln cos 1/n).

Solución

Nuevamente, por la equivalencia ln un ∼ un − 1, cuando un → 1, resul-


ta:
sen(π/2 + 2/n) sen(π/2 + 2/n) − cos 1/n
L = lı́m n2 ln = lı́m n2
n→∞ cos 1/n n→∞ cos 1/n
cos 2/n − cos 1/n cos(1/n + 1/n) − cos 1/n
= lı́m n2 = lı́m n2
n→∞ cos 1/n n→∞ cos 1/n
2 2
cos 1/n − sen 1/n − cos 1/n
= lı́m n2
n→∞ cos 1/n
2
n cos 1/n(cos 1/n − 1) sen2 1/n
= lı́m − lı́m n2
n→∞ cos 1/n n→∞ cos 1/n
2
n (−1)(1/n) 2 1 1 3
= lı́m − lı́m =− −1=− .
n→∞ 2 n→∞ cos 1/n 2 2
En la última lı́nea aplicamos las equivalencias sen(1/n) ∼ 1/n y 1−cos(1/n) ∼
(1/n)2 /2.

PROBLEMA 8.30.

Calcular el lı́mite de la sucesión de término general


(3n2 + 1)(1 − cos(1/n)
an = .
(n2 − 2) ln[1 + (1/n2 )]

363
Solución

1 (1/n)2
Teniendo en cuenta las equivalencias de los infinitésimos 1 − cos ∼
n 2
y ln(1 + un ) ∼ un , cuando un → 0, tenemos:
1/n2
3n2 + 1 1 − cos(1/n) 3
L = lı́m 2 lı́m 2
= 3 lı́m 2 2 = .
n→∞ n − 2 n→∞ ln(1 + (1/n )) n→∞ 1/n 2

PROBLEMA 8.31.

Calcular el lı́mite de la sucesión de término general


r ! 2n−1
1 − n 1+3n
an = .
1 − 2n

Solución

Calculando directamente los lı́mites de la base y el exponente, tenemos


que
p 2/3 1
L= 1/2 = √3
.
2

PROBLEMA 8.32.

Calcular el lı́mite de la sucesión de término general


n + 1 n−3
 
an = .
n−1

Solución

Tenemos una indeterminación del tipo 1∞ . Tomamos logaritmos y utilizamos


la equivalencia ln un ∼ un − 1 cuando un → 1:
 
n+1
ln L = lı́m (n − 3) −1
n→∞ n−1
(n − 3)(n + 1 − n + 1) 2n − 6
= lı́m = lı́m = 2 =⇒ L = e2 .
n→∞ n−1 n→∞ n − 1

364
PROBLEMA 8.33.

Calcular el lı́mite de la sucesión de término general


 2 (n2 −1)/n
n +3
an = .
n2 + 4n

Solución

De forma similar al anterior, tenemos:

n2 − 1 n2 + 3 (n2 − 1)(n2 + 3 − n2 − 4n)


 
ln L = lı́m − 1 = lı́m
n→∞ n n2 + 4n n→∞ n(n2 + 4n)
3n2 − 4n3 − 3 + 4n
= lı́m = −4 =⇒ L = e−4 .
n→∞ n3 + 4n2

PROBLEMA 8.34.

Calcular el lı́mite de la sucesión de término general


 n32+2
n2 + 3n − 2

2n +1
an = .
n2 + n

Solución

Como la indeterminación es del tipo 1∞ , tenemos:

n3 + 2 n2 + 3n − 2
ln L = lı́m ln
n→∞ 2n2 + 1 n2 + n
n + 2 n + 3n − 2 − n2 − n
3 2 2n4 + . . .
= lı́m · = lı́m = 1.
n→∞ 2n2 + 1 n2 + n n→∞ 2n4 + . . .

Por tanto, L = e1 = e.

365
PROBLEMA 8.35.

Calcular el lı́mite de la sucesión de término general


 2n2 −3
n2 − 3n + 5 n+1

an = 1 + ln .
n2 − 9n

Solución

Con la misma indeterminación anterior, deberemos aplicar dos veces la equi-


valencia ln un ∼ un − 1 cuando un → 1:

2n2 − 3 n2 − 3n + 5 2n2 − 3 n2 − 3n + 5
 
ln L = lı́m ln = lı́m −1
n→∞ n + 1 n2 − 9n n→∞ n + 1 n2 − 9n
2n2 − 3 n2 − 3n + 5 − n2 + 9n
= lı́m ·
n→∞ n + 1 n2 − 9n
2
(6n + 5)(2n − 3) 12n3 + . . .
= lı́m = lı́m = 12 =⇒ L = e12 .
n→∞ (n2 − 9n)(n + 1) n→∞ n3 + . . .

PROBLEMA 8.36.

Calcular el lı́mite de la sucesión de término general


ln(n + a) n ln n
 
an = .
ln n

Solución

Debido a que

ln(n + a) ln n(1 + a/n) ln n + ln(1 + a/n) ln(1 + a/n)


= = =1+ → 1,
ln n ln n ln n ln n
tenemos una indeterminación del tipo 1∞ . Por tanto, si llamamos L al lı́mite
de la sucesión, resulta
 
ln(n + a)
ln L = lı́m n ln n − 1 = lı́m n ln(1 + a/n)
n→∞ ln n n→∞

= lı́m n ln(1 + a/n) = lı́m ln(1 + a/n)n = ln ea = a.


n→∞ n→∞

366
Queda en definitiva que L = ea .

PROBLEMA 8.37.

Calcular el lı́mite de la sucesión de término general


an = [1 + ln(n2 − 5n + 3) − ln(n2 + 3n − 5)]2n−5 .

Solución

2n−5
n2 − 5n + 3

Debido a que an = 1 + ln 2 → 1∞ , resulta:
n + 3n − 5

n2 − 5n + 3
 2 
n − 5n + 3
ln L = lı́m (2n − 5) ln 2 = lı́m (2n − 5) −1
n→∞ n + 3n − 5 n→∞ n2 + 3n − 5
(2n − 5)(−8n + 8)
= lı́m = −16.
n→∞ n2 + 3n − 5

En definitiva, L = e−16 .

PROBLEMA 8.38.

Calcular el lı́mite de la sucesión de término general


2n+1
12 + 32 + 52 + · · · + (2n − 1)2

an = 3 · .
4n3

Solución

Calcularemos en primer lugar la expresión 12 + 32 + 52 + · · · + (2n − 1)2


k(k + 1)(2k + 1)
utilizando la fórmula 12 + 22 + · · · + k 2 = , ∀k ∈ N:
6
12 + 32 + 52 + · · · + (2n − 1)2 = 12 + 22 + 32 + 42 + 52 + · · · + (2n − 1)2 + (2n)2
−[22 + 42 + · · · + (2n)2 ]
= 12 + 22 + 32 + 42 + · · · + (2n − 1)2 + (2n)2 − 22 (12 + 22 + · · · + n2 )
2n(2n + 1)(4n + 1) n(n + 1)(2n + 1) 8n3 − 2n
= − 22 = .
6 6 6

367
2n+1 2n+1
3 8n3 − 2n 4n2 − 1
 
De este modo an = · = → 1∞ .
4n3 6 4n2
Entonces hacemos

4n2 − 1
 
−(2n + 1)
ln L = lı́m (2n + 1) − 1 = lı́m = 0.
n→∞ 4n2 n→∞ 4n2

Por tanto L = e0 = 1.

PROBLEMA 8.39.

Calcular el lı́mite de la sucesión de término general


√ !√ 3
n3 +n2 /2
2
4n − 7n
an = √ 3
.
8n3 + 4n2

Solución

Tenemos un lı́mite de la forma 1∞ . Llamando L = lı́m an , si utilizamos la


n→∞
equivalencia ln un ∼ un − 1, tenemos:
√ √
p
3 4n2 − 7n − 3 8n3 + 4n2
ln L = lı́m +n3 n2 /2
· √3
n→∞ 8n3 + 4n2
r
3 2
3 n + n /2
p p 
6 2 − 7n)3 − 6 (8n3 + 4n2 )2
= lı́m (4n
n→∞ 8n3 + 4n2
1  p p 
= lı́m 6 (4n2 − 7n)3 − 6 (8n3 + 4n2 )2 .
2 n→∞
p p
Si llamamos ahora a = 6 (4n2 − 7n)3 y b = 6 (8n3 + 4n2 )2 y utilizamos la
identidad a6 − b6 = (a − b)(a5 + a4 b + · · · + b5 ), tenemos

1 (4n2 − 7n)3 − (8n3 + 4n2 )2


ln L = lı́m p p
2 n→∞ 6 (4n2 − 7n)15 + · · · + 6 (8n3 + 4n2 )10
1 −400n5 + 572n4 − 343n3 1 −400 −25
= lı́m p p = · 30/6
= .
2 n→∞ 6 30 6
(2n) + . . . + · · · + (2n)30 2 6·2 24

En definitiva, L = e−25/24 .

368
PROBLEMA 8.40.

Calcular el lı́mite de la sucesión de término general


cos(a + 1/n) n
 
an = .
cos a

Solución

Como tenemos una indeterminación 1∞ , hacemos:


 
cos(a + 1/n) cos(a + 1/n) − cos a
ln L = lı́m n − 1 = lı́m n · .
n→∞ cos a n→∞ cos a
A+B A−B
Aplicamos la fórmula cos A − cos B = −2 sen sen y tenemos:
2 2
1 1 1
 
−2 sen a + 2n sen 2n −2 sen a + 2n 1
ln L = lı́m n · = lı́m · lı́m n sen
n→∞ cos a n→∞ cos a n→∞ 2n
−2 sen a 1 1
= · lı́m n · = −2 tg a · = − tg a.
cos a n→∞ 2n 2
Queda en definitiva L = e− tg a .

PROBLEMA 8.41.

Calcular el lı́mite de la sucesión de término general


1 n
 
an = cos √ .
n

Solución

Análogamente al anterior tenemos:


 
1
ln L = lı́m n cos √ − 1 .
n→∞ n
Utilizaremos la fórmula cos 2x = cos2 x − sen2 x = 1 − 2 sen2 x =⇒ cos 2x −
1 = −2 sen2 x y resulta:
   2
2 1 1
ln L = lı́m n −2 sen √ = lı́m (−2n) √
n→∞ 2 n n→∞ 2 n
−2n −1
= lı́m = .
n→∞ 4n 2

369
En definitiva, L = e−1/2 .

PROBLEMA 8.42.

Calcular el lı́mite de la sucesión de término general


1 + tg 1/n n
 
an = .
1 − tg 1/n

Solución

Como tenemos una indeterminación del tipo 1∞ , hacemos lo siguiente:


   
1 + tg 1/n 2 tg 1/n
ln L = lı́m n − 1 = lı́m n .
n→∞ 1 − tg 1/n n→∞ 1 − tg 1/n

Como 1/n → 0, podemos sustituir tg 1/n por 1/n. Ası́:

2/n 2
ln L = lı́m n · = lı́m = 2.
n→∞ 1 − tg 1/n n→∞ 1 − tg 1/n

Por tanto, L = e2 .

PROBLEMA 8.43.

Calcular el lı́mite de la sucesión de término general


an = (cos φ/n + a sen φ/n)n .

Solución

Como tenemos un lı́mite de la forma 1∞ , usamos las equivalencias ln un ∼


un − 1, 1 − cos un ∼ u2n /2, sen un ∼ un , cuando un → 0 y resulta:

ln L = lı́m n (cos φ/n − 1 + a sen φ/n)


n→∞
= lı́m n (cos φ/n − 1) + lı́m n · a sen φ/n
n→∞ n→∞
φ2
= lı́m −n · + lı́m n · a · (φ/n) = φ · a.
n→∞ 2n2 n→∞

370
Luego L = eφa .

PROBLEMA 8.44.

Calcular el lı́mite de la sucesión de término general


an = (1/n + sen 1/n + cos 1/n)cotg 1/n .

Solución

Tomando logaritmos, y usando las equivalencias tg 1/n ∼ 1/n, 1 − cos 1/n ∼


(1/n)2
2 , tenemos:

ln L = lı́m cotg 1/n(1/n + sen 1/n + cos 1/n − 1)


n→∞
1/n + sen 1/n + cos 1/n − 1
= lı́m
n→∞ tg 1/n
1/n sen 1/n cos 1/n − 1
= lı́m + lı́m + lı́m
n→∞ tg 1/n n→∞ tg 1/n n→∞ tg 1/n
1/n −(1/n)2
= lı́m + lı́m cos 1/n + lı́m = 1 + 1 − 0 = 2.
n→∞ 1/n n→∞ n→∞ 2(1/n)

de donde L = e2 .

PROBLEMA 8.45.

Calcular el lı́mite de la sucesión de término general


1/n n
1/n
!
a1 + · · · + ak
an = .
k

Solución

Debido a la indeterminación 1∞ , usamos la equivalencia a1/n ∼ (1/n) ln a


con lo que:
" 1/n 1/n
# 1/n 1/n
a1 + · · · + ak (a − 1) + · · · + (ak − 1)
ln L = lı́m n − 1 = lı́m n 1
n→∞ k n→∞ k
(1/n) ln a1 + · · · + (1/n) ln ak ln(a1 . . . ak )
= lı́m n = .
n→∞ k k

371

De donde, L = k a1 . . . ak .

PROBLEMA 8.46.

Calcular el lı́mite de la sucesión de término general


 √ n2
n+ na
an = √ .
n+ nb

Solución

Procediendo como en el problema anterior, tenemos:


√ √ √ √
n2 (n + n a − n − n b) n2 ( n a − n b)
ln L = lı́m √ = lı́m √
n→∞ n+ nb n→∞ n+ nb
√ √ √ √
( n a − 1) − ( n b − 1) n
a−1 n
b−1
= lı́m √
n
= lı́m √
n
− lı́m √
n→∞ 1/n + b/n2 n→∞ 1/n + b/n2 n→∞ 1/n + n b/n2
(1/n) ln a (1/n) ln b
= lı́m √
n
− lı́m √
n→∞ 1/n + b/n 2 n→∞ 1/n + n b/n2

ln a ln b a
= lı́m √ − lı́m √ = ln a − ln b = ln .
n→∞ 1 + n b/n n→∞ 1 + n b/n b

Por tanto, L = a/b.

PROBLEMA 8.47.

Calcular el lı́mite de la sucesión de término general



n
√ √ !n
1 + n 2 + ··· + n p
an = .
p

372
Solución

Utilizaremos la equivalencia aun − 1 ∼ un ln a, cuando un → 0:



n

n √
2 + ··· + n p
1+
ln L = lı́m n ln
n→∞ p
nh √ n

n √ i
= lı́m ( 1 − 1) + ( 2 − 1) + · · · + ( p − 1)
n
n→∞ p
n √ n n √
= lı́m ( 1 − 1) + · · · + lı́m ( n p − 1)
n→∞ p n→∞ p
n 1 n 1
= lı́m · ln 1 + · · · + lı́m · ln p
n→∞ p n n→∞ p n
ln 1 + · · · + ln p ln(1 . . . p) p
= = = ln p p!
p p

p √
Por tanto, L = eln p! = p
p!

PROBLEMA 8.48.

Calcular el lı́mite de la sucesión de término general


" #n
p · a1/n + q · b1/n + r · c1/n
an = .
p+q+r

Solución

Como tenemos un lı́mite de la forma 1∞ , hacemos lo siguiente:


" #
p · a1/n + q · b1/n + r · c1/n
ln L = lı́m n −1
n→∞ p+q+r
" #
p(a1/n − 1) + q(b1/n − 1) + r(c1/n − 1)
= lı́m n
n→∞ p+q+r
1 h
1/n 1/n 1/n
i
= p lı́m n(a − 1) + q lı́m n(b − 1) + r lı́m n(c − 1) .
p + q + r n→∞ n→∞ n→∞

Tal como hemos visto en el problema anterior,

lı́m n(a1/n − 1) = ln a, lı́m n(b1/n − 1) = ln b, lı́m n(c1/n − 1) = ln c.


n→∞ n→∞ n→∞

373
Luego,
1
ln L = (p ln a + q ln b + r ln c)
p+q+r
1 1
= ln(ap bq cr ) = ln(ap bq cr ) p+q+r .
p+q+r

y finalmente L = p+q+r ap bq cr .

PROBLEMA 8.49.

Calcular el lı́mite de la sucesión de término general


 1/ ln(3/n)
1
an = .
n

Solución

En este caso tenemos una indeterminación 00 . Ası́ pues:


 1/ ln(3/n)
1 1 1
ln L = ln lı́m = lı́m ln
n→∞ n n→∞ ln(3/n) n
− ln n − ln n/ ln n
= lı́m = lı́m = 1.
n→∞ ln 3 − ln n n→∞ ln 3/ ln n − ln n/ ln n

Como ln L = 1 =⇒ L = e.

PROBLEMA 8.50.

Calcular el lı́mite de la sucesión de término general


4
n + 2 1/ ln(n −3)
 
an = .
3n3 − 1

374
Solución

Debido a la indeterminación 00 , tomamos logaritmos y aplicamos las equi-


n+2 n
valencias ln(n4 − 3) ∼ ln n4 y ln 3 ∼ ln 3 = ln n−2 :
3n − 1 n
1 n+2 1
ln L = lı́m ln = lı́m ln n−2
n→∞ ln(n4− 3) 3n − 1 n→∞ ln n4
3

−2 ln n 1
= lı́m =− .
n→∞ 4 ln n 2
Se deduce entonces que L = e−1/2 .

PROBLEMA 8.51.

Calcular el lı́mite de la sucesión de término general


 1/(1+ln n)
n+1
an = .
n2 + n + 5

Solución

Tomando logaritmos, tenemos:


1 n+1 ln(n + 1) − ln(n2 + n + 5)
ln L = lı́m ln 2 = lı́m
n→∞ 1 + ln n n + n + 5 n→∞ 1 + ln n
ln n(1 + 1/n) − ln n2 (1 + 1/n + 5/n2 )
= lı́m
n→∞ 1 + ln n
ln n + ln(1 + 1/n) − 2 ln n − ln(1 + 1/n + 5/n2 )
= lı́m
n→∞ 1 + ln n
ln n − 2 ln n ln(1 + 1/n) − ln(1 + 1/n + 5/n2 )
= lı́m + lı́m
n→∞ 1 + ln n n→∞ 1 + ln n
− ln n −1 1
= lı́m + 0 = lı́m = −1 =⇒ L = e−1 = .
n→∞ 1 + ln n n→∞ 1/ ln n + 1 e

PROBLEMA 8.52.

Calcular el lı́mite de la sucesión de término general


an = (2 + 3n4 )1/[3+2 ln(n+1)] .

375
Solución

La indeterminación es en este caso del tipo ∞0 . Ası́ pues:


1
ln L = lı́m ln(2 + 3n4 ).
n→∞ 3 + 2 ln(n + 1)

Teniendo en cuenta las equivalencias ln(2 + 3n4 ) ∼ ln n4 y ln(n + 1) ∼ ln n,


resulta:
4 ln n 4
ln L = lı́m = lı́m = 2.
n→∞ 3 + 2 ln n n→∞ 3/ ln n + 2

Por tanto, L = e2 .

PROBLEMA 8.53.

Calcular el lı́mite de la sucesión de término general


an = (1 − 1/22 )(1 − 1/32 ) . . . (1 − 1/n2 ).

Solución

Desarrollando cada factor y simplificando, tenemos:

22 − 1 32 − 1 n2 − 1
L = lı́m · . . .
n→∞ 22 32 n2
(2 − 1)(2 + 1) (3 − 1)(3 + 1) (n − 1)(n + 1)
= lı́m 2
· 2
...
n→∞ 2 3 n2
n+1 1
= lı́m = .
n→∞ 2n 2

PROBLEMA 8.54.

Calcular el lı́mite de la sucesión de término general


1 1 1 1
an = + + ··· + + .
1·2 2·3 (n − 1)n n(n + 1)

376
Solución

Si descomponemos cada fracción en fracciones simples tenemos:


1 A B A(k + 1) + Bk
= + = =⇒ 1 = A(k + 1) + Bk.
k(k + 1) k k+1 k(k + 1)
1 1 1
Entonces A = 1, B = −1, de donde = − .
k(k + 1) k k+1
Aplicando lo anterior a cada sumando de la sucesión dada tenemos:
       
1 1 1 1 1 1 1 1 1
an = − + − +· · ·+ − + − = 1− .
1 2 2 3 n−1 n n n+1 n+1
Es evidente entonces que

L = lı́m an = 1.
n→∞

PROBLEMA 8.55.

ln n
Demostrar que → 0 cuando n → ∞. Deducir de lo anterior
√ n
que n n → 1.

Solución

ln n
Por ser ln n > 0 y n > 0, entonces lı́m ≥ 0.
n→∞ n

Por otro lado, si llamamos an = ln n, tenemos:


ln n an an an
L = lı́m = lı́m an ≤ lı́m an = lı́m
n→∞ n a n →∞ e a n →∞ 2 a n →∞ (1 + 1)an
a an
= lı́m  n  ≤ lı́m an 
an →∞ 1 + an + an + . . . an →∞
2 3 2
an 2
= lı́m = lı́m = 0.
an →∞ an (an − 1)/2! an →∞ an − 1

ln n
Esto prueba entonces que lı́m = 0.
n→∞ n
Si ahora aplicamos la fórmula ab = eb ln a , podemos calcular:

lı́m n n = lı́m n1/n = elı́mn→∞ 1/n ln n = e0 = 1.
n→∞ n→∞

377
PROBLEMA 8.56.

Calcular el lı́mite de la sucesión de término general



n √
an = nn+1 ( n a − 1).

Solución

Teniendo en cuenta la equivalencia a1/n − 1 ∼ (1/n) ln a y sabiendo que



n
n → 1, resulta:

1
L = lı́m n(n+1)/n (a1/n − 1) = lı́m n(n+1)/n ln a
n→∞ n→∞ n

= lı́m n(n+1)/n · n−1 · ln a = lı́m n n ln a = ln a.
n→∞ n→∞

PROBLEMA 8.57.

Calcular el lı́mite de la sucesión de término general


p
n
an = n3 + an2 + bn + c.

Solución

Debido a que

an n3 + an2 + bn + c
lı́m = lı́m = 1,
n→∞ an−1 n→∞ (n − 1)3 + a(n − 1)2 + b(n − 1) + c

se deduce por el criterio del cociente-raı́z que L = 1.

PROBLEMA 8.58.

Calcular el lı́mite de la sucesión de término general


p
3n
an = n 3 − 1.

378
Solución

√ √ 1/3
Por el mismo criterio anterior, si escribimos 3n n3 − 1 = n n3 − 1 ,
n3 − 1 √
n
como lı́m = 1, entonces lı́m n3 − 1 = 1. Por tanto, L = 1.
n→∞ (n − 1)3 − 1 n→∞

PROBLEMA 8.59.

Calcular el lı́mite de la sucesión de término general


r
n (a + 1)(a + 2) . . . (a + n)
an = .
n!

Solución

Calculamos también el lı́mite del cociente entre dos términos consecuti-


vos:
(a+1)(a+2)...(a+n)
n! (a + n)
L = lı́m = lı́m = 1.
n→∞ (a+1)(a+2)...(a+n−1) n→∞ n
(n−1)!

PROBLEMA 8.60.

Calcular el lı́mite de la sucesión de término general


q √
an = n nn+1 ( a − 1).

Solución

Por el criterio del cociente-raı́z, tenemos:



nn+1 ( a − 1) nn+1
L = lı́m √ = lı́m
n→∞ (n − 1)n ( a − 1) n→∞ (n − 1)n
 n
n
= lı́m n = +∞ · e = +∞.
n→∞ n−1

379
PROBLEMA 8.61.

Calcular el lı́mite de la sucesión de término general



n
nln n
an = .
ln a

Solución

Utilizamos el mismo criterio de los problemas anteriores y tenemos:

1 √n 1 nln n
L = lı́m nln n = lı́m .
ln a n→∞ ln a n→∞ (n − 1)ln(n−1)

Teniendo en cuenta la equivalencia ln n ∼ ln(n − 1), resulta:

 ln n
1 n 1 n→∞ lı́m ln n· n−n+1
n−1
L = lı́m = e
ln a n→∞ n − 1 ln a
1 n→∞ ln n
lı́m n−1 1 0 1
= e = e = .
ln a ln a ln a

PROBLEMA 8.62.

Calcular el lı́mite de la sucesión de término general


p
an = n (1 + 1/n)p . . . (1 + n/n)p .

Solución

Por el criterio del cociente-raı́z nuevamente, resulta:

(1 + 1/n)p . . . (1 + n/n)p
L = lı́m .
n→∞ [1 + 1/(n − 1)]p . . . [1 + (n − 1)/(n − 1)]p

380
Escribimos ahora
 
1 1 n·n
1+ = 1+ ,
n−1 n (n + 1)(n − 1)
 
2 2 n · (n + 1)
1+ = 1+ ,
n−1 n (n + 2)(n − 1)
 
3 3 n · (n + 2)
1+ = 1+ ,
n−1 n (n + 3)(n − 1)
...  
n−2 n−2 n · (2n − 3)
1+ = 1+ ,
n−1 n (2n − 2)(n − 1)
 
n−1 n−1 n · (2n − 2)
1+ = 1+ ,
n−1 n (2n − 1)(n − 1)

y obtenemos
 p
(1 + 1/n) . . . (1 + (n − 1)/n)(1 + n/n)
L = lı́m  n·(2n−2)

n→∞ n·n
(1 + 1/n) (n+1)(n−1) . . . (1 + (n − 1)/n) (n−1)(2n−1)
 p
(1 + n/n)
= lı́m  
n→∞ nn−1 n(n+1)...(2n−3)(2n−2)
(n−1)n−1 (n+1)(n+2)...(2n−1)
"  n−1 #p
n−1 2n − 1
= lı́m (1 + n/n) = (4e−1 )p = (4/e)p .
n→∞ n n

PROBLEMA 8.63.

Calcular el lı́mite de la sucesión de término general


√ √
1 + 2 + ··· + n n
an = .
n

Solución

Por el criterio de la media aritmética



n
n
L = lı́m n = lı́m = 1.
n→∞ n→∞ n − 1


n np
En general lı́m np = lı́m = 1.
n→∞ n→∞ (n − 1)p

381
PROBLEMA 8.64.

Calcular el lı́mite de la sucesión de término general


ln(n!)
an = .
n

Solución

Por el criterio de Stolz (o el de la media aritmética), tenemos:


ln 1 + ln 2 + · · · + ln n
L = lı́m = lı́m ln n = ∞.
n→∞ n n→∞

PROBLEMA 8.65.

Calcular el lı́mite de la sucesión de término general


√ √ √
1 + 2 + 3 3! + · · · + n n!
an = .
n2

Solución

Aplicamos el criterio de Stolz y utilizamos la fórmula de Stirling:


√ √ √ √ √ p
1 + 2 + 3 3! + · · · + n n! − (1 + 2 + 3 3! + · · · + n−1 (n − 1)!)
L = lı́m
n→∞ n2 − (n − 1)2

n
p
n

n! nn e−n 2πn
= lı́m 2 = lı́m
n→∞ n − n2 + 2n − 1 n→∞ 2n − 1
n √ √
q q
n n n 1
= lı́m lı́m 2πn = lı́m lı́m 2πn = .
n→∞ e(2n − 1) n→∞ n→∞ 2en − e n→∞ 2e

PROBLEMA 8.66.

Calcular el lı́mite de la sucesión de término general


n n−1 n−2 2 1
1 + 2 + 3 + ··· + n−1 + n
an = .
ln n!

382
Solución

Aplicamos el criterio de Stolz por dos veces:


h i
n n−1 2 1 n−1 n−2 2 1
1 + 2 + · · · + n−1 + n − 1 + 2 + · · · + n−2 + n−1
L = lı́m
n→∞ ln n! − ln(n − 1)!
1 1 1
1 + 2 + · · · + n−1 + n 1 + 12 + · · · + n−1
1
+ n1
= lı́m = lı́m
n→∞ ln[n!/(n − 1)!] n→∞ ln n
h i
1 1 1 1 1
1 + 2 + · · · + n−1 + n − 1 + 2 + · · · + n−1
= lı́m
n→∞ ln n − ln(n − 1)
1/n 1 1
= lı́m = lı́m  n = lı́m = 1.
n→∞ ln(n/n − 1) n→∞ n→∞ ln e
ln n n−1

PROBLEMA 8.67.

Calcular el lı́mite de la sucesión de término general


n
P √
ln[arc tg(1/ k) + 1]
an = k=1 n .
P √
1/ 3k + 2
k=1

Solución

Aplicamos en primer lugar el criterio de Stolz:


n
P √ n−1
P √
ln[arc tg(1/ k) + 1] − ln[arc tg(1/ k) + 1]
k=1 k=1
L = lı́m
n→∞ n
P √ n−1
P √
1/ 3k + 2 − 1/ 3k + 2
k=1 k=1

Teniendo en cuenta que arc tg 1/ n → 0 y ln(un + 1) ∼ un cuando un → 0,
tenemos
√ √
3n + 2 √
r
ln[arc tg(1/ n) + 1] 1/ n
L = lı́m √ = lı́m √ = lı́m = 3.
n→∞ 1/ 3n + 2 n→∞ 1/ 3n + 2 n→∞ n

383
PROBLEMA 8.68.

Calcular el lı́mite de la sucesión de término general


 
1 1 1
an = √ √ + ··· + √ .
n 1 n

Solución

Por el criterio de Stolz tenemos directamente:


√ √ √
1/ n n+ n−1
L = lı́m √ √ = lı́m √
n→∞ n − n − 1 n→∞ n(n − n + 1)
√ √
n+ n−1
= lı́m √ = 2.
n→∞ n

PROBLEMA 8.69.

Calcular el lı́mite de la sucesión de término general


√ √
1 + 2 2 + ··· + n n
an = √ .
n2 n

Solución

Nuevamente por el criterio de Stolz tenemos:


√ √ √ √
n n n n(n2 n + (n − 1)2 n − 1)
L = lı́m 2 √ √ = lı́m
n→∞ n n − (n − 1)2 n − 1 n→∞ n4 · n − (n − 1)4 · (n − 1)
√ √
n4 + n(n − 1)2 n2 − n n4 + n8 + . . . 2
= lı́m = lı́m = .
n→∞ n5 − (n − 1)5 n→∞ 5n4 + . . . 5

PROBLEMA 8.70.

Calcular el lı́mite de la sucesión de término general


hp √ p 3
√ p
n
√ i
ln 1 + 2 · 1 + 3 2... 1 + n 2
an = .
sen 1 + sen(1/2) + · · · + sen(1/n)

384
Solución

Aplicamos el criterio de Stolz y tenemos en cuenta la equivalencia sen 1/n ∼


1/n:
p √ p3
√ p
n

ln 1 + 2 + ln 1 + 3 2 + · · · + ln 1 + n 2
L = lı́m
n→∞ sen 1 + sen(1/2) + · · · + sen(1/n)
p
n

ln 1 + n 2 (1/n) ln(1 + 21/n )
= lı́m = lı́m
n→∞ sen(1/n) n→∞ sen(1/n)
(1/n) ln(1 + 21/n )
= lı́m = lı́m ln(1 + 21/n ) = ln 2.
n→∞ (1/n) n→∞

PROBLEMA 8.71.

Calcular el lı́mite de la sucesión de término general


ln 1 − ln 2 + ln 3 − · · · + ln(2n − 1) − ln(2n)
an = .
ln n

Solución

Por el criterio de Stolz y la equivalencia ln un ∼ un − 1, cuando un → 1,


tenemos:
ln(2n − 1) − ln(2n) ln(2n − 1)/(2n)
L = lı́m = lı́m
n→∞ ln n − ln(n − 1) n→∞ ln n/(n − 1)
2n−1
−1 1−n 1
= lı́m 2nn = lı́m =− .
n−1 − 1
n→∞ n→∞ 2n 2

PROBLEMA 8.72.

Calcular el lı́mite de la sucesión de término general


12 + 22 + · · · + n 2
an = n n n .
  
1 + 2 + ··· + n

385
Solución

n  
X n
Recordando que 2n = (1 + 1)n = 1k · 1n−k y aplicando sucesivamen-
k
k=0
te el criterio de Stolz, tenemos:

12 + 22 + · · · + n 2 n2
L = lı́m = lı́m
n→∞ (1 + 1)n − 1 n→∞ 2n − 1 − 2n−1 + 1

n 2 n2 n2
= lı́m n = lı́m = lı́m
n→∞ 2 − 2n−1 n→∞ 2n (1 − 1/2) n→∞ 2n−1

n2 − (n − 1)2 2n − 1 2n − 1 − (2n − 3)
= lı́m n−1 n−2
= lı́m n−2
= lı́m
n→∞ 2 −2 n→∞ 2 n→∞ 2n−2 − 2n−3
2
= lı́m n−3 = 0.
n→∞ 2

PROBLEMA 8.73.

Calcular el lı́mite de la sucesión de término general


√ √ p
1 · 2 · 3 + 2 · 3 · 4 + · · · + n(n + 1)(n + 2)
an = √ .
n2 n

Solución

Por el criterio de Stolz se obtiene directamente:


p
n(n + 1)(n + 2)
L = lı́m√ √
n→∞ n2 n − (n − 1)2 n − 1
√ √ √
n3 + 3n2 + 2n(n2 n + (n − 1)2 n − 1)
= lı́m
n→∞ n5 − (n − 1)5
√ √ √
n3 + . . .( n5 + n5 + . . .) 2
= lı́m = .
n→∞ 5n4 + . . . 5

386
PROBLEMA 8.74.

Sabiendo que lı́m un = a, calcular


n→∞
u1 + 2u2 + 3u3 + · · · + nun
a) L1 = lı́m .
n→∞ n2
u1 /1 + u2 /2 + · · · + un /n
b) L2 = lı́m .
n→∞ ln n

Solución

a) Por el criterio de Stolz,


nun n a
L1 = lı́m = lı́m un · lı́m = .
n→∞ n2 − (n − 1)2 n→∞ n→∞ 2n − 1 2

b) Si aplicamos nuevamente el criterio de Stolz,

un /n un
L2 = lı́m = lı́m
n→∞ ln n − ln(n − 1) n→∞ n ln n
n−1
1 1
= lı́m un lı́m  n = a · = a.
n→∞ n→∞ n
ln n−1 ln e

PROBLEMA 8.75.

Calcular el lı́mite de la sucesión de término general


1 2 32 43 (n + 1)n
 
an = 2 + + 2 + ··· + .
n 1 2 3 nn−1

Solución

Por el criterio de Stolz tenemos:


(n+1)n n
nn−1 n n+1
n
L = lı́m 2 = lı́m
n→∞ n − (n − 1)2 n→∞ 2n − 1

1 n 1
 
n e
= lı́m lı́m 1 + = ·e= .
n→∞ 2n − 1 n→∞ n 2 2

387
PROBLEMA 8.76.

Calcular el lı́mite de la sucesión de término general



an = n2 e− n
.

Solución

Tomando logaritmos, resulta:


√ √
 
2 ln n
ln L = lı́m [2 ln n − n] = lı́m n √ −1 .
n→∞ n→∞ n
2 ln n
Aplicamos ahora el criterio de Stolz para resolver el lı́mite de cn = √ :
n
n
2 ln n − 2 ln(n − 1) 2 ln n−1
lı́m cn = lı́m √ √ = lı́m n−(n−1)
n→∞ n→∞ n− n−1 n→∞ √ √
n+ n−1
√n+√n−1
√ √

n n
= 2 lı́m ( n + n − 1) ln = 2 lı́m ln
n→∞ n−1 n→∞ n−1
√ √
 (n−1) n+ n−1
1 n−1
= 2 lı́m ln 1 +
n→∞ n−1
√ √
n+ n−1
lı́m
= 2 ln e n→∞ n−1
= 2 ln e0 = 0.

Resulta entonces que



ln L = lı́m n[cn − 1] = −∞.
n→∞

y L = e−∞ = 0.

PROBLEMA 8.77.

Calcular el lı́mite de la sucesión de término general


(ln n)2
an = .
n

388
Solución

Aplicando el criterio de Stolz tenemos:

(ln n)2 − [ln(n − 1)]2


L = lı́m = lı́m [ln n − ln(n − 1)][ln n + ln(n − 1)]
n→∞ n − (n − 1) n→∞
 
n n
= lı́m ln[n(n − 1)] ln = lı́m ln(n2 − n) −1
n→∞ n − 1 n→∞ n−1
2
ln(n − n)
= lı́m .
n→∞ n−1
Aplicamos nuevamente el criterio de Stolz, y obtenemos:

ln(n2 − n) − ln[(n − 1)2 − (n − 1)]


L = lı́m
n→∞ n − 1 − (n − 2)
2
n −n
= lı́m ln 2 = ln 1 = 0.
n→∞ n − 3n + 2

PROBLEMA 8.78.

Calcular el lı́mite de la sucesión de término general


an = n(cos x)n , 0 < x < π/2.

Solución

En el intervalo 0 < x < π/2, 0 < cos x < 1 y (cos x)n → 0, por lo que
tenemos un lı́mite indeterminado de la forma ∞ · 0. Resulta:
 
n ln n
ln L = lı́m ln n(cos x) = lı́m [ln n + n ln cos x] = lı́m n + ln cos x
n→∞ n→∞ n→∞ n
 
ln n
= lı́m n · lı́m + ln cos x .
n→∞ n→∞ n

Por una parte, según el criterio de Stolz,

ln n ln n − ln(n − 1) n
lı́m = lı́m = lı́m ln = 0.
n→∞ n n→∞ n − (n − 1) n→∞ n−1

Por otra parte, como 0 < cos x < 1, entonces −∞ < ln cos x < 0, con lo
que

389
 
ln n
lı́m + ln cos x = k < 0 y
n→∞ n
 
ln n
ln L = lı́m n lı́m + ln cos x = +∞ · k = −∞.
n→∞ n→∞ n

En definitiva, L = e−∞ = 0.

PROBLEMA 8.79.

Calcular el lı́mite de la sucesión de término general


ln 1 + ln 2 + · · · + ln n
an = .
n ln n

Solución

Aplicando el criterio de Stolz,


ln n ln n
L = lı́m = lı́m
n→∞n ln n − (n − 1) ln(n − 1) n→∞ ln n − ln(n − 1)n−1
n

ln n ln n
= lı́m n n = lı́m n−1
n→∞ ln n→∞

(n−1)n−1 n
ln n n−1
ln n 1
= lı́m  n−1 = n→∞
lı́m  n−1 .
n→∞ n 1 n
ln n + ln n−1 1+ ln n ln n−1

 n−1  n−1
n 1 1
Como = 1+ → e, L = 1+0 = 1.
n−1 n−1

PROBLEMA 8.80.

Calcular el lı́mite de la sucesión de término general


1p
an = n (n + 1)(n + 2) . . . (n + n).
n

390
Solución

En primer lugar tomamos logaritmos:


r
n (n + 1)(n + 2) . . . (n + n)
ln L = lı́m ln
n→∞ nn
1 (n + 1)(n + 2) . . . (n + n)
= lı́m ln .
n→∞ n nn
Aplicamos ahora el criterio de Stolz y resulta:
ln (n+1)(n+2)...(n+n)
nn − ln n(n+1)...(n−1+n−1)
(n−1)n−1
ln L = lı́m
n→∞ n − (n − 1)
(n − 1)n−1
 
(n + 1)(n + 2) . . . (2n)
= lı́m ln ·
n→∞ nn n(n + 1) . . . (2n − 2)
n−1
 
(2n − 1)2n (n − 1)
= lı́m ln ·
n→∞ n nn
(2n − 1)2n (n − 1)n−1
 
= lı́m ln ·
n→∞ n2 nn−1
n − 1 n−1
 
(2n − 1)2n
= ln lı́m + ln lı́m
n→∞ n2 n→∞ n
= ln 4 + ln e−1 = ln(4e−1 ).
Queda por tanto, L = 4e−1 .

PROBLEMA 8.81.

Calcular el lı́mite de la sucesión de término general


1 22 · 33 · 44 . . . (n − 1)n−1 · nn
an = ln 2 3 4 .
n 5 · 6 · 7 . . . (n + 2)n−1 · (n + 3)n

Solución

Agrupando términos y aplicando las propiedades de los logaritmos,


 2  3   4
n − 1 n−1
   n
1 2 3 4 n
L = lı́m ln ...
n→∞ n 5 6 7 n+2 n+3
"   #
2 2 3 3 n
   
1 n
= lı́m ln + ln + · · · + ln
n→∞ n 5 6 n+3
 
1 2 3 n
= lı́m 2 ln + 3 ln + · · · + n ln .
n→∞ n 5 6 n+3

391
Aplicando ahora el criterio de Stolz, obtenemos:
n
n ln n+3 n
L = lı́m = lı́m n ln
n→∞ n − (n − 1) n→∞ n+3
 
n −3n
= lı́m n − 1 = lı́m = −3.
n→∞ n+3 n→∞ n + 3

PROBLEMA 8.82.

Calcular el lı́mite de la sucesión de término general


s    
n2 n n n
an = ... .
1 2 n

Solución

Tomamos logaritmos y aplicamos el criterio de Stolz:


n
 n n

ln 1 2 ... n
ln L = lı́m 2
 n
n→∞
n n n n−1 n−1 n−1
    
2ln 1 ...− ln
n 1 2 ... n−1
= lı́m 2 2
n→∞ n − (n − 1)
(n1 )(n2 )...(nn)
ln n−1 n−1 ... n−1
( 1 )( 2 ) (n−1)
= lı́m .
n→∞ n2 − (n − 1)2

n
 n!  
k (n−k)!k! n n
Sabiendo que n−1 = (n−1)!
= y = 1, resulta:
n−k

k
n
(n−k−1)!k!

n n−1
n n
ln n−1 · n−2 . . . n2 · n
1
ln (n−1)! ln(nn /n!)
ln L = lı́m = lı́m = lı́m .
n→∞ 2n − 1 2n − 1 n→∞ n→∞ 2n − 1


Aplicamos ahora la fórmula de Stirling n! ∼ nn e−n 2πn y tenemos:
n√ n n
ln nn e−n 2πn
ln √e2πn
ln L = lı́m = lı́m
n→∞ 2n − 1 n→∞ 2n − 1
n
√ √
ln e − ln 2πn n ln 2πn
= lı́m = lı́m − lı́m .
n→∞ 2n − 1 n→∞ 2n − 1 n→∞ 2n − 1

392
Volvemos a aplicar el criterio de Stolz:
√ p √
1 ln 2πn − ln 2π(n − 1) 1 1 2πn
ln L = − lı́m = − lı́m ln p
2 n→∞ 2n − 1 − (2n − 3) 2 n→∞ 2 2π(n − 1)
r
1 1 n 1 1 1
= − lı́m ln = − ln 1 = .
2 2 n→∞ n−1 2 2 2

Tenemos entonces L = e1/2 = e.

PROBLEMA 8.83.

Calcular el lı́mite de la sucesión de término general


n[2 · 4 · 6 · . . . (2n − 2)]2
an = .
[1 · 3 · 5 · . . . (2n − 1)]2

Solución

Escribiremos la sucesión en términos de factoriales y aplicaremos la fórmula


de Stirling:
2
2 · 4 . . . (2n − 2) 2
   n−1
2 · 1 · 2 . . . (n − 1) · 2 · 4 . . . 2n
L = lı́m n = lı́m n
n→∞ 1 · 3 . . . (2n − 1) n→∞ 1 · 2 · 3 . . . (2n − 1) · 2n
 n−1 2  n−1 2
2 (n − 1)! · 2 · 4 . . . 2n 2 · (n − 1)! · 2n · n!
= lı́m n = lı́m n
n→∞ (2n)! n→∞ (2n)!
 2n−1 2
2 · (n − 1)! n!
= lı́m n
n→∞ (2n)!
" √ #2
22n−1 (n − 1)n−1 e−n+1 2π(n − 1) · nn e−n 2πn
p
= lı́m n √
n→∞ (2n)2n e−2n 2π · 2n
" √ p #2
(n − 1)n−1 e π n(n − 1) n · e2 π 2 n(n − 1)(n − 1)2n−2
= lı́m n √ = lı́m
n→∞ 2nn n n→∞ 4 · n2n · n
(n − 1)2n−1 e2 π 2 n − 1 2n−1 π 2 e2
 
= lı́m = lı́m
n→∞ 4n2n−1 n→∞ n 4
2 2
  2n−1 2 2 −2 2
π e 1 π e e π
= lı́m 1 − = = .
4 n→∞ n 4 4

393
PROBLEMA 8.84.
Calcular el lı́mite de la sucesión de término general
2n −n √
 
an = 4 n.
n

Solución

Por la fórmula de Stirling, tenemos:


√ √ √
(2n)! 4−n n (2n)2n e−2n 4πn 4−n n
L = lı́m = lı́m
n→∞ (n!)2 n→∞ n2n e−2n · 2πn
√ √
4n n2n e−2n 2n · 4−n π π
= lı́m 2n −2n
= .
n→∞ n e · 2πn π

PROBLEMA 8.85.
Calcular el lı́mite de la sucesión de término general
2 · 4 . . . (2n − 2) 2
 
an = n .
1 · 3 . . . (2n − 1)

Solución

Aplicando nuevamente la fórmula de Stirling,


2
√

√ (n − 1)! 2n−1 n−1 [(n − 1)!]2 2n−1 2

n · 2
L = lı́m  n · (2n−1)!
 = lı́m
n→∞
n−1
n→∞ (2n − 1)!
2 (n−1)!
"√ #2
n · 22n−2 e−2n+2 (n − 1)2n−2 [ 2π(n − 1)]2
p
= lı́m p
n→∞ e−2n+1 (2n − 1)2n−1 2π(2n − 1)
"√ #2
n · 22n−2 e · (n − 1)2n−1 2π
= lı́m p
n→∞ (2n − 1)2n−1 2π(2n − 1)
 2n−1 2
2 (n − 1)2n−1 e2 nπ 2
= lı́m ·
n→∞ (2n − 1)2n−1 4πn − 2π
" 2n−1 #2
e2 nπ 2

2n − 2
= lı́m · lı́m
n→∞ 2n − 1 n→∞ 4πn − 2π

π2 π
= (e−1 )2 · e2 = .
4π 4

394
PROBLEMA 8.86.

Calcular el lı́mite de la sucesión de término general



22n (n!)2 n
an = .
(2n + 1)!

Solución

Utilizamos la fórmula de Stirling en el numerador y denominador:



22n e−2n n2n 2πn n
L = lı́m −2n−1 √
n→∞ e (2n + 1)2n+1 4πn + 2π

22n n2n 2πn n
= lı́m −1 √
n→∞ e (2n + 1)2n+1 4πn + 2π
 2n √
2n 2πn n
= lı́m · e · lı́m √
n→∞ 2n + 1 n→∞ (2n + 1) 4πn + 2π

−1 2π π
= e ·e· √ = .
2 4π 2

PROBLEMA 8.87.

Calcular el lı́mite de la sucesión de término general


n 2
(2n!)2 en /(n!) − 1

an = .
nn

Solución

nn nn
Debido a que A = lı́m = lı́m = 0, resulta la equivalen-
n→∞ (n!)2 n→∞ n2n e−2n 2πn
n 2 nn
cia de infinitésimos en /(n!) − 1 ∼ . Luego
(n!)2

4(n!)2 nn
L = lı́m = 4.
n→∞ nn (n!)2

395
PROBLEMA 8.88.

Calcular el lı́mite de la sucesión de término general


√n
n!
an = .
n

Solución

Si aplicamos la fórmula de Stirling, tenemos:


p √ √
n
nn e−n 2πn n · e−1 2n 2πn √
= lı́m e−1 2πn.
2n
L = lı́m = lı́m
n→∞ n n→∞ n n→∞
√ √ p √
Ahora bien, como lı́m 2n 2πn = lı́m 2n 2π n n = 1, resulta que L = e−1 .
n→∞ n→∞

PROBLEMA 8.89.

Calcular el lı́mite de la sucesión de término general


22n (n!)2
an = √ .
n[(2n)!]

Solución

Aplicando la fórmula de Stirling tenemos:



22n (nn e−n 2πn)2 22n n2n e−2n · 2πn
L = lı́m √ p = lı́m √ √
n→∞ n(2n)2n e−2n 2π(2n) n→∞ n · 22n n2n e−2n 4πn
2πn √
= lı́m √ √ √ = π.
n→∞ n·2 π n

PROBLEMA 8.90.

Dada la sucesión {an } definida por a1 = 2, an = 5an−1 + 3, calcular


an
lı́m .
n→∞ 5n

396
Solución

Vamos a obtener el término general de la sucesión dando valores a n:

an = 5an−1 + 3;
an−1 = 5an−2 + 3 =⇒ 5an−1 = 52 an−2 + 5 · 3;
an−2 = 5an−3 + 3 =⇒ 52 an−2 = 53 an−3 + 52 · 3;
...
a3 = 5a2 + 3 =⇒ 5n−3 a3 = 5n−2 a2 + 5n−3 · 3;
a2 = 5a1 + 3 =⇒ 5n−2 a2 = 5n−1 a1 + 5n−2 · 3.

Sumando miembro a miembro, an = 5n−1 a1 +3(5n−2 +· · ·+52 +5+1).


El último paréntesis corresponde a la suma de los términos de una progresión
geométrica, con lo que

5n−1 − 1 11 · 5n−1 − 3
an = 5n−1 · 2 + 3 · = .
5−1 4
Entonces,

11 · 5n−1 − 3
 
an 11 3 11
L = lı́m n = lı́m n
= lı́m − n
= .
n→∞ 5 n→∞ 4·5 n→∞ 20 4·5 20

397
B. EJERCICIOS PROPUESTOS.

Calcular el lı́mite de las sucesiones de término general

  n2 +2
n+1 n−3
1.- an = .
n−1
Resp.: L = e2 .

pn2 −1
3n2 − 2

2.- an = .
3n2 − 1
Resp.: L = −p/3.

s 
n 2n
3.- an = .
n
Resp.: L = 4.

(2n + 1)3 − (2n − 1)3


4.- an = .
3n2 + 1
Resp.: L = 8.

nπ − 2n(4n − n3 ) 1 + n−1
5.- an = · .
5 − n3 1+n
Resp.: L = −2.


3

3
6.- an = n3 + an2 − n3 − an2 .
Resp.: L = 2a/3.

p √ p √
n2 + n − n2 − n
7.- an = p √ p √ .
n 3 n3 + n − 3 n3 − n
Resp.: L = 0.


n cos(tg n)
8.- an = .
n!
Resp.: L = 0.

398
9.- an = n sen(π/n).
Resp.: L = π.

p
n
10.- an = n2 + n.
Resp.: L = 1.

 
n+1
11.- an = (4n + 3) ln .
n+2
Resp.: L = −4.

  3n+4
5n + 2 9n−5
12.- an = .
15n − 4

Resp.: L = 1/ 3 3.


n2
13.- an = n!
Resp.: L = 0.

1pn
14.- an = (n + 1)(n + 2) . . . 2n.
n
Resp.: L = 4/e.


n
n!
15.- an = .
n
Resp.: L = 1/e.

1m + 2m + · · · + n m
16.- an = , (m > −1).
nm+1
Resp.: L = 1/(1 + m).


q p
17.- an = 2 2 . . . 2 (n veces).
Resp.: L = 2.


q p
18.- an = 2 + 2 + . . . 2 (n veces).
Resp.: L = 2.

399
√ √
3

n
1+ 2! + 3! + · · · + n!
19.- an = .
n2
Resp.: L = 1/e.

1 1 1 1
20.- an = √ +√ +√ + ··· + √ .
n2 +1 n2 +2 n2 +3 n2+n
Resp.: L = 1.

400
CAPÍTULO IX.
SERIES NUMÉRICAS

SECCIONES
A. Series de términos no negativos.
B. Ejercicios propuestos.

401
A. SERIES DE TÉRMINOS NO NEGATIVOS.

Dada una sucesión {a1 , a2 , . . . ,P


an , . . . }, se llama serie de término general
an , y que representaremos por an , a la sucesión de sumas parciales {Sn }
n≥1
definida por S1 = a1 , S2 = a1 + a2 , . . . , Sn = a1 + a2 + · · · + an , . . . .
P
Si existe S = lı́m Sn , la serie an se dice convergente y tiene suma S y
n→∞ n≥1
P
se escribe an = S.
n≥1
P
Si dicho lı́mite es infinito o no existe, la serie an es divergente.
n≥1

Enunciaremos a continuación los criterios generales para estudiar el carácter


(convergente o divergente) de una serie. Nos limitaremos a las series de
términos no negativos (an ≥ 0) aunque el primer criterio es válido para
series generales.
1. Condición del resto.
P
Si una serie an es convergente, entonces lı́m an = 0.
n≥1 n→∞

De aquı́ se deduce que si el término general de una serie no converge


a cero, dicha serie es divergente.
2. Criterio de comparación.
P P P
Dadas dos series an y bn , si an ≤ bn , ∀n y bn converge,
P n≥1 n≥1 n≥1
entonces an converge.
n≥1

Recı́procamente, si una serie es divergente y todos sus términos son


mayores o iguales que los de otra serie, esta última es también diver-
gente.
3. Criterio de comparación por paso al lı́mite.
an
a) Si lı́m = L (L finito y L 6= 0), entonces
n→∞ bn

X X
an converge ⇐⇒ bn converge.
n≥1 n≥1

an
b) Si lı́m = 0, entonces
n→∞ bn
X X
bn converge =⇒ an converge.
n≥1 n≥1

402
an
c) Si lı́m = ∞, entonces
n→∞ bn
X X
an converge =⇒ bn converge.
n≥1 n≥1

Para utilizar los criterios de comparación es conveniente conocer la


convergencia de las siguientes series:
X
- Serie armónica: La serie 1/np es convergente cuando p > 1 y
n≥1
divergente cuando p ≤ 1.
X
-Serie geométrica: La serie a · rn es convergente cuando |r| < 1
n≥1
y divergente cuando |r| ≥ 1.
4. Criterio del cociente (D’Alembert).
an+1
Sea L = lı́m . Entonces,
n→∞ an
P
a) si L < 1, an converge;
n≥1
P
b) si L > 1, an diverge.
n≥1

5. Criterio de la raı́z (Cauchy).



Sea L = lı́m n an . Entonces,
n→∞
P
a) si L < 1, an converge;
n≥1
P
b) si L > 1, an diverge.
n≥1

6. Criterio de Raabe.
 
an+1 P
a) Si lı́m n · 1 − > 1, entonces an converge.
an
 
an+1 P
b) Si lı́m n · 1 − < 1, entonces an diverge.
an
Nota: Este criterio puede ser conveniente en los casos en que los cri-
terios del cociente o de la raı́z no son concluyentes.
7. Criterio de la integral.
Sea f : [1, ∞) → R una función decreciente y f (x) > 0, ∀x. Entonces
X Z ∞
f (n) converge ⇐⇒ f (x)dx converge.
n≥1 1

403
8. Criterio del producto (Pringsheim).
a) Si lı́m np an = L ≥ 0, para algún p > 1, entonces
P
an converge.
b) Si lı́m np an = L > 0, para algún p ≤ 1, entonces
P
an diverge.
9. Criterio logarı́tmico.
log 1/an
Si lı́m = L, entonces
log n
P
a) an converge cuando L > 1.
P
b) an diverge cuando L < 1.

PROBLEMA 9.1.

P
Estudiar el carácter de la serie an de término general
n(n + 1)
an = .
n2 + 2n

Solución

n(n + 1)
Como lı́m = 1 6= 0, la serie es divergente.
n2 + 2n

PROBLEMA 9.2.

Sabiendo que la suma de los n primeros términos de una serie


es
5n2 − 3n + 2
Sn = ,
n2 − 1
hallar el término general y estudiar su naturaleza.

Solución

Aplicamos la fórmula an = Sn − Sn−1 y obtenemos:

5n2 − 3n + 2 5(n − 1)2 − 3(n − 1) + 2 3n2 − 17n + 10


an = − = .
n2 − 1 (n − 1)2 − 1 n4 − 2n3 − n2 + 2n

404
5n2 − 3n + 2
Como además lı́m Sn = lı́m = 5, la serie es convergente.
n2 − 1
Observación: No confundir con la condición necesaria de convergencia en la
que debe ser cero el lı́mite del término general de la serie an , no del término
general de la sucesión de sumas parciales Sn . En este caso, como lı́m Sn = 5,
quiere decir que la suma de la serie es precisamente 5.

PROBLEMA 9.3.

Hallar el mayor valor entero que debe tomar k para que la serie
P nk
an de término general an = sea convergen-
(n + 1)(n + 2)(n + 3)
te.

Solución

Aplicando el criterio logarı́tmico,

log(1/an ) log (n+1)(n+2)(n+3)


nk log(n + 1)(n + 2)(n + 3) − log nk
lı́m = lı́m = lı́m
log n log n log n
3 2
log(n + 6n + 11n + 6) − k log n
= lı́m
log n
log(n3 )(1 + 6/n + 11/n2 + 6/n3 ) − k log n
= lı́m
log n
3 log n + log(1 + 6/n + 11/n2 + 6/n3 ) − k log n
= lı́m
log n
log(1 + 6/n + 11/n2 + 6/n3 )
 
= lı́m 3 − k + = 3 − k.
log n

Para que sea convergente, debe ser 3 − k > 1, y como k debe ser entero, el
mayor valor que hace la serie convergente es k = 1.

PROBLEMA 9.4.

P
Estudiar el carácter de la serie an de término general
1 1
an = √ −√ .
n−1 n+1

405
Solución

Tenemos que
√ √
1 1 n+1− n+1 2
√ −√ = = .
n−1 n+1 n−1 n−1
2/(n − 1) P
Por el criterio de comparación, como lı́m = 2 y la serie 1/n es
1/n
divergente, la serie dada es divergente.

PROBLEMA 9.5.

P
Estudiar el carácter de la serie an de término general
n
an = √ .
2n3 + 1

Solución

Aplicamos el criterio de Prinsgheim, y tenemos:


n nα+1
lı́m nα √ = lı́m √ .
2n3 + 1 2n3 + 1
Para que dicho lı́mite sea real debe ser el grado del numerador igual al grado
del denominador. En este caso α + 1 = 3/2 =⇒ α = 1/2. Como α < 1, la
serie es divergente.

PROBLEMA 9.6.

P
Estudiar el carácter de la serie an de término general
r
n
an = 4
.
n +1

Solución

Aplicando el criterio de Pringsheim, tenemos:


nα+1/2
r
α n
lı́m n = lı́m √ .
n4 + 1 n4 + 1

406
Dicho lı́mite es un número real no nulo cuando α = 3/2. Como es mayor
que uno, la serie es convergente.

PROBLEMA 9.7.

P
Estudiar el carácter de la serie an de término general
1
an = .
1 + np

Solución

1
Según el criterio de Pringsheim, si α = p, lı́m nα = 1. De este modo,
1 + np
cuando p > 1, la serie es convergente y cuando p ≤ 1, la serie es divergente.

PROBLEMA 9.8.

P
Estudiar el carácter de la serie an de término general

x+n−1
an = √ .
x2 + n2 + 1

Solución

Aplicamos nuevamente el criterio de Pringsheim y debemos determinar el


valor de α para que lı́m nα an sea un número real no nulo. Tenemos que

α x+n−1
lı́m n √ = 1 cuando α = 1/2.
x2 + n2 + 1
Como es un valor menor que uno, se deduce que la serie es divergente.

PROBLEMA 9.9.

P
Estudiar el carácter de la serie an de término general
√ √
an = n + 1 − n.

407
Solución

Aplicamos en este caso el criterio de Pringsheim:


√ √ nα
lı́m nα ( n + 1 − n) = lı́m √ √ .
n+1+ n

Este lı́mite es finito cuando α = 1/2 por lo que la serie es divergente.

PROBLEMA 9.10.

P
Estudiar el carácter de la serie an de término general
1
an = √
n
.
n+1

Solución

Como
1 1 n
lı́m an = lı́m √
n
= lı́m n+1 = lı́m = 1 6= 0,
n+1 n
n + 1

la serie es divergente.

PROBLEMA 9.11.

P
Estudiar el carácter de la serie an de término general
n+1
an = ln .
n

Solución

n+1 n+1 1
Debido a la equivalencia de los infinitésimos ln ∼ −1= y
P n n n
como la serie 1/n es divergente, la serie dada también diverge.

408
PROBLEMA 9.12.

P
Estudiar el carácter de la serie an de término general
n!
an = .
n2

Solución

n!
Si calculamos el lı́mite del término general se obtiene que lı́m 2 = ∞ por
n
lo que la serie es divergente.

PROBLEMA 9.13.

P
Estudiar el carácter de la serie an de término general
5 · loga n
an = .
3 · logb n

Solución

Aplicando la fórmula del cambio de base de logaritmos, podemos escri-


bir
5 · (ln n/ ln a) 5 ln b
an = = · .
3 · (ln n/ ln b) 3 ln a
Como el término general es constante, no tiende a cero, por lo que la serie
es divergente.

PROBLEMA 9.14.

P
Estudiar el carácter de la serie an de término general
ln n
an = .
n

409
Solución

ln n 1 P
Por el criterio de comparación, como > y la serie armónica 1/n es
n n
divergente, la serie dada también es divergente.

PROBLEMA 9.15.

Demostrar que las series u1 + u2 + · · · + un + . . . y ln(1 + u1 ) + ln(1 +


u2 ) + · · · + ln(1 + un ) + . . . tienen el mismo carácter si un > 0 y
lı́m un = 0.
n→∞

Solución

Utilizando el criterio de comparación tenemos:


ln(1 + un )
lı́m = lı́m ln(1 + un )1/un = ln lı́m(1 + un )1/un = ln e = 1 6= 0.
un
Esto asegura que ambas series tienen el mismo carácter.

PROBLEMA 9.16.

P
Estudiar el carácter de la serie an de término general

an = arc sen(1/ n).

Solución


arc sen(1/ n)
Debido a que lı́m √ = 1, la serie dada es equivalente a la serie
P √ 1/ n
armónica 1/ n, la cual es divergente.

PROBLEMA 9.17.

P
Estudiar el carácter de la serie an de término general
1 + sen2 n
an = .
n2

410
Solución

1 + sen2 n 2
2/n2 es convergente, por el criterio
P
Como 0 ≤ 2
≤ 2 y la serie
n n
de comparación se deduce la convergencia de la serie dada.

PROBLEMA 9.18.

P
Estudiar el carácter de la serie an de término general
n!
an = .
nn

Solución

Aplicamos el criterio del cociente de D’Alembert:


n−1
n!/nn n!(n − 1)n−1

n−1
lı́m = lı́m = lı́m = e−1 .
(n − 1)!/(n − 1)n−1 nn (n − 1)! n

Como el lı́mite es menor que uno, la serie es convergente.

PROBLEMA 9.19.

P
Estudiar el carácter de la serie an de término general
nn
an = .
3n · n!

Solución

Aplicando el criterio del cociente:

an nn 3n−1 · (n − 1)! 1 nn−1


lı́m = lı́m · = lı́m ·
an−1 3n · n! (n − 1)n−1 3 (n − 1)n−1
 n−1  n−1
1 n 1 1 e
= lı́m = lı́m 1 + = < 1.
3 n−1 3 n−1 3

411
Por tanto la serie dada es convergente.

PROBLEMA 9.20.

P
Estudiar el carácter de la serie an de término general
1 a
an = n
tg n .
2 2

Solución

Aplicando el criterio de D’Alembert:

an+1 tg(a/2n+1 ) 2n 1 a a
lı́m = lı́m n+1
· n
= lı́m tg n+1 · cotg n
an 2 tg(a/2 ) 2 2 2
1 a 2 n 1
= lı́m n+1 · = < 1.
2 2 a 4
Esto prueba que la serie es convergente.

PROBLEMA 9.21.

P
Estudiar el carácter de la serie an de término general
2n x2n
an = respecto a los diversos valores de x.
1 + x2n

Solución

2n
En primer lugar, si x2 = 1 =⇒ an = → ∞ y la serie será divergen-
1+1
te.
x2n
Si x2 > 1 =⇒ lı́m an = lı́m 2n · lı́m = ∞ · 1 = ∞. La serie es
1 + x2n
divergente.
Para x2 < 1 aplicamos el criterio de D’Alembert:

an 2n x2n 1 + x2(n−1) 2x2 (1 + x2n−2 )


lı́m = lı́m · = lı́m = 2x2 ,
an−1 1 + x2n 2n−1 x2(n−1) 1 + x2n

412
pues x2n → 0 y x2n−2 → 0 cuando x2 < 1.

La serie es convergente cuando 2x2 < 1, es decir√ cuando |x| < 2/2 y
divergente cuando 2x2 > 1, es decir cuando |x| > 2/2.
Para el caso en que 2x2 = 1 tenemos x2 = 1/2, de donde:
2n (1/2n ) 1
an = n
= →1
1 + (1/2 ) 1 + (1/2n )

con lo que la serie es también es divergente cuando |x| = 2/2.

PROBLEMA 9.22.

P
Estudiar el carácter de la serie an de término general
n2 + 2n + 2
an = ln .
n2 − 2n + 2

Solución

Si aplicamos el criterio de Pringsheim resulta:


n2 + 2n + 2
 2 
α α n + 2n + 2 4n
lı́m n ln 2 = lı́m n 2
− 1 = lı́m nα 2 .
n − 2n + 2 n − 2n + 2 n − 2n + 2

Si hacemos α = 1, el lı́mite da como resultado 4. De aquı́ se concluye que la


serie es divergente.

PROBLEMA 9.23.

P
Estudiar el carácter de la serie an de término general
2n − 1
an = √ .
( 2)n

Solución

Por el criterio de la raı́z:


s
2n − 1 1 √ 1 2n − 1 1
lı́m n √ = lı́m √ n 2n − 1 = √ lı́m =√ .
( 2)n 2 2 2n − 3 2

413
Como el lı́mite es menor que uno, la serie es convergente.

PROBLEMA 9.24.

P
Estudiar el carácter de la serie an de término general
1
an = .
(ln n)ln n

Solución

Aplicando el criterio logarı́tmico tenemos:

ln (ln n)ln n

ln(1/an ) ln n ln(ln n)
lı́m = lı́m = lı́m = lı́m ln(ln n) = ∞ > 1.
ln n ln n ln n
Esto indica que la serie es convergente.

PROBLEMA 9.25.

P
Estudiar el carácter de la serie an de término general
n+1 a
 
an = ln .
n−1

Solución

 a
2
Comparamos esta serie con la de término general bn = , con lo
n−1
que tenemos:
h  ia    a
2 2
an ln 1 + n−1 ln 1 + n−1
lı́m = lı́m  a = lı́m  2

bn 2
n−1
n−1
"   n−1 #a
2 2
= lı́m ln 1 + = (ln e)a = 1a = 1.
n−1

414
Esto quiere decir que las dos
 series
atienen el mismo carácter y como la serie
2
de término general bn = es una serie armónica, es convergente
n−1
cuando a > 1 y divergente cuando a ≤ 1.

PROBLEMA 9.26.

P
Estudiar el carácter de la serie an de término general
logn a
an = .
loga n

Solución

Aplicando la fórmula del cambio de base en los logaritmos podemos escri-


bir
ln a 2
 
ln a/ ln n
an = = .
ln n/ ln a ln n
Aplicando el criterio logarı́tmico:

ln n 2

ln(1/an ) ln ln a 2 ln(ln n) − 2 ln(ln a)
lı́m = lı́m = lı́m
ln n ln n ln n
ln(ln n) ln(ln a)
= 2 lı́m − 2 lı́m .
ln n ln n
El segundo lı́mite da como resultado cero y para calcular el primero, aplica-
mos el criterio de Stolz:

ln n
ln(ln n) ln(ln n) − ln[ln(n − 1)] ln ln(n−1)
lı́m = lı́m = lı́m  
ln n ln n − ln(n − 1) ln n−1n
 
1 ln n
= lı́m   −1
ln n−1 n ln(n − 1)

1 ln n − ln(n − 1)
= lı́m  ·
ln n ln(n − 1)
n−1
 
n
1 ln n−1 1
= lı́m  · = lı́m = 0.
ln n ln(n − 1) ln(n − 1)
n−1

415
Como el lı́mite es menor que uno, la serie es divergente.

PROBLEMA 9.27.

P
Estudiar el carácter de la serie an de término general
 2n−1
n
an = .
3n − 1

Solución

Por el criterio de la raı́z de Cauchy:


s 2n−1   2n−1
n n n n
lı́m = lı́m = (1/3)2 = 1/9.
3n − 1 3n − 1
Como el lı́mite es menor que uno, la serie es convergente.

PROBLEMA 9.28.

P
Estudiar el carácter de la serie an de término general
n+1 n
 
an = .
2n − 1

Solución

Aplicamos nuevamente el criterio de la raı́z:

s n
n n+1 n+1 1
lı́m = lı́m = < 1.
2n − 1 2n − 1 2
Se deduce que la serie es convergente.

PROBLEMA 9.29.

P
Estudiar el carácter de la serie an de término general
 sen a n
an = (a fijo).
n

416
Solución

Por el criterio de Raabe,


 
sen a n

(n − 1)n sen a
 
n
lı́m n 1 −  n−1  = lı́m n 1 −
 
sen a nn (n − 1)
n−1

n − 1 n sen a
   
= lı́m n 1 − = ∞ · 1 = ∞.
n n−1
Como el lı́mite es mayor que uno, la serie es convergente.

PROBLEMA 9.30.

P
Estudiar el carácter de la serie an de término general
 
b
an = tg n a + con 0 < a < π/2.
n

Solución

Aplicamos el criterio de la raı́z:


 
√ b
lı́m n
an = lı́m tg a + = tg a.
n
De aquı́ se deduce que si 0 < a < π/4, la serie es convergente pues el lı́mite
anterior es menor que uno.
Si π/4 < a < π/2, el citado lı́mite es mayor que uno por lo que la serie es
divergente.
Para a = π/4 se tiene:
tg(π/4) + tg(b/n) n
   
π
n b
lı́m an = lı́m tg + = lı́m
4 n 1 − tg(π/4) tg(b/n)
 n
1 + tg(b/n)
= lı́m = eL ,
1 − tg(b/n)
donde
 
1 + tg(b/n) 2 tg(b/n)
L = lı́m n − 1 = lı́m n ·
1 − tg(b/n) 1 − tg(b/n)
2
= lı́m n tg(b/n) · lı́m = 2b.
1 − tg(b/n)

417
Por lo tanto, lı́m an = e2b 6= 0 y la serie es divergente.

PROBLEMA 9.31.

P
Estudiar el carácter de la serie an de término general
nln n
an = .
(ln n)n

Solución

Aplicamos el criterio de Cauchy o de la raı́z:

√ nln n/n
lı́m n
an = lı́m .
ln n
Tomando logaritmos resulta:

(ln n)2
   
√ ln n
lı́m ln n
an = lı́m ln n − ln(ln n) = lı́m − ln(ln n) .
n n

Utilizamos el criterio de Stolz para calcular el lı́mite del primer suman-


do:

(ln n)2 (ln n)2 − [ln(n − 1)]2


lı́m = lı́m
n n − (n − 1)
= lı́m[ln n + ln(n − 1)][ln n − ln(n − 1)]
n
= lı́m ln n(n − 1) ln
n−1
ln(n2 − n)
 
n
= lı́m ln(n2 − n) − 1 = lı́m
n−1 n−1
2 2
ln(n − n) − ln[(n − 1) − (n − 1)]
= lı́m
n − 1 − (n − 1 − 1)
2
n −n
= lı́m ln 2 = ln 1 = 0.
n − 3n + 2

Como el lı́mite del segundo sumando es lı́m ln(ln n) = +∞, resulta que
√ √
lı́m ln n
an = −∞ =⇒ lı́m n
an = 0 < 1,

de modo que la serie es convergente.

418
PROBLEMA 9.32.

P
Estudiar el carácter de la serie an de término general
1
an = √ n.
(1 + 1/ n)

Solución

Aplicamos el criterio logarı́tmico:



n
 √ n  n
√1

ln 1 + √1 ln 1+ n
ln(1/an ) n
lı́m = lı́m = lı́m
ln n ln n ln n
√ √ n
√ n

n ln 1 + √1 √ 
n n 1
= lı́m = lı́m lı́m ln 1 + √ .
ln n ln n n
Es evidente que el lı́mite del segundo factor es 1. Utilizaremos el criterio de
Stolz para calcular el lı́mite del primer factor:
√ √ √
n n− n−1 n − (n − 1) 1
lı́m = lı́m = lı́m √ √ · n
ln n ln n − ln(n − 1) n + n − 1 ln n−1
1 n−1
= lı́m √ √   = lı́m √ √ = +∞.
( n + n − 1) n − 1 n+ n−1
n−1

ln(1/an )
En definitiva, lı́m = +∞ > 1 y la serie es convergente.
ln n

PROBLEMA 9.33.

P
Estudiar el carácter de la serie an de término general
" #−n
n + 1 n+1 n + 1

an = − .
n n

Solución

Por el criterio de la raı́z:


v" #−n
u  n+1
u
n n + 1 n + 1 1 1
lı́m t − = lı́m = < 1.
n n n+1 n+1 n+1 e−1

n − n

419
Esto muestra que la serie es convergente.

PROBLEMA 9.34.

P
Estudiar el carácter de la serie an de término general
 n −n
n+1 2n + 1
an = + .
n n

Solución

Aplicando el criterio de la raı́z:


√ 1 1 1
lı́m n
an = lı́m n+1 n
 2n+1
= lı́m 1 n
 2n+1
= < 1.
n + n 1+ n + n
e+2

La serie es convergente.

PROBLEMA 9.35.

P
Estudiar el carácter de la serie an de término general
nn
an = .
1 · 3 · 5 · . . . (2n − 3)(2n − 1)

Solución

Aplicando el criterio del cociente de D’Alembert:

an nn 1 · 3 · 5 · . . . (2n − 3)
lı́m = lı́m ·
an−1 1 · 3 · 5 · . . . (2n − 3)(2n − 1) (n − 1)n−1
1 nn n nn−1
= lı́m · = lı́m ·
2n − 1 (n − 1)n−1 2n − 1 (n − 1)n−1
 n−1
n n 1
= lı́m lı́m = · e > 1.
2n − 1 n−1 2

Por tanto la serie es divergente.

420
PROBLEMA 9.36.

P
Estudiar el carácter de la serie an de término general
ln 2 · ln 3 . . . ln n
an = .
n!

Solución

Aplicando el criterio del cociente de D’Alembert:


an ln 2 · ln 3 . . . ln n (n − 1)!
lı́m = lı́m ·
an−1 n! ln 2 · ln 3 . . . ln(n − 1)
ln n
= lı́m = 0 < 1.
n
Entonces se trata de una serie convergente.

PROBLEMA 9.37.

P
Estudiar el carácter de la serie an de término general
n!
an = .
(a + 1)(a + 2) . . . (a + n)

Solución

Aplicamos el criterio del cociente:

n! (a + 1)(a + 2) . . . (a + n − 1) n
lı́m · = lı́m = 1.
(a + 1)(a + 2) . . . (a + n) (n − 1)! a+n
El criterio no permite decidir sobre la convergencia de la serie por lo que
aplicamos el criterio de Raabe:
 
n an
lı́m n 1 − = lı́m = a.
a+n a+n

Resulta que si a < 1, la serie es divergente; si a > 1, la serie es convergen-


te.

421
Cuando a = 1, sustituimos este valor en la serie y obtenemos
X n! X 1
=
2 · 3 · · · · · (n + 1) n+1
la cual es evidentemente divergente.

PROBLEMA 9.38.

P
Estudiar el carácter de la serie an de término general
1 · 3 · 5 · · · · · (2n − 1)
an = .
2 · 4 · 6 · · · · · (2n + 2)

Solución

Aplicaremos el criterio de D’Alembert:


1·3·5·····(2n−1)
an 2·4·6·····(2n+2) 2n − 1
lı́m = lı́m 1·3·5·····(2n−3)
= lı́m = 1.
an−1 2n + 2
2·4·6·····(2n)

Como este criterio no decide el carácter de la serie, aplicamos el criterio de


Raabe:
   
an 2n − 1 3n 3
lı́m n 1 − = lı́m n 1 − = lı́m = .
an−1 2n + 2 2n + 2 2
Como el lı́mite es mayor que uno, la serie es convergente.

PROBLEMA 9.39.

P
Estudiar el carácter de la serie an de término general
−n2 x
an = e según los valores de x.

Solución

Por el criterio de Raabe, tenemos:


2
!
e−n x  2 2
  
lı́m n 1 − −(n−1)2 x = lı́m n 1 − e−n x+n x+x−2nx = lı́m n 1 − ex(1−2n) .
e

422
P
Cuando x = 0, la serie dada es 1 que es evidentemente divergente.
Cuando x < 0, lı́m n 1 − ex(1−2n) = −∞ < 1 por lo que la serie es diver-


gente.
Cuando x > 0, lı́m n 1 − ex(1−2n) = +∞ > 1 por lo que la serie es conver-


gente.

PROBLEMA 9.40.

P
Estudiar el carácter de la serie an de término general
s
α(α + 1) . . . (α + n − 1)
an = según los valores de α y β .
β(β + 1) . . . (β + n − 1)

Solución

Por el criterio de Raabe:


 q 
  α(α+1)...(α+n−1)  r 
an β(β+1)...(β+n−1) α+n−1
lı́m n 1 − = lı́m n 1 −
 q  = lı́m n 1 −
an−1 α(α+1)...(α+n−2) β+n−1
β(β+1)...(β+n−2)
√ √ 
β+n−1− α+n−1
= lı́m n √
β+n−1
β+n−1−α−n+1
= lı́m n · √ √ √
β + n − 1( β + n − 1 + α + n − 1)
n(β − α) β−α
= lı́m √ = .
2
β + n − 1 + n + ... 2

De aquı́ se deduce que si β − α > 2, la serie es convergente. Si β − α < 2, la


serie es divergente.

sque β −α = 2, es decir β = α+2, al sustituir en la serie original


En el caso en
X α(α + 1)
resulta . Aplicando ahora el criterio de Pringsheim,
(α + n)(α + n + 1)
s
α(α + 1)
resulta que lı́m np es finito y no nulo cuando p = 1 lo
(α + n)(α + n + 1)
que hace que la serie sea divergente.
En definitiva, la serie es convergente si y sólo si β − α > 2.

423
PROBLEMA 9.41.


X 1
Calcular la suma de la serie √ .
n=1
n2 − 2 2n + 1

Solución

Si descomponemos el término general en fracciones simples, obtenemos:


1 A B
√ = √ + √ .
n2
− 2 2n + 1 n− 2−1 n− 2+1
√ √
Esto implica que 1 = A(n − 2 + 1) + B(n − 2 − 1) por lo que A = 1/2 y
B = −1/2.
Sumando ahora los n primeros términos de la sucesión tenemos:
1/2 1/2
an = √ − √
n− 2−1 n− 2+1
1/2 1/2
an−1 = √ − √
n− 2−2 n− 2
1/2 1/2
an−2 = √ − √
n− 2−3 n− 2−1
...
1/2 1/2
a2 = √ − √
1− 2 3− 2
1/2 1/2
a1 = √ − √
− 2 2− 2
 
1 1 1 1 1
Sn = √ + √ − √ − √ .
2 1− 2 − 2 n− 2+1 n− 2
 
X 1 1 1
En definitiva, S = an = lı́m Sn = √ + √ .
2 1− 2 − 2

PROBLEMA 9.42.

n + 12
Dada la serie de término general an = , demostrar
n3 + 5n2 + 6n
que es convergente y sumarla.

424
Solución

np (n + 12)
Por el criterio de Pringsheim, lı́m np an = lı́m = 1 cuando p =
n3 + 5n2 + 6n
2 > 1, por lo que la serie es convergente.
Para sumar la serie descomponemos el término general en fracciones sim-
ples:
n + 12 A B C
an = = + +
n3 + 5n2 + 6n n n+2 n+3
A(n + 2)(n + 3) + Bn(n + 3) + Cn(n + 2)
=
n(n + 2)(n + 3)
=⇒ n + 12 = A(n + 2)(n + 3) + Bn(n + 3) + Cn(n + 2).
Para n = 0, 12 = 6A =⇒ A = 2.
Para n = −2, 10 = −2B =⇒ B = −5.
Para n = −3, 9 = 3C =⇒ C = 3.
De aquı́ obtenemos:
2 5 3
an = − +
n n+2 n+3
2 5 3
an−1 = − +
n−1 n+1 n+2
2 5 3
an−2 = − +
n−2 n n+1
2 5 3
an−3 = − +
n−3 n−1 n
...
2 5 3
− +
a4 =
4 6 7
2 5 3
a3 = − +
3 5 6
2 5 3
a2 = − +
2 4 5
2 5 3
a1 = − +
1 3 4
2 2 3 3 2 2
Sn = − − + − + + =⇒ S = lı́m Sn = −1 + 1 + 2 = 2.
n+1 n+2 n+3 3 2 1

PROBLEMA 9.43.

1 1 1
Sumar la serie + + + . . ..
1·3·5 3·5·7 5·7·9

425
Solución

1
El término general de la serie es an = . Al descom-
(2n − 1)(2n + 1)(2n + 3)
ponerlo en fracciones simples resulta:
A B C
an = + +
2n − 1 2n + 1 2n + 3
A(2n + 1)(2n + 3) + B(2n − 1)(2n + 3) + C(2n − 1)(2n + 1)
=
(2n − 1)(2n + 1)(2n + 3)
=⇒ A(2n + 1)(2n + 3) + B(2n − 1)(2n + 3) + C(2n − 1)(2n + 1) = 1
=⇒ A = 1/8, B = −1/4, C = 1/8.

Por tanto,
 
1 1 2 1
an = − +
8 2n − 1 2n + 1 2n + 3
 
1 1 2 1
an−1 = − +
8 2n − 3 2n − 1 2n + 1
 
1 1 2 1
an−2 = − +
8 2n − 5 2n − 3 2n − 1
...  
1 1 2 1
a2 = − +
8 3 5 7
 
1 1 2 1
a1 = − +
8 1 3 5
 
1 1 1 1
Sn = − +1− .
8 2n + 3 2n + 1 3
 
1 1 1
Tenemos entonces que S = lı́m Sn = 1− = .
8 3 12

PROBLEMA 9.44.


X 1
Sumar la serie n+3
.
n=1 3

426
Solución

3!
Escribimos el término general en la forma an = y lo
(n + 3)(n + 2)(n + 1)
descomponemos en fracciones simples:

6 A B C
= + + .
(n + 3)(n + 2)(n + 1) n+3 n+2 n+1

Esto implica que 6 = A(n + 2)(n + 1) + B(n + 3)(n + 1) + C(n + 3)(n + 2) lo


que al resolver produce los valores A = 3, B = −6, C = 3. Sumando ahora
los n primeros términos de la sucesión:

3 6 3
an = − +
n+3 n+2 n+1
3 6 3
an−1 = − +
n+2 n+1 n
3 6 3
an−2 = − +
n+1 n n−1
...
3 6 3
a2 = − +
5 4 3
3 6 3
a1 = − +
4 3 2
3 6 3 3 6 3
Sn = − + + − + .
n+3 n+2 n+2 3 3 2

Entonces S = lı́m Sn = 1/2.

PROBLEMA 9.45.


X ln n+1
n
Sumar la serie .
ln n ln(n + 1)
n=2

Solución

Escribimos el término general como

ln(n + 1) − ln n 1 1
an = = − .
ln n · ln(n + 1) ln n ln(n + 1)

427
Sumando los primeros términos de la sucesión resulta:
1 1
an = −
ln n ln(n + 1)
1 1
an−1 = −
ln(n − 1) ln n
...
1 1
a3 = −
ln 3 ln 4
1 1
a2 = −
ln 2 ln 3
1 1
Sn = − .
ln 2 ln(n + 1)

Entonces S = lı́m Sn = 1/ ln 2.

PROBLEMA 9.46.
 
X 1
Sumar la serie ln 1 − 2 .
n
n≥2

Solución

Escribimos el término general de la forma:

n2 − 1 (n + 1)(n − 1)
an = ln 2
= ln = ln(n + 1) − 2 ln n + ln(n − 1).
n n2
Dando valores decrecientes a n tenemos:

an = ln(n + 1) − 2 ln n + ln(n − 1)
an−1 = ln n − 2 ln(n − 1) + ln(n − 2)
an−2 = ln(n − 1) − 2 ln(n − 2) + ln(n − 3)
...
a4 = ln 5 − 2 ln 4 + ln 3
a3 = ln 4 − 2 ln 3 + ln 2
a2 = ln 3 − 2 ln 2 + ln 1.
n+1
Sn = ln(n + 1) − ln n − ln 2 = ln − ln 2.
n
La suma de la serie es S = lı́m Sn = ln 1 − ln 2 = − ln 2.

428
PROBLEMA 9.47.

Estudiar el carácter y hallar la suma de la serie


X 2n + 1
.
7n
n≥1

Solución

Aplicando el criterio de D’Alembert,

an 2n + 1 7n−1 1 2n + 1 1
lı́m = lı́m n
· = lı́m · = < 1.
an−1 7 2(n − 1) + 1 7 2n − 1 7
La serie es convergente.
3 5 2n + 1
Para hallar su suma escribimos Sn = + 2 + · · · + . Los términos
7 7 7n
de la serie resultan de multiplicar los términos de la progresión aritmética
3, 5, . . . 2n+1 por los correspondientes de la progresión geométrica 1/7, 1/72 , . . . 1/7n .
Estas series, llamadas aritmético-geométricas, se suman de la siguiente for-
ma:
3 5 2n − 1 2n + 1
Sn = + 2 + · · · + n−1 +
7 7 7 7n
1 3 5 2n − 1 2n + 1
Sn = 2 + 3 + · · · + + n+1
7 7 7 7n 7
Restando:
6 3 2 2 2 2n + 1
Sn = + 2 + 3 + · · · + n − n+1
7 7 7 7 7 7
2 2
3 n+1 − 2 2n + 1
= + 7 1 7
− n+1 .
7 7 − 1 7
2n + 1
Como lı́m = 0, resulta que la suma de la serie es:
7n+1
6 3 2/49 10 5
S= + = =⇒ S = .
7 7 6/7 21 9

PROBLEMA 9.48.

n2 xn , 0 < x < 1.
P
Sumar la serie
n≥1

429
Solución

El proceso que seguiremos es el siguiente:

Sn = x + 4x2 + 9x3 + · · · + (n − 1)2 xn−1 + n2 xn


xSn = x2 + 4x3 + · · · + (n − 2)2 xn−1 + (n − 1)2 xn + n2 xn+1 .

Restando miembro a miembro:

(1 − x)Sn = x + 3x2 + 5x3 + · · · + (2n − 1)xn − n2 xn+1


x(1 − x)Sn = x2 + 3x3 + · · · + (2n − 3)xn + (2n − 1)xn+1 − n2 xn+2 .

Restando nuevamente las dos últimas igualdades:

(1 − x)2 Sn = x + 2x2 + 2x3 + · · · + 2xn − (n2 + 2n − 1)xn+1 + n2 xn+2


xn+1 − x2
= x+2· − (n2 + 2n − 1)xn+1 + n2 xn+2 .
x−1

Como 0 < x < 1, (n2 + 2n − 1)xn+1 → 0 y n2 xn+2 → 0 cuando n →


∞. Resulta entonces que si llamamos S = lı́m Sn a la suma de la serie,
tenemos:
2x2 x2 + x
(1 − x)2 S = x − =⇒ S = .
x−1 (1 − x)3

430
B. EJERCICIOS PROPUESTOS.

1.- Estudiar la convergencia de las siguientes series:


X nn
a) .
(2n + 1)n
Resp.: Convergente (raı́z).

X 24n−3
b) .
(4n − 3)!
Resp.: Convergente (cociente).

X n
c) .
en
Resp.: Convergente (cociente).

X 2n
d) .
1 · 3 · 5 . . . (2n + 1)
Resp.: Convergente (cociente).

X cos2 n
e) .
n2
1/n2 ).
P
Resp.: Convergente (comparación con


3
X n+2
f) .
n3 +1
1/n8/3 ).
P
Resp.: Convergente (comparación con

X n2
g) .
n!
Resp.: Convergente (cociente).

X nn · n!
h) .
(3n)!
Resp.: Convergente (cociente).

X (2n)!
i) .
(n!)2

431
Resp.: Divergente (cociente).

X 1
j) .
(ln n!) + n2
1/n2 ).
P
Resp.: Convergente (comparación con

1 1·3 1·3·5
k) + + + ...
3 3·6 3·6·9
Resp.: Convergente (cociente).

X 1 · 3 . . . (2n − 1)
l) .
2 · 4 . . . 2n
Resp.: Divergente (Raabe).

X 1
m) p .
n(n + 1)
P
Resp.: Divergente (comparación con 1/n).

X 2n
n) .
n
Resp.: Divergente (cociente).

X 2 · 5 · 8 . . . (3n − 1)
o) .
1 · 5 · 9 . . . (4n − 3)
Resp.: Convergente (cociente).

X nn/2 · 5n
p) .
n!
Resp.: Convergente (raı́z).

X 1
q) .
(3n − 2)(3n + 1)
1/n2 ).
P
Resp.: Convergente (comparación con

X 1
r) .
n ln 1 + n1
Resp.: Divergente (lı́m an 6= 0).

432
X 1 · 11 · 21 . . . (10n − 9)
s) .
(2n − 1)!
Resp.: Divergente (cociente).

X n!
t) .
nn
Resp.: Convergente (raı́z).


X 2n senn 3
u) .
en n2
Resp.: Convergente (raı́z).

X 1
v) .
n ln n
Resp.: Divergente (integral).

2.- Calcular la suma de las siguientes series:


X 3n + 5
a) .
2n
n≥1

Resp.: S = 11.

X
b) n(n − 1)xn para |x| < 1.
n≥1

2x2
Resp.: S = .
(1 − x)3

√ √
X n+1− n
c) √ .
n≥1
n2 + n

Resp.: S = 1.

X 1
d) .
(3n + 2)(3n + 8)
n≥1

Resp.: S = 13/240.

X  n − 1 2
e) .
en
n≥2

433
e2 + 1
Resp.: S = .
(e2 − 1)3

X 2n + 3
f) .
(n − 1)n(n + 2)
n≥2

Resp.: S = 65/36.

X n
g) .
(4n2 − 1)2
n≥1

Resp.: S = 1/8.

X
h) nen .
n≥1

Resp.: S = ∞.

X 2n2 + n − 1
i) .
en
n≥1

2e2 − 2e + 9
Resp.: S = .
(e − 1)3

434
CAPÍTULO X.
INTEGRACIÓN
DEFINIDA

SECCIONES
A. Definición de función integrable. Primeras propiedades.
B. Teoremas fundamentales del cálculo integral.
C. Ejercicios propuestos.

1
A. DEFINICIÓN DE FUNCIÓN INTEGRABLE. PRIMERAS
PROPIEDADES.

El concepto de integral definida tiene su origen en el problema de calcular


áreas de figuras planas limitadas por lı́neas curvas. En este capı́tulo nos li-
mitaremos a definir y establecer las principales propiedades de la integral
definida y en el siguiente capı́tulo veremos de qué manera aplicar este con-
cepto al cálculo de áreas. Los pasos fundamentales para definir este nuevo e
importante concepto se esbozan a continuación.
Se define partición de un intervalo cerrado [a, b] a cualquier conjunto orde-
nado de puntos P = {x0 , x1 , . . . , xn } que verifiquen la propiedad a = x0 <
x1 < . . . < xn = b.
Toda partición determina n subintervalos [x0 , x1 ], [x1 , x2 ], . . . , [xn−1 , xn ].
Una partición P de [a, b] es regular si dos puntos consecutivos cualesquiera
son equidistantes, es decir cuando xi −xi−1 = (b−a)/n, i = 1, 2, . . . , n.

INTEGRAL DE FUNCIONES ESCALONADAS


Una función y = s(x) definida en [a, b] se dice escalonada cuando existe
una partición P de [a, b] tal que s es constante en cada subintervalo abierto
de P , es decir cuando existen constantes s1 , . . . , sn tales que s(x) = si , si
x ∈ (xi−1 , xi ), para i = 1, . . . , n.
Se define la integral de s entre a y b como la cantidad
Z b n
X
s(x) dx = si · (xi − xi−1 ).
a i=1

A los números a y b se les llama lı́mites o extremos de integración y a la


función y = s(x) se le llama función integrando.
Observación. Si la función es no negativa en [a, b], es decir si si ≥ 0, ∀i, la
definición de integral coincide precisamente con la suma de las áreas de los
rectángulos que la función determina con el eje OX. Este hecho, que motiva
la definición de integral, lo aplicaremos en el capı́tulo siguiente para calcular
áreas de figuras planas.
Para que tengan sentido integrales con lı́mites de integración arbitrarios,
utilizaremos en lo sucesivo los siguientes convenios:
Z b Z a Z a
s(x) dx = − s(x) dx, s(x) dx = 0.
a b a

2
PROPIEDADES. Sean s y t dos funciones escalonadas en [a, b].
Z b Z b Z b
1. Aditiva: [s(x) + t(x)] dx = s(x) dx + t(x) dx.
a a a
Z b Z b
2. Homogénea: k · s(x) dx = k · s(x) dx.
a a
Z b Z b
3. Monotonı́a: Si s(x) ≤ t(x), ∀x ∈ [a, b], entonces s(x) dx ≤ t(x) dx.
a a
Z b Z c Z b
4. Unión: Si a < c < b, entonces s(x) dx = s(x) dx + s(x) dx.
a a c
Z b Z b+c
5. Traslación: ∀c ∈ R, s(x) dx = s(x − c) dx.
a a+c
Z b Z kb
6. Cambio de escala: ∀k 6= 0, s(x) dx = (1/k) s(x/k) dx.
a ka

INTEGRAL DE FUNCIONES ACOTADAS ARBITRARIAS


Una función y = f (x) definida y acotada en [a, b] es integrable en [a, b]
Z b Z b
cuando existe un único número I tal que s(x) dx ≤ I ≤ t(x) dx, para
a a
cada par de funciones escalonadas s y t tales que s(x) ≤ f (x) ≤ t(x),
∀x ∈ [a, b].
Z b
Al valor I = f (x) dx se le llama integral de f desde a hasta b. La función
a
f se llama integrando (o función sub-integral) y a, b son los lı́mites o extremos
de integración.
Otra definición de función integrable se expresa en términos de las integrales
superior e inferior, definidas de la siguiente manera:
Sea f una función acotada en [a, b]. Definimos los conjuntos
Z b 
S = s(x) dx : s es una función escalonada con s(x) ≤ f (x), ∀x ,
a
Z b 
T = t(x) dx : t es una función escalonada con t(x) ≥ f (x), ∀x
a

que, al ser no vacı́os y acotados, tienen supremo e ı́nfimo. Llamamos entonces


integral inferior de f al número real I(f ) = sup S e integral superior de f a
S(f ) = ı́nf T . Es evidente que S ≤ T . Decimos entonces que f es integrable
en [a, b] cuando I(f ) = S(f ), es decir cuando coinciden las integrales inferior
y superior de f .

3
Una simplificación muy común en la práctica consiste en considerar una
partición arbitraria de [a, b] y sustituir los conjuntos S y T por
n
X
I(f, P ) = (xk − xk−1 ) · f (uk ), donde uk = ı́nf{f (x), x ∈ (xk−1 , xk )},
k=1
Xn
S(f, P ) = (xk − xk−1 ) · f (vk ), donde vk = sup{f (x), x ∈ (xk−1 , xk )}.
k=1

De este modo, la función s(x) = f (uk ), ∀x ∈ (xk−1 , xk ), k = 1, . . . , n, es


escalonada con s ≤ f y, también, la función t(x) = f (vk ), ∀x ∈ (xk−1 , xk ), k =
1, . . . , n, es escalonada con f ≤ t, como se observa en las figuras adjun-
tas.

En este contexto, las integrales superior e inferior se definen como

I(f ) = sup {I(f, P ) : P es partición de [a, b]} ,


S(f ) = ı́nf {S(f, P ) : P es partición de [a, b]} .

Entre los ejemplos de funciones integrables podemos destacar las funciones


continuas en [a, b] y las funciones monótonas. Incluso las funciones monóto-
nas a trozos (monótonas en subintervalos de [a, b]) son integrables.

4
Las propiedades ya enunciadas 1 a 5 de las integrales de funciones escalo-
nadas son válidas también para las funciones integrables. En algunos de los
problemas que siguen se demuestran algunas de estas y otras propiedades
de las funciones integrables.

PROBLEMA 10.1
Calcular las integrales de las siguientes funciones en los intervalos
que se indican:
a) f (x) = [x] en [0, n], con n ∈ N.
b) f (x) = [x]2 en [0, n], con n ∈ N.
c) f (x) = [x2 ] en [0, 2].

d) f (x) = [ x] en [0, 9].
e) f (x) = [ex ] en [0, 2].

Solución
a) Como f (x) = k cuando x ∈ [k, k + 1), donde k = 0, 1, . . . , n − 1, se trata
de una función escalonada. Por tanto, su integral vale:

n n−1
n(n − 1)
Z X
[x] dx = k · 1 = 1 + 2 + · · · + (n − 1) = .
0 2
k=0

b) Análogamente al caso anterior, tenemos una función escalonada que


toma los valores f (x) = k 2 en los intervalos x ∈ [k, k + 1), con k =
0, 1, . . . , n − 1. Entonces,

n n−1
n(n − 1)(2n − 1)
Z X
2
[x] dx = k 2 = 12 + 22 + · · · + (n − 1)2 = ,
0 6
k=0

resultado que se puede probar por inducción (ver capı́tulo 1).


c) En primer lugar debemos determinar los sub-intervalos de [0, 2] donde
la función es constante. Estos son los siguientes:

0≤x<1 =⇒ 0 ≤ x2 < 1 =⇒ [x2 ] = 0;



1≤x< 2 =⇒ 1 ≤ x2 < 2 =⇒ [x2 ] = 1;
√ √
2≤x< 3 =⇒ 2 ≤ x < 3 =⇒ [x2 ] = 2;

3≤x<2 =⇒ 3 ≤ x2 < 4 =⇒ [x2 ] = 3.

5
La integral se puede descomponer entonces en la suma siguiente:
Z 2 √ √ √ √ √ √
[x2 ] dx = 1 · ( 2 − 1) + 2 · ( 3 − 2) + 3 · (2 − 3) = 5 − 2 − 3.
0

d) Descomponemos nuevamente el intervalo de integración en sub-intervalos


donde la función sea constante:
√ √
0 ≤ x < 1 =⇒ 0 ≤ x < 1 =⇒ [ x] = 0;
√ √
1 ≤ x < 4 =⇒ 1 ≤ x < 2 =⇒ [ x] = 1;
√ √
4 ≤ x < 9 =⇒ 2 ≤ x < 3 =⇒ [ x] = 2.

La integral es ahora
Z 3

[ x] dx = 1 · (4 − 1) + 2 · (9 − 4) = 13.
0

e) Como es también una función parte entera, es escalonada; los intervalos


donde es constante son los siguientes:

0 ≤ x < ln 2 =⇒ 1 ≤ ex < 2 =⇒ [ex ] = 1;


ln 2 ≤ x < ln 3 =⇒ 2 ≤ ex < 3 =⇒ [ex ] = 2;
ln 3 ≤ x < ln 4 =⇒ 3 ≤ ex < 4 =⇒ [ex ] = 3;
..
.
ln 7 ≤ x < 2 =⇒ 7 ≤ ex < e2 =⇒ [ex ] = 7.

(Téngase en cuenta que el intervalo de integración es [0, 2] y ln 7 <


2 < ln 8.)
La integral es la siguiente:
Z 2 6
X
x
[e ] dx = k · [ln(k + 1) − ln k] + 7 · (2 − ln 7)
0 k=1
= 6 · ln 7 − ln 6 − ln 5 − · · · − ln 2 + 14 − 7 · ln 7 = 14 − ln(7!).

PROBLEMA 10.2
b
|x|
Z
Calcular la integral dx, donde a < b.
a x

6
Solución
(
|x| 1 si x > 0
La función integrando es escalonada porque = . Podemos
x −1 si x < 0
distinguir tres casos:
Z b
|x|
i) a < b ≤ 0: dx = (−1) · (b − a) = a − b.
a x

ii) a < 0 ≤ b: descomponemos la integral en dos sumandos. Ası́:

b 0 b
|x| |x| |x|
Z Z Z
dx = dx + dx = (−1) · (0 − a) + 1 · (b − 0) = a + b.
a x a x 0 x

b
|x|
Z
iii) 0 ≤ a < b: dx = 1 · (b − a) = b − a.
a x

PROBLEMA 10.3
Hallar I(f, P ) y S(f, P ) en los siguientes casos:

a) f (x) = x, x ∈ [0, 1], P = {0, 1/25, 4/25, 9/25, 16/25, 1}.
b) f (x) = x2 , x ∈ [−1, 1], P = {−1, −1/4, 1/4, 1/2, 1}.

Solución

a) Como la función es creciente, el ı́nfimo se alcanza en el extremo izquierdo


y el supremo en el extremo derecho de cada subintervalo de P . De esta
forma,

I(f, P ) = 1/25 · f (0) + (4/25 − 1/25) · f (1/25) + (9/25 − 4/25) · f (4/25)


+(16/25 − 9/25) · f (9/25) + (1 − 16/25) · f (16/25)
= 3/25 · 1/5 + 5/25 · 2/5 + 7/25 · 3/5 + 9/25 · 4/5 = 14/25;
S(f, P ) = 1/25 · f (1/25) + (4/25 − 1/25) · f (4/25) + (9/25 − 4/25) · f (9/25)
+(16/25 − 9/25) · f (16/25) + (1 − 16/25) · f (1)
= 1/25 · 1/5 + 3/25 · 2/5 + 5/25 · 3/5 + 7/25 · 4/5 + 9/25 · 1 = 19/25.

b) La función y = x2 es decreciente cuando x ∈ (−1, 0) y creciente cuando


x ∈ (0, 1). Además en el intervalo (−1/4, 1/4), el ı́nfimo de la función
se alcanza cuando x = 0 y el supremo cuando x = 1/4. Por tanto, en

7
este caso tenemos:

I(f, P ) = (1 − 1/4) · f (−1/4) + (1/4 + 1/4) · f (0) + (1/2 − 1/4) · f (1/4)


+(1 − 1/2) · f (1/2)
= 3/4 · 1/16 + 1/2 · 0 + 1/4 · 1/16 + 1/2 · 1/4 = 3/16;
S(f, P ) = (1 − 1/4) · f (−1) + (1/4 + 1/4) · f (1/4) + (1/2 − 1/4) · f (1/2)
+(1 − 1/2) · f (1) = 3/4 · 1 + 1/2 · 1/16 + 1/4 · 1/4 + 1/2 · 1 = 43/32.

Como se puede observar, en ambos casos se verifica que I(f, P ) ≤


S(f, P ), lo cual es siempre cierto.

PROBLEMA 10.4
Dada la función f (x) = 1 + 2x, si P = {x0 , x1 , . . . , xn } es una parti-
ción regular de [a, b], calcular I(f, P ) y S(f, P ). Utilizar lo anterior
Z b
para calcular (1 + 2x) dx.
a

Solución

Como la función y = 1 + 2x es creciente, el ı́nfimo en cada subintervalo


(xi−1 , xi ) se alcanza en xi−1 y el supremo se alcanza en xi . Además, por
tratarse de una partición regular, los puntos son equidistantes y

b−a b−a
xi − xi−1 = , xi = a + i , ∀i = 1, . . . , n. De este modo, por defi-
n n
nición:

n n
X X b−a
I(f, P ) = (xi − xi−1 ) · f (xi−1 ) = · (1 + 2xi−1 )
n
i=1 i=1
n n  !
b−a X X b−a
= 1+2 a + (i − 1) ·
n n
i=1 i=1
n
!
b−a b−aX
= n + 2an + 2 (i − 1)
n n
i=1
b − a 2 n(n − 1)
 
= (b − a) + 2a(b − a) + 2 ·
n 2
 
(b − a)(n − 1)
= (b − a) 1 + 2a + ;
n

8
n n
X X b−a
S(f, P ) = (xi − xi−1 ) · f (xi ) = · (1 + 2xi )
n
i=1 i=1
n n  !
b−a X X b−a
= 1+2 a+i·
n n
i=1 i=1
n
!
b−a b−aX
= n + 2an + 2 i
n n
i=1
b − a 2 n(n + 1)
 
= (b − a) + 2a(b − a) + 2 ·
n 2
 
(b − a)(n + 1)
= (b − a) 1 + 2a + .
n
Basta calcular el lı́mite de cualquiera de las expresiones obtenidas, cuando
n → ∞, para obtener el valor de la integral propuesta. Ası́ tenemos
Z b  
(b − a)(n + 1)
(1 + 2x) dx = lı́m (b − a) 1 + 2a + = (b − a)(1 + a + b).
a n→∞ n

PROBLEMA 10.5
(
x si x ∈ Q
Probar que la función f (x) = no es integrable en
0 si x ∈
6 Q,
[a, b], con a, b > 0.

Solución
Debemos comprobar que las integrales superior e inferior no coinciden. Pa-
ra ello consideramos cualquier partición regular P = {x0 , x1 , . . . , xn } del
intervalo [a, b]. En cualquier subintervalo (xi−1 , xi ) de P hay infinitos núme-
ros racionales e infinitos números irracionales. Esto quiere decir que, en
(xi−1 , xi ), el ı́nfimo de la función es cero y el supremo es f (xi ) = xi pues la
función, restringida a los racionales, es creciente en dicho intervalo. Tenemos
entonces que
n
X
I(f, P ) = 0 · (xi − xi−1 ) = 0;
i=1
n n  
X b−aX b−a
S(f, P ) = xi · (xi − xi−1 ) = a+i·
n n
i=1 i=1
   
b−a b − a n(n + 1) (b − a)(n + 1)
= n·a+ · = (b − a) a + .
n n 2 2n

9
De lo anterior se deduce que la integral inferior es cero y la integral superior
es
 
(b − a)(n + 1) (b − a)(b + a)
S(f ) = lı́m (b − a) a + = 6= 0.
n→∞ 2n 2

Como I(f ) 6= S(f ), la función no es integrable en dicho intervalo.

PROBLEMA 10.6
a) Sea f una función integrable y no negativa en [a, b] tal que
Z b
f (x) dx = 0. Demostrar que f (x) = 0 en cada punto de conti-
a
nuidad de f .
b) Sea f una función continua y no negativa en [a, b]. Supongamos
Z b
que existe c ∈ [a, b] tal que f (c) > 0. Probar que f (x) dx > 0.
a

Solución

a) Procederemos por reducción al absurdo: si fuera f (c) 6= 0 para algún


c ∈ (a, b) donde f es continua, entonces necesariamente
Z c+ε f (x) > 0,
∀x ∈ (c − ε, c + ε). Pero esto indica que f (x) dx > 0 lo que
c−ε
Z b
contradice la hipótesis de que f (x) dx = 0.
a

b) Como f (c) > 0, por ser f continua, existe un intervalo (c − δ, c + δ) tal


que f (x) > 0, ∀x ∈ (c − δ, c + δ). Consideramos ahora una partición
P = {x0 , x1 , . . . , xn } de [a, b] de tal manera que existan dos puntos
xi , xi+1 ∈ (c − δ, c + δ) (en caso contrario siempre se pueden añadir
dos puntos ası́ a la partición). Sea uk ∈ [xk−1 , xk ] el valor para el cual
f alcanza el mı́nimo en el subintervalo [xk−1 , xk ]. Por definición,

n
X
I(f, P ) = f (uk ) · (xk − xk−1 ) > 0
k=1

porque en (c−δ, c+δ) la función es estrictamente positiva y en el resto


es no negativa. Como I(f ) = supP I(f, P ), también será I(f ) > 0, de
Z b
donde se deduce que f (x) dx > 0.
a

10
PROBLEMA 10.7
Z 1 Z 2 Z 5
Sabiendo que f (x) dx = 6, f (x) dx = 4, f (x) dx = 1, cal-
0 0 2
cular:
Z 5
a) f (x) dx.
0
Z 2
b) f (x) dx.
1
Z 1
c) f (x) dx.
5
Z 1
d) f (2x) dx.
0

Solución
a) Debido a la propiedad aditiva de la integral, tenemos:
Z 5 Z 2 Z 5
f (x) dx = f (x) dx + f (x) dx = 4 + 1 = 5.
0 0 2

Z b Z a
b) Teniendo en cuenta que f (x) dx = − f (x) dx, si descomponemos
a b
nuevamente la integral en dos sumandos, resulta:
Z 2 Z 0 Z 2 Z 1 Z 2
f (x)dx = f (x)dx+ f (x)dx = − f (x)dx+ f (x)dx = −6+4 = −2.
1 1 0 0 0

c) Utilizando el resultado de a) y descomponiendo la integral, obtenemos:


Z 1 Z 0 Z 1 Z 5 Z 1
f (x) dx = f (x) dx+ f (x) dx = − f (x) dx+ f (x) dx = −5+6 = 1.
5 5 0 0 0

Z b Z b/c
d) Por la propiedad de escala f (x) dx = c f (cx) dx con c = 2,
a a/c
resulta: Z 1 Z 2
f (2x) dx = (1/2) f (x) dx = 4/2 = 2.
0 0

PROBLEMA 10.8
Z 2
2 2
Probar que √
4
≤ ex −x dx ≤ 2e2 .
e 0

11
Solución
Calcularemos en primer lugar el máximo y el mı́nimo de la función integran-
do en el intervalo [0, 2]:
2 −x 2 −x
f (x) = ex =⇒ f 0 (x) = (2x − 1) · ex ,
0
f (x) = 0 ⇐⇒ 2x − 1 = 0 ⇐⇒ x = 1/2.
Además como f 0 (x) < 0 para x < 1/2, f decrece en (0, 1/2); por otra parte,
como f 0 (x) > 0 cuando x > 1/2, f crece en (1/2, 2).
Esto quiere decir que el mı́nimo de la función corresponde a x = 1/2 y toma
el valor f (1/2) = e−1/4 y el máximo estará en alguno de los extremos del
intervalo. Ahora bien, como f (0) = 1 y f (2) = e2 , el máximo es el punto
(2, e2 ).
Lo anterior permite escribir la desigualdad e−1/4 ≤ f (x) ≤ e2 , ∀x ∈ (0, 2)
(como se observa en la figura). Como esta desigualdad sigue siendo válida
al calcular las integrales respectivas (propiedad de monotonı́a), obtenemos
en definitiva que
Z 2 Z 2 Z 2 Z 2
−1/4 x2 −x −1/4 2
e dx ≤ e dx ≤ 2
e dx =⇒ 2e ≤ ex −x dx ≤ 2e2 ,
0 0 0 0
como querı́amos demostrar.

PROBLEMA 10.9
Sean f y g dos funciones continuas en [a, b]. Probar que
Z b 2 Z b  Z b 
f (x)g(x) dx ≤ f 2 (x) dx · g 2 (x) dx
a a a

(llamada desigualdad de Schwarz).

12
Solución
Si g es la función nula, la igualdad es evidentemente cierta (ambos miembros
de la desigualdad son nulos).
Sea pues g 6= 0 y llamamos λ a un número real cualquiera. Como la función
(f + λg)2 es no negativa, entonces su integral será también no negativa.
Desarrollándola tenemos:
Z b Z b Z b Z b
2 2 2 2
0≤ (f +λg) (x) dx = f (x) dx+λ g (x) dx+2λ f (x)g(x) dx.
a a a a
Rb
− f (x)g(x) dx
Si sustituimos en la desigualdad el valor λ = Ra b , resulta:
2
a g (x) dx
Rb !2 Z
Z b f (x)g(x) dx b
2
0 ≤ f (x) dx + Rb a
· g 2 (x) dx
2
a a g (x) dx a
Rb Z b
f (x)g(x) dx
−2 · aR b · f (x)g(x) dx
2
a g (x) dx a
R 2 R 2
b b
Z b
a f (x)g(x) dx a f (x)g(x) dx
= f 2 (x) dx + Rb −2· Rb
2 2
a a g (x) dx a g (x) dx
R 2
b
Z b
a f (x)g(x) dx
= f 2 (x) dx − Rb
2
a a g (x) dx
Z b 2 Z b  Z b 
2 2
=⇒ f (x)g(x) dx ≤ f (x) dx · g (x) dx .
a a a

PROBLEMA 10.10
Sea f una función integrable en [a, b]. Probar las siguientes propie-
dades:
Z b Z b

a)
f (x) dx ≤ |f (x)| dx.
a a

b) Si m ≤ f (x) ≤ M en todo [a, b], entonces existe algún k ∈ [m, M ]


Z b
tal que f (x) dx = (b − a) · k .
a

c) (Teorema del valor medio para integrales.) Si f es continua en


[a, b], entonces existe algún c ∈ [a, b] tal que
Z b
f (x) dx = (b − a) · f (c).
a

13
Solución
Z b Z b
a) Aplicaremos la propiedad f ≤ g =⇒ f (x) dx ≤ g(x) dx y el hecho
a a
de que |x| ≤ A ⇐⇒ −A ≤ x ≤ A. Tenemos pues:
Z b Z b Z b
−|f (x)| ≤ f (x) ≤ |f (x)| =⇒ − |f (x)| dx ≤ f (x) dx ≤ |f (x)| dx
a a a
Z b Z b

=⇒
f (x) dx ≤ |f (x)| dx.
a a

b) Aplicamos de nuevo la propiedad anterior, con lo que:


Z b Z b Z b
m ≤ f (x) ≤ M =⇒ m dx ≤ f (x) dx ≤ M dx
a a a
Z b
=⇒ m(b − a) ≤ f (x) dx ≤ M (b − a)
a
Z b Z b
1 1
=⇒ m ≤ f (x) dx ≤ M =⇒ f (x) dx = k,
b−a a b−a a

para algún k ∈ [m, M ].


c) Por ser f continua, alcanza sus valores máximo y mı́nimo, es decir,
m ≤ f (x) ≤ M con m = f (c0 ) y M = f (c1 ). Procediendo como en
Z b
1
el apartado anterior, de la desigualdad m ≤ f (x) dx ≤ M y
b−a a
aplicando la propiedad de Darboux (ver capı́tulo 4), se deduce que
Z b
1
existe c ∈ [a, b] tal que f (c) = f (x) dx.
b−a a
Geométricamente, esta propiedad indica que, en el caso de ser f no
negativa en [a, b], el área limitada por la función y el eje X en el
intervalo [a, b] coincide con el área de un rectángulo de base b − a y
cuya altura es el valor de la función en algún punto c ∈ [a, b].

PROBLEMA 10.11
Sean f y g dos funciones continuas en [a, b] donde además g no
cambia de signo. Probar que existe algún c ∈ [a, b] tal que
Z b Z b
f (x)g(x) dx = f (c) g(x) dx
a a

(teorema generalizado del valor medio para integrales).

14
Solución
Por ser f continua en un intervalo cerrado [a, b], es acotada y existen dos
constantes m y M tales que m ≤ f (x) ≤ M , ∀x ∈ [a, b]. Suponemos además
que g(x) ≥ 0 en [a, b] (en caso contrario, cambia sólo el sentido de las
desigualdades siguientes). Entonces:

m · g(x) ≤ f (x) · g(x) ≤ M · g(x)


Z b Z b Z b
=⇒ m · g(x) dx ≤ f (x) · g(x) dx ≤ M · g(x) dx
a a a
Rb Rb
a f (x) · g(x) dx f (x) · g(x) dx
=⇒ m ≤ Rb ≤ M =⇒ ∃r ∈ [m, M ] : r = a R b .
a g(x) dx a g(x) dx

Aplicando ahora la propiedad de Darboux, como r ∈ [m, M ], existe c ∈ [a, b]


tal que f (c) = r, lo que prueba la propiedad buscada.
Se observa que si g es la función identidad, la propiedad se reduce al teorema
del valor medio probado en el problema anterior.

PROBLEMA 10.12
Sea f una función continua en [a, b]. Comparar la cantidad (b −
Z b
a)f (b) con f (x) dx en los siguientes casos:
a

a) f constante en [a, b].


b) f creciente en [a, b].
c) f decreciente en [a, b].

Solución
a) Si f es constante, entonces f (x) = f (b), ∀x ∈ [a, b]. Integrando miembro
a miembro, resulta:
Z b Z b Z b
f (x) dx = f (b) dx =⇒ f (x) dx = (b − a)f (b).
a a a

b) Si f es creciente, f (x) ≤ f (b), ∀x ∈ [a, b]. Por la propiedad de mono-


tonı́a,
Z b Z b Z b
f (x) dx ≤ f (b) dx =⇒ f (x) dx ≤ (b − a)f (b).
a a a

15
c) Si f es decreciente, f (x) ≥ f (b), ∀x ∈ [a, b]. Nuevamente tenemos:
Z b Z b Z b
f (x) dx ≥ f (b) dx =⇒ f (x) dx ≥ (b − a)f (b).
a a a

B. TEOREMAS FUNDAMENTALES DEL CÁLCULO INTEGRAL.

El concepto de integral definido en el apartado anterior está ı́ntimamente


relacionado con el cálculo de integrales indefinidas o primitivas, como mues-
tran los llamados teoremas fundamentales del cálculo integral. Estos son:

PRIMER TEOREMA FUNDAMENTAL DEL CÁLCULO INTEGRAL


Dada una función Zy = f (x) definida en un intervalo [a, b], se define la función
x
primitiva F (x) = f (t) dt, para todo x ∈ [a, b].
a

a) Si f es integrable en [a, b], entonces F es continua en [a, b].


b) Si f es continua en [a, b], entonces F es derivable y su derivada es
precisamente F 0 (x) = f (x), ∀x ∈ [a, b].
El resultado de este teorema justifica el nombre dado a F de función primi-
tiva de f pues su derivada es la propia f .

Consecuencia del teorema anterior es el llamado


SEGUNDO TEOREMA FUNDAMENTAL DEL CÁLCULO INTEGRAL
Si y = f (x) es una función continua en [a, b] y g es cualquier primitiva de
Z b
f , entonces f (x) dx = g(b) − g(a).
a
Este resultado, que también es válido si f es cualquier función integrable
en [a, b], permite calcular la integral de una función sin más que realizar los
pasos siguientes:
1) Calcular una primitiva de f , es decir una función g tal que g 0 (x) =
f (x).
2) Sustituir g en los puntos x = a y x = b.

16
Z b
3) La resta g(b) − g(a) es precisamente el valor de f (x) dx.
a

En forma esquemática se suele representar este proceso como

Z b  b
f (x) dx = g(x) a = g(b) − g(a).
a

Algunas variaciones de los resultados anteriores, que permiten calcular deri-


vadas de funciones cuya variable está en algún extremo de integración, son
las siguientes:
Z a
a) Si F (x) = f (t) dt, entonces F 0 (x) = −f (x).
x

Z h(x)
b) Si F (x) = f (t) dt, entonces F 0 (x) = f (h(x)) · h0 (x).
a

Z h2 (x)
c) Si F (x) = f (t) dt, entonces F 0 (x) = f (h2 (x)) · h02 (x) − f (h1 (x)) · h01 (x).
h1 (x)

El segundo teorema fundamental también permite extender los métodos de


integración. Podemos destacar los siguientes:

a) Método de sustitución o cambio de variable:

Z b Z h(b)
0 0
f (h(x)) · h (x) dx = f 0 (t) dt = f (h(b)) − f (h(a)).
a h(a)

b) Integración por partes:

Z b b
Z b
f (x)g 0 (x) dx = f 0 (x)g(x) dx

f (x)g(x) a −
a a
Z b
= f (b)g(b) − f (a)g(a) − f 0 (x)g(x) dx.
a

Otros resultados y aplicaciones se presentan en los problemas que siguen.

17
PROBLEMA 10.13
Resolver:
Z 1  
d 1
a) dx.
−1 dx 1 + 21/x
Z x2 p
d
b) 1 + t2 dt.
dx 0
Z x3
d dt
c) √ .
dx x2 1 + t4
 Z x Z y  
d 3
d) sen sen sen t dt dy .
dx 0 0

Solución
a) Por la definición de integral:
Z 1    1
d 1 1 1 1 1 1 1
dx = = − = − =− .
−1 dx 1 + 21/x 1 + 21/x −1 1 + 2 1 + 2−1 3 3/2 3

b) Aplicamos en este caso el primer teorema fundamental del cálculo inte-


gral pues la variable independiente está en el lı́mite superior de inte-
gración. Tenemos ası́:
Z x2
d p p p
1 + t2 dt = 1 + (x2 )2 · 2x = 2x 1 + x4 .
dx 0

c) Análogamente al apartado anterior,


Z x3
d dt 1 1
√ = 3x2 · p − 2x · p
dx x2 1 + t4 3
1 + (x )4 1 + (x2 )4
3x2 2x
= √ −√ .
1+x 12 1 + x8
Z x Z y  Z x 
3 0 3
d) Llamando u(x) = sen sen t dt dy, entonces u (x) = sen sen t dt .
0 0 0
De aquı́ resulta:
d
(sen u(x)) = cos u(x) · u0 (x)
dx Z x Z y   Z x 
3 3
= cos sen sen t dt dy · sen sen t dt .
0 0 0

18
PROBLEMA 10.14
√ √
Dadas las funciones f (x) = x + sen2 πx, g(x) = 1 + x4 ,
Z f (x)
h(x) = g(x) dx, calcular h0 (1).
0

Solución
Aplicando el teorema fundamental del cálculo integral, h0 (x) = g(f (x)) ·
f 0 (x). Por otra parte, si aplicamos las reglas usuales de derivación, obtene-
1 + 2 sen(πx) cos(πx) · π
mos que f 0 (x) = √ . En definitiva,
2 x + sen2 πx

h0 (1) = g(f (1)) · f 0 (1) = g(1) · (1/2) = 2/2.

PROBLEMA 10.15
Determinar todas las funciones continuas f para las cuales
Z x+1
g(x) = f (t) dt es constante.
x−1

Solución
Para que g sea constante debe ser g 0 ≡ 0. Como g 0 (x) = f (x + 1) − f (x − 1),
g será constante cuando f (x + 1) = f (x − 1) para todo x, o, lo que es
equivalente, f (x) = f (x + 2), ∀x. Esta propiedad corresponde precisamente
a las funciones periódicas de perı́odo 2.

PROBLEMA 10.16
R x2 √
tg t dt
Calcular lı́m 0 .
x→0 x − sen x

Solución
Tenemos una indeterminación del tipo 0/0 por lo que aplicaremos la regla de
L’Hôpital y las equivalencias de infinitésimos tg f (x) ∼ f (x) y 1−cos f (x) ∼
[f (x)]2 /2 cuando f (x) → 0. Ası́

2x tg x2 2x · |x| 4|x|
L = lı́m = lı́m 2 = lı́m ,
x→0 1 − cos x x→0 x /2 x→0 x

19
lo que tiene diferentes valores según x sea positivo o negativo. En concreto,
4|x| 4|x|
lı́m = 4 y lı́m = −4.
x→0+ x x→0− x

PROBLEMA 10.17

Dadas las funciones f (x) = x2 , g(x) = x2 + 1 · sen(πx) y
Z f (x)
h(x) = g(t) dt, calcular los máximos y mı́nimos relativos de
0
h.

Solución En primer lugar calculamos los puntos crı́ticos, es decir aquellos


puntos en que h0 (x) = 0:
p
h0 (x) = g(f (x)) · f 0 (x) = x4 + 1 · sen(πx2 ) · 2x;

h0 (x) = 0 ⇐⇒ x = 0 ó πx2 = kπ, k ∈ Z ⇐⇒ x = ± k, k ∈ N ∪ {0}.

Para saber si corresponden a posibles máximos o mı́nimos, calculamos la


derivada de segundo orden:
p 2x · sen(πx2 ) · 4x3 p
h00 (x) = 2x 1 + x4 ·cos(πx2 )·2πx+ √ +2 1 + x4 ·sen(πx2 ).
2 1 + x4
De aquı́ se deduce que si k es par, como cos(πk) = 1, sen(πk) = 0, enton-
ces √ √ p √ p
h00 ( k) = 2 k · 1 + k 2 · 2π k = 4πk 1 + k 2 > 0,

lo que implica que x = k corresponde a un mı́nimo relativo.
Por otra parte, si k es impar, cos(πk) = −1 y sen(πk) = 0, con lo que
√ √ p √ p
h00 ( k) = −2 k · 1 + k 2 · 2π k = −4πk 1 + k 2 < 0,

lo que implica que x = k corresponde a un máximo relativo.
Por último, si k = 0, h00 (0) = 0 pero, en un entorno reducido de x = 0,
h0 (x) > 0, si x > 0 y h0 (x) < 0 si x < 0, lo que indica que x = 0 corresponde
a un mı́nimo relativo.

PROBLEMA 10.18
Z x
2
Probar que la función f (x) = e−1/t dt, definida en el intervalo
1
(0, ∞), tiene inversa derivable. Calcular (f −1 )0 (0).

20
Solución
2
La derivada de la función es f 0 (x) = e−1/x , que es siempre positiva. Esto
quiere decir que la función es creciente en (0, ∞) y, por tanto, tiene inversa.
1
Por la regla de derivación de la función inversa, (f −1 )0 (x0 ) = 0 −1 ,
f (f (x0 ))
y sabiendo que f (1) = 0 equivale a que f −1 (0) = 1, tenemos:
1 1 1
(f −1 )0 (0) = = = = e.
f 0 (f −1 (0)) f 0 (1) e−1

PROBLEMA 10.19
Sea y = g(x) una función
Rx continua y positiva en [0, ∞). Probar que
tg(t) dt
la función f (x) = R0x es creciente en (0, ∞).
0 g(t) dt

Solución
La función será creciente donde su derivada sea positiva. Aplicando la regla
de derivación del cociente tenemos:
Rx Rx
0 xg(x) 0 g(t) dt − g(x) 0 tg(t) dt
f (x) = R x 2
0 g(t) dt
Rx Rx Rx
g(x) 0 xg(t) dt − g(x) 0 tg(t) dt g(x) 0 (x − t)g(t) dt
= R x 2 = R x 2 .
0 g(t) dt 0 g(t) dt

El denominador es evidentemente positivo. Además, como g es positiva, para


todo x > 0, si t ∈ (0, x), entonces g(x) > 0 y (x − t)g(t) > 0. Sabiendo que
la integral de una función positiva es positiva, se obtiene también que el
numerador es positivo. De esto se deduce que la función es creciente cuando
x > 0.

PROBLEMA 10.20
Demostrar que, si f es una función continua, se tiene la siguiente
igualdad
Z x Z x Z u 
(x − u)f (u)du = f (t) dt du.
0 0 0

21
Solución
Si llamamos
Z x Z x Z x
F (x) = (x − u)f (u)du = x f (u)du − uf (u)du,
Z0 x Z u  0 0

G(x) = f (t) dt du,


0 0

debemos comprobar que F 0 (x) = G0 (x) y que en algún punto x0 , F (x0 ) =


G(x0 ). Ahora bien:
Z x Z x
0
F (x) = f (u)du + xf (x) − xf (x) = f (u)du;
Z0 x 0

G0 (x) = f (t) dt.


0

Esto indica que F y G se diferencian en una constante F (x) − G(x) = C.


Pero como F (0) = 0 = G(0), resulta que C = 0, con lo que F = G.

PROBLEMA 10.21
f 0 (x)
Z 3
Calcular 2
dx.
−1 1 + f (x)

Solución
f 0 (x)
Recordando que D(arc tg f (x)) = , obtenemos:
1 + f 2 (x)
3
f 0 (x)
Z
dx = [arc tg f (x)]3−1 = arc tg f (3) − arc tg f (−1).
−1 1 + f 2 (x)

PROBLEMA 10.22
Hallar, mediante integrales definidas, los siguientes lı́mites:
 
1 2 n−1
a) lı́m + 2 + ··· + .
n→∞ n2 n n2
 
1 π 2π (n − 1)π
b) lı́m sen + sen + · · · + sen .
n→∞ n n n n
1p + 2p + · · · + n p
c) lı́m , (p > 0).
n→∞ np+1
 
1 1
d) lı́m n + ··· + 2 .
n→∞ 12 + n2 n + n2

22
Solución
En todos los casos debemos construir una función adecuada y determinar un
intervalo de modo que la definición de integral de dicha función en el intervalo
corresponda al lı́mite buscado. Utilizaremos la definición de integral
Z b n
b−aX
f (x) dx = lı́m f (ui ), ui ∈ [xi−1 , xi ],
a n→∞ n
i=1

donde xi = a + i(b − a)/n, i = 0, 1, 2, . . . , n es un punto genérico de una


partición regular del intervalo [a, b], para lo cual escribiremos las sumas
dadas como en el caso general.
a) Escribimos
 
1 2 n−1 1 0 1 2 n−1
+ + ··· + = + + + ··· + .
n2 n2 n2 n n n n n
Consideramos la función f (x) = x en el intervalo [0, 1]. Una partición
regular del intervalo es P = {0, 1/n, 2/n, . . . , (n − 1)/n, 1}. Como la
función es creciente, la integral inferior en el intervalo [0, 1] es, por
definición,
n n n
X X i−1 1 X i−1
I(f ) = lı́m f (xi−1 )(xi −xi−1 ) = lı́m · = lı́m .
n→∞ n→∞ n n n→∞ n2
i=1 i=1 i=1

Por otra parte, como la función es integrable, podemos aplicar el se-


gundo teorema fundamental, con lo que dicho lı́mite es igual a
n Z 1  2 1
1 Xi−1 x 1
lı́m = I(f ) = I = x dx = = .
n→∞ n n 0 2 0 2
i=1

b) Aplicamos el procedimiento anterior a la función f (x) = sen(πx) en el


intervalo [0, 1].
n   Z 1
1X i−1 1 1 2
lı́m f = sen(πx) dx = − cos(πx) 0 = .
n→∞ n n 0 π π
i=1

c) Escribimos la suma dada como


1p + 2p + · · · + np 1
p+1
= [(1/n)p + (2/n)p + · · · + (n/n)p ] ,
n n
lo que sugiere considerar la función f (x) = xp en [0, 1]. Procediendo
como en los casos anteriores, tenemos:
n   Z 1
1X i 1  p+1 1 1
lı́m f = xp dx = x 0
= .
n→∞ n n 0 p+1 p+1
i=1

23
d) Nuevamente debemos adaptar la suma dada para que tenga la forma de
una suma de Riemann. Para ello dividimos numerador y denominador
por n2 con lo que
   
1 1 1 1 1
n + ··· + 2 = + ··· + .
12 + n 2 n + n2 n 1 + (1/n)2 1 + (n/n)2

1
El lı́mite de esta suma corresponde a la integral de la función f (x) =
1 + x2
en [0, 1]. De este modo,
n   Z 1
1X i 1  1
lı́m f = 2
dx = arc tg x 0
= π/4.
n→∞ n n 0 1+x
i=1

PROBLEMA 10.23
Sea f una función que verifica f (0) = 1, f (2) = 3, f 0 (2) = 5. Hallar
Z 1
xf 00 (2x) dx.
0

Solución
Integramos por partes haciendo u = x, dv = f 00 (2x) dx. De este modo,
du = dx, v = (1/2)f 0 (2x), con lo que
Z Z
xf 00 (2x) dx = x·(1/2)f 0 (2x)− (1/2)f 0 (2x) dx = (1/2)xf 0 (2x)−(1/4)f (2x).

Aplicando ahora el teorema fundamental tenemos que


Z 1 h i1
xf 00 (2x) dx = (1/2)xf 0 (2x) − (1/4)f (2x)
0 0
f 0 (2) f (2) f (0) 5 3 1
= − + = − + = 2.
2 4 4 2 4 4

PROBLEMA 10.24
Z x √
dt π
Resolver la ecuación √ √ = cuando x > 2.
2
2
t t −1 12

24
Solución
Calculamos primero la integral indefinida para lo que hacemos el cambio de
variable t2 − 1 = u2 . Queda entonces:
Z Z
dt du p
√ = = arc tg u = arc tg t2 − 1.
t t2 − 1 u2 + 1

Si sustituimos ahora en los extremos de integración,


Z x
π dt p p π
= √ √ = arc tg x2 − 1 − arc tg 1 = arc tg x2 − 1 −
12 2
2 t t −1 4
p π π π p √
=⇒ arc tg x2 − 1 = + = =⇒ x2 − 1 = tg π/3 = 3
12 4 3
=⇒ x2 = 4 =⇒ x = 2,

pues debe ser x > 2.

PROBLEMA 10.25
Hallar un polinomio p(x) tal que p(0) = p(−2) = 0, p(1) = 15,
Z 0
3 p(x) dx = 4.
−2

Solución
Como se proporcionan cuatro condiciones, probamos como solución un poli-
nomio de grado 3, p(x) = a0 + a1 x + a2 x2 + a3 x3 . De acuerdo con los datos,
tenemos el sistema de ecuaciones siguiente:

p(0) = 0 =⇒ a0 = 0
p(−2) = 0 =⇒ a0 − 2a1 + 4a2 − 8a3 = 0
p(1) = 15 =⇒ a0 + a1 + a2 + a3 = 15
0 0
a1 x2 a2 x3 a3 x4
Z 
4
3 p(x) dx = 4 =⇒ a0 x + + + =
−2 2 3 4 3
 −2
8a2 4
=⇒ − −2a0 + 2a1 − + 4a3 = .
3 3

Al resolver el sistema, obtenemos la solución

a0 = 0, a1 = 4, a2 = 8, a3 = 3.

25
PROBLEMA 10.26
Dada una función integrable f , probar las siguientes propieda-
des:
Z b Z b+c
a) f (x) dx = f (x − c) dx.
a a+c
Z b Z 1
b) f (x) dx = (b − a) f [a + (b − a)x] dx.
a 0

Solución
a) Haciendo en la segunda integral el cambio de variable x − c = t, el
intervalo de integración x ∈ (a + c, b + c) se transforma en t ∈ (a, b).
Como además dx = dt, la integral queda ahora:
Z b+c Z b Z b
f (x − c) dx = f (t) dt = f (x) dx.
a+c a a

b) Al igual que el caso anterior, hacemos en la segunda integral el cambio


de variable t = a + (b − a)x; de aquı́, cuando x = 0, es t = a y cuando
x = 1, es t = b. Además dt = (b − a) dx, con lo que
Z 1 Z b Z b
(b − a) f [a + (b − a)x] dx = f (t) dt = f (x) dx.
0 a a

PROBLEMA 10.27
Sea f una función integrable en [a, b] que verifica f (a + b − x) =
Z b
a+b b
Z
f (x), ∀x. Probar que xf (x) dx = f (x) dx.
a 2 a

Solución
Si hacemos en la primera integral el cambio de variable u = a + b − x,
obtenemos:
Z b Z a
xf (x) dx = (a + b − u)f (a + b − u)(−du)
a b
Z b Z b Z b Z b
= (a + b)f (u) du − uf (u) du = (a + b)f (x) dx − xf (x) dx.
a a a a
Agrupando términos iguales y despejando, resulta:
Z b Z b Z b
a+b b
Z
2 xf (x) dx = (a + b) f (u)du =⇒ xf (x) dx = f (x) dx.
a a a 2 a

26
PROBLEMA 10.28
Z 1 Z 1
n m
Probar que x (1 − x) dx = xm (1 − x)n dx, ∀n, m ∈ Z.
0 0

Solución
Hacemos en la primera integral el cambio de variable t = 1 − x. De este
modo:
Z 1 Z 0 Z 1
n m n m
x (1 − x) dx = (1 − t) t (− dt) = tm (1 − t)n dt,
0 1 0

que es la igualdad buscada.

PROBLEMA 10.29
Sea f una función continua en el intervalo [−a, a].
Z a Z a
a) Si f es par, probar que f (x) dx = 2 f (x) dx.
−a 0
Z a
b) Si f es impar, probar que f (x) dx = 0.
−a

Solución
En ambos casos descomponemos la integral en suma del siguiente modo:
Z a Z 0 Z a
f (x) dx = f (x) dx + f (x) dx,
−a −a 0

y en el primer sumando hacemos el cambio de variable x = −t.


a) Teniendo en cuenta que, al ser f par, f (−x) = f (x), ∀x, tenemos:
Z 0 Z 0 Z a Z a
f (x) dx = f (−t)(− dt) = f (−t) dt = f (t) dt.
−a a 0 0

Sustituyendo este resultado en la primera igualdad, se obtiene en de-


finitiva que
Z a Z a Z a Z a
f (x) dx = f (x) dx + f (x) dx = 2 f (x) dx.
−a 0 0 0

27
b) Procedemos análogamente al caso anterior, pero, al ser f impar, utili-
zamos la propiedad f (−x) = −f (x), ∀x. Ası́:
Z 0 Z 0 Z a Z a
f (x) dx = f (−t)(− dt) = −f (t) dt = − f (t) dt,
−a a 0 0
Z a
por lo que, al sustituir, se deduce que f (x) dx = 0.
−a

PROBLEMA 10.30
Sean f y g dos funciones integrables en R con las siguientes carac-
terı́sticas:
Z x f es impar, g es par, f (5) = 7, f (0) = 0, g(x) = f (x + 5),
f (x) = g(t) dt para todo x. Demostrar:
0

a) f (x − 5) = −g(x) para todo x.


Z 5
b) f (t) dt = 7.
0
Z x
c) f (x) dx = g(0) − g(x).
0

Solución

a) Aplicando que f es impar y que g es par, obtenemos:

f (x − 5) = −f (5 − x) = −g(−x) = −g(x).

b) Por la propiedad de traslación, haciendo t = x + 5 y aplicando el pro-


blema anterior, resulta:
Z 5 Z 0 Z 0 Z 5
f (t) dt = f (x + 5) dx = g(x) dx = g(x) dx = f (5) = 7.
0 −5 −5 0

c) Hacemos el cambio de variable x = u + 5 y aplicamos los resultados


anteriores. Ası́ tenemos que
Z x Z x−5 Z x−5 Z 0 Z x−5
f (x) dx = f (u + 5) du = g(u) du = g(u) du + g(u) du
0 −5 −5 −5 0
Z 5
= g(u) du + f (x − 5) = f (5) + f (x − 5) = g(0) − g(x).
0

28
PROBLEMA 10.31
Probar que si f es integrable
Z a+T Z T en todo R y periódica de perı́odo T ,
entonces f (x) dx = f (x) dx, ∀a ∈ R.
a 0

Solución
Por ser f periódica de perı́odo T , se verifica que f (x) = f (x + nT ), ∀x ∈
R, n ∈ Z.
Z T Z (n+1)T
Probaremos en primer lugar que f (x) dx = f (x) dx, para cual-
0 nT
quier entero n. Para ello basta hacer en la segunda integral el cambio de
variable t = x − nT , con lo que,
Z (n+1)T Z T Z T
(∗) f (x) dx = f (t + nT ) dt = f (t) dt.
nT 0 0

Si a es cualquier número real, por la propiedad arquimediana de los números


reales, existe n ∈ Z tal que nT ≤ a < (n + 1)T . Hacemos pues la descompo-
sición Z a+T Z (n+1)T Z a+T
f (x) dx = f (x) dx + f (x) dx.
a a (n+1)T

En el segundo sumando hacemos el cambio de variable u = x− T . Ası́:


Z a+T Z a Z a
f (x) dx = f (u + T ) du = f (u) du.
(n+1)T nT nT

Sustituyendo este resultado en la igualdad anterior y aplicando (*), obtene-


mos:
Z a+T Z (n+1)T Z a Z (n+1)T Z T
f (x) dx = f (x) dx+ f (u) du = f (x) dx = f (x) dx.
a a nT nT 0

La siguiente figura ilustra la situación planteada en el problema.

29
PROBLEMA 10.32
(
x − [x] − 1/2 si x no es entero
Sea f (x) = y se define la fun-
0 si x es entero,
Z x
ción P (x) = f (t) dt, para todo x real.
0

a) Dibujar la gráfica de f en el intervalo [−3, 3] y probar que f (x+1) =


f (x) para todo x.
x2 − x
b) Demostrar que P (x) = , si 0 ≤ x ≤ 1, y que P es periódica
2
de perı́odo 1.
Z 1
c) Determinar una constante c tal que [P (t) + c] dt = 0.
0
Z x
d) Sea ahora Q(x) = [P (t) + c] dt. Demostrar que ∀x ∈ [0, 1],
0

x3 x2 x
Q(x) = − + y que Q es periódica de perı́odo 1.
6 4 12

Solución
a) Si n es cualquier entero y x ∈ (n, n + 1), entonces [x] = n, con lo que
f (x) = x − n − 1/2, lo que corresponde a una recta de pendiente uno
y que corta al eje X en x = n + 1/2. La gráfica es pues de la forma:

De la misma construcción se deduce que f (x + 1) = f (x), ∀x, lo que


quiere decir que la función es periódica de perı́odo 1. Analı́ticamente,
si x ∈ [n, n + 1), x + 1 ∈ [n + 1, n + 2), y [x + 1] = n + 1. Entonces,

f (x+1) = x+1−[x+1]−1/2 = x+1−(n+1)−1/2 = x−n−1/2 = f (x).

b) Si x ∈ (0, 1), [x] = 0 y f (x) = x − 1/2, con lo que:


x x
t2 x2 x x2 − x
Z 
t
P (x) = (t − 1/2) dt = − = − = .
0 2 2 0 2 2 2

30
Además, es evidente que P (0) = 0 y, por ser f integrable, P es continua
y
x2 − x
P (1) = lı́m P (x) = lı́m = 0.
x→1− x→1− 2
Por otra parte, al ser f periódica, del problema anterior se deduce que
para todo x,
Z x+1 Z 1
f (t) dt = f (t) dt = P (1) = 0.
x 0

Por tanto, si x ∈ [n, n + 1), con n ∈ Z,


Z x+1 Z x Z x+1 Z x
P (x+1) = f (t) dt = f (t) dt+ f (t) dt = f (t) dt = P (x),
0 0 x 0

lo que prueba que P es periódica de perı́odo 1.


c) Cuando x ∈ [0, 1], P (x) = (x2 − x)/2. Por tanto,
1 1
1 1 2 1 t3 t2
Z Z 
1
0 = [P (t) + c] dt = (t − t) dt + [ct]0 = − +c
0 2 0 2 3 2 0
 
1 1 1 1 1
= − + c = − + c =⇒ c = .
2 3 2 12 12

d) Procediendo como en b), si x ∈ [0, 1], tenemos:


Z x x
1 t3 t2 x3 x2

t x
Q(x) = [P (t) + c] dt = − + = − + .
0 2 3 2 12 0 6 4 12

Además, como P (t) + c es también periódica de perı́odo 1,


Z x+1 Z x Z x+1
Q(x + 1) = [P (t) + c] dt = [P (t) + c] dt + [P (t) + c] dt
0 0 x
Z x Z 1
= [P (t) + c] dt + [P (t) + c] dt = Q(x).
0 0

31
C. EJERCICIOS PROPUESTOS.

1. Calcular las integrales de las siguientes funciones en los inter-


valos que se indican:
a)f (x) = x + [x] en [0, 2].
Resp.: I = 3.

πx
b) f (x) = [x] sen en [0, 6].
6
6
Resp.: I = [5 + cos(5π/6) + cos(2π/3) + cos(π/3) + cos(π/6)] = 30/π.
π

c) f (x) = [x] · [2x] en [−2, 2].


Resp.: 11.

2. Sea y = f (x) una función acotada en [−1, 1] y P , P1 dos particio-


nes de dicho intervalo con P ⊂ P1 . Responder justificadamente si
las siguientes afirmaciones pueden ser verdaderas o son siempre
falsas:
a) I(f, P ) = 3, S(f, P ) = 2.
Resp.: Falsa. Siempre debe ser I(f, P ) ≤ S(f, P ).

Z 1
b) I(f, P ) = 3, S(f, P ) = 6, f (x) dx = 2.
−1
Z 1
Resp.: Falsa pues debe cumplirse que I(f, P ) ≤ f (x) dx ≤ S(f, P ).
−1

Z 1
c) I(f, P ) = 3, S(f, P ) = 6, f (x) dx = 10.
−1

Resp.: Falsa por la misma razón que (b).

d) S(f, P ) = 4, S(f, P1 ) = 5.
Resp.: Falsa porque si P ⊂ P1 , entonces S(f, P ) ≥ S(f, P1 ).

e) I(f, P ) = 5, I(f, P1 ) = 4.

32
Resp.: Falsa porque si P ⊂ P1 , entonces I(f, P ) ≤ I(f, P1 ).

3. Hallar los errores en los siguientes cálculos:


Z √3  √3
dx 1 2x π
a) 2
= arc tg 2
=− .
0 1 + x 2 1 − x 0 6
Resp.: Hemos obtenido un resultado negativo al integrar una fun-
ción positiva debido a que la primitiva utilizada no está definida
en x = 1, que es un punto interior del intervalo de integración.

Z π Z π
dx dx
b) =
0 1 + 2 sen2 x 0 cos 2 x + 3 sen2 x

Z π
dx/ cos2 x 1 h √ iπ
= 2 = √ arc tg( 3 tg x) = 0.
0 1 + 3 tg x 3 0

Resp.: El resultado aparentemente contradice el apartado b) del


problema 10.6. Sin embargo la primitiva obtenida no está definida
en x = π/2 que pertenece al intervalo de integración.

(
x + [x] si x ∈ Q
4. Probar que la función f (x) = no es integrable
0 si x ∈
6 Q,
en [0, 2].
Sugerencia: Proceder como en el problema 10.5.

5. Hallar, mediante integrales definidas, los siguientes lı́mites:


 
b b 2b nb
a) lı́m sen + sen + · · · + sen .
n→∞ n n n n
Resp.: L = 1−cos b (basta aplicar la definición de integral definida
a la función y = sen x en el intervalo [0, b] o a la función y =
sen(bx) en el intervalo [0, 1].

√ √
n

n √
n
n
e+ e2 + e3 + · · · + en
b) lı́m .
n→∞ n
Z 1
Resp.: L = e pues dicho lı́mite es precisamente ex dx.
0

6. Calcular la derivada respecto a x de las siguientes funciones:

33
Z x
dt
a) .
1 t
Resp.: 1/x.

Z x2
dt
b) √
.
1 1 + t2
2x
Resp.: √ .
1 + x4
Z 1+x2
c) f (t) dt.
0

Resp.: 2x · f (1 + x2 ).
Z tg x
d) f (t) dt.
x

Resp.: sec2 x · f (tg x) − f (x).

Z x3
e) sen3 t dt.
a

Resp.: 3x2 sen3 x3 .


Rx
0 ln(1 + sen t) dt
7. Calcular lı́m .
x→0 x2
Resp.: L = 1/2 (ver problema 10.16).

8. Sea f continua en [0, 1]. Demostrar que


Z π
π π
Z
xf (sen x) dx = f (sen x) dx.
0 2 0
Sugerencia: Descomponer las integrales en dos sumandos correspon-
dientes a los intervalos [0, π/2] y [π/2, π]. Al hacer el cambio de va-
riable x = π − t, tanto el primer miembro como el segundo dan como
Z π/2
resultado π f (sen x) dx.
0

9. La tangente a la curva y = f (x) forma un ángulo de π/3 con el


Z b
eje OX en x = a y de π/4 en x = b. Hallar f 00 (x) dx.
a
√ √
Resp.: 1 − 3 (los datos indican que f 0 (a) = 3, f 0 (b) = 1).

34
10. Probar:
Z b Z b
a) f (t) dt = f (a + b − x) dx.
a a
Sugerencia: Hacer el cambio de variable t = a + b − x.

Z b Z b/c
b) f (x) dx = c f (cx) dx.
a a/c

Sugerencia: Hacer el cambio de variable x = ct.

11. Sea f una función con derivada de primer orden continua en


Z b
[a, b] y tal que f (a) = f (b) = 0. Si f 2 (x) dx = 1, probar que
Z b a
0
xf (x)f (x) dx = −1/2.
a

Sugerencia: Basta integrar por partes haciendo u = xf (x) y dv =


f 0 (x) dx.

Z 1 Z 1/x
dt dt
12. Probar que = , para x > 0.
x 1 + t2 1 1 + t2
Sugerencia: Basta ver que sus derivadas son iguales y que para x = 1
las funciones coinciden.
Z x
sen t
13. Hallar los máximos y mı́nimos de la función F (x) = dt,
0 t
en el intervalo (0, 2π).
Resp.: Tiene un máximo relativo en x = π.

14. Se considera la función f (x) = x2 ex .


Z x
f (t)
a) Hallar la derivada de G(x) = dt en el intervalo (1, ∞).
1 t3
Resp.: G0 (x) = ex /x.

b) Estudiar el crecimiento y la concavidad G(x) en el intervalo


(1, ∞).
Resp.: G es creciente y cóncava hacia arriba en (1, ∞).

35
CAPÍTULO XI.
APLICACIONES DE LA
INTEGRAL DEFINIDA

SECCIONES
A. Áreas de figuras planas.
B. Cálculo de volúmenes.
C. Longitud de curvas planas.
D. Ejercicios propuestos.

37
A. ÁREAS DE FIGURAS PLANAS.

En Geometrı́a Elemental se conocen las fórmulas para hallar el área de cual-


quier región limitada por una poligonal cerrada. Ahora bien, si una región
está limitada por alguna lı́nea curva, como es el cı́rculo, el área se expresa
como un lı́mite de las áreas de poligonales “próximas”. El procedimiento
descrito en el capı́tulo anterior para definir el concepto de integral de una
función consiste precisamente en aproximar la función por funciones esca-
lonadas; si consideramos una función y = f (x) no negativa en un intervalo
[a, b], la integral inferior es el lı́mite de la suma de las áreas de los rectángu-
los inscritos en la región limitada por la curva y = f (x), el eje OX y las
rectas x = a y x = b, y la integral superior es el lı́mite de las áreas de los
rectángulos circunscritos a dicha región. De este modo podemos definir el
área de dicha región como la integral de la función f en el intervalo [a, b].
En general,
Dada una función y = f (x) integrable en un intervalo [a, b], el área de la
región limitada por la función, el eje OX y las rectas x = a y x = b se define
como Z b
A= |f (x)| dx.
a

Observación: El valor absoluto de la función es debido a que en los inter-


valos donde la función es negativa, la integral también es negativa y su valor
es opuesto al del área correspondiente.
En la práctica, para eliminar el valor absoluto en el integrando, debemos
determinar los intervalos de [a, b] donde la función es positiva o negativa y
descomponer la integral en suma de integrales correspondientes a cada uno
de los intervalos indicados colocando el signo adecuado. Ası́, en la figura
adjunta, el área se expresa como
Z r Z s Z b
A= f (x) dx − f (x) dx + f (x) dx.
a r s

38
En particular, si la función está expresada en forma paramétrica x = x(t), y =
y(t), el área viene expresada como
Z b Z t1
A= y dx = y(t) · x0 (t) dt,
a t0

donde a = x(t0 ), b = x(t1 ).


Regiones más generales que las descritas son aquellas que están limitadas
por dos funciones y = f (x), y = g(x) entre dos rectas verticales x = a y
x = b. En este caso el área se expresa mediante la fórmula
Z b
A= |f (x) − g(x)| dx.
a

En el ejemplo de la figura, el área se descompone como:


Z r Z s Z b
A= [g(x) − f (x)] dx + [f (x) − g(x)] dx + [g(x) − f (x)] dx.
a r s
Si la región está limitada por dos curvas y = f (x), y = g(x) entre dos
rectas horizontales y = c e y = d, consideramos las funciones inversas
e integramos respecto a la variable y. El área se expresa entonces como
Z d
A= |f −1 (y) − g −1 (y)| dy.
c

En el ejemplo de la figura, dicha integral se descompone como


Z r Z d
−1 −1
A= [f (y) − g (y)] dy + [g −1 (y) − f −1 (y)] dy.
c r

39
En los ejercicios que siguen veremos ejemplos de todas las situaciones plan-
teadas. Al ser válidas aquı́ todas las propiedades de las integrales obtenidas
en el capı́tulo anterior, aplicaremos siempre los teoremas fundamentales de
la integral. Omitiremos en la mayorı́a de los casos el cálculo de las primi-
tivas pues ya se han realizado en el capı́tulo 7. Nos limitaremos a escribir
el resultado de dicha primitiva y a indicar las sustituciones en los extremos
de integración. Sı́ es muy conveniente tener una idea aproximada de la re-
presentación gráfica de las funciones involucradas para conocer la posición
relativa de las mismas y los intervalos de integración. Es importante tam-
bién observar las simetrı́as de las figuras para ası́ poder escribir fórmulas
más sencillas para el área de las mismas.

PROBLEMA 11.1
Calcular el área de la región limitada por la gráfica de la función
f y el eje X en el intervalo indicado:
a) f (x) = |x| − |x − 1| en [−1, 2].
b) f (x) = x(ln x)2 en [1, e].
c) f (x) = e−x | sen x| en [0, 2π].

Solución

a) El área de la región (que es la parte sombreada de la figura) viene dada


Z 2

por la fórmula A = |x| − |x − 1| dx.
−1

Teniendo en cuenta el signo de la función, la integral se descompone


ası́:
Z 0 Z 0,5 Z 1 Z 2
5
A= 1 · dx + −(2x − 1) dx + (2x − 1) dx + 1 · dx = .
−1 0 0,5 1 2

40
b) La función y = x(ln x)2 es no negativa en el intervalo [1, e].

El área es entonces, integrando por partes,


Z e  2 e
2 x 2 x2 x2 e2 − 1
A= x(ln x) dx = · (ln x) − · ln x + = .
1 2 2 4 1 4

Z 2π
c) Nuevamente la función es no negativa, por lo que A = e−x | sen x| dx.
0

Para integrar descomponemos en dos sumandos y tenemos:


Z 2π Z π Z 2π
−x −x
A = e | sen x| dx = e sen x dx + −e−x sen x dx
0 0 π
 −x π 2π
e−x (e−π + 1)2

e
= − (sen x + cos x) + (sen x + cos x) = .
2 0 2 π 2

PROBLEMA 11.2
Hallar el área de la figura limitada por la función f (x) = x(x −
1)(x − 2) y el eje OX .

Solución
Como la curva corta al eje OX enZ los puntos de abscisa x = 0, x = 1 y
2
x = 2, el área viene dada por A = |f (x)| dx.
0

41
Ahora bien, en el intervalo [0, 1] la curva queda por encima del eje X mientras
que en el intervalo [1, 2] queda por debajo del mismo. Tenemos pues
Z 1 Z 2 Z 1 Z 2
1
A= f (x) dx+ −f (x) dx = (x3 −3x2 +2x) dx− (x3 −3x2 +2x) dx = .
0 1 0 1 2

PROBLEMA 11.3
Hallar el área del menor de los sectores que la recta x = 3 deter-
mina en la circunferencia de ecuación x2 + y 2 = 25.

Solución

Teniendo en cuenta la simetrı́a de la figura basta calcular el área de la región


contenida en el primer cuadrante. Tenemos
Z 5p
A = 2 25 − x2 dx
3
x 5 25π
 p 
x 2
25 3
= 2 25 − x + arc sen = − 12 − 25 arc sen .
2 2 5 3 2 5

42
PROBLEMA 11.4
Hallar el área de la figura limitada por la recta x = 2a y la hipérbola
x2 y 2
− 2 = 1.
a2 b

Solución

De acuerdo con la figura, el área se obtiene como


Z 2a p
A = 2 b (x/a)2 − 1 dx
a
x + √x2 − a2 2a
" #
bx p 2 √ √
= x − a2 − ab ln = ab[2 3 − ln(2 + 3)].

a a


a

PROBLEMA 11.5
Hallar el área limitada por la curva y 2 = x4 (4 + x).

Solución

Como la figura está determinada por el intervalo x ∈ [−4, 0] y es simétrica


respecto al eje X, el área será
0

Z 0   2
2 3/2 (4 + x) 8(4 + x) 16 4096
A=2 x 4 + x dx = 4(4 + x) − + = .
−4 7 5 3 −4 105

43
PROBLEMA 11.6
Hallar el área limitada por la curva x4 − ax3 + b2 y 2 = 0.

Solución

La curva está definida cuando x ∈ [0, a] y es simétrica respecto a OX. El


área viene dada por:
Z a p
x
A = 2 ax − x2 dx = (cambio (a/2) cos t = x − a/2)
0 b

a3 π a3 t sen 2t sen3 t πa3
Z 
2
= sen t · (1 + cos t) dt = − + = .
4b 0 4b 2 4 3 0 8b

PROBLEMA 11.7
Hallar el área de la figura limitada por la curva (x/5)2 + (y/4)2/3 =
1.

Solución

El área de la figura, teniendo en cuenta sus simetrı́as, es


Z 5 Z π/2
2 3/2
A = 4 4(1 − x /25) dx = (cambio x = 5 cos t) = 16 5 sen4 t dt
0 0
Z π/2  π/2
2 3t sen 4t
= 20 (1 − cos 2t) dt = 20 − sen 2t + = 15π.
0 2 8 0

44
PROBLEMA 11.8
Hallar el área limitada por la curva x = (y 2 + x)2 .

Solución

p√
En forma explı́cita, la ecuación de la curva es y = ± x − x. Como la
gráfica es simétrica respecto al eje OX, el área viene dada por
Z 1q
√ 1 √ sen t
A = 2 x − x dx = (cambio − x = )
0 2 2
π/2
1 π/2 sen 2t cos3 t
Z 
2 1 t π
= cos t · (1 − sen t) dt = + + = .
2 −π/2 2 2 4 3 −π/2 4

PROBLEMA 11.9
x2 2
Hallar el área encerrada por la curva y 2 = (a − x2 ).
a2

Solución

De acuerdo con la figura y gracias a la simetrı́a, tenemos:


Z a p Z π/2
x 2
A = 4 2 2
a − x dx = (cambio x = a sen t) = 4a cos2 t · sen t dt
0 a 0
π/2
cos3 t 4a2

2
= 4a − = .
3 0 3

45
PROBLEMA 11.10
Hallar el área de la figura limitada por la cardioide de ecuación
x(t) = a(2 cos t − cos 2t), y(t) = a(2 sen t − sen 2t).

Solución

Como la figura es simétrica respecto al eje OX, el área viene dada por
Z a Z 0
A = 2 y · dx = 2 y(t)x0 (t) dt
−3a π
Z 0
= 2 a(2 sen t − sen 2t)2a(sen 2t − sen t) dt
π
sen 2t sen 4t 0
 
2 −3t 3
= 4a + 2 sen t + + = 6πa2 .
2 2 8 π

PROBLEMA 11.11
Hallar el área comprendida entre un lazo de la cicloide x = a(t −
sen t), y = a(1 − cos t) y el eje OX .

Solución

2πa
Integrando respecto a la variable t, como un lazo de la cicloide se encuentra
en el intervalo t ∈ [0, 2π], resulta:
Z 2πa Z 2π
A = y(t) dx(t) = a(1 − cos t)a(1 − cos t) dt
0 0
sen 2t 2π
 
2 3t
= a − 2 sen t + = 3πa2 .
2 4 0

46
PROBLEMA 11.12
Hallar el área encerrada por la astroide de ecuación (ax)2/3 +
(by)2/3 = (a2 − b2 )2/3 .

Solución
Escribimos la ecuación en forma paramétrica como x(t) = (c2 /a) cos3 t,
y(t) = (c2 /b) sen3 t, donde c2 = a2 − b2 .
c2 /b

c2 /a

Teniendo en cuenta la simetrı́a de la figura podemos escribir el área co-


mo
Z c2 /a Z 0
A = 4 y · dx = 4 (c2 /b) sen3 t · (c2 /a)(−3 cos2 t sen t) dt
0 π/2
π/2 π/2
12c4 12c4 t sen 4t sen3 2t 3πc4
Z 
4 2
= sen t cos t dt = − − = .
ab 0 ab 16 64 48 0 8ab

PROBLEMA 11.13
Hallar el área de la figura limitada por la curva y 3 = x, la recta
y = 1 y la vertical x = 8.

Solución

47
Como la recta y = 1 corta a la curva en el punto de abscisa x = 1 y en
el intervalo [1, 8] la curva queda por encima de la recta, el área viene dada
por
Z 8 " #8
1/3 3 x4/3 17
A= (x − 1) dx = −x = .
1 4 4
1

PROBLEMA 11.14
Calcular el área limitada por la curva y = e2x y las rectas y = e2 ,
x = 0.

Solución
En este caso, la recta y = e2 queda por encima de la curva y = e2x en la
región comprendida entre los valores x = 0 y x = 1.

e2

El área se obtiene como


Z 1 1
e2x e2 1 e2 + 1

2 2x 2
A= (e − e ) dx = e x − = e2 − + = .
0 2 0 2 2 2

PROBLEMA 11.15
Hallar el área de la región y ≥ x2 − 9, x2 + (y − 3)2 ≥ 9, y ≤ −x + 3.

48
Solución

El centro de la circunferencia es el punto (0, 3) por el cual pasa la recta


y = −x + 3. Esto quiere decir que la recta es un diámetro y el área de la
figura sombreada es la diferencia entre el área de la región comprendida entre
dicha recta y la parábola y el área del semicı́rculo de radio 3. Los puntos de
intersección de la parábola y la recta se obtienen del sistema

y = x2 − 9, y = −x + 3 =⇒ x2 + x − 12 = 0 =⇒ x = 3, x = −4.

Tenemos entonces:
Z 3 Z 3
2 9π 9π
A = [(−x + 3) − (x − 9)] dx − = (−x2 − x + 12) dx −
−4 2 −4 2
 2 3
3
x x 9π 343 9π
= 12x − − − = − .
2 3 −4 2 6 2

PROBLEMA 11.16
Calcular el área de la figura limitada por las curvas y = ex , y = e−x
y la recta x = 1.

49
Solución

Como en el intervalo x ∈ [0, 1] la curva y = ex queda por encima de la curva


y = e−x , el área viene dada por
Z 1 1
(ex − e−x ) dx = ex + e−x 0 = e + e−1 − 2.

A=
0

PROBLEMA 11.17
Hallar el área comprendida entre las parábolas y 2 = 2px, x2 = 2py .

Solución

Como los puntos de intersección de ambas parábolas son (0, 0) y (2p, 2p), el
área viene dada por la integral:
" #2p
2p p
x2 2x3/2 x3 4p2
Z 
p
A= 2px − dx = 2p · − = .
0 2p 3 6p 3
0

50
PROBLEMA 11.18
Dada la curva de ecuación y = x3 y la recta y = λx (ver figura),
demostrar que la región S1 limitada por la curva y la recta en el
intervalo x ∈ [0, a] tiene la misma área que la región S2 limitada
por la curva y el eje X en el mismo intervalo.

Solución
Como la recta pasa por el punto (a, a3 ), se debe cumplir que a3 = λa, es
decir λ = a2 .

Al calcular cada una de las áreas mencionadas obtenemos


Z a  2 a
3 λx x4 2λa2 − a4 a4
S1 = (λx − x ) dx = − = = ,
0 2 4 0 4 4
Z a  4 a 4
x a
S2 = x3 dx = = ,
0 4 0 4

lo que prueba el enunciado.

PROBLEMA 11.19
Hallar el área de la figura encerrada por la parábola y = x2 /4 y la
8
curva de Agnesi y = 2 .
x +4

51
Solución

Los puntos de intersección de ambas curvas son solución del sistema formado
por ambas ecuaciones. Tenemos que:
x2 8 √
= 2 ⇐⇒ x4 + 4x2 = 32 ⇐⇒ x2 = −2 ± 4 + 32 = −2 ± 6.
4 x +4
Como la solución x2 = −8 no es real, sólo es posible x2 = 4 ⇐⇒ x = ±2. El
área es entonces, teniendo en cuenta la simetrı́a de la figura,
Z 2 Z 2
x2 x2
 
8 8
A = 2
− dx = 2 − dx
−2 x + 4 4 0 x2 + 4 4
2
x x3

4
= 2 4 arc tg − = 2π − .
2 12 0 3

PROBLEMA 11.20
πx
Calcular el área limitada por las curvas y = x2 , y = sen .
2

Solución
Como se observa en la figura, la región que limitan dichas curvas se encuentra
πx
en el intervalo [0, 1] en el cual la función y = sen queda por encima de
2
y=x . 2

El área es entonces
Z 1h 1
πx x3

πx 2
i 2 1 2
A= sen − x dx = − cos − =− + .
0 2 π 2 3 0 3 π

52
PROBLEMA 11.21
Calcular el área de los dos trozos en que la circunferencia x2 + (y +
R)2 = 2R2 divide a la circunferencia x2 + y 2 = R2 .

Solución
Los puntos de intersección de ambas curvas son:
x2 +y 2 = R2 , x2 +y 2 +2Ry+R2 = 2R2 =⇒ 2Ry = 0 =⇒ y = 0 =⇒ x = ±R,
y las regiones que limitan son las indicadas en la figura.

Las áreas de ambas regiones son:


Z R p p 
A1 = R2 − x2 − 2R2 − x2 + R dx
−R
" √ #R
x R2 − x2 R2 x
= + arc sen
2 2 R
−R
" √ #R
x 2R2 − x2 x
− 2
+ R arc sen √ + [Rx]R 2
−R = R ;
2 R 2 −R
2 2
A2 = πR − A1 = (π − 1)R .

PROBLEMA 11.22
Calcular el área comprendida entre las curvas y = sen3 x, y =
1/ sen x, para x ∈ [π/4, π/2].

53
Solución

En el intervalo indicado, la curva y = 1/ sen x queda por encima de y =


sen3 x.

π/4 π/2 π

Z π/2  
1 3
A = − sen x dx
π/4 sen x
π/2 √
cos3 x √

5 2
= ln | cosec x − cotg x| + cos x − = − ln( 2 − 1) − .
3 π/4 12

PROBLEMA 11.23
Calcular el área comprendida entre las curvas y = 1/ cos2 x, y =
sen6 x para x ∈ [0, π/4].

Solución

En este caso también la curva y = 1/ cos2 x queda por encima de y =


sen6 x. Bastará pues integrar la resta de ambas funciones en el intervalo
indicado.

π/4 π/2 π

54
Z π/4
A = (sec2 x − sen6 x) dx
0
 π/4
5 1 3 1 59 5π
= tg x − x + sen 2x − sen 4x − sen3 2x = − .
16 4 64 48 0 48 64

PROBLEMA 11.24
Hallar el área de la figura comprendida entre la hipérbola equilátera
x2 − y 2 = 9, el eje OX y la recta que une el origen con el el punto
(5, 4).

Solución
El área de la región se puede obtener como la resta entre el área del triángulo
de vértices O(0, 0), A(5, 0) y B(5, 4) y el área de la región limitada por la
hipérbola y el eje OX en el intervalo [3, 5].

Tenemos pues:
Z 5p
5·4
A = − x2 − 9 dx
2 3
" √ √ !#5
x x2 − 9 9 x+ x2 − 9 9
= 10 − − ln = ln 3.
2 2 3 2
3

PROBLEMA 11.25
Determinar el área de la parte común a las dos elipses
x2 y 2 x2 y 2
+ = 1, + 2 = 1 con a > b.
a2 b2 b2 a

55
Solución

Debido a la simetrı́a de la región (ver figura), basta calcular el área de la


región comprendida en el primer cuadrante.

ab
El punto de intersección de las elipses tiene abscisa x = √ , con lo
a2+ b2
que el área pedida es

√ ab
Z Z b
a2 +b2
p p
A = 4 b 1− x2 /a2 dx + 4 a 1 − x2 /b2 dx
√ ab
0
a2 +b2
 √ ab b
4b a2 4a b2
 
x xp 2 a2+b2 x xp 2
= arc sen + a − x2 + arc sen + b −x2
a 2 a 2 0 b 2 b 2 √ ab
a2+b2
 
b a π
= 2ab arc sen √ − arc sen √ + .
2
a +b 2 2
a +b 2 2

PROBLEMA 11.26
Calcular el área de la región limitada por las gráficas de f (x) =
|x − 1| y g(x) = x2 − 2x.

Solución

Los puntos de intersección de las curvas son:

y = |x − 1|, y = x2 − 2x =⇒ |x − 1| = x2 − 2x
( ( √
x − 1 = x2 − 2x si x > 1 x= 3+ 5
2√ ,
=⇒ =⇒ 1− 5
−x + 1 = x2 − 2x si x < 1 x= 2 .

56
Debido a la simetrı́a de la figura, el área se puede expresar como:

Z √
3+ 5 Z √
3+ 5 √
2
2
2
2 7+5 5
A= √ [|x−1|−(x −2x)] dx = 2 [(x−1)−(x −2x)] dx = .
1− 5
1 6
2

PROBLEMA 11.27
Calcular el área de la figura limitada por la parábolas y = x2 ,
y = x2 /2 y la recta y = 2x.

Solución

La primera parábola y = x2 corta a la recta en el punto de abscisa x = 2


mientras que la segunda parábola y = x2 /2 corta a la recta en el punto de
abscisa x = 4.

El área se descompone entonces como suma de integrales de la siguiente


forma: Z 2 Z 4
2 2
A= (x − x /2) dx + (2x − x2 /2) dx = 4.
0 2

57
PROBLEMA 11.28
Calcular el área de la región limitada por las gráficas de f y g en
el intervalo que se indica en cada caso:

a) f (x) = x, g(x) = x2 en [0, 2].
b) f (x) = x(x2 − 1), g(x) = x en [−1, 2].

Solución

a) Los puntos de intersección de las curvas son


y= x, y = x2 =⇒ x = x4 =⇒ x = 0, x = 1.

El área se descompone entonces como la suma

1 2

√ √ 10 − 4 2
Z Z
A= ( x − x2 ) dx + 2
(x − x) dx = .
0 1 3

b) Los puntos de intersección de las curvas son:

√ √
y = x(x2 − 1), y = x =⇒ x(x2 − 1) = x =⇒ x = 0, x = 2, x = − 2.

58
El área se obtiene entonces como:

Z 2
A = |x(x2 − 1) − x| dx
−1

Z 0 Z 2 Z 2
3 3 11
= (x − 2x) dx + (2x − x ) dx + √ (x3 − 2x) dx = .
−1 0 2 4

PROBLEMA 11.29
Calcular el área limitada por las regiones y ≤ x2 + 1, y ≥ x2 − 9,
y ≤ 3 − x.

Solución

Calculamos los puntos de intersección de las curvas:

y = x2 + 1, y = 3 − x =⇒ x2 + x − 2 = 0 =⇒ x = −2, x = 1;
y = x2 − 9, y = 3 − x =⇒ x2 + x − 12 = 0 =⇒ x = −4, x = 3.

59
El área queda entonces como la suma de las siguientes integrales:

Z −2 Z 1
2
A = [(3 − x) − (x − 9)] dx + [(x2 + 1) − (x2 − 9)] dx
−4 −2
Z 3
[(3 − x) − (x2 − 9)] dx
+
1
Z −2 Z 1 Z 3
2 158
= (−x − x + 12) dx + 10 dx + (−x2 − x + 12) dx = .
−4 −2 1 3

PROBLEMA 11.30
Calcular el área comprendida entre las cuatro parábolas
y 2 = x, y 2 = 2x, x2 = y , x2 = 2y .

Solución

Los distintos puntos de intersección son los siguientes:

x2 = 2y, y 2 = x =⇒ x = 0, x = 41/3 ;
x2 = y, y 2 = x =⇒ x = 0, x = 1;
2 2
x = y, y = 2x =⇒ x = 0, x = 41/6 ;
x2 = 2y, y 2 = 2x =⇒ x = 0, x = 2.

60
El área es entonces
Z 41/6 √
Z 41/3 √ √
Z 2 √
2 1
A= [x − x] dx + [ 2x − x] dx + [ 2x − x2 /2] dx = .
1 41/6 41/3 3

PROBLEMA 11.31
Calcular el área de la figura interior a la circunferencia x2 + (y −
1)2 = 5 y a la parábola x = 2(y − 1)2 .

Solución

Los puntos de intersección de ambas curvas son:

x2 + (y − 1)2 = 5, x/2 = (y − 1)2 =⇒ 2x2 + x − 10 = 0 =⇒ x = 2, x = −5/2.

Como la parábola está definida en x ≥ 0, sólo es posible la solución x = 2


lo que da los puntos (2, 0) y (2, 2).

Como debemos descomponer la integral en dos sumandos para integrar res-

61
pecto a la variable x, integramos respecto a y, lo que da lugar a:
Z 2 hp i
A = 5 − (y − 1)2 − 2(y − 1)2 dy
0
 2
5 y − 1 y − 1p 2 3 1 2
= arc sen √ + 5 − (y − 1)2 − (y − 1) = 5 arc sen √ + .
2 5 2 3 0 5 3

PROBLEMA 11.32
Encontrar el área de la región común a las circunferencias C1 :
x2 + y 2 = 4, C2 : x2 + y 2 = 4x.

Solución

√ √
Los puntos de intersección de las circunferencias son (1, 3) y (1, − 3), de
modo que, si integramos respecto a la variable y, el área puede expresarse
como la integral
√ √
Z 3 p p Z 3 p
A = 2 2 2
[ 4 − y − (2 − 4 − y )] dy = 4 ( 4 − y 2 − 1) dy
0 0
hy p y i√3 8π √
= 4 4 − y 2 + 2 arc sen −y = − 2 3.
2 2 0 3

PROBLEMA 11.33
Sea f la función indicada en la figura adjunta.
Z 1
Hallar f y también el área de la región comprendida entre la
0
función f y el eje X .

62
Solución

El área será la suma de las áreas de los triángulos que la función determina
con el eje OX. Resulta entonces la siguiente serie geométrica:
∞   ∞
X 1 1 1 X 1 1 1 1/2 1
A= · n−1
− n ·1= · n = · = .
2 2 2 2 2 2 1 − 1/2 2
n=1 n=1

Para calcular la integral, debemos sumar las áreas de los triángulos que que-
den por encima del eje OX y restarle la suma de las áreas de los triángulos
que quedan por debajo del mismo. Tenemos nuevamente las series geométri-
cas,
Z 1 ∞   X∞  
X 1 1 1 1 1 1
f = · − − · −
0 2 22n 22n+1 2 22n−1 22n
n=0 n=1
∞ ∞
X 1 X 1 1/4 1/8 1
= − = − = .
22n+2 22n+1 1 − 1/4 1 − 1/4 6
n=0 n=1

B. CÁLCULO DE VOLÚMENES.

El concepto de integral también puede aplicarse para calcular volúmenes de


ciertos sólidos. Los distintos casos y métodos utilizados son los que expone-
mos a continuación.

63
B.1.- VOLÚMENES DE SÓLIDOS DE SECCIÓN CONOCIDA.

Supongamos que un sólido está limitado por dos planos paralelos entre sı́ y
perpendiculares a un eje fijo t en los puntos t = t0 y t = t1 . Supongamos
además que las secciones producidas en el sólido por planos perpendiculares
al eje t son regiones cuya área se puede escribir como una función A(t)
integrable en [t0 , t1 ]. Entonces el volumen de dicho sólido verifica la fórmula
de Cavalieri
Z t1
(1) V = A(t) dt.
t0

En particular,Zsi las secciones son perpendiculares al eje OX entre los valores


x1
x0 y x1 , V = A(x) dx.
x0

Ası́, en el ejemplo de la figura tenemos una pirámide de base b y altura h y


las secciones perpendiculares al eje OX son cuadrados.
Para calcular el lado de un cuadrado genérico escribimos la ecuación de la
recta que une el origen con el punto (h, b) y calculamos su valor en el punto
de abscisa x. Resulta pues y = bx/h con lo que la función a integrar será el
área del cuadrado A(x) = (2y)2 = (2bx/h)2 y el volumen es

h  h
4b2 x3 4b2 h
Z
2
V = (2bx/h) dx = 2 = .
0 h 3 0 3

64
B.2.- VOLÚMENES DE SÓLIDOS DE REVOLUCIÓN.

El sólido de revolución es la figura obtenida al girar una región plana al-


rededor de un eje fijo (eje de revolución o eje de giro). Esto quiere decir
que las secciones perpendiculares a dicho eje son cı́rculos (o coronas circula-
res). El volumen se obtiene según el caso con los siguientes métodos: B.2.1.-
MÉTODO DE LOS DISCOS.
Consiste en interpretar el volumen como lı́mite de la suma de los volúmenes
de los discos que se obtienen al cortar la figura por planos perpendiculares
al eje de giro. Podemos distinguir dos casos:
(*) El eje de giro forma parte del contorno de la región plana.
Si consideramos la región plana limitada por la curva y = f (x), el eje de
giro y las rectas x = a, x = b, las secciones perpendiculares al eje de giro
son cı́rculos con lo que debemos integrar la función que corresponda al área
de los mismos en el intervalo correspondiente.

Ası́, si el eje de giro es el eje OX, tenemos la fórmula


Z b
(2) V =π [f (x)]2 dx.
a

Si el eje de giro es la recta y = r, el radio del cı́rculo en un punto de abscisa


x es |f (x) − r| y el volumen queda entonces:
Z b
(3) V =π [f (x) − r]2 dx.
a

En otros casos se procede de forma similar.


(**) El eje de giro no forma parte del contorno de la región pla-
na.

65
Consideramos ahora la región limitada por las curvas y = f (x), y = g(x) y
dos rectas perpendiculares al eje de giro, siendo éste exterior a la región. En
este caso, las secciones perpendiculares al eje de giro son coronas circulares.
Debemos pues restar el área del cı́rculo exterior menos el área del cı́rculo
interior.

Si el eje de giro es el eje OX,

Z b
(4) V =π ([f (x)]2 − [g(x)]2 ) dx.
a

Análogamente, si el eje de giro es la recta y = r,

Z b
(5) V =π ([f (x) − r]2 − [g(x) − r]2 ) dx.
a

Será necesario conocer la posición relativa de las funciones f y g para lo cual


es fundamental tener una idea de las gráficas de las mismas.

B.2.2.- MÉTODO DE LOS TUBOS.

Este método consiste en interpretar el volumen como lı́mite de la suma de


los volúmenes de los tubos obtenidos al girar alrededor del eje de giro las
franjas de espesor infinitesimal que determina en la región una partición del
intervalo. Este método será apropiado cuando al intentar aplicar el método
de los discos se deba descomponer la integral en varios sumandos.

66
Como el volumen de cada uno de estos tubos es 2π· radio medio · altura, el
volumen obtenido al girar la región comprendida entre la función y = f (x),
el eje X y las rectas x = a, x = b tiene las siguientes fórmulas.
Cuando el eje de giro es el eje OY :
Z b
(6) V = 2π x · f (x) dx.
a

Cuando el eje de giro es la recta vertical x = r:


Z b
(7) V = 2π |x − r| · f (x) dx.
a

Fórmulas análogas se obtienen para regiones comprendidas entre dos funcio-


nes o para ejes horizontales. En los siguientes problemas se realizan ejemplos
de todos los casos indicados.

PROBLEMA 11.34
Hallar el volumen de la figura engendrada al girar la curva y 2 = x3
alrededor del eje X a lo largo del intervalo x ∈ [0, 1].

67
Solución

De acuerdo con la figura, y aplicando la fórmula (2), tenemos:

1 1
x4
Z 
3 π
V =π x dx = π = .
0 4 0 4

PROBLEMA 11.35
Hallar el volumen del cuerpo engendrado por la rotación, alrededor
del eje OX , de la superficie limitada por el eje OX y la parábola
y = ax − x2 (a > 0).

Solución

Aplicamos directamente el método de los discos integrando en el intervalo


[0, a] que corresponde a los valores de x que limitan la superficie dada.

Ası́:
a a
πa5
Z Z
V =π (ax − x2 )2 dx = π (a2 x2 + x4 − 2ax3 ) dx = .
0 0 30

68
PROBLEMA 11.36
Calcular el volumen del sólido engendrado por la rotación de la
región limitada por los ejes coordenados y la curva de ecuación
√ √ √
x + y = a (a > 0) alrededor del eje OX .

Solución

√ √
De la ecuación de la curva se obtiene que y 2 = ( a − x)4 = a2 + x2 +
6ax − 4a3/2 x1/2 − 4a1/2 x3/2 . El volumen buscado es pues
Z a Z a
πa3
V =π y 2 (x) dx = π (a2 + x2 + 6ax − 4a3/2 x1/2 − 4a1/2 x3/2 ) dx = .
0 0 15

PROBLEMA 11.37
Los semiejes positivos y un cuadrante de la astroide de ecuación
x = a cos3 t, y = a sen3 t delimitan una región cuya área designare-
mos por S . Se pide:
i) El volumen del cuerpo de revolución engendrado por S al girar en
torno al eje OX .
ii) El volumen del cuerpo de revolución engendrado por S al girar en
torno al eje OY .

69
Solución
i)

Por el método de los discos, si integramos respecto al parámetro t,


como los valores extremos x = 0 y x = a corresponden a t = π/2 y
t = 0, respectivamente, tenemos:
Z a Z 0
V = π y 2 (t) dx(t) = π a2 sen6 t · (−3a cos2 t sen t) dt
0 π/2
π/2  3 π/2
cos t 3 cos5 t 3 cos7 t cos9 t 16πa3
Z
3 7 2 3
= 3πa sen t cos tdt = −3πa − + − =
0 3 5 7 9 0 105

ii)

Utilizaremos en este caso el método de integración por tubos. El vo-


lumen es
Z a Z 0
V = 2π x(t)y(t) dx(t) = 2π a cos3 t · a sen3 t · (−3a cos2 t sen t) dt
0 π/2
π/2 π/2
sen5 t 2 sen7 t sen9 t 16πa3
Z 
3 5 4 3
= 6πa cos t sen t dt = 6πa − + = .
0 5 7 9 0 105
El resultado es el mismo debido a las simetrı́as de la figura.

PROBLEMA 11.38
Hallar el volumen engendrado por la rotación alrededor del eje
OY del área limitada por el primer arco de la cicloide de ecuación
x = t − sen t, y = 1 − cos t.

70
Solución

De acuerdo con la figura, si aplicamos el método de los tubos e integramos


respecto al parámetro t, tenemos:
Z 2π Z 2π
V = 2π x(t)y(t) dx(t) = 2π (t − sen t)(1 − cos t)(1 − cos t) dt
0 0
Z 2π
= 2π (t − 2t cos t + t cos2 t − sen t + 2 sen t cos t − cos2 t sen t) dt
0
2π
3t2 cos3 t 3 cos 2t 7t sen t

= 2π − cos t + − − = 6π 3 .
4 3 8 4 0

PROBLEMA 11.39
Calcular el volumen del sólido obtenido al girar la región limitada
por la curva f (x) = sen x + cos x y el eje X en el intervalo [0, π]
alrededor del eje X .

Solución

Si aplicamos el método de los discos, resulta:


Z π  π
1
V =π (sen x + cos x) dx = π x − cos 2x = π 2 .
2
0 2 0

La siguiente figura da una idea de la forma del sólido obtenido.

71
PROBLEMA 11.40
Se considera el área S de la región limitada por un cuadrante de
una circunferencia de radio R y las tangentes en sus extremos.
Hallar el volumen que engendra S cuando gira en torno a una de
las tangentes.

Solución
Tomamos como eje OX el eje de giro y como eje OY la recta que, pasando
por
el centro de la circunferencia, es paralela a la otra tangente. De este
√ modo la
ecuación de la circunferencia será x2 + (y + R)2 = R2 =⇒ y = R2 − x2 −
R.

El volumen pedido viene expresado por:


Z R Z R p
2
V = π y (x) dx = π ( R2 − x2 − R)2 dx
0 0
R
x3 πR3

2 3 x
= π 2R x − − R arc sen = (10 − 3π).
3 R 0 6

72
PROBLEMA 11.41
Calcular el volumen engendrado por un segmento circular de ángu-
lo central 2α (ver figura) con α < π/2 y radio R al girar alrededor
de su cuerda.

Solución

Tomando como eje OX la cuerda AB y como eje OY la perpendicular a


esta cuerda que pase por el centro de la circunferencia, debido a que OB =
R sen α y |OC| = R cos α, la ecuación de la√circunferencia es x2 + (y +
R cos α)2 = R2 , de donde y = −R cos α + R2 − x2 . De esta forma, el
volumen pedido es
Z R sen α Z R sen α p
2
V = π y dx = 2π (R2 cos2 α + R2 − x2 − 2R cos α R2 − x2 ) dx
−R sen α 0
2πR3
= (2 sen α − 3α cos α + cos2 α sen α).
3

PROBLEMA 11.42
Se considera el arco OAB de la parábola de ecuación y = x(x − a),
con OA = a > 0 y OC = c > a. Determinar c de tal manera que
el volumen de revolución engendrado por la zona sombreada de la
figura, al girar en torno a OX , sea igual al volumen engendrado
por el triángulo OCB girando en torno al mismo eje.

73
Solución

El volumen engendrado por la zona sombreada es


Z a Z c Z a Z c
2 2 2 2
V = π y (x) dx + π y (x) dx = π x (x − a) dx + π x2 (x − a)2 dx
0 a 0 a
πc3
= (6c2 − 15ca + 10a2 ).
30

Como OC = c, BC = c(c − a) y el volumen del cono engendrado por el


triángulo OCB es

πc2 (c − a)2 · c πc3 (c − a)2


V0 = = .
3 3

Igualando los valores de V y V 0 se deduce que c = 5a/4.

PROBLEMA 11.43

x
Al girar alrededor del eje OX la curva de ecuación y = se
1 + x2
obtiene en el intervalo [0, x] un sólido cuyo volumen designaremos
1
por V (x). Determinar el valor de a para que V (a) = lı́m V (x).
2 x→∞

Solución
El volumen V (x) se calcula mediante la fórmula:
x x
−1 x π x2
Z Z  
2 x dx π
V (x) = π y (x) dx = π = = · .
0 0 (1 + x2 )2 2 1 + x2 0 2 1 + x2

74
π π a2 1 π
Ahora bien, como lı́m V (x) = , deberá cumplirse · 2
= · de
x→∞ 2 2 1+a 2 2
donde a = 1 (no es válido a = −1 pues no está en el dominio de la función).

PROBLEMA 11.44
Un sólido de revolución está generado por la rotación de la gráfica
de y = f (x) para [0, a] alrededor del eje X . Si para a > 0 el volumen
es a3 + a, hallar la función f .

Solución

Por la fórmula del volumen tenemos que


Z a
3
a +a=V =π [f (x)]2 dx.
0

Si llamamos G a una primitiva de f 2 , es decir tal que G0 (x) = f 2 (x), enton-


ces
a3 + a
V = π[G(a) − G(0)] = a3 + a =⇒ G(a) = + G(0).
π

x3 + x
Esto sugiere definir G(x) = . De este modo, G(0) = 0 y
π
r
3x2 + 1 3x2 + 1
G0 (x) = = f 2 (x) =⇒ f (x) = .
π π

PROBLEMA 11.45
Hallar el volumen de la figura engendrada al girar la superficie
comprendida entre la parábola y 2 = x y la circunferencia y 2 =
2x − x2 alrededor del eje X .

Solución

Los puntos de intersección de ambas curvas son (1, 1) y (1, −1).

75
Utilizando el método de integración por discos y descomponiendo la integral
en dos sumandos, tenemos

1 2 1 2
x2 x3
Z Z  
2 2 7π
V =π x dx + π (2x − x ) dx = π +π x − = .
0 1 2 0 3 1 6

PROBLEMA 11.46

Se considera la parábola de ecuación y = x2 2/a, con a > 0, y la
circunferencia x2 +y 2 = a2 . Determinar el volumen engendrado por
la zona sombreada de la figura al girar en torno al eje OX .

Solución

Resolviendo el sistema formado por √ las ecuaciones de la parábola y de la


circunferencia, se tiene que OC = a 2/2. Como el radio de la circunferencia

76
es a, el volumen pedido será


Z a 2/2 Z a
V = π 2x4 /a2 dx + π √ (a2 − x2 ) dx
0 a 2/2
a√2/2 a
5 x3 πa3 √
 
2x 2
= π 2
+ π a x − √
= (20 − 11 2).
5a 0 3 a 2/2 30

PROBLEMA 11.47
Determinar el volumen del sólido obtenido al girar alrededor del
eje OY la región limitada por las parábolas y = ax2 , y = b − cx2 ,
con a, b, c > 0.

Solución

Los puntos de intersección de las parábolas sepobtienen resolviendo el sistema


formado por sus ecuaciones. Ası́ se tiene A( b/(a + c), ab/(a + c)).

Calculamos el volumen por el método de los discos para lo cual debemos


integrar respecto a y en los intervalos (0, ab/(a+c)) y (ab/(a+c), b). Resulta
ası́:
Z ab/(a+c) Z b
y b−y πb2
V =π dy + π dy = .
0 a ab/(a+c) c 2(a + c)

77
PROBLEMA 11.48
Hallar el volumen generado por la rotación del área limitada por
la parábola y 2 = 8x y la ordenada correspondiente a x = 2
i) en torno al eje X ;
ii) en torno al eje Y ;
iii) en torno a la recta x = 2.

Solución
i) Dividiendo el área en franjas verticales,
√ al girar alrededor del eje X se
obtienen discos de radio y = 8x en el intervalo x ∈ [0, 2].

Aplicando la fórmula de integración por discos se obtiene:


Z 2
V =π 8x dx = 16π.
0

ii) Aplicaremos nuevamente el método de los discos para lo cual debemos


integrar respecto a la variable y en el intervalo [−4, 4].

78
Como un disco genérico tiene radio exterior 2 y radio interior x = y 2 /8,
el volumen viene dado por
4 4
y5
Z 
2 2 2 128π
V =π [2 − (y /8) ] dy = π 4y − = .
−4 320 −4 5

iii) Aplicaremos en este caso el método de los tubos. Como √ se observa


√ en
la figura, la altura de un cilindro genérico es 2y = 2 8x = 4 2x y su
distancia al eje de giro es 2 − x.

El volumen pedido será


Z 2 √ √ Z 2 256π
V = 2π 4 2x(2 − x) dx = 8 2π (2x1/2 − x3/2 ) dx = .
0 0 15

PROBLEMA 11.49
¿Cuál es el volumen del sólido que se obtiene al girar alrededor
del eje X la figura limitada por la curva y = ex y las rectas x = 0,
y = e?

79
Solución

Como la recta y = e queda por encima de la curva y = ex en el intervalo


[0, 1], si aplicamos la fórmula (4), el volumen viene dado por:

1
1 2x 1 e2 + 1
Z  
2 2x 2
V =π (e − e ) dx = π e x − e =π· .
0 2 0 2
Una idea del sólido obtenido se expresa en la siguiente figura.

PROBLEMA 11.50
Se considera la región del plano formada por los puntos (x, y) que
satisfacen las desigualdades 0 ≤ x ≤ 2, x2 /4 ≤ y ≤ 1. Calcular el
volumen del sólido obtenido al girar esta región alrededor del eje
Y , alrededor del eje X , alrededor de la recta x = 2, y alrededor de
la recta y = 1.

80
Solución
a)

Al girar alrededor del eje Y , el volumen (por el método de los discos)


es Z 1
 1
V =π 4y dy = π 2y 2 0 = 2π.
0

b)

Nuevamente por el método de los discos, si integramos respecto a x,


tenemos:
Z 2 2
x4 x5
 

V =π 1− dx = π x − = .
0 16 80 0 5

c)

Aplicando en esta ocasión el método de los tubos tenemos:


2 2
x2 x3 x4
Z 
2 10π
V = 2π (2 − x)(1 − x /4) dx = 2π 2x − − + = .
0 2 6 16 0 3

81
d)

Integrando por el método de los discos, tenemos por último que


Z 2 2
x3 x5

16π
V =π (1 − x2 /4)2 dx = π x − + = .
0 6 80 0 15

PROBLEMA 11.51
Hallar el volumen generado por la rotación del área limitada por
y = −x2 − 3x + 6, x + y − 3 = 0 alrededor de la recta
i) y = 0;
ii) x = 3.

Solución
i) Los puntos de intersección de las curvas son

y = −x2 − 3x + 6, y = 3 − x =⇒ −x2 − 2x + 3 = 0 =⇒ x = −3, x = 1.

82
Si aplicamos el método de los discos, como la parábola queda por
encima de la recta en el intervalo x ∈ [−3, 1], el volumen es:
Z 1 Z 1
V = π (yp2 − yr2 ) dx = π [(−x2 − 3x + 6)2 − (3 − x)2 ] dx
−3 −3
Z1
1792π
= π (x4 + 6x3 − 4x2 − 30x + 27) dx = .
−3 15

ii) La recta x = 3 es exterior a la región que gira. Aplicamos en este caso


el método de las tubos. La altura de un cilindro genérico es yp − yr =
(−x2 − 3x + 6) − (3 − x) = −x2 − 2x + 3 y el radio es 3 − x (distancia
del eje de giro a un punto de la región).

El volumen es pues
Z 1 Z 1
2 256π
V = 2π (3−x)(−x −2x+3) dx = 2π (x3 −x2 −9x+9) dx = .
−3 −3 3

PROBLEMA 11.52
Calcular el volumen del sólido obtenido al girar la región limitada
por las gráficas de f (x) = b(x/a)2 y g(x) = b|x/a| alrededor de y = 0.

Solución
Los puntos de intersección de ambas curvas son:
bx2 bx bx2 bx
x≥0:y= 2
, y = =⇒ 2
= =⇒ x2 − ax = 0 =⇒ x = 0, x = a.
a a a a

83
Debido a la simetrı́a de la figura, como la recta queda por encima de la
parábola, el volumen es:
Z a 2 2 a
b2 x4 b2 x3 x5 4πb2 · a
 
b x
V = 2π − dx = 2π · − = .
0 a2 a4 a2 3 5a2 0 15

PROBLEMA 11.53
Calcular el volumen engendrado por la región que delimitan las
parábolas y 2 = 2px, x2 = 2py (p > 0), al girar en torno a OX .

Solución
Se obtiene fácilmente que los puntos de intersección de las parábolas son
(0, 0) y (2p, 2p).

Por el método de los discos, el volumen es:


Z 2p Z 2p 4
x 12
V =π 2px dx − π 2
dx = πp3 .
0 0 4p 5

PROBLEMA 11.54
Calcular el volumen del sólido obtenido al girar la región limitada
por las gráficas de f (x) = sen x y g(x) = cos x en el intervalo [0, π/2]
alrededor del eje X .

84
Solución

π/4 π/2

Aplicando el método de los discos, debido a la posición relativa de las cur-


vas, debemos descomponer la integral en los intervalos [0, π/4] y [π/4, π/2].
Ası́ tenemos:

Z π/4 Z π/2
V = π (cos2 x − sen2 x) dx + π (sen2 x − cos2 x) dx
0 π/4
 π/4  π/2
sen 2x sen 2x
= π −π = π.
2 0 2 π/4

PROBLEMA 11.55
Calcular el volumen del sólido obtenido al girar la región limitada
por las gráficas de f (x) = x2 − 4x + 4 y g(x) = 4 − x alrededor de
y = −1.

Solución

Los extremos de integración serán los puntos de intersección de las curvas.


Estos son:

y = x2 − 4x + 4, y = 4 − x =⇒ x2 − 3x = 0 =⇒ x = 0, x = 3.

85
Si aplicamos el método de los discos (fórmula (5)), teniendo en cuenta que
el radio exterior es re = yr + 1 = 4 − x + 1 y el radio interior es ri = yp + 1 =
x2 − 4x + 4 + 1, resulta:

Z 3
V = π [(4 − x + 1)2 − (x2 − 4x + 4 + 1)2 ] dx
0
3
x3 (x − 2)2 2(x − 2)3

117π
= π − 5x2 + 25x − −x− = .
3 5 3 0 5

Una sección del sólido obtenido tiene la forma de la figura adjunta.

86
PROBLEMA 11.56
Determinar el volumen del sólido que se obtiene al girar alrededor
del eje de abscisas la región del primer cuadrante limitada por las
curvas y = 1/x2 , y = sen(πx/2) y las rectas x = 0, y = e.

Solución

Los puntos de intersección de las curvas son

πx πx 1
y = 1/x2 , y = sen =⇒ sen = 2 =⇒ x = 1;
2 2 x √
2
y = 1/x , y = e =⇒ x2 = 1/e =⇒ x = 1/ e.

Aplicando el método de los discos, tenemos:


Z 1/ e Z 1  
h πx i
2 2 1 2 πx
V = π e − sen dx + √
π 4 − sen dx
0 2 1/ e x 2
√ √
x sen πx 1
 
h
2 x sen πx i1/ e −1 (8e e − 5)π
= π e x− + +π − + = .
2 2π 0 3x3 2 2π 1/√e 6

87
PROBLEMA 11.57
Se considera la hipérbola de ecuación x2 /a2 − y 2 /b2 = 1 y las dos
rectas perpendiculares al eje OX de ecuaciones x = p, x = p + h
(p > a).
Determinar el volumen del cuerpo de revolución engendrado por la
región ABCD indicada en la figura (siendo OB una de las ası́nto-
tas) al girar en torno al eje OX .

Solución

Sabiendo que la ecuación de la ası́ntota OB es y = bx/a, el volumen del


sólido indicado viene dado por
Z p+h " 2 #
πb2 p+h 2 2 2
 2 Z
bx 2 x
V =π −b 2
−1 dx = 2 (x −x +a ) dx = πb2 h.
p a a a p

PROBLEMA 11.58
Hallar el volumen generado por el área comprendida entre la parábo-
la y = 4x − x2 y el eje X al girar alrededor de la recta y = 6.

88
Solución

Utilizando el método de los discos, como la región está comprendida en el


intervalo [0, 4], el volumen, dado por la fórmula (5), es
Z 4 Z 4
2 2
V = π [6 − (6 − y) ] dx = π [36 − (6 − 4x + x2 )2 ] dx
0 0
Z 4
1408π
= π (48x − 28x2 + 8x3 − x4 ) dx = .
0 15

PROBLEMA 11.59
Un servilletero se obtiene practicando un agujero cilı́ndrico en una
esfera de modo que el eje de aquél pase por el centro de ésta.
Si la longitud del agujero es 2h, demostrar que el volumen del
servilletero es πah3 , siendo a un número racional.

Solución
Si llamamos r al radio de la esfera, el radio del agujero cilı́ndrico será k =

r 2 − h2 .

89
De este modo, y de acuerdo con la figura, el sólido obtenido viene dado al
girar alrededor del eje X la región limitada por las curvas x2 + y 2 = r2 e
y = k. Tenemos entonces:
h h
x3 4πh3
Z 
2 2 2 2 2
V =π (r − x − k ) dx = π (r − k )x − = .
−h 3 −h 3

Como 4/3 es racional, el resultado obtenido prueba el enunciado.


Una sección de la figura obtenida es la siguiente:

PROBLEMA 11.60
x2 y 2
Se considera la elipse de ecuación 2 + 2 = 1 y la cuerda F C
a b
paralela al eje OX . Determinar OA = h de manera que el volumen
engendrado por la región sombreada de la figura al girar en torno a
OX sea la mitad del volumen del elipsoide engendrado por el área
que limita la elipse dada girando en torno al mismo eje.

90
Solución

Designaremos por V1 y V2 a los volúmenes del cuerpo engendrado por la


región sombreada y del elipsoide engendradop por la elipse, respectivamen-
p
te. Como los puntos C y F tienen abscisa a 1 − h2 /b2 y −a 1 − h2 /b2 ,
respectivamente, dichos volúmenes se obtienen por integración mediante las
fórmulas:
Z √ 2 2
a 1−h /b
V1 = π √ [b2 (1 − x2 /a2 ) − h2 ] dx
−a 1−h2 /b2

Z a√1−h2 /b2 a 1−h2 /b2
b2 x3

2 2 2 2 2 2
= 2π [b (1 − x /a ) − h ] dx = 2π (b − h )x −
0 3a2 0
4 p
= πa(b2 − h2 ) 1 − h2 /b2 ;
3Z
a
4
V2 = π b2 (1 − x2 /a2 ) dx = πab2 .
−a 3

Como debe ser V1 = V2 /2, al resolver esta ecuación se obtiene que

4 2 2
p 2 2
q √
3
2 2
πa(b − h ) 1 − h /b = πab =⇒ h = b 1 − 1/ 4.
3 3

PROBLEMA 11.61
Calcular el volumen del toro, que es el sólido de revolución engen-
drado al girar un cı́rculo de radio r alrededor de un eje situado en
su plano y a una distancia b de su centro (b ≥ r).

91
Solución

Si hacemos que OX sea el eje de giro y el centro de la circunferencia el punto


(0, b), ésta tiene por ecuación x2 + (y − b)2 = r2 . El volumen, aplicando el
método de los discos, vendrá dado por:

Z r  p 2  p 2 
V = π b+ r2 − x2 − b − r2 − x2 dx = (cambio x = r sen t)
−r
Z π/2  π/2
2 2 1 2
= 4bπ r cos t dt = 2br π t + sen 2t = 2br2 π 2 .
−π/2 2 −π/2

PROBLEMA 11.62
Hallar el volumen de un cono recto de altura h, cuya base es una
elipse de eje mayor 2a y eje menor 2b.

Solución

La sección determinada en el cono por un plano paralelo a la base y de


altura OP = z es una elipse de eje mayor 2x y eje menor 2y. Su área es pues
πxy.

92
Por semejanza de triángulos, se deduce de la figura que

4 4 PC PM x h−z
M P C ∼ M OA =⇒ = es decir = ;
OA OM a h
4 4 PD PM y h−z
M P D ∼ M OB =⇒ = es decir = .
OB OM b h

πab(h − z)2
El área de la sección es entonces πxy = . Luego,
h2
Z h
πab πabh
V = 2 (h − z)2 dz = .
h 0 3

PROBLEMA 11.63
Un sólido tiene una base circular de radio 2. Cada sección produ-
cida por un plano perpendicular a un diámetro fijo es un triángulo
equilátero. Calcular el volumen del sólido.

Solución

Si expresamos por la ecuación x2 +y 2 = 4 a la base del sólido y consideramos


las secciones perpendiculares
p al eje X, √ el lado de√un triángulo genérico es
l = 2y y la altura es h = l2 − l2 /4 = l 3/2 = y 3.

93
El volumen será entonces

√ √
Z 2
2y · y 3 √ Z 2 2 32 3
V = dx = 3 (4 − x ) dx = .
−2 2 −2 3

PROBLEMA 11.64
Un cilindro cuya base es una elipse se corta por un plano inclinado
que pasa por el eje menor de la misma. Hallar el volumen del sólido
restante.

Solución

Supongamos que la ecuación de la elipse es x2 /a2 + y 2 /b2 = 1 y llamamos H


a la altura del cilindro (que corresponde al punto (a, 0)). Cortando el sólido
por planos perpendiculares al eje OY obtenemos triángulos rectángulos se-
mejantes. En un punto arbitrario (x, y) el área de uno de dichos triángulos
(ver figura) es

x·h x2 · tg α x2 · H
A= = = .
2 2 2a

94
Como (x, y) verifica la ecuación de la elipse, escribimos el área en función
a2 (1 − y 2 /b2 ) · H
de y como A(y) = . El volumen será entonces
2a

b b b
a(1 − y 2 /b2 ) · H y3
Z Z 
2abH
V = A(y) dy = 2 dy = aH y − 2 = .
−b 0 2 3b 0 3

PROBLEMA 11.65
Un sólido tiene una base en forma de elipse cuyos ejes mayor y
menor miden 10 y 8 unidades respectivamente. Hallar su volumen
sabiendo que toda sección del mismo perpendicular al eje mayor
es un triángulo isósceles de altura igual a 6.

Solución

Escribimos la ecuación de la elipse como x2 /25 + y 2 /16 = 1.

95
El triángulo obtenido por la sección perpendicular
p al eje OX por un punto
2
x tiene área A(x) = 2y · h/2 = 6y = 24 1 − x /25, y el volumen del sólido
(aplicando los métodos usuales de integración) es
Z 5 Z 5p
V = A(x) dx = 24 1 − x2 /25 dx
−5 −5
 5
24 25 x xp
= arc sen + 25 − x2 = 60π.
5 2 5 2 −5

PROBLEMA 11.66
La sección de un cierto sólido por cualquier plano perpendicular
al eje OX es un cuadrado tal que los extremos de una diagonal
pertenecen respectivamente a las parábolas y 2 = 4x, x2 = 4y . Hallar
el volumen del sólido.

Solución
La región que limitan ambas curvas viene indicada en la figura y los puntos
de corte son (0, 0) y (4, 4).

96
Como indica el enunciado, la diagonal de un cuadrado genérico une los

puntos (x, y1 ) y (x, y2 ) y su longitud, en función de x es d = 2 x − x2 /4.

Como el área del cuadrado es A(x) = d2 /2 = (2 x − x2 /4)2 /2, el volumen
pedido es:

4 √ " #4
(2 x − x2 /4)2 5 7/2
Z
1 x 2x 144
V = dx = 2x2 + − = .
0 2 2 80 7 35
0

C. LONGITUD DE CURVAS PLANAS.

Dada la función y = f (x), definida en un intervalo [a, b], a cada partición


P = {x0 = a, x1 , . . . , xn−1 , xn = b} de [a, b] le corresponde una poligonal de
vértices Pk = (xk , f (xk )), k = 0, 1, . . . , n, como indica la figura.

La longitud del arco de la curva entre los puntos A y B de abscisas x = a y


x = b se define como el supremo de los perı́metros de todas las poligonales. Si
es finito, se dice que la curva es rectificable; si no, la curva no es rectificable
(tiene longitud infinita). El resultado fundamental que aplicaremos en esta
sección es el siguiente:
Teorema. Si una función y = f (x) tiene derivada de primer orden continua
en [a, b], entonces es rectificable y la longitud del arco viene dada por la
fórmula Z bp
l = AB = 1 + [f 0 (x)]2 dx.
a

97
Si la función viene expresada en coordenadas paramétricas x = x(t), y =
y(t), la fórmula queda de la forma
Z t1 p
l= [x0 (t)]2 + [y 0 (t)]2 dt,
t0

siendo t0 y t1 los parámetros correspondientes a los puntos inicial y final de


la curva.
En la mayorı́a de los casos no es posible encontrar expresiones explı́citas de
la longitud de un arco de curva. Por ello se deben crear nuevas funciones,
como es el caso de las integrales elı́pticas (que expresan longitudes de arcos de
elipses), o utilizar métodos aproximados para calcular arcos de curva.

PROBLEMA 11.67
Hallar la longitud del arco de la parábola x2 = 2py , con p > 0,
comprendida en el intervalo [0, a].

Solución
Si calculamos la derivada de la función, tenemos
p
0
p p x2 + p2
y = x/p =⇒ 1 + y 02 = 1 + (x/p)2 = .
p
La longitud del arco pedido queda entonces
x + px2 + p2 a
" p #
1 ap 2 2 + p2
Z
p x x
l = x + p2 dx = + ln

p 0 2 p 2 p


# 0
" p p
p a a2 + p2 a + a2 + p2

= 2
+ ln .
2 p p

PROBLEMA 11.68
(
x cos(π/x) si x 6= 0
Probar que la curva f (x) = no es rectificable
0 si x = 0
en [0, 1].

98
Solución
(−1)n
Si consideramos los puntos xn = 1/n, con n ∈ N, sabemos que f (xn ) =
n
y la longitud de la poligonal de vértices xn es
s 2 
X 1 (−1)n (−1)n+1 2 X 2

X 1
ln = − + − = ,
n n+1 n n+1 n
n≥1 n≥1 n≥1

que es una serie divergente. Esto prueba que la curva no es rectificable en


[0, 1].

PROBLEMA 11.69
Calcular la longitud del arco de curva y = ln(cos x) en el intervalo
[0, π/3].

Solución
Como la derivada de la función es y 0 = − tg x, la longitud pedida es
Z π/3 p  π/3 √
l= 1 + tg2 x dx = ln(sec x + tg x) 0 = ln(2 + 3).
0

PROBLEMA 11.70
Hallar la longitud de la curva de ecuación 8a2 y 2 = x2 (a2 − 2x2 ).

Solución
√ √
−a/ 2 a/ 2

x p 2
Si escribimos la ecuación en forma explı́cita, tenemos y = ± √ a − 2x2 ,
2 2a
(a2 − 4x2 )2 p 3a2 − 4x2
de donde y 02 = 2 2 2
y 1 + y 02 = √ √ .
8a (a − 2x ) 2 2a a2 − 2x2
La longitud del arco será:

99

a/ 2
3a2 − 4x2
Z
1
L = 4· √ √ dx
2 2a 0 a2 − 2x2
" √ √ #a/√2
x 2 2 p
= 2a · arc sen + · x · a2 − 2x2 = πa.
a a
0

PROBLEMA 11.71
Hallar la longitud de la astroide de ecuación x2/3 + y 2/3 = a2/3 .

Solución

Escribiendo la ecuación en forma paramétrica como x = a cos3 t, y =


a sen3 t y teniendo en cuenta la simetrı́a de la figura, la longitud viene dada
por:
Z π/2 p Z π/2
L=4 x0 (t)2 + y 0 (t)2 dt = 4 3a sen t cos t dt = 6a.
0 0

PROBLEMA 11.72
Hallar la longitud de un lazo de la cicloide x = a(t − sen t), y =
a(1 − cos t).

Solución

2aπ 4aπ

100
Como un lazo de la cicloide es el arco de curva comprendido en el intervalo
t ∈ [0, 2π], la longitud es:
Z 2π p Z 2π p
L = x0 (t)2 + y 0 (t)2 dt = a (1 − cos t)2 + sen2 t dt
0 0
√ Z 2π √ √ Z 2π √
= a 2 1 − cos t dt = a 2 2 sen(t/2) dt = 8a.
0 0

PROBLEMA 11.73
Hallar la longitud de la curva cuya ecuación en forma paramétrica
es x(t) = a cos3 t, y(t) = a sen t(1 + cos2 t).

Solución

Debido a la simetrı́a de la figura, por la fórmula de la longitud de arco


tenemos:
Z π/2 p Z π/2 p
L = 4 x0 (t)2 + y 0 (t)2 dt = 4 a cos t 4 − 3 sen2 t dt = (cambio
0 0
√ √
sen 2u π/3 2a(4π + 3 3)
Z π/3  
3 16a 2 8a
sen t = sen u) = √ cos u du = √ u + = √ .
2 3 0 3 2 0 3 3

101
D. EJERCICIOS PROPUESTOS.

1. Encontrar una fórmula que permita calcular el área de cada una


de las regiones I, II, III y IV de la figura siguiente:

g(b) − f (0)
Resp.: Si llamamos r(x) = f (0) + · x a la recta que pasa
b
por los puntos (0, f (0)) y (b, g(b)), tenemos:
Z b
AI = [f (x) − g(x)] dx;
a
Z a Z b
AII = [h(x) − g(x)] dx + [h(x) − f (x)] dx;
0 a
Z a Z b
AIII = [f (x) − r(x)] dx + [g(x) − r(x)] dx;
0 a
Z a
AIV = [g(x) − f (x)] dx.
0

2. Hallar el área de la figura limitada por la hipérbola equilátera


xy = a2 , el eje OX y las rectas x = a, x = 2a.
Resp.: A = a2 ln 2.

3. Hallar el área encerrada por la recta y = 1 y la curva y = ln2 x.


Resp.: A = 4/e.

4. Calcular el área limitada por las curvas y = (x − 4)2 , y = 16 − x2 .

102
Resp.: A = 64/3.

5. Hallar el área limitada por la curva y = x2 − 2x + 2, su tangente


en el punto (3, 5), el eje OX y el eje OY .
Resp.: A = 23/8.

6. Calcular el área de la figura del primer cuadrante limitada por las


parábolas x2 = 2py , y 2 = 2px en el interior de la circunferencia
x2 + y 2 = 3p2 , (p > 0).
p2 √ √
Resp.: A = (4 2 + 9π − 36 arc sen 1/ 3).
24

7. Calcular el área de la región limitada por las curvas y = −x2 + 6,


(y − 2)2 + x2 = 4, y = x.
1
Resp.: A = (49 − 6π).
6

8. Hallar el área de la región limitada por la curva y = (x2 + 2x)e−x


y el eje OX en el tercer cuadrante.
Resp.: A = 4.

x
9. Hallar el área de la región limitada por la curva y = · arc sen x
(1 − x2 )2
y las rectas x = 0, x = 1/2, y = 0.
π 1
Resp.: A = − √ .
9 2 3

10. Calcular el área de la región limitada por las curvas y = 5 − x2 ,


y = (x − 1)2 .
Resp.: A = 9.

x2 y 2
11. Calcular el área de la elipse + 2 = 1.
a2 b
Resp.: A = πab.

12. Calcular el área de la región limitada por las curvas x2 + y 2 = 2,


y = x2 , y = x + 6.
45 + π
Resp.: A = .
2

103
13. Calcular el área de la región limitada por las gráficas de f (x) =
x2 − 4x + 4 y g(x) = 4 − x.
Resp.: A = 9/2.

14. Calcular el área de la figura limitada por la curva y = x3 , la recta


y = 8 y el eje OY .
Resp.: A = 12.

15. Hallar el área limitada por la curva y 2 = x2 − x4 .


Resp.: A = 4/3.

16. Hallar el área de la superficie interior a la circunferencia x2 +


y 2 = 16 y por encima de la parábola x2 = 12(y − 1).

16π + 4 3
Resp.: A = .
3

17. Hallar el área limitada por la curva y = cos 2x + cos x y el eje X


entre las dos ordenadas que corresponden a una distancia igual
a un perı́odo de la curva.

Resp.: A = 3 3.

18. Hallar el área encerrada por el bucle de la curva x3 = a(x2 − y 2 ).


8a2
Resp.: A = .
15

x2 y 2
19. Dada la hipérbola de ecuación − 2 = 1, determinar el área A
a2 b √ √
del triángulo mixtilı́neo AP Q, siendo A(a, 0), P (a 2, b), Q(a 2, 0).

ab √ √
Resp.: A = [ 2 − ln(1 + 2)].
2

20. Hallar el área del segmento circular de centro O y radio r com-


prendido entre las rectas x = a, x = b.
p b p a
Resp.: A = b r2 − b2 + r2 arc sen − a r2 − a2 − r2 arc sen .
r r

104
21. Hallar el área del segmento parabólico comprendido entre y 2 =
2px las rectas x = a, x = b.

4 2p 3/2
Resp.: A = (b − a3/2 ).
3

22. Hallar el volumen del sólido de revolución engendrado por la


figura limitada por la curva y = xex y las rectas y = 0, x = 1 al
girar alrededor del eje OX .
πe2
Resp.: V = .
4

23. Calcular el volumen del sólido engendrado al girar alrededor del


eje OX la región interior a la circunferencia x2 + y 2 = 1 y a la
parábola y 2 = 3x/2.
Resp.: 19π/48.

24. Calcular el volumen del sólido obtenido al girar


√ alrededor del eje
OX la región limitada por la curva y = 2 − 1 − x2 y el eje OX .
π
Resp.: V = (28 − 6π).
3

25. Calcular el volumen del sólido limitado por las curvas x2 −y 2 = 4,


y = 2, y = −2 al girar alrededor del eje OX .
32π √
Resp.: V = (2 2 − 1).
3

26. Calcular el volumen del sólido limitado por las curvas y = sen x,
y = 2x/π al girar alrededor del eje OX .
Resp.: V = π 2 /6.

27. Calcular el volumen del sólido


√ obtenido al girar la región limitada

por las gráficas de f (x) = 4 − x2 y g(x) = 1 en el intervalo [0, 3]
alrededor de y = 0.

Resp.: V = 2π 3.

28. Sea R la región interior a la circunferencia


√ de centro (1, −1) y
radio 2 y por encima de la recta y = 3 − 1.
a) Determinar el área de R.

105
b) Calcular el volumen del sólido obtenido al girar la región R
alrededor del eje OX .
2π √ 2π √
Resp.: A = − 3; V = (2 + 3 3 − 2π).
3 3

29. Sea R la región limitada por las curvas x + y = 2y 2 , y = x3 . Cal-


cular el área de R y el volumen que engendra R al girar alrededor
del eje OX .
Resp.: A = 7/12 (pensar x como función de y); V = 11π/21 (método
de los tubos).

x2
30. Sea R la región limitada por las curvas y = + 2 y 5x+8y −14 =
4
0. Calcular el área de R y el volumen de la figura obtenida al girar
R alrededor del eje OX .
891π
Resp.: A = 27/192; V = (método de los discos).
1280

31. Sea R la región limitada por las curvas y = 4x − x2 y 2x − y = 0.


Calcular el área de R y el volumen de la figura obtenida al girar
R alrededor del eje OX .
Resp.: A = 4/3; V = 32π/5.

1 x2
32. Sea R la región limitada por las curvas y = 2
e y = . Cal-
1+x 2
cular el área de R y el volumen de la figura obtenida al girar R
alrededor de los ejes OX y OY .
π π
Resp.: A = (3π − 2)/6; VX = (5π + 8) (discos); VY = (4 ln 2 − 1)
20 4
(tubos).

33. Se considera la región R limitada por las curvas x2 + (y − 1)2 = 5,


x = 2(y − 1)2 .
a) Calcular el área de R.
b) Calcular el volumen obtenido al girar la región R alrededor
del eje OY .
c) Calcular el volumen obtenido al girar la región R alrededor de
la recta y = 1.

106

1 2 116π 10 5 − 19
Resp.: A = 5 arc sen √ + ; VY = (discos); Vy=1 = ·π
5 3 15 3
(tubos).

34. Dada la región limitada por las curvas y = 4x2 , y = x2 /9, y = 2,


calcular el área de la región y el volumen obtenido al girar dicha
región alrededor de los ejes OX y OY .
√ √
Resp.: A = 20 2/3; VX = 16π 2 (tubos); VY = 35π/2 (discos).

35. Dada la región limitada por las curvas y = x2 + 1, y − 1 = x,


calcular el área de la región y el volumen obtenido al girar dicha
región alrededor del eje OY .
Resp.: A = 1/6; VY = π/6.

36. Dada la región limitada por las curvas x2 + y 2 = 12, x2 = 4y ,


y 2 = 4x, calcular el área de la región y el volumen obtenido al
girar dicha región alrededor del eje OY .

4 2 p π √
Resp.: A = + 12 arc sen 2/3 − 3π; VY = (256 5 − 200).
3 15

37. Se considera la región limitada por la curva y = sen(πx/2) +


cos(πx/2) + 1 y las rectas x = 0, x = 1 e y = 0. Hallar el área de
dicha región y el volumen del sólido obtenido al girar alrededor
del eje OX .
4
Resp.: A = 1 + ; V = 2π + 10.
π

38. Calcular el volumen del tronco de cono con radios de las bases r
y R y altura h.
πh 2
Resp.: V = (r + rR + R2 ).
3

39. Calcular la longitud del arco de la curva y = ex/2 + e−x/2 entre


los puntos de abscisa x = 0 y x = 2.
Resp.: L = e − e−1 .

ex + 1
40. Calcular la longitud del arco de la curva y = ln entre x = 1
ex − 1
y x = 2.
Resp.: L = ln(e2 + 1) − 1.

107
CAPÍTULO XII.
INTEGRALES
IMPROPIAS

SECCIONES
A. Integrales impropias de primera especie.
B. Integrales impropias de segunda especie.
C. Aplicaciones al cálculo de áreas y volúmenes.
D. Ejercicios propuestos.

109
A. INTEGRALES IMPROPIAS DE PRIMERA ESPECIE.

El concepto de integral definida se refiere a funciones acotadas en intervalos


cerrados [a, b], con a, b ∈ R. Este concepto se puede extender eliminando
estas restricciones. Ello da lugar a las integrales impropias.

Llamaremos integral impropia de primera especie aquella cuyo intervalo de


integración es infinito, ya sea de la forma (a, ∞), (−∞, b) o bien (−∞, ∞),
pero la función está acotada. Para cada uno de los casos indicados se defi-
ne
Z ∞ Z B
f (x) dx = lı́m f (x) dx,
a B→∞ a
Z b Z b
f (x) dx = lı́m f (x) dx,
−∞ A→−∞ A
Z ∞ Z B
f (x) dx = lı́m f (x) dx, 1
−∞ A→−∞ A
B→∞

y se dice que la integral impropia correspondiente es convergente si el lı́mite


existe y es finito y divergente en caso contrario. Las siguientes propiedades
son análogas a las correspondientes en las integrales propias (sólo conside-
raremos el caso del intervalo (a, ∞) pues el segundo caso se puede reducir
al primero con el cambio de variable t = −x y el tercer caso es combinación
de los dos anteriores al descomponer la integral en dos sumandos).

PROPIEDADES.

(1) La convergencia de la integral no depende del lı́mite de integración real.


Z ∞ Z ∞
Es decir, f (x)dx converge ⇐⇒ f (x)dx converge.
a b
Z ∞ Z ∞
(2) Homogénea. Si f es convergente, entonces λf es convergente,
a a
para todo λ ∈ R y se cumple:
Z ∞ Z ∞
λf = λ f.
a a

Z ∞ Z ∞ Z ∞
(3) Aditiva. Si f, g convergen, entonces (f + g) converge y
a a a
además Z ∞ Z ∞ Z ∞
(f + g) = f+ g.
a a a

110
(4) Integración por partes. Si f y g tienen derivadas de primer orden
continuas en [a, ∞) y dos de los tres lı́mites
Z b Z b
0
lı́m f (x)g (x) dx, lı́m f 0 (x)g(x) dx, lı́m [f (b)g(b) − f (a)g(a)]
b→∞ a b→∞ a b→∞

existen, entonces el tercero también existe y se tiene que


Z ∞ Z ∞
0
f (x)g (x) dx = lı́m [f (b)g(b) − f (a)g(a)] − f 0 (x)g(x) dx.
a b→∞ a
Z ∞ Z ∞
(5) Si |f | converge, entonces f converge.
a a
Esta última propiedad permite definir el concepto de convergencia ab-
soluta para el caso en que la función integrando no tenga signo cons-
tante en [a, ∞).
Z ∞
Dada una función f integrable en [a, x], para todo x > a, se dice que
Z ∞ Z ∞ a

converge absolutamente si la integral |f | converge, y que f converge


Z ∞ a Z a

condicionalmente si f converge pero |f | diverge.
a a

En los casos en que no sea posible (o no sea necesario) calcular explı́citamente


la integral, su convergencia se puede deducir por alguno de los siguientes
criterios (observar el paralelismo que mantienen algunos de estos criterios
con sus correspondientes para la convergencia de series).

CRITERIOS DE CONVERGENCIA.
(1) Criterio de comparación. Si f y g son funciones
Z ∞ Z ∞en [a, ∞)
continuas
y 0 ≤ f (x) ≤ g(x), ∀x > a, entonces 0 ≤ f (x) dx ≤ g(x) dx.
Z ∞ a Z a

Por tanto, si g(x) dx converge, entonces f (x) dx converge.
a a

(2) Comparación por paso al lı́mite. Sean f y g continuas y no nega-


tivas en [a, ∞).
f (x)
a) Si lı́m = λ 6= 0, λ finito, entonces
x→∞ g(x)
Z ∞ Z ∞
f (x) dx converge ⇐⇒ g(x) dx converge.
a a
f (x)
b) Si lı́m = 0, entonces
x→∞ g(x)
Z ∞ Z ∞
g(x) dx converge =⇒ f (x) dx converge.
a a

111
Z ∞
1
En muchos casos, debido a que dx converge si α > 1 y diverge
1 xα
si α ≤ 1 (ver problema 12.1), se aplica el criterio anterior con g(x) =
1/xα . Este queda entonces ası́:
(3) Sea f una función continua y no negativa en [a, ∞).
a) Si lı́m xα f (x) = λ 6= 0, λ finito, entonces
x→∞
Z ∞
f (x) dx converge ⇐⇒ α > 1.
a
Z ∞
α
b) Si lı́m x f (x) = 0 y α > 1, entonces f (x) dx converge.
x→∞ a
Z ∞
c) Si lı́m xα f (x) = ∞ y α ≤ 1, entonces f (x) dx diverge.
x→∞ a

(4) Criterio de Dirichlet. Sean f una función continua con primitiva


F acotada ∀x ≥ a y g una función decreciente con derivada primera
Z ∞
continua ∀x ≥ a. Si lı́m g(x) = 0, entonces f (x)g(x) dx converge.
x→∞ a

(5) Criterio de la serie asociada. Sea f una función decreciente y no


negativa ∀x ≥ a, y tal que lı́m f (x) = 0. Entonces
x→∞
Z ∞ X
f (x) dx converge ⇐⇒ f (n) converge.
a

PROBLEMA 12.1
Z ∞
Calcular xn dx con a > 0.
a

Solución
Para n 6= −1,
b b
xn+1
Z 
n 1
F (b) = x dx = (bn+1 − an+1 ).
=
a a n+1
n + 1
Z ∞
Si n > −1, entonces lı́m F (b) = ∞, con lo que xn dx diverge.
b→∞ a
Si n < −1, entonces la integral converge y
Z ∞
an+1
lı́m F (b) = xn dx = − .
b→∞ a n+1

112
Para n = −1,
Z b
dx
F (b) = = ln b − ln a
a x
y, como lı́m F (b) = ∞, la integral diverge.
b→∞

PROBLEMA 12.2
Z 0
Calcular ex dx.
−∞

Solución
Resolvemos directamente la integral:
Z 0 Z 0
x
 0
e dx = lı́m ex dx = lı́m ex a = lı́m (1 − ea ) = 1.
−∞ a→−∞ a a→−∞ a→−∞

PROBLEMA 12.3
Z ∞
1
Estudiar la convergencia de la integral √ dx.
0 ex

Solución
Calcularemos directamente la integral aplicando la definición de integral
impropia.

Z ∞ Z b
1 b
e−x/2 dx = lı́m −2e−x/2 0 = lı́m (−2e−b/2 +2) = 2,

√ dx = lı́m
0 ex b→∞ 0 b→∞ b→∞

de lo que se deduce que la integral es convergente.

PROBLEMA 12.4
Z ∞
Estudiar la convergencia de la integral e−a|x| dx, a ∈ R.
−∞

Solución
En primer lugar, si a = 0, e0 = 1 y la integral diverge.

113
Si a 6= 0, descomponemos la integral en dos sumandos y obtenemos:
Z 0 Z ∞ Z 0 Z m
−ax
I = ax
e dx + e dx = lı́m ax
e dx + lı́m e−ax dx
−∞ 0 k→−∞ k m→∞ 0
  0 m
1 ax1 −ax
= lı́m + lı́m − e
e
k→−∞ k
m→∞ a a 0
    (
1 1 ak 1 −am 1 2/a si a > 0,
= lı́m − e + lı́m − e + =
k→−∞ a a m→∞ a a ∞ si a < 0.

Resulta en definitiva que la integral propuesta es convergente cuando a > 0


y divergente cuando a ≤ 0.

PROBLEMA 12.5
Z ∞
Calcular xe−x dx.
0

Solución
Utilizaremos la propiedad (4), relacionada con la integración por partes para
integrales impropias. Para ello, tomando f (x) = x, g 0 (x) = e−x , tenemos que
f 0 (x) = 1, g(x) = −e−x y
Z ∞ Z b
xe−x dx = lı́m xe−x dx
0 b→∞ 0
b
Z b b
xe−x 0 e−x dx = lı́m − e−x
 
= lı́m − + lı́m 0
= 1,
b→∞ b→∞ 0 b→∞

b −b
− xe−x = lı́m −be−b = lı́m

debido a que lı́m 0
= 0.
b→∞ b→∞ b→∞ eb

PROBLEMA 12.6
Z ∞
dx
Hallar x −x
.
−∞ e + e

Solución
Como ambos lı́mites de integración son infinitos, descomponemos la integral
1 ex
en dos sumandos. Si escribimos el integrando como x = , te-
e + e−x 1 + e2x

114
nemos:
b Z 0 x
ex dx
Z
e dx
I = lı́m 2x
+ 0 lı́m
b→∞ 0 1 + e b →−∞ b0 1 + e2x
b 0
= lı́m arc tg ex 0 + 0 lı́m arc tg ex b0
  
b→∞ b →−∞
0 π π π π
= lı́m (arc tg eb − π/4) + 0 lı́m (π/4 − arc tg eb ) = − + −0= .
b→∞ b →−∞ 2 4 4 2

PROBLEMA 12.7
Z ∞
1
Estudiar la convergencia de la integral dx.
2 x(ln x)8

Solución
Si calculamos directamente la integral, tenemos:
∞ b
b
(ln x)−7
Z Z 
1 −8
dx = lı́m (1/x)(ln x) dx = lı́m
2 x(ln x)8 b→∞ 2 b→∞ −7 2
 
−1 1 1
= lı́m + = ,
b→∞ 7(ln b)7 7(ln 2)7 7(ln 2)7

de modo que la integral es convergente.

PROBLEMA 12.8
Z ∞
x
Estudiar la convergencia de la integral ex−e dx.
−∞

Solución
Resolvemos en primer lugar la integral indefinida haciendo el cambio de
variable ex = t:
Z Z Z
x x x
ex−e dx = ex · e−e dx = e−t dt = −e−t = −e−e .

Calculamos a continuación la integral impropia y tenemos:


Z ∞ Z b
x x b a
ex−e dx = lı́m ex−e dx = lı́m (−e−e + e−e ) = 0 + 1 = 1;
−∞ a→−∞ a a→−∞
b→∞ b→∞

de lo que se deduce que la integral es convergente.

115
Z ∞
PROBLEMA 12.9 Hallar e−x sen x dx.
0

Solución
El lı́mite superior de integración es infinito con lo que, al integrar por partes,
obtenemos:
Z b  b
−x 1 −x
I = lı́m e sen x dx = lı́m − e (sen x + cos x)
b→∞ 0 b→∞ 2 0
1 −b 1
= lı́m − e (sen b + cos b) + .
b→∞ 2 2

Cuando b → ∞, e−b → 0, mientras que | sen b + cos b| ≤ 2, luego I = 1/2.

PROBLEMA 12.10
Z ∞
Calcular In = xn e−x dx, para n ∈ N.
0

Solución
Integrando por partes, obtenemos que
Z Z
x e dx = −x e + n xn−1 e−x dx.
n −x n −x

Recordando además que lı́m bn e−b = 0, resulta:


b→∞
Z b Z b
n −x n −b
In = lı́m x e dx = lı́m −b e + n lı́m xn−1 e−x dx = n · In−1 .
b→∞ 0 b→∞ b→∞ 0

Z recurrencia, se llega a que In = n(n − 1)In−2 = · · · = n! · I0


Procediendo por

y como I0 = e−x dx = 1, obtenemos que In = n!
0

PROBLEMA 12.11
Z +∞
dx
Hallar 2
.
0 x +4

116
Solución
Por definición de integral impropia, tenemos:
b
arc tg(x/2) b
Z  
dx π
I = lı́m 2
= lı́m = .
b→∞ 0 x + 4 b→∞ 2 0 4

PROBLEMA 12.12

x2 − x + 2
Z
Calcular la integral dx.
−∞ x4 + 10x2 + 9

Solución
Por definición de integral impropia
∞ B
x2 − x + 2 x2 − x + 2
Z Z
dx = lı́m dx.
−∞ x + 10x2 + 9
4 A→−∞ A x4+ 10x2 + 9
B→∞

Resolvemos en primer lugar la integral indefinida para lo cual aplicamos el


método de integración por fracciones simples. Como
" r #
x2 − x + 2 x2 + 9
Z
1 7 x
dx = ln + arc tg x + arc tg ,
x4 + 10x2 + 9 8 x2 + 1 3 3

la integral propuesta valdrá


 
1 π 7 π π 7 π 5π
I= ln 1 + + · − ln 1 + + · = .
8 2 3 2 2 3 2 12

PROBLEMA 12.13
Z ∞
dx
Demostrar que es convergente, para todo m ∈ N.
0 (1 + x2 )m

Solución
En efecto, si hacemos el cambio de variable x = tg t, dx = sec2 t dt, los
lı́mites de integración son ahora t = 0 (correspondiente a x = 0) y t = π/2
(cuando x = ∞). La integral queda ahora
π/2 π/2 π/2
sec2 t dt
Z Z Z
2−2m
= sec t dt = cos2m−2 t dt,
0 (1 + tg2 t)m 0 0

117
la cual es evidentemente convergente para m natural.

PROBLEMA 12.14
Determinar
Z ∞ el valor de C paraque sea convergente la integral im-
x C
propia 2
− dx. Hallar el valor de dicha inte-
1 2x + 2C x+1
gral.

Solución
Si escribimos la función integrando como cociente de polinomios,
x C x2 + x − 2Cx2 − 2C 2 (1 − 2C)x2 + x − 2C 2
− = = ,
2x2 + 2C x+1 (2x2 + 2C)(x + 1) (2x2 + 2C)(x + 1)
observamos que el denominador tiene grado 3. Para que la integral sea con-
vergente, el grado del numerador debe ser menor que 2. De aquı́ se deduce
que 1 − 2C = 0, es decir C = 1/2.
Para este valor, la integral queda:
Z ∞  Z b Z b 
x 1/2 x 1/2
− dx = lı́m dx − dx
1 2x2 + 1 x + 1 b→∞ 2
1 2x + 1 1 x+1
 b
1 2 1
= lı́m ln(2x + 1) − ln(x + 1)
b→∞ 4 2 1
 
1 2 1 1 1
= lı́m ln(2b + 1) − ln 3 − ln(b + 1) + ln 2
b→∞ 4 4 2 2
1 2
4(2b + 1) 1 8
= lı́m · ln = · ln .
b→∞ 4 3(b + 1)2 4 3

PROBLEMA 12.15
Hallar los valores de los parámetros a y b para que
Z ∞ 2 
2x + bx + a
− 1 dx = 1.
1 x(2x + a)

Solución
Al igual que en el problema anterior, escribimos el integrando como una frac-
ción para comparar los grados del numerador y denominador. Como
2x2 + bx + a (b − a)x + a
−1= ,
x(2x + a) x(2x + a)

118
la integral será convergente cuando b − a = 0, es decir a = b.
En este caso, si integramos por fracciones simples, obtenemos que
x k
Z ∞ 2   
2x + bx + a
I = − 1 dx = lı́m ln
1 x(2x + a) k→∞ 2x + a 1
k 1 1 1
= lı́m ln − ln = ln − ln .
k→∞ 2k + a 2+a 2 2+a
1 1
Como debe ser 1 = ln − ln , resulta que a = b = 2e − 2.
2 2+a

PROBLEMA 12.16
Z ∞
ln x
Estudiar la convergencia de la integral dx.
1 x2

Solución
Resolvemos la integral indefinida por partes haciendo u = ln x y dv =
dx/x2 . Ası́ du = dx/x, v = −1/x y:
Z Z
ln x ln x dx ln x 1 1 + ln x
2
dx = − + 2
=− − =− .
x x x x x x
La integral impropia queda entonces:

1 + ln x b
Z ∞ Z b    
ln x ln x 1 + ln b
dx = lı́m dx = lı́m − = lı́m − + 1 = 1,
1 x2 b→∞ 1 x2 b→∞ x 1 b→∞ b

pues lı́m ln b/b = 0 (se puede aplicar por ejemplo la regla de L’Hôpital).
b→∞

Otra posibilidad, en la que no se calcula directamente la integral, es utilizar


el criterio de comparación. Debido a que:
ln x/x2 ln x 1/x 2
lı́m = lı́m 1/2 = lı́m = lı́m 1/2 = 0,
x→∞ 1/x3/2 x→∞ x x→∞ (1/2)x−1/2 x→∞ x
Z ∞
1
e dx es convergente, se deduce la convergencia de la integral pro-
1 x3/2
puesta.

PROBLEMA 12.17

x2 + 3x + 1
Z
Estudiar la convergencia de la integral √ dx.
1 x4 + x3 + x

119
Solución
En primer lugar observamos que la función integrando es positiva en el
intervalo de integración. Como la diferencia de grados entre el denominador
y el numerador es 2, comparamos el integrando con la función 1/x2 . Debido
a que
x2 +3x+1

x4 +x3 + x x4 + 3x3 + x2
lı́m = lı́m √ = 1,
x→∞ 1/x2 x→∞ x4 + x3 + x
Z ∞
y la integral impropia dx/x2 es convergente, la integral propuesta tam-
1
bién es convergente.

PROBLEMA 12.18
Z ∞
dx
Estudiar la convergencia de la integral √
3
.
1 2x + x + 1 + 5

Solución
Análogamente al problema anterior, la función es positiva en el intervalo
[1, ∞). Además, cuando x → ∞, es un infinitésimo del mismo orden que
1/x, es decir
√1
2x+ 3 x+1+5
lı́m = 1/2.
x→∞ 1/x
Z ∞
dx
Como es divergente, la integral propuesta también lo será.
1 x

PROBLEMA 12.19
Z ∞
x
Estudiar la convergencia de la integral √ dx.
0 x4+1

Solución
La convergencia de la integral dada equivale a la convergencia de la inte-
gral
Z ∞
x
√ dx porque, en el intervalo [0, 1], el integrando es acotado y la
1 x4 + 1
integral es propia.
Como la función integrando es positiva en el intervalo de integración, pode-
mos aplicar el criterio de comparación. Ası́ tenemos que

x/ x4 + 1 x2
lı́m = lı́m √ = 1,
x→∞ 1/x x→∞ x4 + 1

120
pues el grado
Z ∞ del numerador coincide con el grado del denominador. Como la
dx
integral es divergente, también es divergente la integral propuesta.
1 x

PROBLEMA 12.20
Z ∞
dx
Investigar la convergencia de la integral √ .
1 x3 + 1

Solución
Como el integrando es positivo aplicamos el criterio de comparación por
paso al lı́mite. Cuando x → ∞, tenemos
1 1 1 1 1
√ =p = ·p ∼ 3/2 .
+1 x3 x3 (1 + 1/x3 ) x3/2 1 + 1/x3 x
Z ∞
dx
Como la integral es convergente, la integral propuesta también lo
1 x3/2
será.

PROBLEMA 12.21

x2 dx
Z
Estudiar la convergencia de la integral .
0 (a2 + x2 )3/2

Solución
Comparamos el integrando con la función y = 1/x. Tenemos ası́:
x2
(a2 +x2 )3/2 x3
lı́m = lı́m = 1.
x→∞ 1/x x→∞ x3 · (a2 /x2 + 1)3/2
Z ∞
dx
Como es divergente, también lo es la integral propuesta.
0 x

PROBLEMA 12.22
Z ∞
x dx
Estudiar la convergencia de la integral √ .
3 x6 + 1

Solución
Comparando los grados del numerador y denominador, obtenemos que g(x) =
1/x2 es un infinitésimo equivalente a la función integrando cuando x → ∞.

121
Z ∞
dx
Como además es convergente, por el criterio de comparación dedu-
3 x2
cimos que la integral propuesta es también convergente.

PROBLEMA 12.23
Z ∞
2
Estudiar la convergencia de la integral e−x dx.
−∞

Solución
En primer lugar descomponemos la integral en tres sumandos. Además, de-
bido a la simetrı́a de la función integrando, podemos escribir:
Z −1 Z 1 Z ∞ Z 1 Z ∞
−x2 −x2 −x2 −x2 2
I= e dx+ e dx+ e dx = e dx+2 e−x dx.
−∞ −1 1 −1 1

Para estudiar la convergencia de esta última integral impropia, como la fun-


ción integrando es positiva, aplicamos el criterio de comparación. Tenemos
2
por un lado que se verifica la acotación e−x ≤ e−x , ∀x ≥ 1, y por otro lado
que
Z ∞ Z b b  
−x
e−x dx = lı́m −e−x 1 = lı́m −e−b + e−1 = e−1 .

e dx = lı́m
1 b→∞ 1 b→∞ b→∞

Esto indica que la integral propuesta es convergente.

PROBLEMA 12.24

x3
Z
Investigar la convergencia de la integral dx.
0 2x

Solución
x3
Debido a que 2x es un infinito de orden superior a x3 , es decir lı́m x = 0,
x→∞ 2
aplicaremos el criterio de comparación por paso al lı́mite con la función
g(x) = 1/2x . Ahora bien, como

x3 /2x
lı́m = lı́m x3 = ∞,
x→∞ 1/2x x→∞

Z ∞
dx
e converge, el criterio no puede aplicarse con esta función.
0 2x

122
Si tomamos una función un poco mayor que g, como h(x) = (2/3)x , tene-
mos:
x3 /2x x3
lı́m = lı́m = 0,
x→∞ (2/3)x x→∞ (4/3)x

y además
∞ b
(2/3)x
Z 
1
(2/3)x dx = lı́m =− .
0 b→∞ ln 2/3 0 ln 2/3
El citado criterio de comparación indica pues que la integral propuesta es
convergente.

PROBLEMA 12.25
Z ∞ √
x
Determinar si la integral dx converge o no.
1 3x

Solución
El integrando es no negativo y decreciente en [1, ∞). Recordamos que, de
acuerdo con el criterio de la integral para series Z
infinitas, si f es una función
∞ X
no creciente y no negativa en [1, ∞), entonces f y f (n) convergen
1 n≥1
ambas o divergen ambas.
X √n
En este caso la convergencia de la serie se puede determinar por el
3n
n≥1
criterio de la raı́z. Tenemos ası́:
√ √
n + 1/3n+1 3n · n + 1 1
lı́m √ n
= lı́m n+1 √ = < 1,
n→∞ n/3 n→∞ 3 · n 3

de modo que la serie converge, con lo que también la integral dada converge.

PROBLEMA 12.26
Z ∞
x
Estudiar la convergencia de la integral dx.
0 ex −1

Solución
x
Aunque la función no está definida en x = 0, como lı́m = 1, la
ex − 1 x→0+
función está acotada para x > 0 y la integral no es impropia enX x = 0. El
n
carácter de esta integral es el mismo que el de la serie asociada .
en − 1

123
n X 1
Aplicando el criterio de Pringsheim, como lı́m n2 · = 0 y es
n→∞ en − 1 n2
convergente, también lo es la serie anterior.

PROBLEMA 12.27

4x3 + 2x + 1
Z
Estudiar la convergencia de la integral dx.
0 ex

Solución
Debido a que la función integrando es positiva en el intervalo de integración
y tiende a cero cuando x → ∞, reducimos el estudio de la convergencia
X 4n3 + 2n + 1
de la integral al de la serie asociada . Por el criterio de la
en
n≥0
raı́z, r
4n3 + 2n + 1
n
lı́m = lı́m 1/e < 1.
n→∞ en n→∞
Entonces la integral es convergente.

PROBLEMA 12.28
Z ∞
ln(1 + x)
Estudiar el carácter de la integral I = dx.
0 ex

Solución
Como la función integrando es no negativa en el intervalo de integración,
X ln(1 + n)
estudiaremos el carácter de la serie asociada .
en
Aplicando el criterio del cociente tenemos:
ln(n+2)
en+1 ln(n + 2) 1
lı́m ln(n+1)
= lı́m = < 1,
e ln(n + 1) e
en

lo que indica que la serie es convergente y, en consecuencia, también es


convergente la integral propuesta.

PROBLEMA 12.29
Z ∞
x dx
Estudiar el carácter de la integral .
0 1 + x2 sen2 x

124
Solución
X n
Como la serie asociada a la integral impropia es , la cual es
1 + n sen2 n
2
X1
equivalente a la serie y esta es divergente, también será divergente la
n
integral dada.

PROBLEMA 12.30
Z ∞
sen kx
Estudiar la convergencia de la integral dx.
0 ex2

Solución
Como la función integrando cambia de signo, estudiamos la convergencia
X | sen kn|
absoluta. La serie asociada a la integral es que es convergente
n≥0
en2
| sen kn| 1
pues n 2 ≤ n2 y, por el criterio de la raı́z,
e e
r
1 1
lı́m n n2 = lı́m n = 0 < 1.
n→∞ e n→∞ e
Lo anterior indica que la integral dada es absolutamente convergente.

PROBLEMA 12.31
Z ∞
sen x
Estudiar la convergencia de la integral dx, para
1 xα
α > 0.

Solución
Como la función f (x) = sen x tiene primitiva F (x) = − cos x acotada y la
función g(x) = 1/xα es derivable y decreciente, con lı́m g(x) = 0, por el
x→∞
criterio de Dirichlet (4) se deduce que la integral es convergente.

PROBLEMA 12.32
Z ∞
cos x
Estudiar el carácter de la integral dx.
1 x2

125
Solución
Como el integrando no es una función positiva en el intervalo de integración,
debemos estudiar la convergencia
Z ∞ absoluta.ZComo | cos x| ≤ 1, ∀x, tenemos
cos x ∞
1 cos x 1
que 2 ≤ 2 de donde dx ≤ dx, la cual es convergen-

x x 1 x2 1 x2
te. Se deduce por el criterio de comparación que la integral propuesta es
absolutamente convergente.
Z ∞
f (x)
Como regla general podemos afirmar que, si en la expresión dx el
1 xn
numerador
Z ∞ está acotado, la integral impropia converge absolutamente si lo
dx
hace n
.
1 x

PROBLEMA 12.33
Z ∞
sen x
Probar que dx converge condicionalmente.
0 x

Solución
sen x
Aunque la función no esté definida en x = 0, está acotada pues lı́m = 1.
x→0 x
Por tanto laZ convergencia de la integral dada equivale a la convergencia de

sen x
la integral dx. Como vimos en el problema 12.31, esta integral es
1 x
convergente.
Z ∞
sen x
1 − cos 2x
Sin embargo, dx diverge pues, como | sen x| ≥ sen2 x = ,
x 2

1
tenemos que
Z ∞
| sen x| 1 ∞ dx 1 ∞ cos 2x
Z Z
dx ≥ − dx.
1 x 2 1 x 2 1 x
Z ∞ Z ∞
dx cos 2x
De las dos últimas integrales, diverge y dx converge,
1 x 1 x
pues, integrando por partes,

sen 2x b
  Z ∞ Z ∞
− sen 2
Z
cos 2x sen 2x sen 2x
dx = lı́m + 2
dx = + dx,
1 x b→∞ 2x 1 1 2x 2 1 2x2

y esta última integral converge absolutamente como se deduce por la acota-


sen 2x
ción ≤ 1 .
2x 2x2
2
Z ∞
sen x
De lo anterior se deduce que dx converge condicionalmente.
1 x

126
PROBLEMA 12.34

sen3 x
Z
Estudiar la convergencia de la integral dx.
0 x

Solución
La integral es impropia por tener un lı́mite de integración infinito. Aunque
sen3 x x3
además la función no está definida en x = 0, como lı́m = lı́m = 0,
x→0+ x x→0+ x
la integral no es impropia en x = 0.
3 sen x sen 3x
Para estudiar la convergencia utilizamos la fórmula sen3 x = − .
4 4
Entonces
Z ∞
sen3 x 3 ∞ sen x 3 ∞ sen 3x
Z Z
dx = dx − dx,
0 x 4 0 x 4 0 3x

y cada uno de los sumandos es convergente como vimos en el problema


anterior. Entonces su suma será también convergente.

B. INTEGRALES IMPROPIAS DE SEGUNDA ESPECIE.

Si una función y = f (x) no está acotada en un intervalo [a, b], no tiene


sentido el concepto de integral definida de f en [a, b]. Esta situación da lugar
a las integrales impropias de segunda especie; para definirlas, distinguimos
los siguientes casos:
a) Si f es integrable en [a, r], ∀r < b, y lı́m f (x) = ±∞, definimos
x→b−

Z b Z r Z b−ε
f (x) dx = lı́m f (x) dx = lı́m f (x) dx.
a r→b− a ε→0+ a

b) Si f es integrable en [s, b], ∀s > a, y lı́m f (x) = ±∞, definimos


x→a+

Z b Z b Z b
f (x) dx = lı́m f (x) dx = lı́m f (x) dx.
a s→a+ s ε→0+ a+ε

127
c) Si existe c ∈ (a, b) tal que f es integrable en [a, r] ∪ [s, b], ∀r < c, s > c
y lı́m f (x) = ±∞, definimos
x→c
Z b Z r Z b
f (x) dx = lı́m f (x) dx + lı́m f (x) dx.
a r→c− a s→c+ s

Al igual que para las integrales impropias de primera especie, se dice que
una integral es convergente si existe el lı́mite o lı́mites que las definen.
Las propiedades 1 a 5 enunciadas para las integrales impropias de primera
especie son válidas también aquı́ con las modificaciones obvias. También
los criterios de convergencia son análogos a los allı́ indicados pues existe
un paralelismo entre ambos tipos de integrales impropias. Ası́, en el primer
caso, si lı́m f (x) = ±∞, al hacer el cambio de variable b − x = 1/t, se
x→b−
tiene: Z b Z ∞
f (x) dx = g(t) dt,
a 1/(b−a)

y resulta una integral impropia de primera especie.


Escribiremos a continuación los criterios especı́ficos para el caso a) aclarando
nuevamente que los demás pueden plantearse de forma similar.
(1) Criterio de comparación. Si f y g son funciones continuas en [a, r],
∀r < b y 0 ≤ f (x) ≤ g(x), ∀x ∈ [a, r], entonces
Z b Z b
g(x) dx converge =⇒ f (x) dx converge.
a a

(2) Comparación por paso al lı́mite. Sean f y g continuas y no nega-


tivas en [a, r], ∀r < b.
f (x)
a) Si lı́m = λ 6= 0, λ finito, entonces
x→b− g(x)
Z b Z b
f (x) dx converge ⇐⇒ g(x) dx converge.
a a

f (x)
b) Si lı́m = 0, entonces
x→b− g(x)
Z b Z b
g(x) dx converge =⇒ f (x) dx converge.
a a

Como aplicación, es común considerar el criterio de comparación con


Z b
1
la función g(x) = 1/(b − x)α pues dx converge si α < 1 y
a (b − x)α
diverge si α ≥ 1 (ver problema 12.35). Entonces tenemos:

128
(3) Sea f una función continua y no negativa en [a, r], ∀r < b.
a) Si lı́m (b − x)α f (x) = λ 6= 0, λ finito, entonces
x→b−
Z b
f (x) dx converge ⇐⇒ α < 1.
a
Z b
α
b) Si lı́m (b − x) f (x) = 0 y α < 1, entonces f (x) dx converge.
x→b− a
Z b
α
c) Si lı́m (b − x) f (x) = ∞ y α ≥ 1, entonces f (x) dx diverge.
x→b− a
En los siguientes ejercicios se muestran también casos en que una integral
debe descomponerse como integral impropia de primera y segunda especie.

PROBLEMA 12.35
Z b
dx
Resolver α
, con α ∈ R, donde a < b.
a (b − x)

Solución
Distinguiremos los siguientes casos:
- Si α = 1, por definición de integral impropia,
Z b Z r
dx dx  r
= lı́m = lı́m − ln(b − x) a
a b−x r→b− a b − x r→b−
= lı́m [− ln(b − r) + ln(b − a)] = ∞.
r→b−

- Si α 6= 1,
r
(b − x)−α+1
Z b Z r 
dx dx
= lı́m = lı́m −
a (b − x)α r→b− a (b − x)α r→b− −α + 1 a
−α+1 −α+1
 (
∞ si − α + 1 < 0

(b − a) − (b − r)
= lı́m = (b−a)−α+1
r→b− −α + 1 −α+1 si − α + 1 > 0.

En definitiva, la integral propuesta es convergente cuando α < 1 y divergente


cuando α ≥ 1.

PROBLEMA 12.36
Z b
dx
Calcular 3/2
donde a < b.
a (x − a)

129
Solución
Como la función no está acotada en x = a, hacemos lo siguiente:
Z b Z b  b
dx dx −2
3/2
= lı́m = lı́m √
a (x − a) c→a+ c (x − a)3/2 c→a+ x−a c
 
−2 2
= lı́m √ +√ = ∞.
c→a + b−a c−a

PROBLEMA 12.37
Z 3
dx
Calcular √ .
0 9 − x2

Solución
El integrando presenta una discontinuidad esencial en x = 3. Resulta enton-
ces:

Z 3 Z 3−ε
dx dx
√ = lı́m √
0 9 − x2 ε→0+ 0 9 − x2
 3−ε 3−ε π
= lı́m arc sen x/3 0 = lı́m arc sen = arc sen 1 = .
ε→0 + ε→0 + 3 2

PROBLEMA 12.38
Z 1
dx
Estudiar la convergencia de la integral p .
0 (1 − x)(2 + x)

Solución
1
El integrando f (x) = p es no negativo y lı́m f (x) = ∞. To-
(1 − x)(2 + x) x→1−
1
mando g(x) = √ , tenemos:
1−x
f (x) 1 1
lı́m = lı́m √ = √ > 0.
x→1− g(x) x→1− 2 + x 3
Por tanto, la integral dada converge si y sólo si converge la integral de g.
Ahora bien,

Z 1 Z b
dx dx  b
√ = lı́m √ = lı́m − 2 1 − x 0 = 2,
0 1 − x b→1− 0 1 − x b→1−

130
luego la integral dada es convergente.

PROBLEMA 12.39
Z 1
dx
Investigar si es convergente la integral √ .
0 1 − x4

Solución
La función integrando tiene una discontinuidad en x = 1. Comparamos la
integral propuesta con la de 1/(1 − x)α con α apropiado. Debido a que

1/ 1 − x4 (1 − x)α (1 − x)α
lı́m α
= lı́m √ = lı́m p = 1/2,
x→1− 1/(1 − x) x→1− 1 − x4 x→1− (1 − x)1/2 (1 + x)(1 + x2 )

Z 1
1
cuando α = 1/2 y además 1/2
dx es convergente, del criterio de
0 (1 − x)
comparación se deduce la convergencia de la integral propuesta.

PROBLEMA 12.40
Z 2
dx
Estudiar la convergencia de la integral √ .
0 (1 + x2 ) 4 − x2

Solución
Z 2
dx
Aplicamos el criterio de comparación con la integral convergente .
0 (2 − x)1/2
Como
1√
(1+x2 ) 4−x2 1 1
lı́m 1 = lı́m = ,
x→2−
(2−x)1/2
x→2− (1 + x2 )(2 + x)1/2 10
la integral es convergente.

PROBLEMA 12.41
Z 1
dx
Estudiar la convergencia de la integral p .
0 (1 − x3 )n

Solución
La integral es impropia porque el integrando tiende a infinito cuando x → 1.
dt
Hacemos el cambio de variable x3 = t, dx = 2/3 . La integral se escribe
3t

131
Z 1
dt
ahora como I = p . A primera vista parece que se ha com-
2/3 (1 − t)n
0 3t
plicado la integral pues ahora es impropia para los dos extremos del intervalo.
Dividimos éste en dos sumandos:
Z 1/2 Z 1
dt dt
I= p + p .
3t 2/3 (1 − t)n 2/3 (1 − t)n
0 1/2 3t
Z 1/2
dt
El primer sumando es convergente pues la integral es equivalente a
0 t2/3
que sabemos es convergente. El segundo sumando, al estar acotado 1/t2/3
en todo el intervalo, será convergente cuando n/2 < 1, es decir n < 2.
Otro método más sencillo serı́a descomponer 1 − x3 de la siguiente forma
1 − x3 = (x2 + x + 1)(1 − x). La integral queda entonces
√ dx
Z 1
(x2 +x+1)n
I= .
0 (1 − x)n/2
Como el numerador está acotado en todo el intervalo y el grado del deno-
minador es n/2, la integral será convergente cuando n/2 < 1.

PROBLEMA 12.42
Z 4
dx
Demostrar que 2
no existe.
0 (x − 1)

Solución
El integrando presenta una discontinuidad esencial en x = 1, valor com-
prendido entre los lı́mites de integración. Descomponemos la integral en dos
sumandos y resulta:
Z 1−ε Z 4
dx dx
I = lı́m 2
+ lı́m 2
ε→0 +
0 (x − 1) 0
ε →0 +
1+ε0 (x − 1)
 1−ε  4
−1 −1
= lı́m + lı́m
ε→0 + x−1 0 0
ε →0 + x − 1 1+ε0
   
1 1 1
= lı́m − 1 + lı́m − + 0 = ∞.
ε→0+ ε ε0 →0+ 3 ε
Si no se hubiera tenido en cuenta el punto de discontinuidad, obtendrı́amos
equivocadamente el resultado:
Z 4  4
dx 1 4
2
= − =−
0 (x − 1) x−1 0 3

132
pues además no es posible que la integral de una función positiva sea nega-
tiva.

PROBLEMA 12.43
Z 1
dx
Estudiar la convergencia de la integral I = √
3
.
−1 x

Solución
Como la función no está acotada en x = 0, descomponemos la integral en
suma: Z 1 Z 0 Z 1
dx dx dx
√3
= √3
+ √
3
.
−1 x −1 x 0 x
Z a
dx
Cada uno de los sumandos es convergente pues tiene la forma α
con
0 x
α < 1. De ello se deduce que la integral es convergente.
El valor de la integral serı́a el mismo si no se tuviera en cuenta la disconti-
nuidad esencial en x = 0, pero no serı́a correcto el proceso seguido.

PROBLEMA 12.44
Z 4
dx
Hallar √
3
.
0 x−1

Solución
Como el integrando presenta una discontinuidad en x = 1, tenemos que
Z 1−ε Z 4
dx dx
I = lı́m √
3
+ lı́m √3
ε→0 +
0 x − 1 ε →0 1+ε0 x − 1
0 +

3 1−ε 3 4
= lı́m (x − 1)2/3 0 + lı́m (x − 1)2/3 1+ε0
ε→0+ 2 ε0 →0+ 2
3 3 √ 3 √ 
= lı́m [(−ε)2/3 − 1] + lı́m [ 9 − (ε0 )2/3 ] =
3 3
9−1 .
ε→0+ 2 ε0 →0+ 2 2

PROBLEMA 12.45
Z 3
dx
Determinar el carácter de la integral p .
2 (3 − x)(x − 2)

133
Solución
La integral es impropia porque el integrando tiende a infinito en los dos extre-
mos del intervalo. Separamos la integral en dos sumandos y tenemos:
Z 2,5 Z 3
dx dx
I= p + p .
2 (3 − x)(x − 2) 2,5 (3 − x)(x − 2)

Aplicaremos el criterio de comparación para estudiar la convergencia de cada


integral. En el caso de que 2 ≤ x ≤ 2,5, deducimos que
p (x − 2)1/2
(3 − x)(x − 2) ≥ (x − 2)/2 =⇒ (3 − x)(x − 2) ≥ √
2

1 2
=⇒ p ≤ .
(3 − x)(x − 2) (x − 2)1/2
Z 2,5

2
Como además 1/2
dx es convergente, también lo será el primer
2 (x − 2)
sumando de la integral dada.
Procediendo análogamente con el segundo sumando obtenemos que, si 2,5 <
x < 3, √
1 2
p ≤
(3 − x)(x − 2) (3 − x)1/2
Z 3 √
2
y sabemos también que 1/2
dx es convergente.
2,5 (3 − x)

En definitiva obtenemos que la integral propuesta es convergente.

PROBLEMA 12.46
Z 1
dx
Determinar la naturaleza de la integral I = p .
0 x(1 − x2 )

Solución
Como la integral es impropia en los dos extremos de integración, la dividimos
en dos sumandos. Ası́ escribimos
Z 1/2 Z 1 Z 1/2 dx Z 1 dx
dx dx (1−x2 )1/2 1/2
I= p + p = + √x .
0 x(1 − x2 ) 1/2 x(1 − x2 ) 0 x1/2 1/2 1 − x2

Los numeradores están acotados en los intervalos correspondientes. Por tan-


Z 1/2
dx
to la primera integral tiene el mismo carácter que que sabemos
0 x1/2

134
es convergente. Con respecto a la segunda integral podemos factorizar el
Z 1 dx Z 1 dx
x 1/2 x1/2
denominador y escribir √ = 1/2 (1 + x)1/2
. Esta inte-
1/2 1 − x2 1/2 (1 − x)
Z 1
dx
gral es equivalente en cuanto a su carácter a la integral 1/2
que
1/2 (1 − x)
es también convergente.

En definitiva, la integral dada es convergente.

PROBLEMA 12.47
Z π/2
cos x
Hallar √ dx.
0 1 − sen x

Solución

El integrando presenta una discontinuidad en x = π/2, de modo que


Z π/2−ε
cos x π/2−ε
dx = lı́m − 2(1 − sen x)1/2 0

I = lı́m √
ε→0+ 0 1 − sen x ε→0+

= −2 lı́m {[1 − sen(π/2 − ε)]1/2 − 1} = 2.


ε→0+

PROBLEMA 12.48
Z 1
ln x
Calcular la integral √ dx.
0 x

Solución

Esta integral es impropia porque el integrando no está acotado en x = 0. Si


realizamos la integral indefinida por partes, tenemos:

1 Z 1 √

Z  
ln x 1 2 x
I = lı́m √ dx = lı́m 2 x ln x a − dx
a→0+ a x a→0+ a x
 √ √ 1 √ √
= lı́m 2 x ln x − 4 x a = −4 − lı́m (2 a ln a − 4 a)
a→0+ a→0+
2 ln a 2/a
= −4 − lı́m = −4 − lı́m = −4.
a→0+ a−1/2 a→0+ (−1/2)a−3/2

135
PROBLEMA 12.49
Z 1
arc sen x
Calcular √ dx.
0 1 − x2

Solución
Por definición de integral impropia, tenemos:
Z 1 Z B
arc sen x arc sen x
√ dx = lı́m √ dx
0 1 − x2 B→1−
0 1 − x2
B
(arc sen x)2 (π/2)2 π2

= lı́m = = .
B→1− 2 0 2 8

PROBLEMA 12.50
π/2
1 − cos x
Z
Determinar los valores de m para que dx sea con-
0 xm
vergente.

Solución
x2 1 − cos x 1
Debido a la equivalencia 1 − cos x ∼ si x → 0, entonces m
∼ m−2
Z π/2 2 Z π/2 x 2x
1 − cos x 1
y las dos integrales m
dx, m−2
dx tienen el mismo carácter
0 x 0 2x
(convergen o divergen a la vez). De aquı́ se deduce que la integral es con-
vergente cuando m − 2 < 1, o bien m < 3, y divergente cuando m ≥ 3.

PROBLEMA 12.51
Z 1
ln x
Estudiar la convergencia de la integral dx.
0 1−x

Solución
Como la función no está acotada en x = 0 ni en x = 1, descomponemos la
integral en dos sumandos ası́:
Z 1 Z α Z 1
ln x ln x ln x
dx = dx + dx, con 0 < α < 1.
0 1−x 0 1−x α 1−x

136
Aplicamos el criterio de comparación para estudiar la convergencia de cada
una de las integrales. Debido a que
ln x
1−x
lı́m =0
x→0 1/x1/2

Z α
dx
y que es convergente, el primer sumando es convergente.
0 x1/2
Análogamente, como
ln x
1−x
lı́m √ =0
x→1 1/ 1 − x
Z 1
dx
y √ es convergente, el segundo sumando es también convergente.
α 1−x
De lo anterior se deduce que la integral propuesta es convergente.

PROBLEMA 12.52
1

1 − x dx
Z
Determinar la naturaleza de la integral según los
0 xa ln x
valores de a > 0.

Solución
Como la función integrando no está definida en x = 0 ni en x = 1, descom-
ponemos la integral en dos sumandos
1/2
√ 1

1 − x dx 1 − x dx
Z Z
I= + = I1 + I2 .
0 xa ln x 1/2 xa ln x

En la segunda integral hacemos el cambio de variable z = 1 − x, con lo


que
Z 1/2 √
z dz
I2 = a ln(1 − z)
.
0 (1 − z)
Debido a la equivalencia de infinitésimos ln(1 − z) ∼ −z cuando z → 0,
Z 1/2 √ Z 1/2
− z dz −dz
podemos comparar la integral con = √ y esta última
0 z 0 z
es convergente.
Estudiamos ahora el primer sumando, que es una integral impropia en x = 0
porque √
1−x x−a −ax−a−1
lı́m a = lı́m = lı́m = ∞.
x→0 x ln x x→0 ln x x→0 1/x

137
Z 1/2
dx
Compararemos la integral con que es convergente si b < 1 y diver-
0 xb
gente si b ≥ 1. Calculando el lı́mite del cociente, obtenemos:
√ (
1 − x/xa ln x xb−a 0 si b ≥ a
lı́m b
= lı́m =
x→0+ 1/x x→0+ ln x ∞ si b < a.

De este modo, si a < 1, elegimos b = a, en cuyo caso el lı́mite del cociente


es cero y la integral I1 es convergente. Por otra parte, si a > 1, elegimos
b = 1 lo que hace que el lı́mite del cociente sea infinito y la integral sea
divergente.
Z 1/2 √
1−x
Estudiaremos por último el caso a = 1. Como dx tiene el
0 x ln x
Z 1/2
1 √
mismo carácter que dx pues 1 − x está acotada en (0, 1/2), y
0 x ln x
además Z 1/2
1  1/2
dx = lı́m ln | ln x| a = ∞,
0 x ln x a→0 +

la integral es también divergente.


En definitiva, obtenemos que la integral propuesta es convergente cuando
a < 1 y divergente cuando a ≥ 1.

PROBLEMA 12.53
Z 1
Estudiar el carácter de la integral I = x3 e1/x dx.
0

Solución

et
Z
Si hacemos el cambio de variable x = 1/t, resulta la integral I = dt.
1 t5
en
Ahora bien, como la sucesión de término general an = 5 es divergente,
X n
(lı́m an = ∞), la serie an es divergente. Por el criterio de la serie asociada,
la integral impropia I es también divergente.

PROBLEMA 12.54
Z π
dx
Estudiar la convergencia de la integral .
0 1 − cos x

138
Solución
El denominador se anula cuando x = 0; por tanto el integrando no está aco-
x2
tado en x = 0. Debido a la equivalencia 1 − cos x ∼ , resulta que la inte-
Z π 2
dx
gral propuesta tiene el mismo carácter que 2
. Como ésta es divergente,
0 x
también lo es la integral propuesta.

PROBLEMA 12.55
Z ∞
1
Estudiar la convergencia de la integral √ dx.
0 x + x4

Solución
Descomponemos la integral en dos sumandos como
Z 1 Z ∞
1 1
I= √ dx + √ dx.
x+x 4 x + x4
0 1

Ası́ tenemos dos integrales impropias: la primera es de segunda especie pues


la función no está acotada en x = 0 y la segunda de primera especie, pues
el intervalo de integración es infinito. Aplicamos el criterio de comparación
en ambos casos. Por una parte,

1/ x + x4 1
lı́m √ = lı́m √ =1
x→0 1/ x x→0 1 + x3
Z 1
1
e √ dx es convergente.
0 x
Por otra parte,

1/ x + x4 x2
lı́m = lı́m √ =1
x→∞ 1/x2 x→∞ x + x4
Z ∞
1
e dx es convergente. Como ambas integrales son convergentes, tam-
1 x2
bién lo será la suma de ambas.

PROBLEMA 12.56


e− x
Z
Estudiar la convergencia de la integral √ dx.
0 x

139
Solución

e− x
Como lı́m √ = ∞, la integral es impropia en ambos extremos de inte-
x→0+ x
gración. Calculando directamente la integral, obtenemos:
√ √

e− x √ e− x √
Z Z h iB
− x
√ dx = −2e =⇒ √ dx = lı́m −2e− x
x 0 x A→0+ A
B→∞
h √ √ i
= lı́m −2e− B
+ 2e− A
= 2.
A→0+
B→∞

PROBLEMA 12.57
Determinar Z ∞los valores de a para los cuales es convergente la in-
xa−1
tegral I = dx.
0 1+x

Solución
Por una parte el intervalo de integración es infinito y por otra, en el caso
de que a − 1 < 0, el integrando no está acotado en x = 0. Debemos pues
descomponer la integral en dos sumandos
1 ∞
xa−1 xa−1
Z Z
I= dx + dx.
0 1+x 1 1+x
Z 1
dx
La primera integral tiene el mismo carácter que , la cual es conver-
0 x1−a
gente cuando 1 − a < 1, es decir a > 0.
Con respecto al segundo sumando, debido
Z ∞ a−1 a la equivalencia
Z ∞ 1+x ∼ Z
x, cuando

x 1
x → ∞, la integral es equivalente a dx = xa−2 dx = 2−a
dx,
1 x 1 1 x
la cual es convergente si 2 − a > 1, o bien a < 1.
En definitiva, las dos condiciones indican que la integral propuesta es con-
vergente cuando 0 < a < 1 y divergente en caso contrario.

PROBLEMA 12.58

e−x − 1
Z
Estudiar la convergencia de la integral dx según los
0 xα
distintos valores de α.

140
Solución
Debido a que la función integrando no está acotada en x = 0 cuando α > 1,
descomponemos la integral en dos sumandos
Z ∞ −x Z 1 −x Z ∞ −x
e −1 e −1 e −1
α
dx = α
dx + dx,
0 x 0 x 1 xα
y estudiamos la convergencia de cada uno −xde ellos.
En el primer sumando,
e − 1
−x
como e − 1 ∼ −x si x → 0, entonces ∼ 1 , de modo que la
xα xα−1
integral es convergente si α − 1 < 1 y divergente si α − 1 ≥ 1.
−x
e − 1
Para el segundo sumando, como lı́m = lı́m 1 , la convergencia
x→∞ xα x→∞ xα
Z ∞
dx
equivale a la de la integral α
. Por tanto, converge si α > 1 y diverge
1 x
si α ≤ 1.
Como la integral propuesta es convergente cuando lo sean ambos sumandos,
tenemos que es convergente cuando α ∈ (1, 2) y divergente en el resto.

PROBLEMA 12.59

tα−1
Z
Probar que la integral impropia dt converge si α > 1 y
0 et − 1
diverge si α ≤ 1.

Solución
Descomponemos la integral en dos sumandos como
Z ∞ α−1 Z 1 α−1 Z ∞ α−1
t t t
t
dt = t
dt + dt,
0 e −1 0 e −1 1 et − 1
y estudiamos la convergencia de cada uno de ellos.
El primer sumando corresponde a una integral impropia de segunda especie.
tα−1 1
Debido a la equivalencia et − 1 ∼ t cuando t → 0, resulta que t ∼ 2−α .
e −1 t
Esto indica que la integral converge cuando 2 − α < 1, es decir α > 1, y
diverge cuando α ≤ 1.
El segundo sumando es siempre
Z ∞ convergente como se deduce al compararlo
dt
con la integral convergente . En efecto:
1 t2

tα−1 tα+1
lı́m t2 · = lı́m = 0.
t→∞ et − 1 t→∞ et − 1

141
La integral propuesta es por tanto convergente cuando α > 1.

PROBLEMA 12.60
Se define la función Γ(x) como:
Z ∞
Γ(x) = tx−1 e−t dt.
0

a) Probar que converge para x > 0 y diverge para x ≤ 0.


b) Probar que Γ(x + 1) = xΓ(x) para x > 0.
c) De lo anterior, deducir que Γ(n) = (n−1)! para cualquier n natural.

Solución
a) Vamos a separar el estudio en tres casos:
- x ≥ 1: La integral es impropia de primera especie pues Z ∞ la función
dt
está acotada. Aplicamos el criterio de comparación con , que es
1 t2
convergente:
t2+x−1
lı́m t2 · tx−1 e−t = lı́m = 0,
t→∞ t→∞ et
como se deduce al aplicar la regla de L’Hôpital sucesivas veces (el
denominador es un infinito de orden superior al del numerador). Esto
indica que la integral impropia es convergente.
- 0 < x < 1: En este caso la integral también es impropia de segunda
especie pues en x = 0 la función no está acotada. Descomponemos la
integral como
Z ∞ Z 1 Z ∞
tx−1 e−t dt = tx−1 e−t dt + tx−1 e−t dt.
0 0 1

El segundo sumando es convergente (se procede como en el caso ante-


rior); para estudiar la convergencia del primer sumando aplicamos de
Z 1
dt
nuevo el criterio de comparación con α
donde elegimos cualquier
0 t
α que cumpla 1 > α > 1 − x. Debido a que

lı́m tα · tx−1 e−t = lı́m tα+x−1 = 0,


t→0+ t→0+
Z 1
dt
y a que es convergente, también la integral propuesta es con-
0 tα
vergente.

142
- x ≤ 0: De nuevo tenemos una integral impropia
Z 1 de segunda especie.
dt
Aplicamos el criterio de comparación con α
, haciendo α = 1−x ≥
0 t
1. Resulta:
lı́m tα · tx−1 e−t = lı́m e−t = 1
t→0+ t→0+
Z 1
dt
y, como es divergente, también lo es la integral propuesta.
0 tα
b) Aplicando el método de integración por partes,
Z b b
Z b
tx e−t dt = lı́m − tx e−t tx−1 e−t dt

Γ(x + 1) = lı́m 0
+ lı́m x
b→∞ 0 b→∞ b→∞ 0
bx
= lı́m + xΓ(x) = xΓ(x).
b→∞ eb

c) Aplicando el apartado b) sucesivas veces, tenemos:

Γ(n) = (n − 1)Γ(n − 1) = · · · = (n − 1)(n − 2) . . . 2 · 1 · Γ(1).


Z ∞
Como además Γ(1) = e−t dt = 1, deducimos que Γ(n) = (n − 1)!
0

C. APLICACIONES AL CÁLCULO DE ÁREAS Y VOLÚME-


NES.

El concepto de integral impropia permite también aplicarlo al cálculo de


áreas y volúmenes de regiones no acotadas. Como veremos en los problemas
siguientes, es posible que regiones no acotadas tengan áreas o volúmenes
finitos, lo cual será debido a la convergencia de las integrales que las definen.

PROBLEMA 12.61
Rx
(arc tg t)2 dt
Resolver lı́m 0 √ .
x→∞ x2 + 1

143
Solución
La integral del numerador es divergente porque lı́m (arc tg t)2 = π 2 /4 6= 0.
t→∞
Como el lı́mite del denominador también es infinito, tenemos una indeter-
minación ∞/∞. Aplicando la regla de L’Hôpital,
Rx
(arc tg t)2 dt (arc tg x)2 π 2 /4
L = lı́m 0 √ = lı́m √ = = π 2 /4.
x→∞ x2 + 1 x→∞ x/ x2 + 1 1

PROBLEMA 12.62
R 2
x t2
0 e dt
Resolver lı́m R x 2t2 .
0 e dt
x→∞

Solución
Z ∞ Z ∞
t2 2
Como las integrales e dt y e2t dt son divergentes (los integran-
0 0
dos son funciones que no están acotadas en (0, ∞)), tenemos una indeter-
minación del tipo ∞/∞. Aplicando por dos veces la regla de L’Hôpital,
resulta:
R 2
x t2 2 Rx 2
0 e dt 2ex 0 et dt
L = lı́m R x 2t2 = lı́m
x→∞
0 e dt x→∞ e2x2
R x t2 2
2 0 e dt 2ex
= lı́m = lı́m = lı́m 1/x = 0.
x→∞ ex2 x→∞ 2xex2 x→∞

PROBLEMA 12.63
1+x2
et
Z
Sea F la función definida en todo R por F (x) = dt.
1 t
a) Estudiar la continuidad y derivabilidad de F .
b) Probar que lı́m F (x) = lı́m F (x) = ∞.
x→∞ x→−∞

Solución
a) Como la función integrando f (x) = ex /x es continua en R \ {0}, será in-
tegrable en cualquier intervalo que no contenga al cero. Esto implica
que F es continua en R pues, al ser 1 + x2 > 0, cualquier punto del

144
intervalo [1, 1 + x2 ] es positivo. Además es también derivable en todo
R, siendo
2
e1+x
f 0 (x) = · 2x, ∀x ∈ R.
1 + x2
Z ∞ t
e
b) Como lı́m F (x) = lı́m F (x) = dt, debemos estudiar la con-
x→∞ x→−∞ 1 t
vergencia de esta integral impropia.
et
Debido a que lı́m = ∞, la función integrando no está acotada, de
t→∞ t
modo que la integral es divergente. Tenemos en definitiva que

lı́m F (x) = lı́m F (x) = +∞.


x→∞ x→−∞

PROBLEMA 12.64
Demostrar la acotación
2 ∞ 2
e−x e−x
Z
≤ −t2
1 e dt ≤ .
2x 1 + 2x2 x 2x

Solución
1 2
Integramos en primer lugar por partes, haciendo u = y dv = 2te−t dt.
2t
Ası́:

∞ ∞
−1 −t2 b
Z Z   Z ∞
2 1 2 1 2
e−t dt = · 2te−t dt = lı́m ·e − 2
· e−t dt
x x 2t b→∞ 2t x x 2t
Z ∞  −x 2
1 2 e
=⇒ 1 + 2 · e−t dt = .
x 2t 2x
1 1
Como x ≤ t, 1 + ≤ 1 + 2 . Por tanto,
2t2 2x
2 Z ∞ Z ∞
e−x
 
1 −t2 1 2
= 1+ 2 ·e dt ≤ 1 + 2 · e−t dt
2x x 2t x 2x
Z ∞ −x 2 Z ∞ 2
−t2 e −t2 e−x
=⇒ e dt ≥  y también e dt ≤ .
x 2x 1 + 2x12 x 2x

Observación. Esta acotación permite estimar el error que se comete al


2
despreciar el área situada bajo la curva y = e−x para valores grandes de x.

145
PROBLEMA 12.65
Hallar el área comprendida entre la estrofoide y 2 (a + x) = x2 (a − x)
y su ası́ntota.

Solución
r
a−x
En forma explı́cita, la ecuación es y = ±x y su ası́ntota es la recta
a+x
x = −a.

De acuerdo con la figura y teniendo en cuenta la simetrı́a, el área es:

0 0
a2 (4 + π)
r r
a−x a−x
Z Z
A=2 x dx = 2 lı́m x dx = .
−a a+x r→−a+ r a+x 2

PROBLEMA 12.66
Hallar el área situada a la derecha de x = 3 y limitada por la curva
1
y= 2 y el eje X .
x −1

146
Solución

Z ∞
dx
De acuerdo con la gráfica, el área viene dada por la fórmula A =
3 −1 x2
que es una integral impropia. Resolviendo la integral indefinida por el méto-
do de fracciones simples, obtenemos:
b
x−1 b
Z  
dx 1
A = lı́m = lı́m ln
b→∞ 3 x2 − 1 2 b→∞ x+1 3
1 b−1 1 1 1 1 − 1/b 1 1
= lı́m ln − ln = lı́m ln + ln 2 = ln 2.
2 b→∞ b + 1 2 2 2 b→∞ 1 + 1/b 2 2

PROBLEMA 12.67
1 x
Calcular el área limitada por las curvas y = , y= en el
x 1 + x2
intervalo x ∈ [1, ∞).

Solución

147
De acuerdo con la gráfica y por definición de integral impropia, tenemos:
Z ∞  Z b

1 x 1 x
A = − −
dx = lı́m dx
1 x 1 + x2 b→∞ 1 x 1 + x2
b

  
1 2 b 1
= lı́m ln x − ln(1 + x ) = lı́m ln √ − ln √ = ln 2.
b→∞ 2 1 b→∞ 1 + b2 2

PROBLEMA 12.68
Hallar el área limitada por la curva x2 y 2 + x2 − y 2 = 0 y sus ası́nto-
tas y el volumen engendrado por dicha área al girar alrededor del
eje X .

Solución
r
x2
a) Si despejamos la variable y, la curva se expresa como y = ± lo
1 − x2
que indica que las ası́ntotas son x = 1 y x = −1.

Teniendo en cuenta que la curva es simétrica respecto a los dos ejes de


coordenadas (lo que se deduce al sustituir x por −x e y por −y), el área
Z 1
x dx
vendrá dada por la fórmula A = 4 √ . Como el integrando
0 1 − x2
presenta una discontinuidad en x = 1, debemos calcular
Z 1−ε
x dx  p 1−ε p
A = 4 lı́m √ = 4 lı́m − 1 − x2 0 = 4 lı́m (1− 2ε − ε2 ) = 4.
ε→0+ 0 1 − x2 ε→0+ ε→0+

b) Aprovechando de nuevo las simetrı́as y aplicando el método de los discos,

148
tenemos:
1 1 1
x2
Z Z Z
2 2
V = π y dx = 2π y dx = 2π 2
dx
−1 0 0 1−x
"  #1−ε
1−ε
x2 1 + x 1/2
Z 
= 2π lı́m dx = 2π lı́m −x + ln = ∞.
ε→0+ 0 1 − x2 ε→0+ 1−x
0

PROBLEMA 12.69
Hallar el área de la región comprendida entre la curva de Agnesi
a3
y= 2 y el eje de abscisas y el volumen engendrado por la
x + a2
misma región al girar alrededor del eje X .

Solución
a3
El eje de abscisas es la ası́ntota de la curva, pues lı́m = 0.
x→∞ x2 + a2

a) Teniendo en cuenta la simetrı́a de la figura, el área viene dada por


∞ Z ∞ Z b
a3
Z
a 2 1/a
A = 2 2
dx = 2 2
dx = lı́m 2a 2
dx
−∞ x + a 0 (x/a) + 1 b→∞ 0 (x/a) + 1
b
lı́m 2a2 arc tg(x/a) 0 = lı́m 2a2 arc tg(b/a) = πa2 .

=
b→∞ b→∞

b) Aplicando el método de los discos, el volumen se obtiene por la fórmula


∞ ∞
a6
Z Z
V =π y 2 (x) dx = 2π dx.
−∞ 0 (x2 + a2 )2

Para realizar la integración aplicamos el cambio de variable x = a tg t,


con lo que x = 0 =⇒ t = 0 y x = ∞ =⇒ t = π/2 y obtenemos:
∞ π/2
a6 a2
Z Z
V = 2π dx = 2π · a sec2 t dt
0 (x2 + a2 )2 0 sec4 t
Z π/2
= 2π a3 cos2 t dt = 2πa3 · π/4 = π 2 a3 /2.
0

149
PROBLEMA 12.70
Se considera la curva y = x−1/4 definida en (0, 1].
a) Hallar el área bajo la curva.
b) Hallar el volumen del sólido obtenido al girar la curva alrededor
del eje X .

Solución

a) Como la función no está acotada en x = 0, el área viene dada por una


integral impropia:
" #1 !
1 1
x3/4 4 4a3/4
Z Z
4
A= x−1/4 dx = lı́m x−1/4 dx = lı́m = lı́m − = .
0 a→0+ a a→0+ 3/4 a→0+ 3 3 3
a

b) Análogamente al apartado anterior,


Z 1 Z 1
−2/4
V = π x dx = π lı́m x−1/2 dx
0 a→0+ a
" #1
x1/2
= π lı́m = 2π lı́m (1 − a1/2 ) = 2π.
a→0+ 1/2 a→0+
a

PROBLEMA 12.71
Se considera la región R limitada por las curvas y(x2 +1)+arc tg x =
0 y x2 y 3 = 1 en el intervalo x ∈ [0, 1].
i) Calcular el área de la región R.
ii) ¿Existe el volumen del sólido obtenido al girar R alrededor del eje
X?

150
Solución

i)

De acuerdo con la figura, el área viene dada por:


" #1
1
x1/3 (arc tg x)2
Z 
−2/3 arc tg x
A = x + 2
dx = lı́m +
0 x +1a→0+ 1/3 2
a
(π/4)2 (arc tg a)2 π2
 
= lı́m 3 + − 3a1/3 − =3+ .
a→0+ 2 2 32

ii) El volumen pedido es el mismo que el de la región comprendida entre la


curva y = x−2/3 y el eje X en el intervalo [0, 1] (basta observar que al
girar esta región ya queda incluida la parte comprendida en el cuarto
cuadrante). Aplicando el método de los discos,
" #1
1 1
x−1/3
Z Z
2 −4/3
V = π y dx = π x dx = lı́m π ·
0 0 a→0+ −1/3
a
−1/3 −1/3
= −3π lı́m (1 −a ) = ∞.
a→0+

PROBLEMA 12.72
Determinar el volumen del sólido obtenido al girar la región limi-
tada por la curva e−y = −x y los ejes de coordenadas alrededor del
eje OX .

151
Solución

De acuerdo con la figura, si aplicamos el método de los tubos, la fórmula del


volumen da: Z ∞ Z ∞
V = 2π (−x)ydy = 2π ye−y dy.
0 0
Como es una integral impropia debemos estudiar su convergencia. Integra-
mos en primer lugar por partes y obtenemos:
Z
ye−y dy = −(y + 1)e−y ,

con lo que
B
V = lı́m 2π −(y + 1)e−y 0 = lı́m −2π(B + 1)e−B + 2π = 2π.

B→∞ B→∞

152
D. EJERCICIOS PROPUESTOS.

Z 0
1. Hallar e2x dx.
−∞

Resp.: I = 1/2.

Z ∞
dx
2. Calcular .
0 1 + x2
Resp.: I = π/2.

Z ∞
dx
3. Calcular √ .
1 x3 x2 − 1
Resp.: I = π/4.


xa−1
Z
4. ¿Para qué valores de a es convergente dx?
0 1+x
Resp.: Diverge para todo a.

Z ∞
dx
5. Calcular .
0 4ex + 9e−x
π 1 2
Resp.: I = − arc tg .
12 6 3
Z ∞
x dx
6. Estudiar la convergencia de la integral √ .
1 x2 + 2 5 x4 + 1
Z ∞
Resp.: Divergente (comparar con dx/x).
1

Z ∞
ln x dx
7. Estudiar la convergencia de la integral .
1 (1 + x2 )2
Z ∞
Resp.: Convergente (comparar con dx/xα con 1 < α < 4).
1

Z ∞
x
8. Estudiar la convergencia de la integral dx.
0 (1 + x2 )2
Z ∞
Resp.: Convergente (comparar con dx/x3 ).
1

153

x2 − 8x − 17
Z
9. Estudiar la convergencia de la integral dx.
2 x4 + 4x3 + 6x2 − 4x − 7
Z ∞
Resp.: Convergente (comparar con dx/x2 ).
2


3e3x/2 − 2ex + 2ex/2
Z
10. Estudiar la convergencia de la integral dx.
2 e2x − 2e3x/2 + 3ex − 4ex/2 + 2
Z ∞
Resp.: Convergente (comparar con e−x/2 dx).
2

Z ∞
dx
11. Estudiar la convergencia y calcular la integral .
0 (1 + x2 )2
Resp.: I = π/4.

Z ∞
12. Estudiar la convergencia de la integral f (x) dx siendo
0
(
√1 si x < 1
f (x) = 1−x .
x−1
si x ≥ 1.
1+x3


Z 1
x−1 1
Resp.: Convergente pues dx/ 1 − x es convergente y 3
∼ 2
0 1+x x
cuando x → ∞.
Z ∞
13. Estudiar la convergencia de la integral x sen x dx.
0

Resp.: Divergente (la función y = x sen x no está acotada en (0, ∞)).

Z ∞ Z ∞
cos x sen x
14. Probar que dx = dx y que una de ellas con-
0 1+x 0 (1 + x)2
verge absolutamente.
Sugerencia: La igualdad se obtiene integrando por partes. Ver proble-
ma 12.32 para estudiar la convergencia.

15. Se considera la función f (x) = ce−2x .


Z ∞
a) Determinar el valor de c para que f (x) dx = 1.
0
Z ∞
b) Calcular xf (x) dx con el valor de c obtenido en a).
0

154
Resp.: a) c = 2; b) I = 1/2.
Z ∞
16. Probar que e−px dx es convergente si p > 0 y divergente si
1
p ≤ 0.
Sugerencia: Resolver la integral.
Z ∞
2 + cos x
17. Estudiar la convergencia de la integral √ dx.
1 x
Z ∞ √
Resp.: Divergente (comparar con dx/ x).
1

Z π/2
18. Demostrar que sec x dx no existe.
0
Resp.: La integral es divergente.
Z 1
dx
19. Calcular .
−1 x
Resp.: La integral es divergente.
Z 3
x dx
20. Calcular .
0 (x2 − 1)3/5

5
Resp.: 5(2 2 − 1)/4.

1
ex dx
Z
21. Estudiar la convergencia de la integral impropia √ .
0 1 − cos x
Z 1
Resp.: Divergente (comparar con dx/x).
0

Z 1
dx
22. Estudiar la convergencia de √ .
0 1− x2 + 2 1 − x2
Z 1
Resp.: Convergente (comparar con dx/(1 − x)1/2 ).
0

Z 3
dx
23. Estudiar la convergencia de la integral . √
1 4x − x2 − 3
√ √
Z c Z 3
Resp.: Convergente (comparar con dx/ x − 1 y con dx/ 3 − x).
1 c

155
Z 1
dx
24. Estudiar la convergencia de la integral .
0 sen2 x
Resp.: Divergente (integración directa).

Z 1
1
25. Estudiar la convergencia de la integral ln dx.
0 1−x
Resp.: Convergente (integración directa).

Z 1
dx
26. Estudiar la convergencia de la integral √ .
0 x + 4x3
Z 1 √
Resp.: Convergente (comparar con dx/ x).
0

Z x
−x2 2
27. Demostrar que lı́m e et dt = 0.
x→∞ 0
Sugerencia: Aplicar la regla de L’Hôpital.

28. Calcular el área de la región limitada superiormente por la curva


xy = 1, inferiormente por la curva y(x2 + 1) = x y a la izquierda
de x = 1.
Resp.: A = ∞.

x−1
29. Calcular el área de la región limitada por las curvas y = √ ,
x2 − 1
x = 1, x = 3 por encima del eje OX .
√ √
Resp.: A = 8 − ln(3 + 8).

x
30. Calcular el área de la región limitada por la curva y 2 =
1 − x3
entre los puntos de abscisa x = 0 y x = 1.
Resp.: A = π/3.

31. Calcular el área comprendida entre y = xe−x y el eje X en (0, ∞).


Z ∞
¿Cuánto vale xe−x dx?
−∞

Resp.: A = 1; I = 0 por ser una función impar y la integral conver-


gente.

156
32. Sea f (x) = e−2x para todo x. Llamamos R a la región limitada
por la curva y el eje X en el intervalo [0, t], con t > 0. Calcular
el área A(t) de R y el volumen V (t) obtenido al girar R alrededor
del eje X . Interpretar los valores de lı́m A(t) y lı́m V (t).
t→∞ t→∞
1 1 π π
Resp.: A(t) = − e−2t + ; V (t) = − e−4t + .
2 2 4 4

157
CAPÍTULO XIII.
SUCESIONES
NUMÉRICAS

SECCIONES
A. Sucesiones convergentes y lı́mites de oscilación. Sucesiones monótonas
y acotadas.
B. Sucesiones recurrentes.
C. Ejercicios propuestos.

159
A. SUCESIONES CONVERGENTES Y LÍMITES DE OSCILA-
CIÓN. SUCESIONES MONÓTONAS Y ACOTADAS.

En el capı́tulo 8 se definió el concepto de convergencia de sucesiones numéri-


cas. Vamos a estudiar aquı́ las sucesiones en un contexto más general, mos-
trando otros conceptos fundamentales de las sucesiones y las relaciones entre
ellos. Estos conceptos son los de convergencia, monotonı́a y acotación, que
pasamos a definir a continuación.

A.1.CONVERGENCIA.
a) Una sucesión {an }n∈N es convergente cuando existe un número real
L tal que

∀ε > 0, ∃N ∈ N : |an − L| < ε, ∀n > N.

Intuitivamente, quiere decir que todos los términos de la sucesión,


excepto quizá los N primeros, caen dentro del intervalo (L−ε, L+
ε), es decir su distancia a L es menor que ε.
En este caso se dice que L es el lı́mite de la sucesión y se escribe
L = lı́m an , o bien an → L (observa que, a diferencia de las
n→∞
funciones reales, no tiene sentido calcular lı́mites de sucesiones
cuando n → x0 6= ∞, pues en un entorno de x0 puede no haber
ningún número natural).
b) Una sucesión es divergente cuando se verifica una de las siguientes
propiedades:

∀M > 0, ∃N ∈ N : an > M, ∀n > N (y se dice que an → ∞).


∀M < 0, ∃N ∈ N : an < M, ∀n > N (y se dice que an → −∞).

c) Una sucesión es oscilante cuando no es convergente ni divergente,


es decir, no hay ningún número (finito ni infinito) en cuyas proxi-
midades se encuentren todos los términos de la sucesión, a partir
de uno de ellos en adelante.
Definimos subsucesión de una sucesión {an } a cualquier sucesión con-
tenida en el conjunto {a1 , a2 , . . . , an , . . . }.

Una caracterización de las sucesiones convergentes la proporciona el


(1) criterio de convergencia de Cauchy: Una sucesión {an }, con
an ∈ R, ∀n, es convergente si y sólo si

∀ε > 0, ∃N ∈ N : |an − am | < ε, ∀n, m > N.

160
Esta condición, aunque teórica, permite comprobar la convergencia
de una sucesión sin necesidad de conocer su lı́mite.
Otras propiedades de interés son las siguientes:

(2) El lı́mite de una sucesión, en caso de existir, es único.

(3) Si an → a y a 6= 0, entonces existe N ∈ N tal que an tiene el


mismo signo que a, ∀n > N .

(4) Si an < bn , ∀n, entonces lı́m an ≤ lı́m bn .

(5) Si an ≤ bn ≤ cn , ∀n y L = lı́m an = lı́m cn , entonces L = lı́m bn .

(6) Si {an } es convergente (o divergente), toda subsucesión de ella


tiene el mismo lı́mite.
Esta propiedad también se puede expresar diciendo que no se altera
el valor del lı́mite de una sucesión al reordenar arbitrariamente sus
términos.

Un concepto más general que el de lı́mite de una sucesión es el de lı́mite


de oscilación. Decimos entonces que un número L es lı́mite de oscilación
de una sucesión {an } si cualquier intervalo de la forma (L − ε, L + ε)
contiene infinitos términos de la sucesión. Llamamos lı́mite superior
de {an } al mayor, si existe, de los lı́mites de oscilación (y utilizamos
la notación L = lı́m sup an ) y el lı́mite inferior de {an } será el menor
de los lı́mites de oscilación, en caso de existir (y utilizamos la notación
análoga L = lı́m inf an ).

A.2. MONOTONIA.
a) Una sucesión {an }n∈N es monótona creciente cuando

an+1 ≥ an , ∀n ∈ N.

b) Análogamente, una sucesión {an }n∈N es monótona decreciente


cuando
an+1 ≤ an , ∀n ∈ N.
En general, diremos que una sucesión es monótona cuando es, o
bien monótona creciente, o bien monótona decreciente.

A.3.ACOTACION.
a) Una sucesión {an }n∈N está acotada superiormente cuando

∃M ∈ R : an ≤ M, ∀n ∈ N.

161
b) Una sucesión {an }n∈N está acotada inferiormente cuando

∃M 0 ∈ R : an ≥ M 0 , ∀n ∈ N.

Decimos en general que una sucesión está acotada si lo está su-


perior e inferiormente.

Los resultados que relacionan estos tres conceptos son los siguientes:

1) Toda sucesión convergente está acotada.

2a) Toda sucesión creciente y acotada superiormente es convergente.


Además el lı́mite es el supremo del conjunto {an }n∈N .

2b) Toda sucesión decreciente y acotada inferiormente es convergente.


En este caso el lı́mite es el ı́nfimo del conjunto {an }n∈N .

3) Toda sucesión contiene alguna subsucesión que es creciente o de-


creciente.

4) Toda sucesión acotada tiene alguna subsucesión convergente. En


particular:

5) Toda sucesión acotada tiene lı́mites superior e inferior finitos.

Ası́ pues, se dice que el lı́mite superior de una sucesión es +∞ si la


sucesión no está acotada superiormente y que el lı́mite inferior es −∞
si no está acotada inferiormente.

En los siguientes problemas se muestran ejemplos de algunas de estas pro-


piedades y la forma de aplicarlas a casos concretos.

PROBLEMA 13.1
Determinar el menor valor de N para el que se verifica lo siguien-
te:
a) |an − 2| < 10−5 , ∀n > N, si an = 4 + 1/n.
p

1 (−1)n
b) + < 10−6 , ∀n > N .
n n2

162
Solución
a) Se trata de resolver la desigualdad |an − 2| < 10−5 . Debido a que an >
2, ∀n, tenemos la siguiente cadena de equivalencias:
| 4 + 1/n − 2| < 10−5 ⇐⇒ 4 + 1/n − 2 < 10−5
p p

4 + 1/n < 10−5 + 2


p
⇐⇒
⇐⇒ 4 + (1/n) < (10−5 + 2)2
⇐⇒ 1/n < 10−10 + 4 + 4 · 10−5 − 4
⇐⇒ n > (10−10 + 4 · 10−5 )−1 = 1010 · (1 + 4 · 105 )−1 .
p
El resultado indica que la sucesión dada por an = 4 + 1/n tiene
lı́mite 2.
b) Procediendo análogamente al caso anterior, tenemos:
1 (−1)n
+ < 10−6 ⇐⇒ n + (−1)n < 10−6 · n2 ⇐⇒ 10−6 · n2 − n − (−1)n > 0.
n n2
Si resolvemos esta inecuación de segundo grado, resulta:
- Cuando n es par:

−6 2 1+ 1 + 4 · 10−6
10 · n − n − 1 > 0 =⇒ n >
2 · 10−6
1 h p i
=⇒ n > 106 + 103 4 + 106 ' 106 .
2
- Cuando n es impar:

−6 2 1+ 1 − 4 · 10−6
10 · n − n + 1 > 0 =⇒ n >
2 · 10−6
1 h p i
=⇒ n > 106 + 103 106 − 4 < 106 .
2
En definitiva, para n > 106 se verifica la desigualdad propuesta.

PROBLEMA 13.2
Demostrar, utilizando la definición de lı́mite, que la sucesión de
4n − 3
término general an = converge a 4.
n+1

Solución
Debemos probar que, dado cualquier número ε > 0, somos capaces de en-
contrar un número natural N tal que la distancia entre an y el lı́mite 4, es
menor que ε, ∀n > N . Ahora bien, dicha distancia es

4n − 3 −7
|an − 4| =
− 4 =
= 7 ,
n+1 n + 1 n + 1

163
7
y será |an − 4| < ε si y sólo si n > − 1. Por tanto, basta elegir cualquier
ε
7
número natural N > − 1 para que se cumpla la definición de lı́mite.
ε

PROBLEMA 13.3

n
Sean a y b positivos. Probar que lı́m an + bn = máx{a, b}.
n→∞

Solución
En primer lugar, si a = b,

n
√n

n
lı́m an + bn = lı́m 2an = lı́m a 2 = a = máx{a, b}.
n→∞ n→∞ n→∞

Supongamos ahora que a > b. Entonces


√n
p p
lı́m an + bn = lı́m n an [1 + (b/a)n ] = lı́m a n 1 + (b/a)n = a = máx{a, b},
n→∞ n→∞ n→∞

debido a que (b/a)n → 0.


Análogamente se deduce el caso en que b > a.

PROBLEMA 13.4
an + 1 n
 
Sabiendo que lı́m = 9, calcular el valor de a.
n→∞ an − 1

Solución
Como se trata de una indeterminación de la forma 1∞ , podemos tomar
an + 1 an + 1
logaritmos y aplicar la equivalencia ln ∼ − 1. De este mo-
an − 1 an − 1
do,  
an + 1 2 2
ln 9 = lı́m n − 1 = lı́m n · = ,
an − 1 an − 1 a
2
con lo que debemos tomar a = .
ln 9

PROBLEMA 13.5
Hallar la relación entre los parámetros a y b para que se verifique
n + a 2n+3 n + 3 bn+4
   
lı́m = lı́m .
n→∞ n + 1 n→∞ n + 2

164
Solución

Ambos lı́mites son de la forma 1∞ . Tomando logaritmos y aplicando la


equivalencia ln un ∼ un − 1, cuando un → 1, obtenemos:
  
n+a n+a
ln L1 = lı́m (2n + 3) ln = lı́m (2n + 3) −1
n→∞ n+1 n→∞ n+1
n+a−n−1
= lı́m (2n + 3) · = 2(a − 1),
 n +
1
n→∞
 
n+3 n+3
ln L2 = lı́m (bn + 4) ln = lı́m (bn + 4) −1
n→∞ n+2 n→∞ n+2
n+3−n−2
= lı́m (bn + 4) · = b.
n→∞ n+2

Para que L1 = L2 , debe cumplirse que e2(a−1) = eb , o bien 2(a − 1) = b.

PROBLEMA 13.6
Probar que la sucesión de término general
1 2 3 n
an = + + + ··· + n
2 4 8 2
es de Cauchy.

Solución

La sucesión {an } es precisamente


X n la sucesión de sumas parciales de la serie
de términos positivos . Como dicha serie es convergente (lo cual se
2n
n≥1
puede probar fácilmente aplicando el criterio de la raı́z), también lo es la
sucesión de sumas parciales y converge a la suma de la serie. Como toda
sucesión convergente es de Cauchy, obtenemos el resultado deseado.

PROBLEMA 13.7
Sea {xn } una sucesión monótona creciente. Probar que también lo
x1 + · · · + xn
es la sucesión de término general yn = .
n

165
Solución
Probaremos que yn+1 ≥ yn , para lo cual utilizamos el hecho de que x1 ≤
x2 ≤ · · · ≤ xn ≤ xn+1 :
x1 + · · · + xn + xn+1 x1 + (1/n)xn+1 + · · · + xn + (1/n)xn+1
yn+1 = =
n+1 n+1
x1 (1 + 1/n) + · · · + xn (1 + 1/n) nyn (1 + 1/n)
≥ = = yn .
n+1 n+1

Análogamente se prueba que si {xn } es decreciente, también lo es la sucesión


de sus medias aritméticas.

PROBLEMA 13.8
Demostrar que es convergente la sucesión de término general
1 1
xn = + ··· + .
2n + 1 3n

Solución
Demostraremos que la sucesión es monótona creciente y acotada superior-
mente, lo que da como consecuencia la convergencia de la sucesión.
*) La sucesión está acotada superiormente pues:
1 1 1 1 n 1
xn = + ··· + < + ··· + = = .
2n + 1 3n 2n 2n 2n 2
*) La sucesión es monótona creciente, como se comprueba al realizar la
siguiente resta:
1 1 1 1 1
xn+1 − xn = + + − −
3n + 3 3n + 2 3n + 1 2n + 2 2n + 1
9n2 + 11n + 4
= > 0, ∀n.
6(n + 1)(3n + 2)(3n + 1)(2n + 1)

Como hemos indicado al principio, se aquı́ se deduce que la sucesión es


convergente.

PROBLEMA 13.9
Encontrar los lı́mites de oscilación de las sucesiones siguientes:
a) an = n2 [1 + (−1)n ].
n nπ
b) an = sen2 .
n+1 4

166
Solución
a) Si n es par, tenemos la sub-sucesión an = 2n2 que es divergente (su
lı́mite es +∞).
Por otra parte, si n es impar, resulta an = 0, que converge a cero.
En definitiva, los lı́mites de oscilación son 0 y +∞.

nπ 2 n
b) En este caso, si n es impar, sen =± , con lo que an =
4 2 2(n + 1)
que converge a 1/2.

Si n es múltiplo de 4, sen = 0 y la subsucesión correspondiente
4
converge a cero.

Si n es de la forma n = 4k + 2, k ∈ N, entonces sen = ±1, con lo
4
n
que an = que converge a 1.
n+1
En definitiva, los lı́mites de oscilación son {0, 1/2, 1}.

PROBLEMA 13.10
Encontrar los lı́mites de oscilación de la sucesión cuyo término
nπ 12 + 2n nπ
general es an = sen + n+1 · cos .
2 2 2

Solución
Si n es par, n = 2k, tenemos la sub-sucesión
12 + 22k k 12 + 2
2k
a2k = sen kπ + · cos kπ = (−1)
22k+1 2 · 22k
12 + 22k
(debido a que sen kπ = 0 y cos kπ = (−1)k ). Como lı́m = 1/2, la
n→∞ 2 · 22k
subsucesión {a2k } es oscilante. Por lo tanto, si k nuevamente es par, la sub-
sucesión correspondiente tiene lı́mite 1/2, y si k es impar, la sub-sucesión
converge a −1/2.
Por otra parte, si n es impar, n = 2k + 1, obtenemos la sub-sucesión
(2k + 1)π 12 + 22k+1 (2k + 1)π
a2k+1 = sen + · cos = (−1)k ,
2 22k+2 2
que también es oscilante pero tiene lı́mites de oscilación 1 y −1.
En definitiva, los lı́mites de oscilación de la sucesión propuesta son −1, −1/2,
1/2 y 1, que son los lı́mites de las subsucesiones {a4k+3 }, {a4k+2 }, {a4k } y
{a4k+1 }, respectivamente.

167
PROBLEMA 13.11
Encontrar los lı́mites de oscilación de la sucesión {m/n : m, n ∈
N, m.c.d.(m, n) = 1} ordenada de modo que m + n vaya de menor
a mayor.

Solución
Como la sucesión consta de todos los números racionales positivos, tiene por
lı́mites de oscilación a todos los reales no negativos pues, como es sabido, en
cualquier entorno de un número real hay siempre una infinidad de números
racionales.

PROBLEMA 13.12
Sea f una función real monótona creciente y acotada en el inter-
valo [0, 1]. Definimos las sucesiones {sn } y {tn } como:
n−1 n
1X 1X
sn = f (k/n), tn = f (k/n).
n n
k=0 k=1

i) Demostrar que
1 1
f (1) − f (0)
Z Z
sn ≤ f (x) dx ≤ tn y 0 ≤ f (x) dx − sn ≤ .
0 0 n

ii) Demostrar que las sucesiones {sn } y {tn } convergen ambas a


Z 1
f (x)dx.
0

Solución
i) Como f es creciente, si P = {0, 1/n, 2/n, . . . , (n − 1)/n, 1} es una parti-
ción cualquiera del intervalo [0, 1], entonces
     
k−1 k k−1 k
f ≤ f (x) ≤ f , ∀x ∈ , , k = 1, . . . , n.
n n n n
De aquı́ se deduce que
Z k/n   Z k/n Z k/n  
k−1 k
f dx ≤ f (x) dx ≤ f dx,
(k−1)/n n (k−1)/n (k−1)/n n
de modo que, sumando a lo largo de Zlos subintervalos de la partición
1
P , se obtiene precisamente que sn ≤ f ≤ tn .
0

168
Si restamos sn a los tres miembros de la desigualdad anterior, obtene-
mos que
" n n−1
#
Z 1
1 X X 1
0≤ f −sn ≤ tn −sn = f (k/n) − f (k/n) = [f (1)−f (0)].
0 n n
k=1 k=0

Si, en vez de restar sn , restamos tn en la misma desigualdad, se obtiene


la desigualdad análoga
Z 1
1
− [f (1) − f (0)] ≤ f − tn ≤ 0.
n 0

ii) De las dos últimas desigualdades, al calcular el lı́mite de los extremos


se obtiene que
Z 1  Z 1
1
0 ≤ lı́m f − sn ≤ lı́m [f (1) − f (0)] = 0 =⇒ lı́m sn = f;
0 n 0
Z 1  Z 1
1
0 = lı́m − [f (1) − f (0)] ≤ lı́m f − tn ≤ 0 =⇒ lı́m tn = f.
n 0 0

PROBLEMA 13.13
Sea X el conjunto de los números reales que son solución de las
ecuaciones n2 x2 − n(3n − 1)x + (2n2 − 3n − 2) = 0 para cada n ∈ N.
Hallar sup X e ı́nf X .

Solución
Al resolver la ecuación obtenemos las raı́ces
p
n(3n − 1) ± n2 (3n − 1)2 − 4n2 (2n2 − 3n − 2)
x =
2n2
2n + 1 n−2
=⇒ an = , bn = .
n n

   
2n + 1 n−2
Resulta entonces el conjunto X = :n∈N ∪ : n ∈ N . Co-
n n
2n + 1
mo la sucesión an = es decreciente y tiene lı́mite 2 y la sucesión
n
n−2
bn = es creciente y tiene lı́mite 1, se deduce que sup X = a1 = 3 e
n
ı́nf X = b1 = −1.

169
B. SUCESIONES RECURRENTES.

Sabemos que para definir una sucesión es necesario dar una regla de forma-
ción de todos sus términos. Esa regla puede ser una fórmula explı́cita que
dé la imagen de cada número natural o una fórmula recurrente, mediante la
cual cada término viene dado en función de uno o varios términos preceden-
tes. Este caso merece especial atención pues no se pueden aplicar las reglas
usuales para el cálculo de lı́mites sino que la convergencia se deduce de for-
ma indirecta. En los siguientes problemas utilizaremos fundamentalmente el
siguiente esquema:
1. Determinar si la sucesión es monótona (bien creciente o decreciente),
mediante la comparación de dos términos consecutivos.
2a. En caso afirmativo, comprobar si la sucesión está acotada. Esto ya da
como consecuencia la convergencia de la sucesión.
2b. En caso negativo, aplicar el criterio de convergencia de Cauchy.
3. Una vez comprobada la existencia de lı́mite, aplicar la propiedad de unici-
dad del mismo. Esto quiere decir que, en particular, L = lı́m an = lı́m an+1 .
Esto da lugar a una ecuación cuya solución es el lı́mite de la sucesión.

PROBLEMA 13.14
Calcular el lı́mite de las sucesiones de término general

q p
a) an = 2 2 2 . . . (n veces).

q p
b) an = 2 + 2 + 2 + . . . (n veces).

Solución
a) Si escribimos el término general como
n n
an = 21/2 · 21/4 · · · · · 21/2 = 21/2+1/4+···+1/2 ,

y sabiendo que el exponente es la suma de los términos de una progre-


sión geométrica de razón 1/2,

1/2 − 1/2n+1
1/2 + 1/4 + · · · + 1/2n = = 1 − 1/2n ,
1 − 1/2
al calcular el lı́mite obtenemos:
n
lı́m an = lı́m 21−1/2 = 2.

170

Otro método: El término general se puede escribir como an = 2an−1
y se puede probar que es monótona creciente y está acotada superior-
mente.
Por inducción se demuestra que an < 2, ∀n:

Como a1 = 2, es evidente que a1 < 2.
Si suponemos ahora que an−1 < 2, probaremos que an < 2:
p √
an = 2an−1 < 2 · 2 = 2.

s
an 2
Por otra parte, como = > 1, se deduce que la sucesión
an−1 an−1
es monótona creciente.
Lo anterior prueba que la sucesión es convergente y, por tanto, de la
relación L = lı́m an = lı́m an−1 resulta que:

L = 2L =⇒ L2 = 2L =⇒ L = 0 ó L = 2.

Como an > 0, ∀n y es creciente, sólo puede ser L = 2.


b) Como no podemos escribir una fórmula explı́cita (simple) para expresar
el término general, aplicamos el segundo método del apartado ante-

rior. Escribimos para ello en forma recurrente an = 2 + an−1 , ∀n y
probamos que la sucesión es monótona y acotada.
- La sucesión está acotada superiormente por 2:

Es evidente que a1 = 2 < 2 y, si suponemos que an−1 < 2, resulta
que p √
an = 2 + an−1 < 2 + 2 = 2.
- La sucesión es monótona creciente. Para ello escribiremos desigual-
dades equivalentes a la que queremos probar:

an+1 ≥ an ⇐⇒ 2 + an ≥ an ⇐⇒ 2 + an ≥ a2n ⇐⇒ a2n − an − 2 ≤ 0
⇐⇒ (an − 2)(an + 1) ≤ 0 ⇐⇒ −1 ≤ an ≤ 2,

lo que es evidentemente cierto.


De lo anterior se deduce nuevamente que la sucesión es convergente. Si
llamamos L a su lı́mite, debido a que L = lı́m an = lı́m an+1 , tenemos:

L = 2 + L =⇒ L2 = 2+L =⇒ L2 −L−2 = 0 =⇒ (L−2)(L+1) = 0 =⇒ L = 2,

ya que los términos de la sucesión son todos positivos y el lı́mite no


puede ser negativo.

171
PROBLEMA 13.15
Se considera la sucesión (an )n≥1 definida por a1 = a2 = a > 0,
1
an = 1 1 , si n ≥ 3. Probar que es monótona y acotada y
an−1 + an−2
calcular su lı́mite.

Solución
Es evidente por la misma construcción del término general que an ≥ 0, ∀n,
lo que indica por una parte que la sucesión está acotada inferiormente por 0.
an−1 · an−2
Veamos que es monótona decreciente. Para ello escribimos an =
an−1 + an−2
y obtenemos que
an an−2
= < 1 =⇒ an < an−1 ,
an−1 an−1 + an−2
como querı́amos probar.
Para calcular el lı́mite tenemos en cuenta que L = lı́m an = lı́m an−1 =
1
lı́m an−2 y planteamos la ecuación L = . Esto implica que L =
1/L + 1/L
L/2 de donde L = 0.

PROBLEMA 13.16
Sea (xn )n∈N la sucesión definida de la siguiente forma:
(
x1 = 1,
1
xn+1 = 1+x 2 · xn .
n

Demostrar que es convergente y que lı́m xn = 0.


n→∞

Solución
Para probar que es convergente, basta ver que está acotada inferiormente y
que es decreciente.
Por una parte, es evidente que xn ≥ 0, ∀n.
xn+1 1
Por otra parte, como = < 1, se deduce que xn+1 < xn , ∀n.
xn 1 + x2n
Sabiendo ya que es convergente, para calcular el lı́mite tendremos en cuenta
que lı́m xn = lı́m xn+1 . Si llamamos L a dicho lı́mite, resulta que
1 1
L= · L =⇒ L = 0 ó = 1 =⇒ L = 0.
1 + L2 1 + L2

172
PROBLEMA 13.17
Sea (an )n∈N la sucesión definida de la siguiente forma:
(
a1 = 1,
an+1 = 2(1+a n)
2+an .

Probar que lı́m an = 2.
n→∞

Solución
(*) Probaremos en primer lugar por inducción que a2n < 2:

a21 = 1 < 2;
1 + 2an + a2n 2 + 4an + 2a2n
Si a2n < 2, a2n+1 = 4 · = 2 · .
4 + 4an + a2n 4 + 4an + a2n

Pero como a2n < 2, entonces 2 + 4an + 2a2n < 4 + 4an + a2n , de donde
a2n+1 < 2.

Queda pues probado que la sucesión está acotada superiormente por 2.
an+1 2 + 2an
(*) Por otra parte, como a2n < 2, entonces = > 1 =⇒ an+1 > an ,
an 2an + a2n
es decir que la sucesión es monótona creciente.
(*) De lo anterior podemos concluir que la sucesión es convergente; si lla-
mamos L = lı́m an , tenemos

2(1 + L) √
L= =⇒ 2L + L2 = 2 + 2L =⇒ L2 = 2 =⇒ L = 2,
2+L
pues an ≥ 0 para todo n.

PROBLEMA 13.18
Sea (xn )n∈N la sucesión definida de la siguiente forma:
(
x1 = 10, 
xn+1 = 2 xn + x1n .
1

Demostrar que es convergente y que lı́m xn = 1.


n→∞

173
Solución

*) Veremos que la sucesión está acotada inferiormente.

Para ello probaremos en primer lugar por inducción que xn > 0, ∀n:

Es evidente que x1 > 0 y, si suponemos que xn > 0, tenemos que xn +1/xn >
0, de donde xn+1 > 0.

Veamos a continuación que además xn ≥ 1, ∀n. Para ello suponemos tam-


bién que xn ≥ 1 y probaremos que xn+1 ≥ 1:

Como (xn − 1)2 ≥ 0, entonces

x2n + 1
x2n + 1 − 2xn ≥ 0 =⇒ x2n + 1 ≥ 2xn =⇒ ≥ 1 =⇒ xn+1 ≥ 1.
2xn

*) La sucesión es monótona decreciente:

Como x2n ≥ 1, se deduce que, para todo n,

x2n + 1
 
1 1 xn+1 1 1
2
≤ 1 =⇒ 1+ 2
≤ 2 =⇒ = 2
= 1+ 2 ≤ 1 =⇒ xn+1 ≤ xn .
xn xn xn 2xn 2 xn

*) De lo anterior se deduce que la sucesión es convergente. Además, si lla-


mamos L al lı́mite, tenemos que

1 L2 + 1
L= · =⇒ 2L2 = L2 + 1 =⇒ L2 = 1 =⇒ L = ±1.
2 L

Como todos los términos son positivos, sólo es posible la solución L = 1.

PROBLEMA 13.19
Sea (xn )n∈N la sucesión definida de la siguiente forma:
(
x1 = 1/3,
xn+1 = √ 1 .
1+ (1/xn )−1

Demostrar que es convergente y calcular su lı́mite.

Solución

- En primer lugar, probaremos por inducción que 0 < xn < 1/2, ∀n:

La propiedad es cierta para n = 1 pues x1 = 1/3 < 1/2.

174
Si suponemos cierta la propiedad para xn , veamos que también lo es para
xn+1 :
p
0 < xn < 1/2 =⇒ 1/xn > 2 =⇒ 1/xn − 1 > 1 =⇒ 1/xn − 1 > 1
p 1
=⇒ 1 + 1/xn − 1 > 2 =⇒ xn+1 = p < 1/2,
1 + (1/xn ) − 1

y además xn+1 > 0.


- Veamos a continuación que la sucesión es creciente, es decir xn+1 ≥ xn , ∀n.
Para ello, escribiremos inecuaciones equivalentes a la que queremos probar
hasta obtener alguna cuya solución sea más asequible:
1 1 p
xn+1 ≥ xn ⇐⇒ p ≥ xn ⇐⇒ ≥ 1 + (1/xn ) − 1
1+ (1/xn ) − 1 xn
r
1 1 1 1 1
⇐⇒ −1≥ − 1 ⇐⇒ − 1 ≥ 1 ⇐⇒ ≥ 2 ⇐⇒ ≥ xn
xn xn xn xn 2

lo cual está ya probado al principio.


- En definitiva, como la sucesión es monótona creciente y acotada supe-
riormente, será convergente. Para calcular su lı́mite, utilizamos la fórmula
L = lı́m xn = lı́m xn+1 y tenemos en cuenta que L ≤ 1/2 (pues 1/2 es una
cota superior de {xn }):
1 p
L= p =⇒ L + L 1/L − 1 = 1 =⇒ L2 (1/L − 1) = (1 − L)2
1+ 1/L − 1
=⇒ L − L2 = 1 − 2L + L2 =⇒ 2L2 − 3L + 1 = 0

3± 9−8
=⇒ L = =⇒ L = 1 ó L = 1/2 =⇒ L = 1/2.
4

Observación. Aunque la sucesión esté definida en forma recurrente, en este


caso es posible encontrar una fórmula explı́cita del término general. Ası́, se
1
puede probar por inducción que xn = , ∀n y es evidente que
1 + 21/2n−1
lı́m xn = 1/2.

PROBLEMA 13.20
Demostrar que la sucesión (xn )n∈N definida por:

x1 = 1/8, xn+1 = x2n + 1/4, n ≥ 1,




es convergente y calcular su lı́mite.

175
Solución
*) Veamos en primer lugar que {xn } es creciente:

xn+1 − xn = x2n − xn + 1/4 = (xn − 1/2)2 ≥ 0, ∀n =⇒ xn+1 ≥ xn , ∀n.

*) A continuación probaremos por inducción que la sucesión está acotada


superiormente por 1/2. En efecto,

x1 = 1/8 < 1/2.

Además, si suponemos xn < 1/2, veamos que xn+1 < 1/2:

xn < 1/2 =⇒ x2n < 1/4 =⇒ x2n + 1/4 < 1/2 =⇒ xn+1 < 1/2.

*) De lo anterior se deduce que {xn } es convergente. Si llamamos L a su


lı́mite, se debe verificar:

L = L2 + 1/4 =⇒ L2 − L + 1/4 = 0 =⇒ (L − 1/2)2 = 0 =⇒ L = 1/2.

PROBLEMA 13.21
Se define la siguiente sucesión:

x1 ∈ (0,√1),
xn+1 = 1 − 1 − xn .

Probar que es convergente y calcular su lı́mite.

Solución
*) Veamos en primer lugar que la sucesión está acotada probando por in-
ducción que xn ∈ (0, 1), ∀n.
Por hipótesis x1 ∈ (0, 1). Si suponemos que xn ∈ (0, 1), resulta:

0 < xn < 1 =⇒ 0 > −xn > −1 =⇒ 1 > 1 − xn > 0 =⇒ 1 > 1 − xn > 0
√ √
=⇒ −1 < − 1 − xn < 0 =⇒ 0 < 1 − 1 − xn < 1,

que es precisamente la condición 0 < xn+1 < 1.


*) La sucesión es decreciente pues, como

√ 1 − (1 − xn ) x
xn+1 = 1 − 1 − xn = √ = √n
1 + 1 − xn 1 + 1 − xn

176

y 1+ 1 − xn > 1, resulta que
xn+1 1
= √ < 1 =⇒ xn+1 < xn , ∀n.
xn 1 + 1 − xn
*) Lo anterior prueba que la sucesión es convergente. Si llamamos L al lı́mite
y utilizamos la propiedad L = lı́m xn = lı́m xn+1 , deducimos que
√ √
L = 1 − 1 − L =⇒ L − 1 = − 1 − L =⇒ (L − 1)2 = 1 − L
=⇒ (L − 1)2 + (L − 1) = 0 =⇒ (L − 1)(L − 1 + 1) = 0.

Esto conduce a las dos posibilidades L = 1 ó L = 0. Al ser la sucesión


decreciente y sus términos menores que 1, debe ser L = 0.

PROBLEMA 13.22
Sea (xn )n∈N la sucesión definida por la ley de recurrencia 7xn+1 =
x3n + 6. Estudiar la convergencia en los siguientes casos:
i) x1 = 1/2.
ii) x1 = 3/2.
iii) x1 = 5/2.

Solución
Estudiaremos en primer lugar el crecimiento en el caso general. Como

x3n 6 x3 − 7xn + 6 (xn − 1)(xn − 2)(xn + 3)


xn+1 − xn = + − xn = n = ,
7 7 7 7
el crecimiento depende del signo de los factores.
Por otra parte, si la sucesión fuera convergente y llamamos L al lı́mite, debe
verificarse que

7L = L3 + 6 ⇐⇒ L3 − 7L + 6 = 0 ⇐⇒ (L − 1)(L − 2)(L + 3) = 0,

con lo que los únicos posibles lı́mites son L = 1, L = 2 ó L = −3.


Veamos las distintas posibilidades según los valores del primer término.
i) Si x1 = 1/2, probaremos por inducción que 0 < xn < 1, ∀n: Efectiva-
mente, si suponemos que 0 < xk < 1, entonces

0 < x3k < 1 =⇒ 6 < x3k + 6 < 7 =⇒ 6/7 < xk+1 < 1,

177
como querı́amos probar.
De lo anterior se deduce que la sucesión es creciente pues xn − 1 <
0, xn − 2 < 0, xn + 3 > 0, de donde xn+1 − xn > 0. Como también
está acotada superiormente, debe ser convergente y, recordando que
0 < xn < 1, el lı́mite debe ser L = 1.
ii) Si x1 = 3/2, probaremos por inducción que 1 < xn < 2, ∀n: Efectiva-
mente, si suponemos que 1 < xk < 2, entonces

1 < x3k < 8 =⇒ 7 < x3k + 6 < 14 =⇒ 1 < xk+1 < 2,

como querı́amos probar.


En este caso, como ahora xn − 1 > 0, xn − 2 < 0, xn + 3 > 0, se
deduce que xn+1 − xn < 0, con lo que la sucesión es decreciente. Como
está acotada inferiormente, también es convergente y su lı́mite sólo
puede ser L = 1.
iii) Si x1 = 5/2, se comprueba también por inducción que xn > 2, ∀n:
Suponemos para ello que xk > 2; entonces

x3k > 8 =⇒ x3k + 6 > 14 =⇒ xk+1 > 2,

como querı́amos probar.


De lo anterior se deduce que la sucesión es creciente pues, como xn −1 >
0, xn −2 > 0 y xn +3 > 0, resulta que xn+1 −xn > 0. Su lı́mite, en caso
de existir, deberı́a ser mayor que 2 lo cual es imposible según hemos
comprobado anteriormente. Esto indica que la sucesión es divergente.

PROBLEMA 13.23
Sea (an )n∈N la sucesión definida por recurrencia de la siguiente
forma:
a1 = −3/2, 3an+1 = 2 + a3n .
a) Comprobar que es convergente y calcular su lı́mite.
b) Modificar el primer término a1 para que el lı́mite sea −2.

Solución
a) - Probaremos en primer lugar que an ∈ (0, 1), ∀n > 2.
En efecto, como 3a2 = 2 + (−3/2)3 =⇒ a2 = −11/24, de donde

2 + (−11/24)3
a3 = = 0,63.
3

178
Si suponemos ahora que 0 < an < 1, se deduce que:

0 < a3n < 1 =⇒ 2 < a3n + 2 < 3 =⇒ 0 < 2/3 < an+1 < 1.

- Probamos a continuación que la sucesión es creciente. Para ello efec-


tuamos la resta entre dos términos consecutivos y obtenemos:

2 + a3n 2 + a3n − 3an (an − 1)2 · (an + 2)


an+1 − an = − an = = > 0,
3 3 3
con lo que, efectivamente, an+1 > an , ∀n.
- Lo anterior implica que la sucesión es convergente y L = lı́m an =
lı́m an+1 . Entonces:

3L = 2 + L3 =⇒ L3 − 3L + 2 = 0 =⇒ (L − 1)2 (L + 2) = 0.

Como an > 0, ∀n > 2, debe ser L = 1.


b) Veamos las distintas posibilidades para a1 :
- Si a1 > −2, entonces se prueba por inducción que an > −2, ∀n.
Debido a que

(an − 1)2 · (an + 2)


an+1 − an = > 0,
3
la sucesión es creciente y el lı́mite no puede ser −2, que es una cota
inferior.
- Si a1 < −2, entonces se prueba análogamente que an < −2, ∀n y que
la sucesión es decreciente; el lı́mite no puede ser −2 que es ahora una
cota superior.
La única posibilidad es pues que a1 = −2. Esto da lugar a una sucesión
constante pues, si an = −2, entonces an+1 = (2 − 8)/3 = −2. En este
caso el lı́mite es evidentemente −2.

PROBLEMA 13.24
Sea (xn )n∈N la sucesión definida de la siguiente forma:

x1 = 2,
xn+1 = 2 − x1n .

Demostrar que es convergente y calcular su lı́mite. ¿Qué pasarı́a


si x1 = 4?

179
Solución
Para que sea convergente basta comprobar que está acotada inferiormente
y es monótona decreciente. Para ello veamos en primer lugar que xn ≥
1, ∀n:
Está claro que x1 ≥ 1. Si además suponemos que xn ≥ 1 para algún n,
entonces
1 1 1
≤ 1 =⇒ − ≥ −1 =⇒ 2 − ≥1
xn xn xn
o bien xn+1 ≥ 1.
Además, como

1 2xn − 1 − x2n (xn − 1)2


xn+1 − xn = 2 − − xn = =− < 0,
xn xn xn

se deduce que xn+1 < xn y la sucesión es efectivamente decreciente.


Ahora bien, si llamamos L al lı́mite de la sucesión, como L = lı́m xn =
lı́m xn+1 , entonces L = 2 − 1/L =⇒ L2 = 2L − 1 =⇒ L = 1.
El mismo procedimiento se aplica al caso en que x1 = 4 pues son válidas las
mismas operaciones hechas anteriormente.
Observemos que en ambos casos podemos escribir una expresión explı́cita
para el término general de la sucesión. Concretamente, en el caso en que
n+1 3n + 1
x1 = 2, es xn = y, si x1 = 4, entonces xn = , lo que se puede
n 3n − 2
probar por inducción.

PROBLEMA 13.25
Sean a1 , a2 > 0, 2an+1 = an + an−1 , para n > 2. Probar que
1
lı́m an = (a1 + 2a2 ).
n→∞ 3

Solución
En primer lugar veremos que la sucesión es de Cauchy:

an + an−1 |an−1 − an |
|an+1 − an | =
− an = .
2 2

Procediendo por recurrencia se obtiene que

|a1 − a2 |
|an+1 − an | = .
2n−1

180
En general, si m > n, por la desigualdad triangular,

|am − an | ≤ |am − am−1 | + |am−1 − am−2 | + · · · + |an+1 − an |


 
1 1 1
= |a1 − a2 | m−2 + m−3 + · · · + n−1
2 2 2
1/2 n−1 1
≤ |a1 − a2 | · = |a1 − a2 | · n−2 .
1 − 1/2 2

De lo anterior se deduce que, si m, n → ∞, |am − an | → 0 y la sucesión es


de Cauchy.
Por el criterio general de convergencia de Cauchy, la sucesión es convergente.
Además,

2an+1 = an + an−1
2an = an−1 + an−2
2an−1 = an−2 + an−3
..
.
2a3 = a2 + a1 .

Sumando miembro a miembro, 2an+1 + 2an = an + 2a2 + a1 y llamando


2a2 + a1
L = lı́m an = lı́m an+1 , resulta que 3L = 2a2 + a1 , de donde L = .
3

PROBLEMA 13.26
Sea (an )n∈N la sucesión definida de la siguiente forma:
√ q

a1 = 2, an+1 = 2 + an , ∀n = 1, 2, . . .

Probar que la sucesión es convergente.

Solución
*) Probaremos en primer lugar por inducción que an ∈ (1, 2), ∀n.
Para n = 1 es cierto por hipótesis. Suponemos por tanto que también es
cierto para n y lo probaremos para n + 1. Tenemos pues:
√ √ √ √
1 < an < 2 =⇒ 1 < an < 2 =⇒ 3 < 2 + an < 2 + 2 < 4
√ q

=⇒ 3 < 2 + an < 2 =⇒ 1 < an+1 < 2.

181
*) Veamos a continuación que la sucesión es de Cauchy. Para ello, calculamos
una cota superior de |an+1 − an |:
q
√ q

|an+1 − an | = 2 + an − 2 + an−1

(2 + √an ) − (2 + √an−1 )

= p √ p √
2 + an + 2 + an−1
|an − an−1 | |a − an−1 |
= p √ √  √ √ < n .
2·2
p
2 + an + 2 + an−1 an + an−1
√ p √
(Esta última desigualdad se deduce de que an > 1 y 2 + an > 1.)
Procediendo por recurrencia se obtiene que

|an − an−1 | |a2 − a1 |


|an+1 − an | < < ... < .
4 4n−1
De lo anterior resulta que, si n, p > 0:

|an+p − an | ≤ |an+p − an+p−1 | + |an+p−1 − an+p−2 | + · · · + |an+1 − an |


 
1 1 1
< |a2 − a1 | n+p−2 + n+p−3 + · · · + n−1
4 4 4

X 1 1/4n−1 4|a2 − a1 |
< |a2 − a1 | · = |a2 − a1 | · = .
4k−1 1 − 1/4 3 · 4n−1
k=n

Como el último término tiende a cero cuando n → ∞, entonces |an+p −an | <
ε, ∀ε > 0. Esto indica que la sucesión es de Cauchy y, por tanto, convergente.

PROBLEMA 13.27
Sean a y b dos números reales que verifican 0 < a < b. Demostrar
que las dos sucesiones (xn )n∈N , (yn )n∈N definidas de la siguiente
forma ( (
x1 = a y1 = b
√ , n,
xn+1 = xn yn yn+1 = xn +y 2

son convergentes y que tienen el mismo lı́mite, llamado media


aritmético-geométrica de a y b.

Solución
Es evidente que xn > 0, yn > 0 para todo n ∈ N.
(*) Probaremos en primer lugar que xn ≤ yn , ∀n:

182
Como

0 ≤ (xn − yn )2 = x2n + yn2 − 2xn yn


= x2n + yn2 + 2xn yn − 4xn yn = (xn + yn )2 − 4xn yn ,

4xn yn
tenemos que (xn + yn )2 ≥ 4xn yn , es decir ≤ 1, ∀n.
(xn + yn )2

x2n+1 xn yn
Ahora bien, como 2 = ≤ 1, resulta que x2n+1 ≤ yn+1
2 , de
yn+1 (xn + yn )2 /4
donde xn+1 ≤ yn+1 , ∀n.

(*) Veamos a continuación que {xn } es creciente y que {yn } es decrecien-


te:
r
xn+1 yn
Como = ≥ 1, entonces xn+1 ≥ xn y {xn } es creciente.
xn xn
xn + yn xn − yn
Además yn+1 − yn = − yn = ≤ 0, es decir {yn } es decre-
2 2
ciente.

En definitiva, la sucesión {xn } es monótona creciente y acotada superiormen-


te por b y la sucesión {yn } es monótona decreciente y acotada inferiormente
por a. Esto implica que ambas son convergentes.

(*) Por último, si llamamos L = lı́m xn , M = lı́m yn , tenemos que L =



LM , de donde L2 = LM y, por ser L > 0, L = M .

PROBLEMA 13.28
Dados los números reales u0 y v0 con u0 < v0 , se construyen las
sucesiones
u0 + v0 u0 + 2v0
u1 = , v1 =
2 3
.. ..
. .
un−1 + vn−1 un−1 + 2vn−1
un = , vn =
2 3
a) Probar que lı́m (un − vn ) = 0.
n→∞

b) Probar que las sucesiones {un } y {vn } son convergentes.

183
Solución

a) Si calculamos la diferencia |un − vn |, obtenemos:


un−1 + vn−1 un−1 + 2vn−1
|un − vn | =

2 3

3un−1 + 3vn−1 − 2un−1 − 4vn−1 |un−1 − vn−1 |
=
= .
6 6

Procediendo por recurrencia, se obtiene que

|un−1 − vn−1 | |un−2 − vn−2 | |u0 − v0 |


0 ≤ |un − vn | = = 2
= ··· = .
6 6 6n

|u0 − v0 |
Como lı́m = 0, se deduce que lı́m |un − vn | = 0.
n→∞ 6n n→∞

b) Para probar que ambas sucesiones son convergentes, basta ver que veri-
fican el criterio general de Cauchy. Por el apartado anterior,


un + vn |vn − un | |u0 − v0 |
|un+1 − un | = − un = = .
2 2 2 · 6n

Si ahora p es un entero positivo cualquiera,

|un+p − un | ≤ |un+p − un+p−1 | + |un+p−1 − un+p−2 | + · · · + |un+1 − un |


 
|u0 − v0 | 1 1
= + ··· + n
2 6n+p−1 6
 
|u0 − v0 | 1 1
≤ + · · · + n+p−1 + . . .
2 6n 6
|u0 − v0 | 1/6n 3
= · = · |u0 − v0 | → 0,
2 1 − 1/6 5 · 6n

cuando n → ∞.

Para demostrar que {vn } es de Cauchy se procede de forma completa-


mente análoga. Se podrı́a razonar también diciendo que si {un − vn } y
{un } son ambas convergentes, también debe serlo {vn } que es su resta.

184
PROBLEMA 13.29
Dadas las constantes k > 0 y a > 0, se definen las sucesiones:
p
a) an+1 = k + an con a1 = a;
k
b) bn+1 = con b1 = a.
1 + bn
Probar que {an } converge a la raı́z positiva de la ecuación x2 −
x − k = 0 y que {bn } converge a la raı́z positiva de la ecuación
x2 + x − k = 0.

Solución
a) Sea L la raı́z positiva de la ecuación x2 − x − k = 0; ası́ L verifica
L2 = L + k. Dividiremos el estudio de la sucesión en tres casos:
1) Caso a1 = a = L. En este caso la sucesión es constante pues, debido
a que L2 = L + k, si suponemos an = L, entonces
p √ √
an+1 = k + an = k + L = L2 = L.

De aquı́ se deduce evidentemente que la sucesión converge a L.


2) Caso a1 = a < L. Probaremos que la sucesión está acotada supe-
riormente por L y que es monótona creciente.
En efecto, procediendo por inducción, como a1 < L, si suponemos que
an < L, entonces
p √
an + k < L + k = L2 =⇒ an + k < L2 =⇒ an+1 < L.

Por otra parte, como an está comprendido entre las dos raı́ces de la
ecuación x2 − x − k = 0, deducimos que
p
a2n − an − k < 0 =⇒ k + an > a2n =⇒ k + an > an =⇒ an+1 > an .

Estas dos propiedades conducen a la convergencia de la sucesión dada.


3) Caso a1 = a > L. Este caso es similar al anterior, pues se prueba
análogamente que la sucesión está acotada inferiormente y es monóto-
na decreciente.
b) Llamamos en este caso M a la raı́z positiva de la ecuación x2 + x −
k = 0. Esta sucesión no es monótona como se deduce de la siguiente
implicación (debemos tener en cuenta que M 2 + M = k, o bien M =
k/(1 + M )):
k k
an > M =⇒ an+1 = < = M.
1 + an 1+M

185
(Análogamente se prueba que si an < M , an+1 > M .) Veamos que la
sucesión es de Cauchy. Supondremos para ello que a1 > M > a2 , pero
la situación serı́a análoga en el caso de que a1 < M < a2 . Tenemos
ası́:

k k − a2n − a22n
a2n+1 − a2n = − a2n = (1)
1 + a2n 1 + a2n
∗ a2n · a2n−1 − a22n a2n
= = · (a2n−1 − a2n ) (2)
1 + a2n 1 + a2n
∗∗ L
< (a2n−1 − a2n ). (3)
1+L

Los pasos (*) y (**) se deducen de que, por una parte,

k
a2n = =⇒ a2n + a2n · a2n−1 = k
1 + a2n−1

y, por otra,

a2n < L =⇒ a2n + a2n · L < L + a2n · L =⇒ a2n (1 + L) < L(1 + a2n )
a2n L
=⇒ < .
1 + a2n 1+L

Además, de (1) se deduce que

k a2n−1 + a22n−1 − k
a2n−1 − a2n = a2n−1 − = (4)
1 + a2n−2 1 + a2n−1
a2n−1 (a2n−1 − a2n−2 )
= < a2n−1 − a2n−2 . (5)
1 + a2n−1

De (1) y (2), procediendo por recurrencia, llegamos a:


 2
L L
a2n+1 − a2n < (a2n−1 − a2n−2 ) < (a2n−3 − a2n−4 )
1+L 1+L
 n−1  n
L L
< ... < (a3 − a2 ) < (a1 − a2 ),
1+L 1+L

y
 n
L
a2n+1 − a2n+2 < a2n+1 − a2n < (a1 − a2 ).
1+L
De estas acotaciones se deduce como en los problemas anteriores que
la sucesión es de Cauchy y, por tanto, también es convergente.

186
PROBLEMA 13.30
Para los antiguos griegos, el rectángulo más estético era aquel cu-
yos lados a y b verifican la llamada “relación áurea”, es decir
a b
= .
b a+b
a) Comprobar que la relación
√ anterior es cierta si y sólo si el cociente
b 1+ 5
L = vale L = .
a 2
b) Probar que dicho número L es el lı́mite de la sucesión

a1 = 1,
an+1 = 1 + 1/an .

xn
c) Probar que si n ≥ 2, an = donde (xn ) es la sucesión de Fibo-
xn−1
nacci siguiente:

1, 2, 3, 5, 8, 13, 21, . . .

es decir, aquella cuyos términos se obtienen sumando los dos


términos anteriores a él.

Solución

a) Escribiendo identidades equivalentes, tenemos

 2
a b 2 b
2 b
= ⇐⇒ a + ab = b ⇐⇒ 1 + =
b a+b a a
 2 √
b b b 1± 1+4
⇐⇒ − − 1 = 0 ⇐⇒ = .
a a a 2

Como a,√b > 0, entonces b/a > 0 y sólo es válida la solución positiva
b 1+ 5
= .
a 2
b) Debido a que 1/an ≥ 0, ∀n, se deduce que an+1 = 1 + (1/an ) ≥ 1.

Por lo anterior, 1/an ≤ 1, de modo que an+1 = 1 + (1/an ) ≤ 2.

Lo anterior prueba que la sucesión está acotada, 1 ≤ an ≤ 2, ∀n.

187
Veremos a continuación que es una sucesión de Cauchy:

1 1 1
|an+1 − an | = 1 +
− an = 1 +
−1−
an an an−1
|an − an−1 | |an−1 − an−2 |
= =
an · an−1 (an · an−1 )(an−1 · an−2 )
|a2 − a1 |
= ··· = .
(an · an−1 )(an−1 · an−2 ) . . . (a2 · a1 )

1 1
Como a2 − a1 = 1 y an+1 · an = an + 1 > 2, es decir < , ∀n,
an+1 · an 2
1
resulta que |an+1 − an | ≤ .
2n−1
En general,

|an+p − an | ≤ |an+p − an+p−1 | + |an+p−1 − an+p−2 | + · · · + |an+1 − an |


1 1 1
≤ + + · · · + n−1
2n+p−2 2n+p−3 2
1 1 1 1/2n−1 1
≤ + + · · · + + · · · = = n−2 ,
2n−1 2n 2n+p−2 1 − 1/2 2

y el último término tiende a cero cuando n → ∞.


Como la sucesión es de Cauchy, es convergente. Si llamamos L a su lı́mite,
tenemos que L = 1 + 1/L, de donde, sucesivamente, obtenemos:
√ √
2 2 1± 1+4 1+ 5
L = L + 1 =⇒ L − L − 1 = 0 =⇒ L = =⇒ L = ,
2 2
debido a que L debe ser positivo.
xn
c) Probaremos por inducción que an = :
xn−1
x2 2
Para n = 2, a2 = 1 + 1 = 2 y = = 2.
x1 1
xn xn+1
Supongamos ahora que an = y veamos que an+1 = :
xn−1 xn
1 xn−1 xn + xn−1 xn+1
an+1 = 1 + =1+ = = ,
an xn xn xn
debido a que xn + xn−1 = xn+1 .

188
C. EJERCICIOS PROPUESTOS.

1. Sea (xn )n∈N una sucesión de números reales. Responder razona-


damente si cada uno de los siguientes apartados es verdadero o
falso.
a) Si (xn )n∈N converge, entonces es creciente o decreciente.
Resp.: Falso (ver por ejemplo la sucesión xn = (−1)n /n).

b) Si (xn )n∈N es creciente o decreciente, entonces converge.


Resp.: Falso (ejemplo xn = n).

c) Si (xn )n∈N converge, entonces está acotada.


Resp.: Verdadero (se deduce de la definición de convergencia).

d) Si (xn )n∈N está acotada, entonces converge.


Resp.: Falso (ejemplo xn = (−1)n ).

e) Si (xn )n∈N es creciente, entonces está acotada superiormente.


Resp.: Falso (ejemplo xn = n).

f) Si (xn )n∈N es creciente, entonces está acotada inferiormente.


Resp.: Verdadero pues x1 ≤ an , ∀n.

g) Si lı́m xn = x, entonces lı́m |xn | = |x|.



Resp.: Verdadero pues |xn | − |x| ≤ |xn − x| → 0.

h) Si (xn )n∈N está acotada y converge, entonces es monótona.


Resp.: Falso (ejemplo xn = (−1)n /n).

2. Sean (an )n∈N y (bn )n∈N dos sucesiones cualesquiera. Responder


razonadamente si cada uno de los apartados siguientes es verda-
dero o falso.
a) Si para todo n ∈ N, an 6= 0, entonces lı́m an bn 6= 0.
n→∞

Resp.: Falso (ejemplo bn = 0, ∀n).

189
b) Si para todo n ∈ N, an 6= 0 y bn 6= 0, entonces lı́m an bn 6= 0.
n→∞

Resp.: Falso (ejemplo an = bn = 1/n).

c) Si lı́m an bn = 0, entonces lı́m an = 0 ó lı́m bn = 0.


Resp.: Verdadero pues si fueran a = lı́m an 6= 0 y b = lı́m bn 6= 0,
entonces a · b = lı́m an · bn 6= 0.
 n
n+a
3. Hallar a tal que lı́m = 4.
n→∞ n−a
Resp.: a = ln 2.

4. Hallar la relación entre a y b para que



n+1 r ! √ 2n √
n+1− n

n+b n+a
lı́m ln = lı́m .
n→∞ n n→∞ n

Resp.: b = ea 2/2
.

5. Hallar la relación que debe existir entre a y b para que


3n + a 2n−1
  p 
lı́m = lı́m 4n2 + bn − 2n .
n→∞ 3n − 4 n→∞

Resp.: e2(a+4)/3 = b/4.

6. Hallar la relación que debe existir entre a y b para que


2n + a 3n−1
  p 
lı́m = lı́m n2 + bn − n .
n→∞ 2n − 1 n→∞

Resp.: b/2 = e3(a+1)/2 .

7. Sean a1 , . . . , ak números realesppositivos. Calcular el lı́mite de la


sucesión de término general n an1 + · · · + ank .
Resp.: L = máx{a1 , . . . , ak }.

8. Sea {an } una sucesión creciente, con an ≥ 0, ∀n. Probar que



la sucesión de término general xn = n a1 · a2 · · · · · an también es
creciente.
Sugerencia: Comprobar que la sucesión {ln xn } es creciente y deducir
que también lo es la sucesión pedida.

190
r q
3 √
9. Probar que la sucesión de término general an = 2 3 . . . n n es
convergente.
Sugerencia: Calcular el logaritmo de an y estudiar la serie resultante.

n
X 1
10. Demostrar que la sucesión de término general an = es
n+k
k=1
convergente.
n
Resp.: Como an < , ∀n, la sucesión está acotada superiormente;
n+1
1
además an+1 − an = > 0 y la sucesión es creciente.
(2n + 1)(2n + 2)

11. Sea (xn )n∈N la sucesión definida de la siguiente forma:


(
x1 =k < 0, 
xn+1 = 12 xn + x1n .

i)Probar que xn ≤ −1, ∀n ≥ 2. (Aplicar el método de inducción.)


ii) Probar que {xn } es convergente y hallar su lı́mite. (La sucesión
es creciente pues xn+1 − xn ≥ 0, ∀n.)
Resp.: L = −1.

12. Se define la sucesión (un )n≥0 por:

un−1 (1 + un−1 )
u0 = 1, u1 = 2/3, . . . , un = .
1 + 2un−1

i) Comprobar que es decreciente.


ii) Demostrar que es convergente y calcular su lı́mite.
Resp.: Es evidente que un /un−1 ≤ 1 porque un ≥ 1, ∀n. Además
un ≥ 0, ∀n.


13. Sea {xn } la sucesión definida por x1 = 1, xn+1 = 2 + xn para n >
1. Demostrar que es monótona creciente y acotada superiormente
por 2. Calcular su lı́mite.
Resp.: L = 2.

191
2an
14. Comprobar que la sucesión definida por an+1 = , a1 = 3, es
an + 3
convergente y calcular su lı́mite.
an+1
Resp.: an > 0, y < 1, ∀n. Además L = 0.
an

2
15. Se define la sucesión a1 ∈ (1, 2), an+1 = 3 − . Estudiar su con-
an
vergencia.
Resp.: La sucesión es creciente y acotada. Su lı́mite es L = 2.

an+1 1
16. Sea {an } una sucesión que verifica an > 0, ∀n y < . Cal-
an 5
cular lı́m an .
a1
Resp.: Como 0 < an < , lı́m an = 0.
5n−1

ab
17. Dados a, b > 0, se define la sucesión u1 = a + b, un = a + b − , n > 1.
un−1
an+1 − bn+1
i) Probar que un = si a 6= b.
an − bn
Resp.: Aplicar el método de inducción.

ii) Calcular lı́m un si a 6= b.


Resp.: lı́m un = máx{a, b}.

iii) Calcular un y lı́m un si a = b.


(n + 1)a
Resp.: un = , lı́m un = a.
n

192
CAPÍTULO XIV.
SERIES NUMÉRICAS
ARBITRARIAS

SECCIONES
A. Series de términos de signo variable.
B. Series dependientes de parámetros.
C. Ejercicios propuestos.

193
A. SERIES DE TÉRMINOS DE SIGNO VARIABLE.

En el capı́tulo 9 se estudiaba la convergencia de las series de términos con


signo constante. Trataremos aquı́ las series arbitrarias, es decir aquellas cu-
yos términos no son todos del mismo signo, más precisamente aquellas que
tienen infinitos términos positivos e infinitos términos negativos. Para estas
series será importante estudiar no sólo su convergencia sino la convergencia
de la serie formada por los valores absolutos de sus términos. Debido a la
propiedad:
X X
(1) Si la serie |an | converge, entonces an también converge y además
n≥1 n≥1
X X
an ≤ |an |


n≥1 n≥1

podemos distinguir los siguientes casos:


X
(a) Una serie an se dice que converge absolutamente si converge
Xn≥1
la serie |an |.
n≥1
X
(b) Una serie an converge condicionalmente si es convergente pero
n≥1
X
diverge la serie |an |.
n≥1
X X
(c) Una serie es divergente si divergen an y |an |.
n≥1 n≥1

Otras propiedades destacables son:


X
(2) Una serie an converge absolutamente si y sólo si son convergentes la
n≥1
serie formada con sus términos positivos y la formada con sus términos
negativos.
X X
(3) Si las series an y bn son absolutamente convergentes, entonces
n≥1 n≥1
X
la serie (αan + βbn ) es absolutamente convergente, ∀α, β ∈ R.
n≥1
X
(4) Si an converge absolutamente, todo reordenamiento de {an } produce
n≥1
X
una serie cuya suma coincide con an .
n≥1

194
X X
(5) Si an y bn son absolutamente convergentes, también lo es la
n≥1 n≥1
X
serie producto pn definida de la siguiente manera:
n≥1

n
X
pn = a1 bn + a2 bn−1 + · · · + an b1 = ak bn−k .
k=1

Un caso particular de las series arbitrarias lo constituyen las se-


ries alternadas, que son aquellas cuyos términos son alternativamente
positivos
X y negativos. Las series alternadas se suelen expresar como
n
(−1) an donde an ≥ 0, ∀n, o de cualquier forma equivalente (por
n≥1
X X
ejemplo sen(nπ/2)an ó cos(nπ)an ). Las series alternadas tienen
n≥1 n≥1
la siguiente propiedad:
X
(6) Si (−1)n an es una serie alternada convergente y llamamos S a la
n≥1
suma de la serie, entonces
n

X
S − (−1)k ak ≤ ak+1


k=1

(esto quiere decir que el error cometido al sumar los n primeros térmi-
nos es menor que el primer término desechado).
Para estudiar la convergencia de las series arbitrarias, aparte de los criterios
ya enunciados en el capı́tulo 9 para series de términos positivos, aplicaremos
los siguientes criterios especı́ficos:
- Criterio de Leibnitz. Si la sucesión de términos positivos X{an } es de-
creciente y tiene lı́mite cero, entonces la serie alternada (−1)n an es
n≥1
convergente.
X
- Criterio del cociente. Dada la serie arbitraria an , llamamos
n≥1

an+1 an+1
L = lı́m sup y l = lı́m inf
an .

an
(a) Si L < 1, la serie converge absolutamente.
(b) Si l > 1, la serie diverge.
X p
n
- Criterio de la raı́z. Dada la serie arbitraria an , llamamos L = lı́m sup |an |.
n≥1

195
(a) Si L < 1, la serie converge absolutamente.
(b) Si L > 1, la serie diverge.
Veremos en los siguientes problemas ejemplos diversos de aplicación de estas
propiedades.

PROBLEMA 14.1
P
Estudiar
√ el carácter de la serie an de término general an =
(−1)n [ n2 − 1 − n] y hallar una cota del error cometido al tomar
como suma la de los cuatro primeros términos.

Solución
−1 (−1)n+1
Si escribimos an = (−1)n √ =√ , vemos que se trata
n2 − 1 + n n2 − 1 + n
de una serie alternada. Aplicaremos el criterio de Leibnitz:
1 1
√ <p =⇒ |an | < |an−1 |.
n2 −1+n 2
(n − 1) + 1 + (n − 1)
1
lı́m an = lı́m √ = 0.
n2 − 1 + n
Como la sucesión es en valor absoluto decreciente y convergente a cero, la
serie es convergente.
Por otra parte, es sabido que al tomar la suma parcial sn como valor de la
serie, el error cometido es menor que el valor absoluto del primer término
despreciado |an+1 |.
√ √
La cota del error pedida en este caso es |a5 | = | 52 − 1 − 5| = 5 − 24.

PROBLEMA 14.2
Probar que la serie
1 3 1 4 1 n+1
1 − ln 2 + − ln + − ln + · · · + − ln + ...
2 2 3 3 n n
es convergente.

Solución
 n  n+1
1 1
A partir de la desigualdad evidente 1+ <e< 1+ , se obtie-
n n
ne que

196
   
1 1 1
n ln 1 + < 1 < (n + 1) ln 1 + =⇒ n < n+1
 <n+1
n n ln n
 
1 n+1 1
=⇒ < ln < ,
n+1 n n

lo que quiere decir que la sucesión de valores absolutos es decreciente.

Además es evidente que dicha sucesión tiende a cero pues

1 n+1
lı́m = 0 y lı́m ln = 0.
n→∞ n n→∞ n

Por el criterio de Leibnitz, la serie es convergente.

Observación. A la suma de la serie anterior se le llama constante de Euler


γ = 0,577215 . . . la cual no se sabe aún si se trata de un número racional o
irracional.

PROBLEMA 14.3
1 1 1
Considerando que 1 + + + · · · + = ln n + γ + εn , donde γ es la
2 3 n
constante de Euler y lı́m εn = 0, hallar la suma de la serie
n→∞

1 1 1 1 1 1 1 1 1 1 1 1 1
1+ + + − − − + + + + − − − + ...
3 5 7 2 4 6 9 11 13 15 8 10 12
formada a partir de la serie alternada
1 1 1 1 1 1
1− + − + − + · · · + (−1)n+1 + . . .
2 3 4 5 6 n
tomando cuatro términos positivos, después tres términos negati-
vos, después cuatro positivos, etc.

Solución

Calculamos la suma de los (4 + 3)n primeros términos de la serie. Tenemos


ası́:

197
1 1 1 1 1 1
S(4+3)n = 1 + + + − − − + ...
3 5 7 2 4 6
1 1 1 1 1 1 1
+ + + + − − −
8n − 7 8n − 5 8n − 3 8n − 1 6n − 4 6n − 2 6n
1 1 1 1 1 1 1
= 1 + + + + ··· + + + +
3 5 7 8n − 7 8n − 5 8n −3 8n − 1
1 1 1 1 1 1
− + + + ··· + + +
2 4 6 6n − 4 6n − 2 6n
1 1 1 1 1 1 1 1
= 1 + + + + ··· + + + + +
2 3 4  8n − 6 8n − 5 8n − 4 8n − 3  −2
8n
1 1 1 1 1 1 1 1
+ + − + + + ··· + + +
8n − 1 8n 2 4 6 8n − 4 8n − 2 8n
 
1 1 1 1 1 1
− + + + ··· + + +
2 4 6 6n − 4 6n − 2 6n

1 1 1 1 1 1
= ln 8n + γ + ε8n − 1 + + + + · · · + +
2 2 3 4 4n − 3 4n − 2
  
1 1 1 1 1 1 1 1
+ + − 1 + + + ··· + + +
4n − 1 4n 2 2 3 3n − 2 3n − 1 3n
1 1
= ln 8n + γ + ε8n − (ln 4n + γ + ε4n ) − (ln 3n + cγ + ε3n )
2 2
1 1 1 1
= ln 8n − ln 4n − ln 3n + ε8n − ε4n − ε3n
2 2 2 2
1 1 1 1 1 1
= ln 8 + ln n − ln 4 − ln n − ln 3 − ln n + ε8n − ε4n − ε3n
2 2 2 2 2 2
1 1 1 1 1
= 3 ln 2 + ln n − ln 2 − ln n − ln 3 − ln n + ε8n − ε4n − ε3n
2 2 2 2 2
1 1 1
= 2 ln 2 − ln 3 + ε8n − ε4n − ε3n .
2 2 2
1 4
Entonces S = lı́m S(4+3)n = 2 ln 2 − ln 3 = ln √ .
2 3

PROBLEMA 14.4
Estudiar la convergencia (absoluta y condicional) de la serie
X n
(−1)n .
ln n
n≥1

Solución
Aplicaremos la regla de L’Hôpital para calcular el lı́mite del término general.
Ası́:

198
n 1
lı́m |an | = lı́m = lı́m = ∞.
ln n 1/n
Por el criterio del resto se deduce que la serie es divergente.

PROBLEMA 14.5
Estudiar la convergencia (absoluta y condicional) de la serie
X 1
(−1)n .
2n + 1
n≥1

Solución
Como se trata de una serie alternada podemos aplicar el criterio de Leib-
nitz.
1
Si llamamos an = , es evidente que la sucesión {an } es decreciente y
2n + 1
tiene lı́mite cero, por lo que la serie es convergente. Sin embargo, la serie
X 1 P
es divergente (basta compararla con la serie 1/n) lo que indica
2n + 1
que la serie dada es condicionalmente convergente.

PROBLEMA 14.6
Estudiar la convergencia (absoluta y condicional) de la serie
X n
(−1)n n .
2
n≥1

Solución
Aplicaremos el criterio del cociente. Como

an+1 n+1
lı́m
= lı́m 2n+1 = lı́m n + 1 = 1 < 1,
n
an 2n 2n 2
la serie es absolutamente convergente.

PROBLEMA 14.7
Estudiar la convergencia (absoluta y condicional) de la serie
X 1
(−1)n−1 √ .
n
n≥1

199
Solución
Aplicaremos el criterio de Leibnitz a la serie alternada.

La sucesión de término general an = 1/ n es decreciente (pues an+1 <
an , ∀n) y lı́m an = 0 lo que indica que la serie es convergente.
Sin embargo, dicha convergencia es condicional porque la serie de valores
X 1
1/nα con α ≤ 1).
P
absolutos √ es divergente (caso particular de
n

PROBLEMA 14.8
Estudiar la convergencia (absoluta y condicional) de la serie
X n
(−1)n−1 .
6n − 5
n≥1

Solución
Como
n 1
lı́m |an | = lı́m = 6= 0,
6n − 5 6
por el criterio del resto se deduce que la serie es divergente.

PROBLEMA 14.9
Estudiar la convergencia (absoluta y condicional) de la serie
X 2n + 1
(−1)n−1 .
n(n + 1)
n≥1

Solución
Del criterio de Leibnitz se obtiene que la serie es convergente pues la sucesión
2n + 1
de término general an = es decreciente y lı́m an = 0. Efectivamen-
n(n + 1)
te:
2n + 3 2n + 1 n(2n + 3) − (n + 2)(2n + 1)
an+1 − an = − =
(n + 1)(n + 2) n(n + 1) n(n + 1)(n + 2)
−2n − 2
= < 0 =⇒ an+1 < an , ∀n;
n(n + 1)(n + 2)
2n + 1
lı́m an = lı́m 2 = 0.
n +n

200
X 2n + 1
La convergencia es condicional pues la serie es divergente (basta
n(n + 1)
P
aplicar el criterio de comparación con la serie 1/n).

PROBLEMA 14.10
Estudiar la convergencia (absoluta y condicional) de la serie
X n2 + 2
(−1)n .
(n + 2)2
n≥1

Solución
Como
n2 + 2
lı́m |an | = lı́m = 1 6= 0,
(n + 2)2
la serie es divergente (criterio del resto).

PROBLEMA 14.11
Estudiar la convergencia (absoluta y condicional) de la serie
X 1
(−1)n .
ln n
n≥1

Solución
1
Por el criterio de Leibnitz, como lı́m = 0 y la sucesión {1/ ln n} es de-
ln n
creciente, la serie es convergente.
X 1
Por otra parte, la serie de valores absolutos es divergente como se
ln n X
1
comprueba aplicando el criterio de comparación con .
n
En definitiva, la serie propuesta es condicionalmente convergente.

PROBLEMA 14.12
Estudiar la convergencia (absoluta y condicional) de la serie
 n
n 2n + 1
X
(−1) .
3n + 1
n≥1

201
Solución
Aplicaremos el criterio de la raı́z. Como
p
n 2n + 1 2
lı́m |an | = lı́m = < 1,
3n + 1 3
la serie es absolutamente convergente.

PROBLEMA 14.13
Estudiar la convergencia (absoluta y condicional) de la serie
X 1 + sen2 n
(−1)n .
n2
n≥1

Solución
X 1 + sen2 n
Estudiaremos la convergencia de la serie de valores absolutos .
n2
1 + sen2 n 2 X 2
Por el criterio de comparación, como ≤ y la serie es
n2 n2 n2
convergente, la serie propuesta es absolutamente convergente.

PROBLEMA 14.14
Estudiar la convergencia (absoluta y condicional) de la serie
X (2n)!
(−1)n sen2 n · 2n .
n
n≥1

Solución
(2n)!
Si llamamos an al término general de la serie, debido a que |an | ≤ , la se-
n2n
X (2n)!
rie dada será absolutamente convergente si es convergente la serie .
n2n
Por el criterio del cociente,
(2n+2)!
bn+1 (n+1)2n+2 (2n + 2)(2n + 1)n2n
lı́m = lı́m = lı́m
bn (2n)! (n + 1)2 (n + 1)2n
n2n
 2n
(2n + 2)(2n + 1) n
= lı́m · lı́m = 4e−2 < 1.
(n + 1)2 n+1
Esto indica que la serie original es absolutamente convergente.

202
PROBLEMA 14.15
Estudiar la convergencia (absoluta y condicional) de la serie
X 1 · 5 · 9 . . . (4n − 3)
cos(nπ).
(3n)! + 1
n≥1

Solución

En primer lugar acotamos en valor absoluto el término general de la se-


rie:
1 · 5 · 9 . . . (4n − 3)
|an | < ,
(3n)!

debido a que | cos(nπ)| = 1 y (3n)! + 1 > (3n)!


X 1 · 5 · 9 . . . (4n − 3)
A continuación probaremos que la serie mayorante
(3n)!
es convergente aplicando el criterio del cociente:

1·5·····(4n−3)(4n+1)
bn+1 (3n+3)! 4n + 1
lı́m = lı́m 1·5·····(4n−3)
= lı́m = 0.
bn (3n + 3)(3n + 2)(3n + 1)
(3n)!

De lo anterior, y aplicando el criterio de comparación, se deduce que la serie


original es absolutamente convergente.

PROBLEMA 14.16
Estudiar la convergencia (absoluta y condicional) de la serie
X (2n)!
cos(nπ).
n2n + n
n≥1

Solución

Se trata de una serie alternada ya que cos(nπ) = (−1)n . Como n2n +n > n2n ,
(2n)!
se verifica que |an | < 2n . En el problema 14.14 se probó que la serie
n
X (2n)!
mayorante es convergente. Esto indica que la serie propuesta es
n2n
absolutamente convergente.

203
PROBLEMA 14.17
Estudiar la convergencia (absoluta y condicional) de la serie
X (−3)2n
.
n≥1
(n + 1) ln2 (n + 1)

Solución
Por el criterio del cociente,
32n+2
32 (n + 1) ln2 (n + 1)

an+1 (n+2) ln2 (n+2)
lı́m = lı́m = lı́m = 9 > 1,
an 32n (n + 2) ln2 (n + 2)
(n+1) ln2 (n+1)

con lo que la serie diverge.

PROBLEMA 14.18
Estudiar la convergencia (absoluta y condicional) de la serie
X 2n sen(nπ/2)
p .
(3n − 2) · 5 n
n≥1

Solución

Como sen = (−1)n+1 , tenemos una serie alternada. Si aplicamos el cri-
2
terio del cociente, resulta:
n+1
√ 2 √
an+1 (3n+1)·5n+1 2 3n − 2 2
lı́m
= lı́m 2 n = lı́m √ · √ = √ <1
an √ n
5 3n + 1 5
(3n−2)·5

y la serie es absolutamente convergente.

PROBLEMA 14.19
Estudiar la convergencia (absoluta y condicional) de la serie
X (−1)n
√ √ .
n≥1
n+ n+1

204
Solución
Aplicaremos el criterio de Leibnitz al tratarse de una serie alternada. Para
1
ello debemos comprobar que la sucesión de término general an = √ √
n+ n+1
converge a cero y es decreciente.
√ √
an+1 n+ n+1
Es evidente que lı́m an = 0. Además, =√ √ < 1, por lo
an n+1+ n+2
que la sucesión es decreciente.
X 1
Sin embargo, la serie de valores absolutos √ √ es divergente,
n+ n+1
X 1
como se deduce al aplicar el criterio de comparación con √ .
n
En definitiva, la serie propuesta es condicionalmente convergente.

PROBLEMA 14.20
Estudiar la convergencia (absoluta y condicional) de la serie
X sen √n
√ .
n≥1
n3 + 1

Solución
Estudiamos
√la serie
de valores absolutos aplicando el criterio de comparación.
sen n
≤ 1 y la serie
X 1
Como √ es convergente, se deduce que la
n3 + 1 n3/2 n3/2
serie original es absolutamente convergente.

PROBLEMA 14.21
X
Sea an una serie hipergeométrica, es decir que verifica la rela-
n≥1
an+1 αn + β
ción = , ∀n, donde α, β, γ son constantes fijas y α, γ
an αn + γ
no nulas a la vez.
γ−β
a) Probar que la serie es convergente si > 1.
α
an (nα + β) − a1 γ
b) Probar que Sn = , ∀n.
α+β−γ
−a1 γ
c) Probar que, en caso de convergencia, la suma de la serie es .
α+β−γ

205
Solución
a) Aplicaremos el criterio de Raabe (observamos que, desde un cierto n en
an+1
adelante, > 0, pues α, β, γ son constantes fijas):
an
 
an+1 αn + γ − αn − β γ−β
lı́m n 1 − = lı́m n · = ,
an αn + γ α
γ−β
lo que indica que la serie converge si > 1.
α
an (nα + β) − a1 γ
b) Probaremos por inducción que Sn = , siendo Sn =
α+β−γ
a1 + a2 + · · · + an .
a1 (α + β) − a1 γ
- Para n = 1, = a1 = S1 .
α+β−γ
an−1 [(n − 1)α + β) − a1 γ
- Si suponemos que Sn−1 = , debemos com-
α+β−γ
an (nα + β) − a1 γ
probar que Sn = .
α+β−γ
Por ser una serie hipergeométrica, se verifica la relación an [α(n − 1) +
γ] = an−1 [α(n − 1) + β]. Utilizando esta igualdad, tenemos:
an−1 [(n − 1)α + β) − a1 γ
Sn = Sn−1 + an = + an
α+β−γ
an [α(n − 1) + γ] − a1 γ
=⇒ Sn = + an
α+β−γ
an [α(n − 1) + γ + α + β − γ] − a1 γ an (αn + β) − a1 γ
= = ,
α+β−γ α+β−γ
como querı́amos demostrar.
X α · nan + βan − γa1
c) Si la serie es convergente, entonces an = lı́m Sn = lı́m .
α+β−γ
n≥1
Ahora bien, recordando que, en una serie convergente, lı́m an = 0 y
−γa1
lı́m nan = 0, dicho lı́mite queda .
α+β−γ

PROBLEMA 14.22
Probar que, si a + b = c, entonces
   
X an X bn X cn
 · = .
n! n! n!
n≥0 n≥0 n≥0

206
Solución
an bn
Por definición de producto de series, si an = y bn = , el término general
n! n!
de la serie producto es

pn = a0 · bn + a1 · bn−1 + a2 · bn−2 + · · · + an · b0
bn bn−1 a2 bn−2 an
= +a· + · + ··· +
n! (n − 1)! 2! (n − 2)! n!
cn
 
1 n n(n − 1) 2 n−2 1
= b + nabn−1 + a ·b + · · · + an = (a + b)n = ,
n! 2! n! n!
como querı́amos probar.
X xn
Observación. Si llamamos f (x) = , hemos probado que f (a) · f (b) =
n!
n≥0
f (a+b) lo que sugiere llamar a f función exponencial (ver capı́tulo siguiente).

PROBLEMA 14.23
 2
X 1 X 1
Probar que   = .
2n · n! n!
n≥0 n≥0

Solución El término n-ésimo del producto es


n n n  
X 1 1 X n! 1 X n 1
pn = i
· n−i = n
= n = ,
2 · i! 2 · (n − i)! 2 · n!(n − i)! · i! 2 · n! i n!
i=0 i=0 i=0

n  
n
X n
debido a que 2 = .
i
i=0

B. SERIES DEPENDIENTES DE PARÁMETROS.

En este apartado se resolverán distintos problemas relacionados con la con-


vergencia de series definidas en función de uno o varios parámetros. Se tra-
tará de determinar los valores que deben tomar dichos parámetros para que
la serie correspondiente sea convergente (tanto absoluta como condicional)
o divergente. El esquema que seguiremos en general es el siguiente:

207
- Aplicar el criterio del cociente o de la raı́z para obtener los valores de los
parámetros que den convergencia absoluta.
- Estudiar la convergencia de la serie que resulta al sustituir los valores de
los parámetros que hacen que el criterio anterior no sea concluyente. Para
ello podemos hacer uso de alguno de los criterios ya indicados, tanto en este
capı́tulo como en el capı́tulo 9.

PROBLEMA 14.24
X 1 · 5 · 10 . . . (n2 + 1) 1
Estudiar el carácter de la serie · según
(2n − 1)! a2n
los diferentes valores de a.

Solución Si aplicamos el criterio del cociente, tenemos:



1·5·10...(n2 +1)[(n+1)2 +1] 1
· a2n+2 (n + 1)2 + 1 1

an+1 (2n+1)! 1
lı́m = lı́m 2 +1)
= lı́m · 2 = .
an 1·5·10...(n
· 1
2n
(2n + 1) · 2n a 4|a|2
(2n−1)! a

1
La serie es absolutamente convergente cuando < 1, es decir cuando
4|a|2
1 1
|a| > y divergente cuando |a| < .
2 2
1
Cuando a = , aplicamos el criterio de Raabe y resulta:
2
(n + 1)2 + 1
   
an+1 −6n − 8 6
lı́m n 1 − = lı́m n 1 − · 4 = lı́m n· 2 = − < 1,
an (2n + 1) · 2n 4n + 2n 4

de modo que la serie es divergente.


1
Cuando a = − , la serie coincide con la anterior de modo que también es
2
divergente.

PROBLEMA 14.25
X an nπ
Estudiar el carácter de la serie √ sen según los dife-
3n − 2 2
n≥1
rentes valores de a.

208
Solución
(
nπ 0 si n = 2k es par
Como sen = si aplicamos el criterio
2 (−1)k+1 si n = 2k − 1 es impar,
de la raı́z, resulta:
p
n |a|
lı́m sup |an | = lı́m p
n
√ = |a|,
3n − 2

de modo que la serie converge absolutamente si |a| < 1 y diverge si |a| >
1.
Cuando a = 1, sustituyendo los valores de sen nπ/2 antes indicados, tenemos
la serie
X 1 nπ X 1 X (−1)k+1
√ · sen = p · (−1)k+1 = √ ,
3n − 2 2 3(2k − 1) − 2 6k − 5
n≥1 k≥1 k≥1

que es una serie alternada condicionalmente convergente (ver problema 14.7).


Cuando a = −1, resulta la serie
X (−1)n nπ X (−1)2k−1 X (−1)k
√ · sen = √ · (−1)k+1 = √ ,
3n − 2 2 6k − 5 6k − 5
n≥1 k≥1 k≥1

que es también condicionalmente convergente.

PROBLEMA 14.26
X en
Estudiar la convergencia de la serie (−1)n según los dife-
nena
n≥1
rentes valores de a.

Solución
Por el criterio de la raı́z,
p e
lı́m n
|an | = lı́m √ = e1−a .
n
n · ea

La serie es absolutamente convergente cuando e1−a < 1, es decir a > 1 y


divergente cuando a < 1.
X (−1)n
Cuando a = 1, queda la serie que es condicionalmente convergen-
n
te.

209
PROBLEMA 14.27
X 1
Estudiar el carácter de la serie según los diferentes
1 + a2n
valores de a.

Solución
La serie es de términos positivos, por lo que podemos aplicar el criterio de
1 X 1
2n
X 1
comparación. Como lı́m 1+a1 = 1, las series 2 n
y tienen
(a2 )n
(a ) 1 + a2n
X 1
el mismo carácter. Ahora bien, la serie es convergente cuando
(a2 )n
a2 > 1, es decir |a| > 1, y divergente cuando |a| < 1. De aquı́ se deduce que
la serie dada es también convergente cuando |a| > 1 y divergente cuando
|a| < 1.
P
Cuando |a| = 1, queda la serie 1/2 que es claramente divergente.

PROBLEMA 14.28
X nan
Estudiar el carácter de la serie según los diferentes valores
en
de a.

Solución
Por el criterio del cociente:
(n+1)an+1
an+1
= lı́m en+1
|a|(n + 1) |a|
lı́m = lı́m = .

na n
an en
en e

Resulta que la serie es absolutamente convergente cuando |a| < e y diver-


gente cuando |a| > e.
P
Cuando Pa = e, la serie queda n que es divergente y cuando a = −e, la
serie es (−1)n n que también es divergente.

PROBLEMA 14.29
X
Estudiar el carácter de la serie an na según los diferentes valores
de a.

210
Solución
Por el criterio del cociente,
n+1 a

an+1 a (n + 1)
lı́m = lı́m = |a|.
an an · na

La serie es absolutamente convergente cuando |a| < 1 y divergente cuando


|a| > 1.
P
Cuando a = 1 obtenemos la serie n que es divergente; cuando a = −1,
X (−1)n
la serie es que, como sabemos, es condicionalmente convergente.
n

PROBLEMA 14.30
X n
1
Estudiar el carácter de la serie a+ según los diferentes
n
valores de a.

Solución
Debido al criterio de la raı́z tenemos:

p
n
1
lı́m |an | = lı́m a + = |a|.
n

La serie es absolutamente convergente cuando |a| < 1 y divergente cuando


|a| > 1.
X 1 n 1 n
  
Cuando a = 1, tenemos la serie 1+ . Como lı́m 1 + = e 6= 0,
n n
dicha serie es divergente.
X 1 n

Cuando a = −1, la serie es −1 + que también es divergente debido
 n n
1
a que lı́m −1 + no existe.
n

PROBLEMA 14.31

X an n + 1
Estudiar el carácter de la serie según los diferentes
2n (n + 2)
valores de a.

211
Solución
Aplicamos también en este caso el criterio del cociente. Tenemos ası́:
n+1 √


a n+2
an+1 2n+1 (n+3) |a|(n + 2) n + 2 |a|
lı́m
= lı́m n √
= lı́m √ = .
an a n+1
2n (n+2)
2(n + 3) n + 1 2

La serie es pues absolutamente convergente cuando |a| < 2 y divergente


cuando |a| > 2.

X n+1
Si a = 2, la serie es que es divergente, como se comprueba al
n+2
X 1
aplicar el criterio de comparación con √ .
n

X n+1
Si a = −2, la serie es ahora (−1)n : dicha serie es condicionalmente
n+2
convergente√pues, según el criterio de Leibnitz, la sucesión de término ge-
n+1
neral an = es decreciente y converge a cero pero la serie de valores
n+2
absolutos, como ya hemos indicado, es divergente.

PROBLEMA 14.32
X (n2 + 1)an
Estudiar el carácter de la serie según los diferentes
(n + 1)!
valores de a.

Solución
Por el criterio del cociente,

[(n+1)2 +1]an+1 2
an+1 (n+2)! = lı́m (n + 2n + 2)|a| = 0.

lı́m = lı́m 2 n
an (n +1)a (n + 2)(n2 + 1)
(n+1)!

La serie es pues absolutamente convergente para cualquier valor del paráme-


tro a.

PROBLEMA 14.33
X
Estudiar el carácter de la serie (a/n)n según los diferentes va-
lores de a.

212
Solución
Aplicando el criterio de la raı́z, resulta:
p
lı́m n |an | = lı́m |a/n| = 0.

Esto indica que la serie es siempre absolutamente convergente.

PROBLEMA 14.34
X n2 + 1
Estudiar el carácter de la serie según los diferentes va-
nan
lores de a.

Solución
Si aplicamos el criterio del cociente, tenemos:

(n+1)2 +1 2
= lı́m (n + 2n + 2) · n = 1 .
an+1 (n+1)an+1
lı́m = lı́m 2
an n +1
n
(n + 1)(n2 + 1) · |a| |a|
n·a

De aquı́ se deduce que la serie es absolutamente convergente cuando |a| > 1


y divergente cuando |a| < 1. En los casos extremos tenemos:
X n2 + 1
- Si a = 1, queda la serie que es divergente porque el término
n
general no tiende a cero.
X n2 + 1
- Si a = −1, la serie es (−1)n que también es divergente por la
n
misma razón que en el caso anterior.

PROBLEMA 14.35
X an
Estudiar el carácter de la serie según los diferentes valores
n!
de a.

Solución
Por el criterio del cociente,
n+1
a
= lı́m n = lı́m |a| = 0,
an+1 (n+1)!
lı́m a
an n! n+1

por lo que la serie es absolutamente convergente para cualquier a ∈ R.

213
PROBLEMA 14.36
X n!
Estudiar el carácter de la serie según
(2 + a)(2 + 2a) . . . (2 + na)
los diferentes valores de a.

Solución
De la definición se observa que la serie no tiene sentido cuando a = −2/n, ∀n ∈
N. Para el resto de valores de a utilizamos el criterio del cociente y obtene-
mos:

(n+1)!
an+1 n+1 = 1 .
(2+a)(2+2a)...[2+(n+1)a]
lı́m
= lı́m
n!
= lı́m
an
(2+a)(2+2a)...(2+na)
2 + (n + 1)a |a|

Resulta entonces que la serie es absolutamente convergente cuando |a| > 1


y divergente cuando |a| < 1.
Con respecto a los valores extremos, para a = −1, como hemos indicado, la
serie no tiene sentido, y para a = 1 queda la serie
X n! X 2 · n! X 2
= = .
3 · 4 . . . (n + 2) (n + 2)! (n + 2)(n + 1)
Esta serie es convergente como se comprueba al aplicar el criterio de com-
X 1
paración con .
n2

PROBLEMA 14.37
X 2n
Estudiar el carácter de la serie sen2n a según los diferentes
n2
valores de a.

Solución
Observamos que se trata de una serie de términos no negativos por lo que
no hay distinción entre convergencia y convergencia absoluta. Si aplicamos
el criterio de la raı́z, resulta:
√ 2
lı́m n
an = lı́m √
n
· sen2 a = 2 sen2 a.
n2
La serie es pues absolutamente convergente cuando sen2 a < 1/2, es de-
√ (4n − 1)π (4n + 1)π
cir | sen a| < 2/2. Esto ocurre cuando <a< , n ∈ Z.
4 4
Además, en los extremos de cada intervalo, es decir en los puntos en que

214
X 1
sen2 a = 1/2, la serie queda de la forma que, como sabemos, es con-
n2
vergente.
En el resto de valores de a la serie es divergente.

PROBLEMA 14.38
X n!
Estudiar el carácter de la serie según
(a + b)(a + 2b) . . . (a + nb)
los diferentes valores de a y b, con a, b > 0.

Solución
Tenemos en este caso una serie de términos no negativos.
Si aplicamos el criterio del cociente, resulta:
(n+1)!
an+1 (a+b)(a+2b)...(a+nb)[a+(n+1)b] n+1 1
lı́m = lı́m n!
= lı́m = .
an (a+b)(a+2b)...(a+nb)
a + (n + 1)b b

La serie es pues convergente cuando b > 1 y divergente cuando b < 1.


X n!
Cuando b = 1 tenemos la serie . Para estudiar
(a + 1)(a + 2) . . . (a + n)
su convergencia aplicamos el criterio de Raabe:
   
an+1 n+1 an
lı́m n 1 − = lı́m n · 1 − = lı́m = a.
an a+n+1 a+n+1

Ası́ pues, si a < 1, la serie es divergente y si a > 1, convergente.


X n! X 1
Por último, si a = b = 1, tenemos la serie = que
(n + 1)! n+1
sabemos es divergente.

PROBLEMA 14.39
X a(a + 1) . . . (a + n − 1)
Estudiar el carácter de la serie según los
n! nb
diferentes valores de a y b, con a 6= b.

Solución
Podemos suponer que se trata de una serie de términos no negativos porque,
desde un cierto N en adelante, a + n − 1 > 0, ∀n ≥ N y el numerador no
cambia de signo.

215
Si aplicamos el criterio del cociente, obtenemos:

a(a+1)...(a+n−1)(a+n)
n + a  n b

an+1 (n+1)!(n+1) b
lı́m a(a+1)...(a+n−1) = lı́m n + 1 · n + 1 = 1.
= lı́m
an
n!nb

Como no podemos decidir la convergencia de la serie con este criterio, apli-


camos el criterio de Raabe:

(n + 1)b+1 − (n + a) · nb
 
an+1
lı́m n · 1 − = lı́m n ·
an (n + 1)b+1
n b+1 + (b + 1)nb + · · · − nb+1 − anb
= lı́m n · = b + 1 − a.
(n + 1)b+1

Cuando b + 1 − a > 1, o bien b > a, la serie será convergente, y divergente


cuando b < a.

PROBLEMA 14.40
X a(a + 1) . . . (a + n − 1)
Estudiar el carácter de la serie según los
b(b + 1) . . . (b + n − 1)
diferentes valores de a y b.

Solución
En primer lugar observamos que debe ser b 6= 0, −1, −2, . . . para que el
denominador no se anule.
Si aplicamos el criterio del cociente, tenemos:

a(a+1)...(a+n−1)(a+n)
an+1 b(b+1)...(b+n−1)(b+n) a + n
lı́m
= lı́m a(a+1)...(a+n−1) = lı́m
= 1,
an b(b+1)...(b+n−1) b + n

por lo que este criterio no es concluyente.


Aplicamos pues el criterio de Raabe:
 
= lı́m n · b + n − (a + n) = b − a.
an+1
lı́m n 1 −
an b+n

Se deduce que la serie es absolutamente convergente cuando b > a + 1 y


divergente cuando b < a + 1.
X a
Cuando b = a + 1, queda la serie que es divergente.
a+n

216
PROBLEMA 14.41
X an
Estudiar el carácter de la serie según los diferentes valores
nb
de a y b.

Solución
Aplicando el criterio del cociente, tenemos:
n+1
a  b
an+1 (n+1)b n
lı́m = lı́m n = lı́m a · = |a|.

an a n + 1
nb

La serie será pues absolutamente convergente cuando |a| < 1 y divergente


cuando |a| > 1.
X 1
Si a = 1, queda la serie (serie de Riemann), que sabemos es conver-
nb
gente cuando b > 1 y divergente cuando b ≤ 1.
X (−1)n
En el caso a = −1, la serie es de la forma ; dicha serie es absoluta-
nb
mente convergente cuando b > 1 (por ser convergente la serie de sus valores
absolutos), es condicionalmente convergente cuando 0 < b ≤ 1 (pues, según
el criterio de Leibnitz, el término general en valor absoluto forma una suce-
sión decreciente y convergente a cero), y es divergente cuando b ≤ 0 porque
el término general no tiende a cero.

PROBLEMA 14.42
Probar que la sucesión {an } de término general

an = (1 − 1/4)(1 − 1/9) . . . (1 − 1/n2 )

es convergente y que su lı́mite es estrictamente positivo.

Solución
Si llamamos bn al logaritmo del término general, obtenemos:
     
1 1 1
bn = ln an = ln 1 − + ln 1 − + · · · + ln 1 − 2 .
4 9 n

Esto quiere decir que bn es el término general de la sucesión de sumas par-


  ∞  
1 X 1
ciales de ln 1 − 2 , con lo que lı́m bn = ln 1 − 2 .
n n
n=2

217
Debido a la igualdad
 
1
ln 1 − 2 = ln(n2 − 1) − ln n2 = ln(n − 1) − 2 ln n + ln(n + 1),
n
tenemos:

bn = ln 1 − 2 ln 2 + ln 3
+ ln 2 − 2 ln 3 + ln 4
+ ln 3 − 2 ln 4 + ln 5
..
.
+ ln(n − 1) − 2 ln n + ln(n + 1)
n+1
= − ln 2 − ln n + ln(n + 1) = − ln 2 + ln .
n

Esto implica que lı́m bn = − ln 2 = ln 1/2 y, como bn = ln an , resulta en


definitiva que lı́m an = 1/2.

218
C. EJERCICIOS PROPUESTOS.

1. Contestar razonadamente si cada uno de los siguientes enuncia-


dos es verdadero o falso:
P
a) Si A es la suma de la serie an , entonces la sucesión
n≥1
(an )n∈N converge a A.
Resp.: Falso si A 6= 0 pues an → 0.

P P
b) Si A es la suma de la serie an , entonces la serie |an |
n≥1 n≥1
converge a |A|.
(−1)n
Resp.: Falso (ejemplo an = ).
n

an+1 P
c) Si lı́m = −2, entonces an converge.
n→∞ an n≥1

Resp.: Falso (ejemplo an = (−2)n ).

an+1 P
d) Si lı́m < 1, entonces an converge.
n→∞ an n≥1

Resp.: Falso (mismo ejemplo anterior).

P
e) Si an converge, entonces la sucesión (an+1 /an )n∈N tiene
n≥1
lı́mite.
1 1
Resp.: Falso (ejemplo a2n = , a2n+1 = n ).
2n 2

an converge, entonces lı́m a2n = 0.


P
f) Si
n≥1 n→∞

Resp.: Verdadero por el criterio del resto.

a2n converge.
P P
g) Si an converge, entonces
n≥1 n≥1

(−1)n
Resp.: Falso (ejemplo an = √ ).
n

219
P P 2
h) Si an converge, entonces an converge.
n≥1 n≥1

Resp.: Falso (mismo ejemplo anterior).

P X a2n
i) Si an converge absolutamente, también lo hace .
n≥1 1 + a2n
n≥1

a2n
Resp.: Verdadero pues < a2n < |an |, desde un cierto n (re-
1 + a2n
cordemos que an → 0).

X xn
j) Si {xn } es una sucesión positiva, la serie es con-
1 + n2 xn
vergente.
1/n2 ).
P
Resp.: Verdadero (aplicar el criterio de comparación con

P P P
k) Si an y bn son divergentes, entonces an bn es diver-
n≥1 n≥1 n≥1
gente.
Resp.: Falso (ejemplo an = 1/n y bn = 1/n).

l) Si lı́m an = 0 y el signo de an es alternativamente positivo y


n→∞ P
negativo, entonces an converge.
n≥1

2 + (−1)n
Resp.: Falso (ejemplo an = (−1)n · ).
n
P
m) Si an < 1/n para todo n, entonces an diverge.
n≥1

Resp.: Falso (ejemplo an = −1/n2 ).

n) Si an < 1/n2 para todo n, entonces


P
an converge.
n≥1

Resp.: Falso (ejemplo an = −1/n).

P
2. Probar que, si la serie an es absolutamente convergente, tam-
Xn+1
bién lo es la serie an .
n
Sugerencia: Aplicar el criterio de comparación.

220
∞  
X 1 1 1
3. Estudiar el carácter de la serie (−1)n+1 1+ + ··· + n .
n+1 2 2
n=1

Resp.: Convergente (aplicar el criterio de Leibnitz).


X (n14 + 5) ln(n2 + 2)
4. Estudiar la convergencia de la serie (−1)n .
en (n4 + 2)
n=1

Resp.: Absolutamente convergente (aplicar el criterio del cociente).

X ln n
5. Estudiar el carácter de la serie (−1)n.
2n
Resp.: Absolutamente convergente (criterio del cociente).


X
n n
6. Estudiar el carácter de la serie (−1) .
n−1
Resp.: Condicionalmente
P √ convergente (criterios de Leibnitz y compa-
ración con 1/ n).

X 1
7. Estudiar el carácter de la serie (−1)n √ .
n + (−1)n
Resp.: Condicionalmente
P √ convergente (criterios de Leibnitz y de com-
paración con 1/ n).

X  p 
3
8. Estudiar el carácter de la serie (−1)n n − n3 − n .

Resp.: Converge condicionalmente


P (usar el criterio de Leibnitz y el de
comparación con 1/n).

9. Estudiar
 la2 convergencia (absoluta y condicional) de la serie
X πn
sen . Resp.: Divergente (se trata de la serie 1+0+1+0+. . . ).
2
n≥1

X √n · an
10. Estudiar el carácter de la serie según los distintos
(n + 1) 2n
n≥1
valores de a ∈ R.
Resp.: Absolutamente convergente si |a| < 2; condicionalmente con-
vergente si a = −2; divergente si a = 2 ó |a| > 2.

221
X (a − 1)n
11. Estudiar el carácter de la serie según los valores de
n(n + 1)
a ∈ R.
Resp.: Absolutamente convergente cuando a ∈ [0, 2]; diverge en el
resto.

X an
12. Estudiar el carácter de la serie √ √ .
n n + 1 + (n + 1) n
Resp.: Absolutamente convergente cuando a ∈ [−1, 1]; diverge en el
resto.

X (a − 5)n
13. Estudiar el carácter de la serie según los valores
(2n + 1) · 5n
de a ∈ R.
Resp.: Converge absolutamente cuando a ∈ (0, 10); converge condicio-
nalmente cuando a = 0; diverge en el resto.

X  a(a + n) n
14. Estudiar el carácter de la serie , con a ∈ R.
n
Resp.: Converge absolutamente cuando a ∈ (−1, 1); diverge en el resto.

X
15. Estudiar el carácter de la serie n3 an según los diferentes va-
lores de a. Resp.: Absolutamente convergente cuando |a| < 1; diver-
gente en el resto.


X n
16. Calcular la suma de la serie (−1)n .
5n
n=1

Resp.: S = 5/36.

X n2
17. Calcular la suma de la serie (−1)n+1 .
5n
n≥1

Resp.: S = 5/54.

222
CAPÍTULO XV.
SUCESIONES Y SERIES
DE FUNCIONES

SECCIONES
A. Campo de convergencia. Convergencia uniforme.
B. Series de potencias. Intervalos de convergencia.
C. Desarrollo de funciones en series de potencias.
D. Aplicaciones al cálculo infinitesimal.
E. Ejercicios propuestos.

223
A. CAMPO DE CONVERGENCIA. CONVERGENCIA UNIFOR-
ME.

Consideramos en este capı́tulo sucesiones {fn } cuyos términos son funciones


reales con dominio I común. Para cada x ∈ I, se construye la sucesión
numérica {fn (x)} formada por las imágenes dePlas funciones en el punto x.
Análogamente, se define la serie de funciones fn como la sucesión {Sn }
n≥1
n
P
de sumas parciales Sn = fk .
k=1

En lo que sigue nos referiremos a series de funciones pues, aunque son un


caso particular de las sucesiones, nuestro interés se centra en el estudio de
las series de potencias (sección B) y el desarrollo de funciones en series de
potencias (sección C).
P
Definimos campo de convergencia de la serie fn como el conjunto S de
Pn≥1
puntos x ∈ I para los que la serie numérica fn (x) converge. Ası́ pues, si
P n≥1P
f (x) = fn (x), con x ∈ S, se dice que la serie fn converge puntualmen-
n≥1 n≥1
n
P
te a f . Como sabemos, esto significa que, llamando Sn (x) = fk (x),
k=1

X
∀x ∈ S, ∀ε > 0, ∃N ∈ N : |Sn (x) − f (x)| = fk (x) < ε, ∀n > N,


k>n

donde N depende de ε y de x. Si dicho N es el mismo


P para todos los valores de
x ∈ S (no depende de x), se dice que la serie fn converge uniformemente
n≥1
a f en S.
De la definición es evidente la siguiente propiedad:
P
1) Si una serie de funciones fn converge uniformemente a f , entonces
n≥1
converge puntualmente a f .
Otras propiedades de interés son las siguientes:
P
2) Criterio de convergencia de Cauchy. La serie fn converge uni-
n≥1
formemente en S si y sólo si
k+p
X
∀x ∈ S, ∀ε > 0, ∃N ∈ N : fn (x) < ε, ∀k > N, p ∈ N.


n=k+1

224
P
3) Continuidad. Si una serie de funciones fn converge uniformemente
n≥1
a f en S y cada fn es continua en x0 ∈ S, entonces f es continua en
x0 .
En sı́mbolos, X X
lı́m fn (x) = lı́m fn (x).
x→x0 x→x0
n≥1 n≥1

4) Derivación. Sea {f
Pn } una sucesión de funciones derivables en (a, b) y
tal que la serie fn (x0 ) converge para algún x0 ∈ (a, b). Si la se-
P 0 n≥1 P
rie fn converge uniformemente en (a, b), entonces fn converge
n≥1 n≥1
uniformemente en (a, b) y
X 0 X
fn (x) = fn0 (x), ∀x ∈ (a, b).
n≥1 n≥1

P
5) Integración. Si una serie de funciones fn converge uniformemente
n≥1
a f en un intervalo [a, b] y cada fn es integrable en [a, b], entonces f
es integrable en [a, b] y
XZ x Z xX
fn (t) dt = fn (t) dt, ∀x ∈ [a, b].
n≥1 a a n≥1

Esto se expresa diciendo que una serie uniformemente convergente se


puede integrar término a término. Un método usual para probar que
una serie es convergente es el siguiente.
P
6) Criterio de Weierstrass. Sea fn una serie de funciones tal que
n≥1 P
|fn (x)| ≤ an , ∀n, ∀x ∈ S, donde an es una serie numérica con-
P n≥1
vergente. Entonces fn converge uniformemente en S.
n≥1

Observación. El criterio de Weierstrass asegura la convergencia uniforme


y absoluta de una serie de funciones, pero en general ambos conceptos no
son equivalentes.

PROBLEMA 15.1
Determinar el campo de convergencia de la serie
X
2n senn x.
n≥0

225
Solución
Aplicando el criterio de la raı́z, la serie es absolutamente convergente cuan-
do:
p
lı́m n |an | < 1 ⇐⇒ lı́m 2| sen x| < 1 ⇐⇒ | sen x| < 1/2
 
(6n − 1)π (6n + 1)π
⇐⇒ x ∈ , , n ∈ Z,
6 6

que son los intervalos donde la serie es absolutamente convergente.


En los extremos de cada intervalo, es decir cuando P | senP
x| = 1/2, donde
el criterio de la raı́z no decide, quedan las series 1 ó (−1)n , que son
claramente divergentes.

PROBLEMA 15.2

X cos nx
Hallar el campo de convergencia de la serie .
enx
n=1

Solución
Descomponemos el problema en varios casos:
- Si x > 0, aplicamos el criterio de comparación; tenemos por un lado que
cos nx 1 X
nx ≤ nx y, aplicando el criterio de la raı́z a la serie 1/enx , resul-
e e
ta: p
lı́m n 1/enx = lı́m 1/ex < 1 pues x > 0.
Como la serie mayorante es convergente, también lo será la serie dada.
P
- Si x = 0, tenemos la serie 1 que es divergente.
cos nx
- Si x < 0, como lı́m enx = 0, entonces no existe lı́m nx , con lo que la
e
serie es también divergente.

PROBLEMA 15.3
Estudiar la convergencia y convergencia uniforme de la serie
X x
en R.
(1 + x)n
n≥1

226
Solución
x
Cuando |1 + x| < 1, es decir −2 < x < 0, tenemos que → ∞.
(1 + x)n
Por el criterio del resto se deduce que la serie no es convergente en R. En
particular, tampoco converge uniformemente.

PROBLEMA 15.4
Estudiar la convergencia y convergencia uniforme de la serie
X
enx sen nx en R.
n≥1

Solución
Cuando x > 0, el término general enx sen nx no tiene lı́mite. Por el criterio
del resto se deduce que la serie no converge en R.

PROBLEMA 15.5
Estudiar la convergencia y convergencia uniforme de la serie
X sen √nx
√ en R.
n n
n≥1

Solución
Aplicaremos el criterio de Weierstrass. Como

sen nx 1
√ ≤ √
n n n n , ∀n

X 1 X 1
y la serie √ = es convergente, se deduce que la serie pro-
n n n3/2
puesta converge absoluta y uniformemente en R.

PROBLEMA 15.6
Estudiar la convergencia y convergencia uniforme de la serie
X
(−1)n−1 xn en [−1/2, 1/2].
n≥1

227
Solución
La serie converge absoluta y uniformemente debido al criterio de Weierstrass
porque, si −1/2 ≤ x ≤ 1/2,
1
|(−1)n−1 xn | ≤ , ∀n
2n
1/2n es convergente.
P
y la serie geométrica

PROBLEMA 15.7
Estudiar la convergencia y convergencia uniforme de la serie
X 2x
arc tg 2 en R.
x + n3
n≥1

Solución
Sea N una constante positiva fija. Teniendo en cuenta que | arc tg x| ≤
|x|, ∀x ∈ R, obtenemos las siguientes acotaciones:

2x 2x 2N
- Si |x| ≤ N , arc tg 2
≤ ≤ 3 , ∀n.
x + n3 x2 + n3 n

2x 2x 2
- Si |x| > N , arc tg 2 3
≤ 2 3
≤ 2 , ∀n > |x|.
x +n x +n n
X 2N X 2
Como las dos series y son convergentes, del criterio de Weiers-
n3 n2
trass se deduce que la serie propuesta es absoluta y uniformemente conver-
gente.

PROBLEMA 15.8
X xn sen nx
Probar que la serie converge uniformemente en [−1, 1]
np
n≥1
si p > 1.

Solución
En efecto, por el criterio de Weierstrass, si p > 1 y −1 ≤ x ≤ 1, tenemos la
acotación n
x sen nx
≤ 1
n p np
X 1
y la serie mayorante es convergente.
np

228
PROBLEMA 15.9
X x2 + n
Probar que la serie (−1)n converge uniformemente en
n2
n≥1
todo [a, b] pero nunca converge absolutamente.

Solución
x2 + n
Si llamamos fn (x) = (−1)n , por el criterio de comparación, como
P n2
|fn (x)| ∼ 1/n y la serie 1/n es divergente, la serie propuesta no es ab-
solutamente convergente. Sin embargo, aplicando el criterio de Leibnitz, se
prueba que converge condicionalmente en R.
Por otra parte, al ser una serie alternada, si llamamos α = máx{|a|, |b|},
tenemos:
x2 + (n + 1) α2 + (n + 1)
|Sn (x) − S(x)| ≤ |fn+1 (x)| = 2
≤ → 0, ∀x ∈ [a, b],
(n + 1) (n + 1)2

lo que indica que la serie converge uniformemente.

PROBLEMA 15.10
X∞
Dada la serie fn (x), donde fn (x) son continuas en [0, 1] para
n=1
n
X
2 n
todo n y verifican la acotación fk (x) − x ≤ 2 , ∀x ∈ [0, 1],

n +5
k=1
∞ Z 1
X
calcular fn (x) dx.
n=1 0

Solución
n
Como → 0 cuando n → ∞ independientemente de x ∈ [0, 1], la
n2 + 5

X
acotación dada indica que la serie fn (x) converge uniformemente a la
n=1
función y = x2 . En consecuencia la serie se puede integrar término a término
y resulta:
∞ Z 1 Z ∞
1X Z 1
X 1
fn (x) dx = fn (x) dx = x2 dx = .
3
n=1 0 0 n=1 0

229
PROBLEMA 15.11
X sen nx
Probar que la serie es convergente en todo R. Si f (x) es
n2
n≥1
su suma, probar que f es continua en [0, π] y que
Z π X 1
f (x) dx = 2 .
0 (2n − 1)3
n≥1

Solución
sen nx 1 X 1
Si llamamos fn (x) = 2
, como |fn (x)| ≤ 2 , ∀n, ∀x ∈ R y la serie
n n n2
es convergente, por el criterio de Weierstrass se deduce que la serie propuesta
es uniformemente convergente en R. Como además las funciones fn (x) son
continuas, también lo será su suma f (x).
Z π XZ π
De la fórmula f (x) dx = fn (x) dx, deducimos entonces que:
0 n≥1 0

Z π XZ π
sen nx X  − cos nx π
f (x) dx = dx =
0 n2 n3
n≥1 0 n≥1 0
X 1 − cos nπ X 2
= = ,
n3 (2n − 1)3
n≥1 n≥1

(
0 si n es par
pues 1 − cos nπ =
2 si n es impar.

B. SERIES DE POTENCIAS. INTERVALOS DE CONVERGEN-


CIA.

X
Una serie de la forma an (x − a)n = a0 + a1 (x − a) + · · · + an (x − a)n + . . .,
n≥0
con an ∈ R, ∀n, se llama serie de potencias de x − a o serie de potencias
centrada en a. Nos referiremos aquı́ a las series de potencias centradas en
el
Xorigen pues basta hacer Xuna traslación x − a = t para reducir la serie
n
an (x − a) a la serie an tn . La siguiente propiedad es básica:
n≥0 n≥0

230
X
1) El campo de convergencia de una serie de potencias an xn es un
n≥0
intervalo centrado en el origen, o todo R, o el origen.
Ası́ pues, para determinar el intervalo de convergencia, basta calcular
la distancia de los extremos del intervalo al origen, lo que llamaremos
radio de convergencia.
2) Fórmula de Hadamard. El radio de convergencia (la mitad de Xla am-
plitud del intervalo de convergencia) de una serie de potencias an xn
n≥0
p
n
es R = 1/L donde L = lı́m sup |an |.
3) Si el campo de convergencia tiene radio R, la serie converge absoluta
y uniformemente ∀x ∈ (−R, R) y diverge si |x| > R. Sin embargo, si
x = R ó x = −R, caben todas las posibilidades.
Para la mayor parte de las series de potencias que consideraremos,
el campo de convergencia puede obtenerse mediante el criterio del
cociente o de la raı́z. Tenemos entonces la siguiente propiedad:
p
4) a) Si existe lı́m n |an | = L, el radio de convergencia es R = 1/L.

an+1
b) Si existe lı́m
= L, el radio de convergencia es R = 1/L.
an
Las operaciones posibles con series de potencias se deducen de las
correspondientes con series arbitrarias. Podemos destacar las siguien-
tes:
5) En el interior de su intervalo de convergencia, toda serie de potencias
puede derivarse término a término.
Es decir, X 0 X
n
an (x − a) = n · an (x − a)n−1
n≥0 n≥1
y la serie obtenida tiene el mismo radio de convergencia que la serie
original.
6) Toda serie de potencias es integrable en su campo de convergencia y la
primitiva se obtiene integrando término a término la serie dada.
Esto se expresa simbólicamente como
Z xX X (x − a)n+1
an (t − a)n dt = an · , ∀x ∈ (a − R, a + R)
a n+1
n≥0 n≥0

y la serie resultante tiene el mismo radio de convergencia que la serie


original (aunque es posible que converja también en algún extremo del
intervalo de convergencia).

231
7) Dadas las series de potencias
X X
f (x) = an (x − a)n y g(x) = bn (x − a)n ,
n≥0 n≥0

convergentes en los intervalos (a − R1 , a + R1 ) y (a − R2 , a + R2 ),


respectivamente, el producto viene dado por la serie
X
f (x) · g(x) = cn (x − a)n , ∀x ∈ (−R, R),
n≥0

n
X
donde cn = ak · bn−k , n ≥ 0 y R = mı́n{R1 , R2 }.
k=0

PROBLEMA 15.12
X xn
Determinar el campo de convergencia de la serie .
2n · n2
n≥0

Solución
Aplicando la fórmula de Hadamard, calculamos el radio de convergencia
como:
1 p 1 1
= lı́m sup n |an | = lı́m √
n
= .
R 2· n 2 2
Por tanto, R = 2 y la serie converge absolutamente cuando x ∈ (−2, 2).
En los extremos del intervalo tenemos:
X 1
- Si x = 2, resulta la serie que es convergente.
n2
X (−1)n
- Si x = −2, resulta la serie que es también absolutamente con-
n2
vergente.

PROBLEMA 15.13
X n! xn
Determinar el campo de convergencia de la serie .
nn
n≥0

Solución
Por la fórmula de Hadamard,

n
p√
−1 · n
1 n! n · e 2πn
= e−1 ,
p
= lı́m sup n |an | = lı́m = lı́m
R n n

232
con lo que R = e y la serie converge absolutamente en (−e, e) y diverge
cuando x ∈ (−∞, −e) ∪ (e, ∞). En los extremos tenemos:
X n! en
- Si x = e, la serie es . Como
nn
n! en √
lı́m = lı́m 2πn = ∞ =
6 0,
nn
la serie es divergente.
X (−1)n n! en
- Si x = −e, la serie es que también es divergente, por la
nn
misma razón del caso anterior.

PROBLEMA 15.14
Determinar el campo de convergencia de la serie
X xn
.
n · 10n−1
n≥1

Solución
Por la fórmula de Hadamard,

1 p 1 1
= lı́m sup n |an | = lı́m √
n
= ,
R n · 10n−1 10

de donde R = 10 y la serie converge absolutamente en (−10, 10) y diverge


en (−∞, −10) ∪ (10, ∞).
X 10
- Si x = 10, la serie resulta que es divergente.
n
X (−1)n 10
- Si x = −10, tenemos la serie que es condicionalmente con-
n
vergente (basta aplicar el criterio de Leibnitz).

PROBLEMA 15.15
X xn
Determinar el campo de convergencia de la serie , donde
an + bn
n≥0
a, b > 0.

233
Solución
Supondremos que a ≥ b pues, en caso contrario, se procede de forma análoga.
Por la fórmula de Hadamard,

1 p 1 1 1
= lı́m sup n |an | = lı́m √
n n n
= lı́m p = ,
R a +b a 1 + (b/a)n
n a

con lo que R = a y la serie converge absolutamente en (−a, a) y diverge en


(−∞, −a) ∪ (a, ∞).
X an
- Cuando x = a, tenemos la serie . Como
an + bn

an 1
lı́m n n
= lı́m = 1 6= 0,
a +b 1 + (b/a)n

la serie es divergente.
- Cuando x = −a, aplicamos el mismo procedimiento anterior y la serie es
también divergente.

PROBLEMA 15.16
Determinar el campo de convergencia de la serie
X  nx n
.
n+1
n≥0

Solución
Por la fórmula de Hadamard,

1 p n
= lı́m sup n |an | = lı́m = 1 =⇒ R = 1,
R n+1

y la serie converge absolutamente en (−1, 1) y diverge en (−∞, −1)∪(1, ∞).


X  n n X  −n n
- Si x = 1, tenemos la serie y si x = −1, . En
n+1 n+1
ambos casos, si llamamos an al término general,
 n
n n −1
lı́m |an | = lı́m = lı́m en·( n+1 −1) = lı́m en· n+1 = e−1 6= 0,
n+1

de modo que ambas series son divergentes.

234
PROBLEMA 15.17
Hallar el intervalo de convergencia de la serie
3(x − 1)2 n+1 (n + 1)(x − 1)
n
(x − 1) − + · · · + (−1) + ...
22 2n

Solución

Por la fórmula de Hadamard,



n
1 p n+1 1
= lı́m sup n |an | = lı́m = =⇒ R = 2.
R 2 2

El intervalo de convergencia es entonces I = (1 − 2, 1 + 2) = (−1, 3).


P
- Para x = −1, tenemos la serie divergente −(n + 1), y para x = 3,
tenemos también la serie divergente (−1)n+1 (n + 1).
P

PROBLEMA 15.18
Determinar el campo de convergencia de la serie
X x4n−1
1+ (−1)n .
4n
n≥1

Solución

Aplicaremos el criterio del cociente considerando la serie como serie numéri-


ca.
4n+3
an+1 x /(4n + 4)
lı́m
= lı́m
= |x|4 .
an x4n−1 /4n

La serie será convergente cuando |x|4 < 1, es decir cuando |x| < 1, y diver-
gente cuando |x| > 1. En los casos extremos tenemos:
X (−1)n
- Si x = 1, la serie 1 + es condicionalmente convergente.
4n
n≥1

X (−1)n
- Si x = −1, la serie es 1 − que es también condicionalmente
4n
n≥1
convergente.

235
PROBLEMA 15.19
Determinar el campo de convergencia de la serie
X n2 +2
[cos(1/n)] n+2 xn .
n≥0

Solución
Por la fórmula de Hadamard,

1 p n2 +2
= lı́m sup n |an | = lı́m [cos(1/n)] n2 +2n = 1,
R
de modo que la serie converge absolutamente en (−1, 1) y diverge en (−∞, −1)∪
(1, ∞).
En los extremos x = 1 y x = −1 las series son divergentes porque, aplicando
el criterio del resto,
n2 +2 n2 n2 −1/n2
lı́m [cos(1/n)] n+2 = lı́m e n+2 ·[cos(1/n)−1] = lı́m e n+2 · 2 = 1 6= 0.

PROBLEMA 15.20
Determinar el campo de convergencia de la serie
n
X
n(−1) xn−1 .
n≥1

Solución
n
Aplicaremos el criterio de comparación, para lo que llamaremos an = n(−1) xn−1 .
n
Tenemos la acotación |an | = n(−1) |x|n−1 ≤ n|x|n−1 . Además la serie n|x|n−1
P
converge si |x| < 1 como se deduce aplicando el criterio del cociente:

(n + 1)|x|n
lı́m = |x|.
n|x|n−1

Lo anterior indica que la serie propuesta es también absolutamente conver-


gente cuando |x| < 1.
Ahora bien, si |x| = 1, lı́m an no existe; por tanto la serie diverge.
Por tratarse de una serie de potencias, la serie debe ser también divergente
cuando |x| > 1.

236
PROBLEMA 15.21
Determinar el campo de convergencia de la serie
X (x − 1)2n
.
n · 9n
n≥1

Solución
Aplicando el criterio de la raı́z,
p
n |x − 1|2 |x − 1|2
lı́m |an | = lı́m √ = .
9· nn 9

Esto quiere decir que la serie converge absolutamente cuando |x − 1|2 < 9,
es decir cuando x ∈ (−2, 4) y diverge cuando x ∈ (−∞, −2) ∪ (4, ∞).
P
Además, tanto para x = −2 como para x = 4, queda la serie 1/n que es
divergente.

PROBLEMA 15.22
Determinar el campo de convergencia de la serie
X (2n − 1)n (x + 1)n
.
2n−1 · nn
n≥1

Solución
Por el criterio de la raı́z tenemos:
p
n (2n − 1) · |x + 1|
lı́m |an | = lı́m = |x + 1|
2(n−1)/n · n
de modo que la serie converge absolutamente cuando |x + 1| < 1, es decir
cuando x ∈ (−2, 0) y diverge cuando x ∈ (−∞, −2) ∪ (0, ∞).
X (2n − 1)n
- Para x = 0 queda la serie . Esta serie es divergente porque
2n−1 · nn
el término general no tiende a cero:

(2n − 1)n 2n − 1 n
 
lı́m n−1 n = lı́m 2 · = 2e−1/2 .
2 ·n 2n
X (2n − 1)n
- Si x = −2, tenemos la serie alternada (−1)n · n−1 n que también
2 ·n
es divergente por la misma razón que en el caso anterior.

237
PROBLEMA 15.23
Determinar el campo de convergencia de la serie
X (x + 5)2n−1
.
2n · 4n
n≥1

Solución
Por el criterio del cociente, obtenemos:
|x+5|2n+1
|x + 5|2 · 2n |x + 5|2

an+1 2(n+1)·4n+1
lı́m = lı́m
|x+5|2n−1
= lı́m = .
an 4 · 2(n + 1) 4
2n·4n

|x + 5|2
Entonces la serie converge absolutamente cuando < 1, es decir cuan-
4
do x ∈ (−7, −3) y diverge cuando x ∈ (−∞, −7) ∪ (−3, ∞).
P
En los extremos del intervalo, x = −3 y x = −7, tenemos la serie 1/4n
que es divergente.

PROBLEMA 15.24
Determinar el campo de convergencia de la serie
X n!
xn−1 .
(a + 1) . . . (a + n)

Solución
Si aplicamos el criterio del cociente, tenemos:

(n+1)!xn
an+1
= lı́m (n + 1)|x| = |x|.
(a+1)...(a+n)(a+n+1)
lı́m
= lı́m

n!x n−1
an a+n+1
(a+1)...(a+n)

Entonces la serie es absolutamente convergente cuando |x| < 1 y divergente


cuando |x| > 1. En los extremos del intervalo de convergencia tenemos:
X n!
- Si x = 1, la serie es . Aplicando el criterio de Raabe,
(a + 1) . . . (a + n)
resulta:  
n+1 an
lı́m n · 1 − = lı́m = a,
a+n+1 a+n+1
con lo que la serie es convergente si a > 1 y divergente si a < 1. Por último,
X 1
si a = 1, la serie es ahora , que es evidentemente divergente.
n+1

238
X n!
- Si x = −1, queda la serie alternada (−1)n−1 · . Como
(a + 1) . . . (a + n)
hemos visto antes, cuando a > 1 es absolutamente convergente. Cuando a ≤
n!
1 aplicamos el criterio de Leibnitz, para lo cual llamamos an = :
(a + 1) . . . (a + n)
an+1 n+1
Como = , la sucesión {an } es decreciente si 0 < a ≤ 1 y
an a+n+1
creciente si a < 0. En el primer caso, 0 < a ≤ 1, además lı́m an = 0.
Veámoslo:
n! 1
Si llamamos L = lı́m = lı́m ,
(a + 1) . . . (a + n) (1 + a)(1 + a/2) . . . (1 + a/n)
al
tomar logaritmos obtenemos:

X
ln L = lı́m −[ln(1 + a) + ln(1 + a/2) + · · · + ln(1 + a/n)] = − ln(1 + a/n).
n=1

Esta última serie es divergente pues ln(1+a/n) ∼ 1/n, con lo que ln L = −∞,
de donde L = e−∞ = 0, como querı́amos probar.
Por último, si a = 0, tenemos la serie divergente (−1)n−1 .
P

En resumen, en el caso x = −1, la serie dada es absolutamente convergente


cuando a > 1; condicionalmente convergente cuando 0 < a ≤ 1 y divergente
cuando a ≤ 0.

PROBLEMA 15.25
Determinar el campo de convergencia de la serie
 
X
n−1 1
x ln 1 + .
n

Solución
Por el criterio del cociente,


an+1 xn · ln n+2
n+1
lı́m = lı́m = |x|.
an xn−1 · ln n+1
n

Tenemos entonces que la serie converge absolutamente cuando |x| < 1 y


diverge cuando |x| > 1. Además,
X
- Si x = 1, tenemos la serie ln(1 + 1/n) que es divergente como se com-
P
prueba al compararla con la serie armónica 1/n.

239
X
- Si x = −1, queda la serie alternada (−1)n−1 ln(1 + 1/n) que es con-
dicionalmente convergente, pues la sucesión {ln(1 + 1/n)} es decreciente y
tiene lı́mite cero.

PROBLEMA 15.26
Determinar el campo de convergencia de la serie
X (2n + 1)!
(−x/e)5n .
1 · 3 · 5 · 6 . . . (4n − 3)(3n)

Solución
Si hacemos el cambio t = (−x/e)5 y aplicamos el criterio del cociente, tene-
mos:
(2n+3)!
an+1 1·3·5·6...(4n−3)(3n)(4n+1)(3n+3) (2n + 3)(2n + 2) |t|
lı́m
= lı́m
(2n+1)!
·|t| = lı́m ·|t| = .
an (4n + 1)(3n + 3) 3
1·3·5·6...(4n−3)(3n)

De aquı́ se deduce √
que la serie converge absolutamente
√ cuando |t| < 3, o
5 5
bien cuando |x| < e 3, y diverge cuando |x| > e 3.
√ X (2n + 1)! · 3n
- Cuando x = e 5 3, la serie queda (−1)5n · .
1 · 3 · 5 · 6 . . . (4n − 3)(3n)
Esta serie es divergente porque el término general no tiende a cero. En
efecto, como

4n2 + 10n + 6

an+1 (2n + 3)(2n + 2) · 3
= = > 1,
an (4n + 1)(3n + 3) 4n2 + 5n + 1

entonces |an+1 | > |an | y lı́m |an | =


6 0.

- Cuando x = −e 5 3, procedemos de manera análoga al caso anterior. Ası́ la
serie es también divergente.

PROBLEMA 15.27
X
Si la serie an z n tiene radio de convergencia 2, encontrar los
n
X X
radios de convergencia de las series akn z n , an z k , (k > 1),
2
X
an z n .

240
Solución
p
Por hipótesis sabemos que 1/2 = lı́m sup n |an |. Aplicando también la fórmu-
la de Hadamard en los demás casos, tenemos:
k 1
q p
lı́m sup n |akn | = lı́m sup n |an | = k .
2
X
De aquı́ se deduce que la serie akn z n tiene radio de convergencia R1 =
2k .
Para el segundo caso, como
n 1 n
lı́m sup k |an | = lı́m sup |an | n · kn = (1/2)0 = 1,
p

n
X
el radio de convergencia de la serie an z k es R2 = 1.
Análogamente, como
h in·(1/n2 )
n2
p
lı́m sup |an | = lı́m sup |an |1/n = (1/2)0 = 1,
2
X
el radio de convergencia de an z n es R3 = 1.

PROBLEMA 15.28
X
Se considera la serie de potencias an xn , donde llamamos
1 · 2...n
an = .
3 · 5 . . . (2n + 1)
a) Probar que su radio de convergencia es 2.
b) Probar que la serie original no converge en x = 2.
c) Sean bn = an 2n y pn = ln bn . Probar que los términos pn son las
sumas parciales de una serie de términos negativos que diverge
hacia −∞.
d) Deducir de c) el carácter de la serie original en x = −2.

Solución
a) Por el criterio del cociente,
1·2...n·(n+1) n+1
an+1 3·5...(2n+1)(2n+3) x n+1 |x|
lı́m = lı́m
1·2...n · n = lı́m · |x| = ,
an 3·5...(2n+1)
x 2n + 3 2

de modo que la serie converge absolutamente cuando |x| < 2.

241
b) Para x = 2 aplicamos el criterio de Raabe:
   
an+1 n+1 1 1
lı́m n· 1 − = lı́m n· 1 − · 2 = lı́m n· = < 1,
an 2n + 3 2n + 3 2

por lo que la serie es divergente.


2 · 4 . . . 2n 2 4 2n
c) Si escribimos bn = an · 2n = = · ... , enton-
3 · 5 . . . (2n + 1) 3 5 2n + 1
ces
2 4 2n
pn = ln bn = ln + ln + · · · + ln
3 5 2n + 1
es una cantidad negativa por ser suma de números negativos (loga-
ritmos de números menores que uno). Además pn es la suma de los
X 2n
n primeros términos de la serie ln . Esta serie es divergen-
2n + 1
n≥1
te
P como se observa al aplicar el criterio de comparación con la serie
1/n:
2n n
ln 2n+1

2n
lı́m = lı́m ln = e−1/2 .
1/n 2n + 1
Esto quiere decir que lı́m pn = −∞, como querı́amos probar.
X
d) La serie original en x = −2 es la serie alternada (−1)n · 2n · an .
Para estudiar su convergencia aplicamos el criterio de Leibnitz. Por
el apartado c), el término general en valor absoluto tiende a cero
pues |(−1)n · 2n · an | = bn = epn → e−∞ = 0. Además la sucesión {bn }
es decreciente pues

bn+1 n+1 2n + 2
=2· = < 1.
bn 2n + 3 2n + 3

De lo anterior resulta que la serie es condicionalmente convergente (la


convergencia no es absoluta pues vimos en el apartado b) que la serie
de valores absolutos no es convergente).

C. DESARROLLO DE FUNCIONES EN SERIES DE POTEN-


CIAS.

Se plantea en esta sección el problema de saber si una función f es la su-


ma de una serie de potencias que converja en cierto intervalo centrado en

242
algún punto x = a. Es decir, queremos encontrar los coeficientes {an } para
que X
f (x) = an (x − a)n , ∀x ∈ (a − R, a + R).
n≥0

1) Una condición necesaria para que exista dicha serie es que f sea infini-
tamente derivable en un entorno de a; en este caso, los coeficientes se
f (n) (a)
obtienen por la fórmula an = (como se deduce al aplicar su-
n!
cesivas veces la propiedad 5 de la sección B). Tenemos ası́ la llamada
serie de Taylor generada por la función f en el punto x = a:
X f (n) (a)
f (x) ∼ (x − a)n ,
n!
n≥0

o, en el caso particular de a = 0, la serie de McLaurin generada por f :


X f (n) (a)
f (x) ∼ xn .
n!
n≥0

Para encontrar alguna condición suficiente que asegure la convergencia


de la serie de Taylor a la función f escribimos la siguiente fórmula de
Taylor con resto:
n
X f (k) (a) f (n+1) (c)
f (x) = (x−a)k +Rn (x, a), donde Rn (x, a) = (x−a)n+1
k! (n + 1)!
k=0

para algún c comprendido entre x y a (Rn (x, a), llamado resto de orden
n de la serie, indica el error cometido al sustituir la función f por la
suma de los n primeros términos de la serie de Taylor asociada). Es
Rn (x, a)
evidente que lı́m = 0, es decir, el resto es un infinitésimo de
x→a (x − a)n
orden superior a n en x = a.
De lo anterior se deduce que:
2) Una condición necesaria y suficiente para que la serie de Taylor converja
a f es que
f (n+1) (c)
lı́m Rn (x, a) = lı́m (x − a)n+1 = 0.
n→∞ n→∞ (n + 1)!

Muchas veces, en la práctica basta encontrar una cota superior de la


derivada de orden n + 1 de la función en un entorno de x = a. Esto da
lugar entonces a:
3) Una condición suficiente para que la serie de Taylor converja a f es
que las derivadas de cualquier orden de la función f estén acotadas en
algún entorno de a.

243
Escribiremos a continuación los desarrollos en serie de las funciones más
comunes, que servirán de base para obtener los desarrollos de otras funcio-
nes.
X xn
1. Función exponencial. ex = ,
n!
n≥0

y la serie converge en todo R.


X x2n+1
2. Funciones trigonométricas. sen x = (−1)n ,
(2n + 1)!
n≥0

que converge en todo R.


X x2n
Análogamente, cos x = (−1)n
(2n)!
n≥0

y converge también en todo R (se puede obtener como derivada de sen x).
X xn
3. Función logarı́tmica. ln(x + 1) = (−1)n−1
n
n≥1

y la serie converge absolutamente en (−1, 1) y condicionalmente en x = 1.

X m
m
4. Serie binómica. (1 + x) = xn ,
n
n≥0
 
m m(m − 1) . . . (m − n + 1)
(donde definimos = , para todo m ∈ R y
n n!
n ∈ N) y la serie es absolutamente convergente en (−1, 1); para ciertos
valores de m la serie también converge en algún extremo del intervalo.

En los siguientes problemas veremos la forma de obtener desarrollos en se-


rie de funciones que se obtienen mediante operaciones algebraicas de las
anteriores.

PROBLEMA 15.29
Desarrollar en serie de McLaurin la función f (x) = (1 + x)e−x y
determinar su intervalo de convergencia.

244
Solución
X (−x)n
Como e−x = , para todo x ∈ R, entonces
n!
n≥0
X (−x)n X (−x)n X (−1)n X (−1)n
(1 + x)e−x = +x = −1 + xn + xn+1
n! n! n! n!
n≥0 n≥0 n≥1 n≥0
X n
(−1) n X (−1)m−1
m
= −1 + x + x
n!(m − 1)!
n≥1 m≥1
 
X
n n 1 1 X 1−n
= −1 + (−1) x − = −1 + (−1)n xn ,
n! (n − 1)! n!
n≥1 n≥1

y el desarrollo es también válido en todo R.

PROBLEMA 15.30
x
Desarrollar la función f (x) = x + √ en serie de potencias
1 + x2
alrededor del origen especificando su intervalo de convergencia.

Solución
X −1/2
2 −1/2
Utilizaremos el desarrollo en serie binómica (1 + x ) = (x2 )n ,
n
 n≥0
−1/2 1 · 3 . . . (2n − 1)
válido cuando |x| < 1; teniendo en cuenta que = (−1)n · ,
n n! · 2n
resulta:
X 1 · 3 . . . (2n − 1) 2n+1
x + x(1 + x2 )−1/2 = x + (−1)n · x
n! · 2n
n≥0
X 1 · 3 . . . (2n − 1) 2n+1
= 2x + (−1)n · x ,
n! · 2n
n≥1

y el desarrollo es igualmente válido cuando |x| < 1 (observar también que la


serie converge condicionalmente cuando x = ±1 procediendo como se hizo
en el problema 15.24).

PROBLEMA 15.31
(
ex −1
x si x 6= 0
Desarrollar la función f (x) = en serie de poten-
1 si x = 0
cias alrededor del origen.

245
Solución
X xn
A partir del desarrollo ex = , obtenemos:
n!
n≥0

X xn ex − 1 X xn−1
ex − 1 = =⇒ =
n! x n!
n≥1 n≥1

y el desarrollo es válido en todo R por serlo el desarrollo de ex .

PROBLEMA 15.32
Obtener el desarrollo en serie de potencias de x de la función
f (x) = (1 + x2 ) arc tg x, especificando su intervalo de convergencia.

Solución

Calculando la derivada de la función y = arc tg x, tenemos el desarro-


llo:
1 X −1 X
0
y = 2
= (x2 )n = (−1)n x2n .
1+x n
n≥0 n≥0

Si integramos ahora término a término, para x ∈ (−1, 1):

X x2n+1
y= (−1)n + C, con C = y(0) = 0.
2n + 1
n≥0

Multiplicando ahora por 1 + x2 , obtenemos en definitiva:

X x2n+1 X x2n+3 X x2n+1


f (x) = (−1)n + (−1)n = (−1)n
2n + 1 2n + 1 2n + 1
n≥0 n≥0 n≥0
2m+1
 
m−1 x 1 1
X X
n−1 2n+1
+ (−1) =x+ (−1) x − +
2m − 1 2n + 1 2n − 1
m≥1 n≥1
X 2
= x+ (−1)n−1 x2n+1 ,
(2n + 1)(2n − 1)
n≥1

y el desarrollo es válido cuando |x| < 1 pues corresponde al intervalo donde es


válido el desarrollo de (1+x2 )−1 (en este caso se puede comprobar fácilmente
que también es convergente cuando x = ±1).

246
PROBLEMA 15.33
Desarrollar en serie de potencias alrededor de x = 0 la función
x
f (x) = especificando su intervalo de convergencia. Escribir
1 + x3 Z x
el desarrollo de la función F (x) = f (t)dt.
0

Solución
A partir del desarrollo de (1 + x3 )−1 resulta:
X −1 X
3 −1
x(1 + x ) = x x3n = (−1)n x3n+1 ,
n
n≥0 n≥0

y la serie converge absolutamente a la función cuando x ∈ (−1, 1).


Como en dicho intervalo la convergencia es absoluta y uniforme, enton-
ces
Z x XZ x
F (x) = f (t)dt = (−1)n t3n+1 dt
0 n≥0 0
3n+2
x
x3n+2

X t X
= (−1)n = (−1)n .
3n + 2 0 3n + 2
n≥0 n≥0

Ahora la serie obtenida converge también (aunque sólo condicionalmente)


cuando x = −1.

PROBLEMA 15.34
Desarrollar la función f (x) = sen2 x en serie de McLaurin.

Solución
1 − cos 2x
Debido a la fórmula sen2 x = y a partir del desarrollo del coseno,
2
el desarrollo de la función dada es:

1 1X (2x)2n X 22n−1 · x2n


f (x) = − (−1)n = (−1)n−1 .
2 2 (2n)! (2n)!
n≥0 n≥1

El intervalo de convergencia coincide pues con el de la serie correspondiente


a cos 2x, es decir todo R.

247
PROBLEMA 15.35
ln(1 + x)
Desarrollar la función f (x) = en serie de McLaurin.
1+x

Solución
Debemos multiplicar las series correspondientes a las funciones y = ln(1+x),
y = (1 + x)−1 . Tenemos pues:
   
X X xn
f (x) =  (−1)n xn  ·  (−1)n−1 .
n
n≥0 n≥1

Para calcular el coeficiente del término general de la serie producto hace-


mos:
n n n
X X 1 X1
pn = an−k · bk = (−1)n−k · (−1)k−1 · = (−1)n−1 , ∀n ≥ 1.
k k
k=0 k=1 k=1

En definitiva, tenemos:
n
!
X X
n−1
f (x) = (−1) · 1/k xn ,
n≥1 k=1

y el desarrollo es válido en (−1, 1) que corresponde a la intersección de los


intervalos de convergencia de las series factores.

PROBLEMA 15.36

Desarrollar alrededor de x = 1 la función f (x) = x.

Solución
Haciendo el√cambio de variable t = x − 1, podemos escribir la función co-
mo f (t) = t + 1. Al desarrollar ésta última como serie binómica, obtene-
mos:
X 1/2 X 1/2
1/2 n
f (t) = (t + 1) = t =⇒ f (x) = (x − 1)n ,
n n
n≥0 n≥0

y el desarrollo es válido cuando −1 < x − 1 < 1, es decir cuando 0 < x < 2.

248
PROBLEMA 15.37
12 − 5x
Desarrollar la función f (x) = en serie de McLaurin.
6 − 5x − x2

Solución
En primer lugar descomponemos la función en fracciones simples. Ası́:
5x − 12 A B (A + B)x + 6A − B
f (x) = = + = =⇒ A = −1, B = 6.
x2
+ 5x − 6 x−1 x+6 (x − 1)(x + 6)
X m
m
Teniendo en cuenta ahora el desarrollo en serie binómica, (1 + x) = xn ,
n
n≥0
x ∈ (−1, 1), escribimos los desarrollos correspondientes a cada sumando co-
mo:
−1 X −1 X
−1
= [1 + (−x)] = (−x)n = xn , x ∈ (−1, 1);
x−1 n
n≥0 n≥0
6 X −1 X xn
= [1 + (x/6)]−1 = (x/6)n = (−1)n n , x/6 ∈ (−1, 1).
x+6 n 6
n≥0 n≥0

Sumando las series en el intervalo (−1, 1), que es la intersección de los in-
tervalos de convergencia de ambas series, obtenemos:
n X (−1)n

nx
X X
n
f (x) = x + (−1) n = 1+ xn .
6 6n
n≥0 n≥0 n≥0

PROBLEMA 15.38
Desarrollar la función f (x) = arc sen x en serie de McLaurin.

Solución
1
Como la derivada de la función es f 0 (x) = √ = (1 − x2 )−1/2 , podemos
1 − x2
escribir el desarrollo de esta última función como:
X −1/2 X 1 · 3 . . . (2n − 1)
0
f (x) = (−x2 )n = x2n , x ∈ (−1, 1).
n 2n · n!
n≥0 n≥0

Integrando ahora término a término en el intervalo de convergencia absoluta,


resulta:
X 1 · 3 . . . (2n − 1) x2n+1
f (x) = · , x ∈ (−1, 1).
2n · n! 2n + 1
n≥0

249
PROBLEMA 15.39
1+x
Desarrollar la función f (x) = en serie de McLaurin.
1 − x3

Solución
Si descomponemos la función en dos fracciones y aplicamos el desarrollo de
X −1
la serie geométrica (1 − x3 )−1 = (−x3 )n , tenemos:
n
n≥0

1 x X −1 X −1
3 n
f (x) = + = (−x ) + x (−x3 )n
1 − x3 1 − x3 n n
n≥0 n≥0
X X X
= x3n + x3n+1 = (x3n + x3n+1 )
n≥0 n≥0 n≥0

y el desarrollo es válido en el intervalo (−1, 1), que corresponde al intervalo


donde convergen ambas series.

PROBLEMA 15.40
ex
Desarrollar la función f (x) = en serie de McLaurin.
1+x

Solución
Multiplicando las series correspondientes a las funciones y = ex e y = (1 +
x)−1 , tenemos:
   
X xn X
f (x) =   ·  (−1)n xn  .
n!
n≥0 n≥0

El coeficiente del término general en la serie producto es


n n
X X (−1)n−k
pn = ak · bn−k =
k!
k=0 k=0
X
y la serie pn xn converge absolutamente en el intervalo (−1, 1) que corres-
n≥0
ponde a la intersección de los intervalos de convergencia de las dos series
factores.

250
PROBLEMA 15.41
r
1+x
Desarrollar la función f (x) = ln en serie de McLaurin.
1−x

Solución
Aplicando las propiedades usuales de los logaritmos, escribimos la función
1
como f (x) = [ln(1 + x) − ln(1 − x)]. Recordando que el desarrollo de ln(1+
2
X (−1)n−1
x) en serie de McLaurin es xn , y el radio de convergencia es 1,
n
n≥1
escribimos los desarrollos correspondientes a cada uno de los sumandos y
obtenemos:
 
1 X (−1)n−1 X (−1)n−1
f (x) = xn − (−x)n 
2 n n
n≥1 n≥1

1 X (−1)n−1 X x2n−1
= [1 − (−1)n ]xn =
2 n 2n − 1
n≥1 n≥1

y la serie converge absolutamente en (−1, 1).

PROBLEMA 15.42
¿Es posible desarrollar
( en serie de potencias alrededor del origen
2
e + e−1/x
x si x 6= 0,
la función f (x) =
1 si x = 0?

Solución
2
La función y = e−1/x tiene todas sus derivadas en el origen nulas (esto se
puede probar por inducción), de modo que f (n) (0) = 1 y podemos escribir
el desarrollo
x2 xn
f (x) ∼ 1 + x + + ··· + + Rn (x).
2! n!
x2 xn
 
Sin embargo, como lı́m 1 + x + + ··· + + Rn (x) = ex + lı́m Rn (x),
n→∞ 2! n! n→∞
−1/x2
si la serie converge a la función, debe ser lı́m Rn (x) = e 6= 0 salvo para
n→∞
x = 0. Esto indica que la función no es desarrollable en serie de McLaurin.

251
D. APLICACIONES AL CÁLCULO INFINITESIMAL.

Debido a que las series de potencias son la generalización inmediata de los


polinomios (donde el número de términos es infinito), el cálculo de las de-
rivadas e integrales es también directo. Además, como muchas funciones
elementales son suma de series de potencias, sus valores en puntos del in-
tervalo de convergencia serán también suma de las series correspondientes a
esos puntos. Esto permite plantear una gran variedad de aplicaciones de los
desarrollos de funciones en serie de Taylor y McLaurin a diversos problemas
de Cálculo Infinitesimal; completamos ası́ las herramientas necesarias para
el cálculo de lı́mites, derivadas, integrales y sumas de series que no eran
posible sin el uso de las series de potencias.

PROBLEMA 15.43
sen2 x − x2
Calcular, mediante series de funciones, lı́m .
x→0 (ex − 1)4

Solución
Teniendo en cuenta el desarrollo en serie de las funciones involucradas, po-
demos escribir las siguientes relaciones:

x3 2x4 2x4
sen x = x− +. . . =⇒ sen2 x = x2 − +. . . =⇒ sen2 x−x2 = − +R4 (x),
3! 3! 3!
R4 (x) 2 2 2x4
donde lı́m = 0, lo cual da lugar a la equivalencia sen x − x ∼ − .
x→0 x4 3!
Procediendo análogamente, resulta:

ex = 1 + x + R1 (x) =⇒ ex − 1 = x + R1 (x) =⇒ (ex − 1)4 ∼ x4 .

Aplicando las equivalencias obtenidas, tenemos:

sen2 x − x2 −2x4 /6 1
lı́m x 4
= lı́m 4
=− .
x→0 (e − 1) x→0 x 3

PROBLEMA 15.44
sen2 x3
Calcular, mediante series de funciones, lı́m .
x→0 (1 − cos x2 )3

252
Solución
Análogamente al problema anterior, tenemos:
sen x3 = x3 + R3 (x) =⇒ sen2 x3 = x6 + R6 (x) =⇒ sen2 x3 ∼ x6 ;
x4 x4 x12
cos x2 = 1 − + R4 (x) =⇒ 1 − cos x2 = + R4 (x) =⇒ (1 − cos x2 )3 ∼ .
2! 2 8

Aplicando las equivalencias anteriores, obtenemos:


sen2 x3 x6 8
lı́m 2 3
= lı́m 12
= lı́m 6 = ∞.
x→0 (1 − cos x ) x→0 x /8 x→0 x

PROBLEMA 15.45
Calcular, mediante series de funciones,
( la derivada de orden k en
sen x
x si x 6= 0
el origen de la función f (x) =
0 si x = 0.

Solución
Debido al desarrollo
sen x x2 x2n
=1− + · · · + (−1)n + ...,
x 3! (2n + 1)!
f (k) (0)
y recordando que el término general del desarrollo verifica la fórmula ak = ,
k!
se obtiene en definitiva que
(
(k) 0 si k = 2n + 1(k es impar)
f (0) = ak · k! = (−1)n (−1)n
(2n+1)! · (2n)! = 2n+1 si k = 2n(k es par).

PROBLEMA 15.46
Z 1
Calcular, mediante series de funciones, sen x2 dx.
0

Solución
X (x2 )2n+1
Como sen x2 = (−1)n y la convergencia es uniforme en R, po-
(2n + 1)!
n≥0
demos integrar término a término:
Z 1 X (−1)n Z 1 X (−1)n
2 1
sen x dx = x4n+2 dx = · .
0 (2n + 1)! 0 (2n + 1)! 4n + 3
n≥0 n≥0

253
PROBLEMA 15.47
Z x
dt
Calcular, mediante series de funciones, .
0 1 + t3

Solución
1 X
A partir del desarrollo en serie = (−1)n · (t3 )n , que es uniforme-
1 + t3
n≥0
mente convergente en (−1, 1), resulta:
Z x Z x 3n+1
dt X
n 3n
X
n x
3
= (−1) t dt = (−1) , ∀x ∈ (−1, 1).
0 1+t n≥0 0 n≥0
3n + 1

PROBLEMA 15.48
Z 1
1
Calcular ln dx.
0 1−x

Solución
Aplicaremos en este caso el desarrollo de la función logaritmo. Como
1 X xn
ln = − ln(1 − x) = , ∀x ∈ (−1, 1),
1−x n
n≥1

y la convergencia es uniforme en dicho intervalo, la integral impropia va-


le
Z 1 Z β
1 1
ln dx = lı́m ln dx
0 1−x β→1 −
0 1−x
X Z β xn X 1
= lı́m dx = = 1.
0 n n(n + 1)
β→1 −
n≥1 n≥1

Para calcular la suma de la última serie, se descompone el término general


en fracciones simples y se obtiene en forma simplificada el término general
de la sucesión de sumas parciales (ver capı́tulo 9).

PROBLEMA 15.49
Z 1 ∞
x X (−1)n−1
Probar que 3
dx = .
0 1+x 3n − 1
n=1

254
Solución

Si escribimos el desarrollo en serie de la función integrando, obtenemos:

x X
n 3n
X
= x (−1) x = (−1)n x3n+1 , ∀x ∈ (−1, 1).
1 + x3
n≥0 n≥0

Como la convergencia de la serie de potencias es uniforme, integramos térmi-


no a término, con lo que:
Z 1 Z 1 X (−1)n X (−1)m−1
x X
dx = (−1)n x3n+1 dx = = .
0 1 + x3 0 3n + 2 3m − 1
n≥0 n≥0 m≥1

PROBLEMA 15.50
X
Probar que la serie x(1 − x)n converge no uniformemente en
n≥0
[0, 2). Sin embargo, se puede integrar término a término en [0, 1].

Solución

Como se trata de una serie geométrica de razón 1 − x, será convergente


P si
|1 − x| < 1, es decir si 0 < x < 2. Además, si x = 0, resultaPla serie 0 que
converge a la función cero, pero si x = 2, resulta la serie (−1) · 2 que es
divergente.

De lo anterior se deduce que el intervalo de convergencia es [0, 2). Para ver


que la convergencia no es uniforme, llamamos {Sn (x)} a la sucesión de sumas
parciales, es decir

n
(
X
k 1 − (1 − x)n+1 1 − (1 − x)n+1 si 0 < x < 2
Sn (x) = x(1−x) = x· =
1 − (1 − x) 0 si x = 0.
k=0

(
0 si x = 0
Entonces S(x) = lı́m Sn (x) =
1 si 0 < x < 2.

Como dicho lı́mite no es una función continua, no puede ser lı́mite uniforme
de funciones continuas.

Por otra parte, para ver que se puede integrar término a término en [0, 1],

255
tenemos:
Z 1
S(x) dx = 1;
0
1 1 Z 1
−x(1 − x)n+1
Z 
1 1
fn (x) dx = + (1 − x)n+1 dx =
0 n+1 0 n+1 0 (n + 1)(n + 2)
∞ Z 1 ∞
X X 1
=⇒ Sn (x) dx = = 1.
0 (n + 1)(n + 2)
n=0 n=0

Como se observa en este problema, la convergencia uniforme no es necesa-


ria para que se pueda integrar término a término una serie aunque, como
sabemos, sı́ es una condición suficiente.

PROBLEMA 15.51
Calcular las integrales de las siguientes funciones en el intervalo
[0, 1]:
 π
a) f (x) = signo sen .
x
b) f (x) = signo (sen ln x).

Solución
 
π 1 1
a) Teniendo en cuenta que sen < 0 cuando x ∈ , , donde
x 2k 2k − 1
k ∈ Z \ {0}, entonces
(
1 1
−1 si 2k < x < 2k−1
f (x) = 1 1
1 si 2k+1 < x < 2k .

Por tanto la integral buscada se descompone como la suma de las series


Z 1 ∞   X ∞  
X 1 1 1 1
f (x) dx = − − −
0 2k 2k + 1 2k − 1 2k
k=1 k=1
∞   X ∞  
X 1 1 1 −1 2 1
= − − = + − .
k 2k + 1 2k − 1 2k − 1 2k 2k + 1
k=1 k=1

Para calcular la suma de esta serie observamos, por un lado, que la su- 
1 1 1
cesión de sumas parciales tiene por término general Sn = 2 − + − + . . . ,
3 4 5

256
X (−1)n−1 1 X (−1)n−1
y por otro que = ln 2, de modo que ln 2 − 1 + = .
n 2 n
n≥1 n≥3
Reuniendo todos estos datos, obtenemos que
Z 1  
1
f (x) dx = 2 − ln 2 .
0 2

b) Análogamente al apartado anterior, determinamos primero el signo de


la función sen ln x. Se obtiene ası́ que f (x) = 1 cuando sen ln x >
0, es decir cuando e−2kπ < x < e(−2k+1)π , con k ∈ N. La integral se
descompone en suma como
Z 1 ∞ h
X i ∞ h
X i
−2kπ
f (x) dx = (−2k+1)π
e −e − e−2kπ − e(−2k−1)π .
0 k=1 k=0

Como las series involucradas son geométricas, sus sumas son, respec-
tivamente,
∞ ∞ ∞
X
(−2k+1)π e−π X
−2kπ e−2π X
(−2k−1)π e−π
e = , e = , e = .
1 − e−2π 1 − e−2π 1 − e−2π
k=1 k=1 k=0

En definitiva, obtenemos:

e−π − e−2π − 1 + e−π (e−π − 1)2


Z 1
f (x) dx = = .
0 1 − e−2π e−2π − 1

PROBLEMA 15.52
X
Probar que la serie ne−nx es uniformemente convergente en
n≥1
[a, ∞) con a > 0, pero no en [0, ∞). Calcular la suma de la serie
para x > 0.

Solución
Si llamamos fn (x) = ne−nxP, cuando x ∈ [a, ∞), entonces fn (x) ≤ ne−an , ∀n.
Además la serie numérica ne−an es convergente cuando e−a < 1 (lo que
se prueba aplicando el criterio del cociente), es decir cuando a > 0.
El criterio de Weierstrass indica que la serie propuesta converge uniforme-
mente en [a, ∞).
P
Haciendo x = 0, nos queda la serie divergente n, por lo que la serie de
funciones no es uniformemente convergente en [0, ∞).

257
Para calcular la suma de la serie, basta tener en cuenta que fn (x) = D(−e−nx ).
X e−x
Como la serie e−nx converge uniformemente si x > 0 y e−nx =
P
,
1 − e−x
n≥1
entonces  −x 
X
−nx e ex
ne = −D = .
1 − e−x (ex − 1)2
n≥1

PROBLEMA 15.53
X xn
¿Qué función representa la serie ?
1 + ··· + n
n≥1

Solución
n(n + 1)
Si recordamos la fórmula 1 + 2 + · · · + n = , entonces
2
xn 2xn 2xn 2xn
= = − , ∀x ∈ (−1, 1).
1 + ··· + n n(n + 1) n n+1
X xn
Sabiendo además que ln(1 − x) = − , entonces
n
n≥1

X xn 1 X xn+1 1
= = [− ln(1 − x) − x].
n+1 x n+1 x
n≥1 n≥1

De aquı́ resulta:

xn
 
X ln(1 − x) + x
f (x) = = 2 − ln(1 − x) − , ∀x ∈ (−1, 1).
1 + ··· + n x
n≥1

PROBLEMA 15.54
Demostrar que para |x| < 1 se verifica lo siguiente:
X 1
a) (−1)n xn = .
1+x
n≥0
X 1
b) (−1)n x2n = .
1 + x2
n≥0

258
Solución
a) Si escribimos el término general de la sucesión de sumas parciales, tene-
mos:

Sn = 1 − x + x2 − · · · + (−1)n xn ;
xSn = x − x2 + x3 − · · · + (−1)n xn+1 . Sumando miembro a miembro,
1 + (−1)n xn+1 1
(1 + x)Sn = 1 + (−1)n xn+1 =⇒ Sn = =⇒ S = lı́m Sn = ,
1+x n→∞ 1+x
cuando |x| < 1, pues lı́m xn+1 = 0.
n→∞

b) Análogamente al anterior,

Sn = 1 − x2 + x4 − · · · + (−1)n x2n ;
x2 Sn = x2 − x4 + x6 − · · · + (−1)n x2n+2 ;
(1 + x2 )Sn = 1 + (−1)n x2n+2
1 + (−1)n x2n+2 1
=⇒ Sn = =⇒ S = lı́m Sn = ,
1 + x2 n→∞ 1 + x2
también cuando |x| < 1.

PROBLEMA 15.55
X 1
Dada la serie de potencias (1 − x)3n , deter-
(3n − 2)(3n + 1) · 8n
minar su campo de convergencia y calcular su suma cuando x =
−1.

Solución
Aplicando el criterio del cociente,

|1−x|3n+3
|1 − x|3

an+1 (3n+1)(3n+4)8n+1 3n − 2
lı́m = lı́m
|1−x|3n
= lı́m |1 − x|3 = .
n→∞ an 8(3n + 4) 8
(3n−2)(3n+1)

De aquı́ se deduce que la serie converge absolutamente cuando |1 − x|3 < 8,


o bien cuando x ∈ (−1, 3). En los extremos del intervalo tenemos:
X 1
- Si x = 3, la serie (−1)n es absolutamente convergen-
(3n − 2)(3n + 1)
te.
X 1
- Si x = −1, la serie es también absolutamente conver-
(3n − 2)(3n + 1)
gente.

259
Para calcular la suma de esta última serie, escribimos el término general
1 1/3 1/3
como an = = − . Ası́, la suma de los n pri-
(3n − 2)(3n + 1) 3n − 2 3n + 1
meros términos vale:
   
1 1 1 1 1 1 1 1 1
Sn = 1 − + − + − ··· + − = 1− .
3 4 4 7 7 3n − 2 3n + 1 3 3n + 1

La suma será entonces S = lı́m Sn = 1.


n→∞

PROBLEMA 15.56
Determinar el intervalo de convergencia de la serie
X 1 · 3 · 5 . . . (2n − 1)
(−x/2)3n .
(n + 1)!
n≥0

Calcular la suma de la serie para x = − 3 4.

Solución
Por el criterio del cociente,
1·3...(2n−1)(2n+1)
(−x/2)3n+3 3 3

= lı́m 2n + 1 · |x| = |x| ,
an+1 (n+2)!
lı́m = lı́m
1·3...(2n−1)

(−x/2)3n
n→∞ an n+2 8 4
(n+1)!

3
de modo que la serie es absolutamente convergente cuando |x| < 4.
En los extremos del intervalo tenemos las series
X 1 · 3 . . . (2n − 1) X 1 · 3 . . . (2n − 1)
y (−1)n .
(n + 1)! · 2n (n + 1)! · 2n

Ambas son absolutamente convergentes como se deduce al aplicar el criterio


de Raabe:
   
an+1
lı́m n · 1 − = lı́m n · 1 − 2n + 1 3
= > 1.
an 2(n + 2) 2

Escribimos la serie en x = − 3 4 como
X 1 · 3 . . . (2n − 1) X (−1/2)(−3/2) . . . [−(2n − 1)/2]
S = = (−1)n
(n + 1)! · 2n n! · (n + 1)
n≥0 n≥0
 
X −1/2 1
= (−1)n .
n n+1
n≥0

260
X m
A partir del desarrollo de la serie binómica xn = (1 + x)m , al inte-
n
n≥0
grar los dos miembros de la igualdad, resulta:
X m Z x Z x X m xn+1 (1 + x)m+1 1
n m
x dx = (1+x) dx =⇒ = − .
n 0 0 n n+1 m+1 m+1
n≥0 n≥0

Haciendo ahora m = −1/2 y x = −1, tenemos:


X −1/2 (−1)n+1
= −2 =⇒ S = 2.
n n+1
n≥0

PROBLEMA 15.57
X (x − 3)3n−1
Calcular la suma de la serie especificando el inter-
(3n − 1) · 8n
n≥1
valo de convergencia de la misma.

Solución
Por el criterio de la raı́z,

p
n |x − 3|3−1/n |x − 3|3
lı́m |an | = lı́m √ = ,
n→∞ 8 · n 3n − 1 8

y la serie converge absolutamente cuando |x−3|3 < 8, o bien x ∈ (1, 5).


X (−1)3n−1
- Cuando x = 1, la serie es que converge condicionalmente
2(3n − 1)
(basta aplicar el criterio de Leibnitz).
X 1
- Cuando x = 5, la serie es divergente.
2(3n − 1)
X (x − 3)3n−1
Para calcular la suma de la serie, si llamamos f (x) = , al
(3n − 1) · 8n
n≥1
derivar obtenemos:
X (x − 3)3n−2 X  (x − 3)3 n
f 0 (x) = = (x − 3)−2 ·
8n 8
n≥1 n≥1
Z x
(x − 3)3 /8 x−3 x−3
= (x − 3)−2 · = =⇒ f (x) = dx
1 − (x − 3)3 /8 8 − (x − 3)3 3 8 − (x − 3)3
ln(5 − x) ln(7 − 4x + x2 )
 
1 π x−2
= √ − arc tg √ − + .
2 3 6 3 6 12

261
PROBLEMA 15.58
X 1
Demostrar que ch 1 = .
(2n)!
n≥0

Solución
Recordando la fórmula 2 ch x = ex + e−x y el desarrollo en serie de cada uno
de los sumandos, obtenemos:
X xn X xn X xn X x2n
2 ch x = + (−1)n = [1 + (−1)n ] =2
n! n! n! (2n)!
n≥0 n≥0 n≥0 n≥0
X x2n
=⇒ ch x = .
(2n)!
n≥0

PROBLEMA 15.59

X xn
Sabiendo que = ex , hallar las sumas de las siguientes se-
n!
n=0
ries:

X n−1
a) .
n!
n=2

X (n − 1)(n + 1)
b) .
n!
n=2

Solución
a) Al descomponer la serie en suma, tenemos:
Xn−1 X 1 X 1 X 1 X 1
= − = − .
n! (n − 1)! n! m! n!
n≥2 n≥2 n≥2 m≥1 n≥2

Ahora bien, como


X 1 X 1 X 1
e= =1+ =1+1+ ,
n! n! n!
n≥0 n≥1 n≥2

resulta en definitiva que S = (e − 1) − (e − 2) = 1.

262
b) Procediendo análogamente al apartado anterior,


X n2 − 1 X n2
X 1 X n X 1
S = = = − −
n! n! n! (n − 1)! n!
n=2 n≥2 n≥2 n≥2 n≥2
X m+1 X 1 X 1 X 1 X 1
= − = + −
m! n! (m − 1)! m! n!
m≥1 n≥2 m≥1 m≥1 n≥2
X 1 X 1 X 1
= + − = e + (e − 1) − (e − 2) = e + 1.
k! m! n!
k≥0 m≥1 n≥2

PROBLEMA 15.60

X 1
Sabiendo que, para |x| < 1, se tiene xn = , calcular cuando
1−x
n=0
sea posible:
X xn+1
a) .
n+1
n≥0

X xn+2
b) .
(n + 1)(n + 2)
n≥0
X
c) nxn−1 .
n≥1
X
d) n(n − 1)xn−2 .
n≥2
X
e) n2 xn .
n≥0

X 1 X n2
f) + .
en+2 (n + 1)(n + 2) πn
n≥0 n≥0

Solución

a) Integrando miembro a miembro, resulta:

XZ x Z x X xn+1
1
xn dx = dx =⇒ = − ln |1−x|, ∀x ∈ (−1, 1).
1−x n+1
n≥0 0 0 n≥0

263
b) Integrando nuevamente el resultado de a),
X Z x xn+1 Z x X xn+2
dx = − ln |1 − x| dx =⇒
n≥0 0 n+1 0 (n + 1)(n + 2)
n≥0
= −x ln |1 − x| + x + ln |1 − x|, ∀x ∈ (−1, 1).

c) Derivamos ahora término a término la serie original. Ası́:


  X
1 1 X
D = D(xn ) =⇒ = n · xn−1 , ∀x ∈ (−1, 1).
1−x (1 − x)2
n≥1 n≥1

d) Derivando nuevamente,
  X
1 n−1 2 X
D = D(nx ) =⇒ = n(n−1)·xn−2 , ∀x ∈ (−1, 1).
(1 − x)2 (1 − x)3
n≥2 n≥2

e) Teniendo en cuenta los resultados de los apartados anteriores,


X X X X X
n2 xn = n(n − 1)xn + nxn = x2 n(n − 1)xn−2 + x · nxn−1
n≥0 n≥1 n≥1 n≥2 n≥1
2 1 x x2 +
= x2 · 3
+x· 2
= .
(1 − x) (1 − x) (1 − x)3

f) Haciendo en b) x = 1/e y en e) x = 1/π, resulta:


X 1 X n2 1

1

1

1

1/π 2 + 1/π
+ = − ln 1 − + +ln 1 − + .
en+2 (n + 1)(n + 2) πn e e e e (1 − 1/π)3
n≥0 n≥0

PROBLEMA 15.61
X n2
Dada la serie (x − 1)n , determinar su intervalo de conver-
22n
n≥1
gencia y calcular su suma cuando x = 0.

Solución
Por el criterio del cociente,
(n+1)2
|x − 1|n+1 (n + 1)2

an+1 22n+2 |x − 1|
lı́m = lı́m
n2
= lı́m |x − 1| · 2 2
= ,
n→∞ an |x − 1|n 2 ·n 4
22n

y la serie converge absolutamente cuando |x−1| < 4, es decir x ∈ (−3, 5).

264
En los extremos, tenemos:
P 2
n es divergente; para x = −3, la serie (−1)n n2 es
P
Para x = 5, la serie
también divergente.
X n2 X n2
Para x = 0 resulta la serie (−1)n n = . Para calcular su suma
4 (−4)n
n≥1 n≥1
aplicaremos el apartado e) del problema anterior haciendo x = −1/4. Queda
ası́
X n2 (−1/4)2 + (−1/4) 12
n
= 3
=− .
(−4) (1 + 1/4) 125
n≥1

PROBLEMA 15.62

X x
Sabiendo que, para |x| < 1, se tiene nxn = , calcular
(1 − x)2
n=1
cuando sea posible:
X nxn+1
a) .
n+1
n≥1
X
b) n2 xn−1 .
n≥1
X
c) n2 (n − 1)xn−2 .
n≥1

X n2 (n − 1)
d) .
en
n≥1

Solución
a) Integrando miembro a miembro, obtenemos:
X nxn+1 X Z x Z x
x 1
= nxn dx = 2
dx = +ln |1−x|−1.
n≥1
n+1 0 n≥1 0 (1 − x) 1−x

b) Si derivamos ahora la fórmula dada,


 
X X x 1+x
n2 xn−1 = D(nxn ) = D = .
(1 − x)2 (1 − x)3
n≥1 n≥1

c) Derivamos nuevamente el resultado de b). Ası́:


 
X X 1+x 2x + 4
n2 (n − 1)xn−2 = D(n2 xn−1 ) = D = .
(1 − x)3 (1 − x)4
n≥1 n≥1

265
d) Si, en el apartado anterior, hacemos x = 1/e, resulta:
X n2 (n − 1) 2/e + 4 e3 (2 + 4e)
= = .
en (1 − 1/e)4 (e − 1)4
n≥1

266
E. EJERCICIOS PROPUESTOS.

1. Contestar razonadamente si cada uno de los siguientes apartados


es verdadero o falso:
X X
a) Si la serie an 6n es convergente, entonces la serie an (−6)n
n≥0 n≥0
es convergente.
(−1)n
Resp.: Falso (considerar el contraejemplo: an = ).
n · 6n

X X
b) Si la serie an 6n es convergente, entonces la serie an (−5)n
n≥0 n≥0
es convergente.
Resp.: Verdadero, pues el radio de convergencia es R ≥ 6.

X
c) Si la serie an xn es convergente para todo x > 0, entonces
n≥0
la serie converge para todo x < 0.
Resp.: Verdadero, pues el radio de convergencia es R = ∞.

X
d) Si f (x) = an xn es una función continua par, entonces
n≥0
a2n+1 = 0, para todo n.
Resp.: Verdadero, pues f (−x) = f (x) =⇒ an = (−1)n an , ∀n.


X
e) Si la serie an xn tiene radio de convergencia R > 0, R es
n=0

X
también el radio de convergencia de la serie an · n · (n − 1)xn−2 .
n=2

Resp.: Verdadero pues la segunda serie es derivada de orden dos


de la primera.


X 1 + cos x
2. Estudiar la convergencia uniforme de la serie en R.
n2
n=1

Resp.: Por el criterio de Weierstrass, |fn (x)| ≤ 2/n2 , ∀n.

267

X
3. Se considera la serie fn donde las funciones fn son conti-
n=1
n
X ln n
nuas en [0, 1] y se tiene además que fk (x) − x2 ≤ 2 ,

n +5
k=1

Z 1X
∀x ∈ [0, 1]. Calcular, si es posible, fn (x) dx.
0 n=1

Resp.: 1/3.

4. ¿Existe una sucesión {fn } de funciones integrables en [0, 1] que


Z 1
2 5 + 3n
converja uniformemente a f (x) = x en [0, 1] y tal que fn (x) dx = ?
0 n
Z 1 Z 1
Resp.: No; si existiera, deberı́a cumplirse que lı́m fn (x) dx = lı́m fn (x) dx.
0 0

5. Sea {fn } una sucesión de funciones derivables en [0, 1] tal que


i) fn → f en [0, 1];
ii) fn (0) = (1 + 1/n)n , ∀n ∈ N;
iii) |fn0 (x) − x| ≤ 7/(3 + n), ∀x ∈ [0, 1], ∀n ∈ N.
Calcular f (1/2).
Z x
Resp.: Como f (x) = lı́m fn0 (x) dx + lı́m fn (0) =⇒ f (1/2) = e + 1/8.
0 n n

6. Estudiar la convergencia de la serie


X 1 1 1 1

+ + + ··· + xn .
1·3 2·4 3·5 n(n + 2)
n≥1

Resp.: Converge absolutamente en [−1, 1]; diverge en el resto.

7. Determinar el intervalo de convergencia de la serie de potencias



X 2 · 4 · 6 . . . 2n
· xn+1 .
3 · 5 · 7 . . . (2n + 1) · 4n+1
n=1

Resp.: Converge absolutamente en (−4, 4); converge condicionalmente


en x = −4; diverge en el resto.

268
∞  2
X 1 + n2 n
8. Obtener el campo de convergencia de la serie ·x .
1 + n3
n=1

Resp.: Converge absolutamente en [−1, 1]; diverge en el resto.


X (−1)n (x − 1)n
9. Estudiar el carácter de la serie con x ∈ R.
2n (3n − 1)
n=1

Resp.: Converge absolutamente cuando x ∈ (−5, 7); diverge en el resto.

X (x − 2)n
10. Obtener el campo de convergencia de la serie .
(2n − 1) · 2n
n≥1

Resp.: Converge absolutamente cuando x ∈ (0, 4); converge condicio-


nalmente cuando x = 0; diverge en el resto.

X (−1)n x2n
11. Determinar el campo de convergencia de la serie .
22n (n!)2
Resp.: Converge absolutamente en R.

X 2n (n!)2 n
12. Determinar el campo de convergencia de la serie (−1)n x .
(2n + 1)!
n≥1

Resp.: Converge absolutamente en (−2, 2); converge condicionalmente


en x = 2; diverge en el resto.

13. Determinar el campo de convergencia de la serie


X 1
(−x/5)3n .
(3n − 2)(3n + 1)
Resp.: [−5, 5].

14. Desarrollar en serie de potencias alrededor del punto indicado y


encontrar el campo de convergencia de la misma:
a) f (x) = ln x, x = 1.
X (−1)n−1
Resp.: f (x) = (x − 1)n , ∀x ∈ (0, 2]. b) f (x) = 1/x2 , x =
n
n≥1

−1.

269
X x
Resp.: f (x) = (n + 1)(x + 1)n , ∀x ∈ (−2, 0). c) f (x) = ,
(1 − x)2
n≥0

x = 0.
X
Resp.: f (x) = (n + 1)xn+1 , ∀x ∈ (−1, 1).
n≥0

15. Desarrollar en serie de McLaurin la función y = sh x y hallar


X 1
.
(2n + 1)!
n≥0

X x2n+1 X 1 e2 − 1
Resp.: sh x = ; = sh 1 = .
(2n + 1)! (2n + 1)! 2e
n≥0 n≥0

16. Escribir los primeros términos


p del desarrollo alrededor de x =
0 de la función f (x) = ln( 1 + x2 − x) y aplicarlo al cálculo de

2
x + ln( 1 + x − x)
lı́m .
x→0 x3
x3
Resp.: f (x) ∼ −x + + R3 (x); L = 1/6.
3!

17. Hallar la suma de las series


X xn
a) .
n
n≥1

Resp.: S = − ln |1 − x|.

X x2n−1
b) .
2n − 1
n≥1
r
1+x
Resp.: S = ln .
1−x
X
c) nxn .
n≥1
x
Resp.: S = .
(1 − x)2


X n2 − 5n + 7
18. Calcular la suma de la serie .
n!
n=1

Resp.: e − 7.

270
Z x ∞X (−1)k x2k+1
sen t
19. Probar que dt = .
0 t (2k + 1)(2k + 1)!
k=0

sen t X (−1)k t2k
Sugerencia: Utilizar el desarrollo = .
t (2k + 1)!
k=0

20. Estudiar la convergencia de la serie 3x/2 + 7x2 /4 + 11x3 /8 + 15x4 /16 + . . .


y sumarla cuando sea posible.
Resp.: Converge absolutamente cuando x ∈ (−2, 2); diverge en el resto.
x2 + 6x
S= .
(2 − x)2

271
CAPÍTULO XVI.
NÚMEROS COMPLEJOS

SECCIONES
A. Definición. Primeras propiedades.
B. Potencia y raı́z de números complejos.
C. Ejercicios propuestos.

273
A. DEFINICIÓN. PRIMERAS PROPIEDADES.

Un número complejo es un par ordenado de números reales. El conjunto de


los número complejos es pues

C = {(a, b) : a, b ∈ R} = R × R.

Si z = (a, b) ∈ C, se llama parte real de z a la primera componente a = Re z


y se llama parte imaginaria de z a la segunda componente b = Im z .
Observación 1. Los números complejos surgen como necesidad de resolver
ecuaciones que involucren raı́ces de números negativos. Ası́, por ejemplo, la
ecuación x2 + 1 = 0 no tiene solución en el sistema de números reales; por
eso, originalmente se representó una de sus raı́ces con la letra i (de imagina-
rio). En este nuevo conjunto se podrá verificar el teorema fundamental del
Algebra mediante el cual todo polinomio con coeficientes complejos tiene
exactamente tantas raı́ces como indica su grado.
Observación 2. De la definición se observa que C puede representarse como
el conjunto de puntos del plano: su primera coordenada corresponde a la
parte real y su segunda coordenada a la parte imaginaria. El punto cuyas
coordenadas son las componentes de un número complejo se llama afijo del
número. Llamamos eje real al eje de abscisas y eje imaginario al eje de
ordenadas.

El conjunto C tiene estructura de cuerpo con las operaciones

(a, b) + (c, d) = (a + c, b + d),


(a, b) · (c, d) = (ac − bd, ad + bc),

siendo (0, 0) el neutro para la suma y (1, 0) el elementounidad. El opuesto



−1 1 a −b
de z = (a, b) es −z = (−a, −b) y el inverso, z = = , ,
z a2 + b2 a2 + b2
si z 6= 0.

274
Se puede considerar a los números reales como subconjunto de los comple-
jos haciendo la identificación a = (a, 0), ∀a ∈ R, pues de este modo las
operaciones anteriores coinciden con la suma y producto de números reales.
Ası́ pues, a los números complejos de la forma a = (a, 0) los llamaremos
reales y un número complejo es imaginario si no es real.
Se llama unidad imaginaria al número i = (0, 1) y un número complejo z es
imaginario puro si z = (0, b) = b · i, b ∈ R.
Observación 3. Ası́ definida, la unidad imaginaria verifica la ecuación x2 +
1 = 0. En general, debido a que i4 = 1, todas las potencias de i se reducen
a cuatro:
i0 = 1, i1 = i, i2 = −1, i3 = −i.
Debido a la descomposición (a, b) = (a, 0) + (0, b), todo número complejo se
puede escribir en forma binómica como (a, b) = a + ib.
Se define conjugado de z = (a, b) al número z = (a, −b). De la definición
se observa que los afijos de dos complejos conjugados son puntos simétricos
respecto al eje real. Podemos destacar las siguientes propiedades:
1) z = z, ∀z ∈ C.
2) z = z ⇐⇒ z ∈ R.
3) z1 + z2 = z1 + z2 , z1 · z2 = z1 · z2 , ∀z1 , z2 ∈ C.
4) −z = − z, z −1 = z −1 , ∀z ∈ C.
5) z + z = 2 Re z; z − z = 2i Im z, ∀z ∈ C.
Llamamos módulo de un número complejo z = (a, b) a la longitud r = |z| =

a2 + b2 . Se verifican las siguientes propiedades:
6) z = 0 ⇐⇒ |z| = 0.
7) |z|2 = z · z, ∀z ∈ C.
8) |z1 · z2 | = |z1 | · |z2 |, ∀z1 , z2 ∈ C.
9) |z1 + z2 | ≤ |z1 | + |z2 |, ∀z1 , z2 ∈ C.
Se llama argumento de un número complejo z = (a, b) al número ϕ = arg z =
arc tg b/a y gráficamente representa el ángulo que forma el segmento OP con
la parte positiva del eje real medido en la dirección contraria al movimiento
de las agujas del reloj. A veces se considera el argumento como alguno de
los valores ϕ + 2kπ, con k ∈ Z, con lo que llamaremos argumento principal
de z, Arg z, al que verifica 0 ≤ Arg z < 2π. Las siguientes propiedades son
fáciles de verificar:
10) arg(αz) = arg z si α > 0; arg(αz) = π + arg z si α < 0.

275
11) arg(z1 · z2 ) = arg z1 + arg z2 , ∀z1 , z2 ∈ C.

12) arg z = − arg z, ∀z ∈ C; arg(1/z) = − arg z, , ∀z 6= 0.

Dado un número complejo z = (a, b), los valores r = |z|, ϕ = arg z consti-
tuyen las llamadas coordenadas polares de z. De la relación entre las coor-
denadas cartesianas y polares podemos escribir z como

z = (a, b) = a + ib = r cos ϕ + ir sen ϕ = r(cos ϕ + i sen ϕ)

con r ≥ 0 y 0 ≤ ϕ < 2π, y la llamaremos forma trigonométrica del número


z (abreviadamente escribiremos z = r cis ϕ). A veces también se utiliza la
forma módulo-argumental o polar z = rϕ .

En los siguientes problemas estudiaremos diferentes aplicaciones de los con-


ceptos y propiedades arriba indicadas.

PROBLEMA 16.1
Dados z1 = x1 + iy1 y z2 = x2 + iy2 , probar que z1 z 2 + z 1 z2 =
2(x1 x2 + y1 y2 ) y z1 z 2 − z 1 z2 = 2i(y1 x2 − x1 y2 ).

Solución

Operando directamente se obtiene:

z1 z 2 = (x1 + iy1 )(x2 − iy2 ) = x1 x2 + y1 y2 + iy1 x2 − ix1 y2 ;


z 1 z2 = (x1 − iy1 )(x2 + iy2 ) = x1 x2 + y1 y2 − iy1 x2 + ix1 y2 .

Al sumar miembro a miembro se obtiene que z1 z 2 + z 1 z2 = 2(x1 x2 + y1 y2 ) y


al restar, se obtiene análogamente que z1 z 2 − z 1 z2 = 2i(y1 x2 − x1 y2 ).

PROBLEMA 16.2
Dados z1 = x1 + iy1 , z2 = x2 + iy2 , expresar en forma binómica el
z1 + z 2
número .
z 1 z2 − 1

Solución

276
Multiplicamos y dividimos por el conjugado del denominador:

z1 + z 2 (z1 + z2 )(z 1 z 2 − 1) |z1 |2 z 2 + |z2 |2 z 1 − (z1 + z2 )


= =
z 1 z2 − 1 |z1 z2 − 1|2 |z1 z2 − 1|2
(x21 + y12 )(x2 − y2 i) + (x22 + y22 )(x1 − y1 i) − (x1 + x2 ) − (y1 + y2 )i
=
|(x1 x2 − y1 y2 − 1) + (x1 y2 + x2 y1 )i|2
(x21 + y12 )x2 + (x22 + y22 )x1 − (x1 + x2 )
=
(x1 x2 − y1 y2 − 1)2 + (x1 y2 + x2 y1 )2
y2 (x21 + y12 ) + y1 (x22 + y22 ) + (y1 + y2 )
−i · .
(x1 x2 − y1 y2 − 1)2 + (x1 y2 + x2 y1 )2

PROBLEMA 16.3
Dado z = x+iy , expresar (z/z)2 −(z/z)2 en forma binómica.

Solución
Haciendo denominador común, tenemos:

z2 z2 z4 − z4
(z/z)2 − (z/z)2 = 2 − 2 =
z z z2z2
z 4 + z 4 − 2z 4 2 Re(z 4 ) − 2z 4
= =
|z|4 (x2 + y 2 )2
2(x − 6x y + y ) − 2[x4 + 4x3 (−iy) + 6x2 (−iy)2 + 4x(−iy)3 + (iy)4 ]
4 2 2 4
=
(x2 + y 2 )2
8x3 yi − 8xy 3 i 8xy(x2 − y 2 )
= = 0 + i · .
(x2 + y 2 )2 (x2 + y 2 )2

PROBLEMA 16.4
Simplificar las siguientes expresiones:
a) (a + ib)2 + (a − ib)2 .
b) (1 + ia)4 + (1 − ia)4 .
a + ib a − ib
c) + .
c + id c − id

277
Solución
a) Desarrollando las potencias,

(a + ib)2 + (a − ib)2 = a2 − b2 + 2iab + a2 − b2 − 2iab = 2(a2 − b2 ).

b) Análogamente al anterior,

(1 + ia)4 + (1 − ia)4 = 1 + 4ia + 6(ia)2 + 4(ia)3 + (ia)4


+ 1 − 4ia + 6(ia)2 − 4(ia)3 + (ia)4 = 2 − 12a2 + 2a4 .

c) Hacemos denominador común y obtenemos

a + ib a − ib (a + ib)(c − id) + (c + id)(a − ib) ac + bd


+ = 2 2
=2· 2 .
c + id c − id c +d c + d2
Se observa que todos los resultados son reales, hecho que se deduce de
que en todos los casos se tenı́a la suma de un número y su conjugado.

PROBLEMA 16.5
(1 + i)(1 − i)3 i(i − 1)(2i − 1)
Calcular B = − .
(1 + 2i)3 (3 + i)2

Solución
Desarrollando las potencias y expresando cada sumando en forma binómica
tenemos:

(1 + i)(1 − i)(1 − i)2 i(i − 1)(2i − 1)


B = −
(1 + 2i)3 (3 + i)2
(1 − i2 )(1 − i)2 (i2 − i)(2i − 1)
= −
(1 + 2i)3 (3 + i)2
2(1 + i2 − 2i) (−1 − i)(2i − 1)
= −
1 + 3 · 2i + 3 · (2i)2 + (2i)3
3 9 + i2 + 6i
−4i 2i − 1 + 2i2 − i −4i −3 + i
= + = +
1 − 12 + 6i − 8i 8 + 6i −11 − 2i 8 + 6i
−4i(−11 + 2i) (−3 + i)(8 − 6i) 8 + 44i −24 + 8i + 18i − 6i2
= + = +
 125   100  125 100
8 18 44 26 −29 + 153i
= − + + i= .
125 100 125 100 250

278
PROBLEMA 16.6

Hallar el módulo y argumento de z = −2 − 2 3i.

Solución
Por la definición de módulo y argumento, obtenemos:
√ q √
|z| = | − 2 − 2 3i| = 22 + (2 3)2 = 4.
√ √
tg α = 2 3/2 = 3.
Como las partes real e imaginaria de z son negativas, el afijo está en el tercer
cuadrante, con lo que arg z = α = π/3 + π.

PROBLEMA 16.7
1 + ix
Hallar el módulo y el argumento del número complejo , don-
1 − ix
de x ∈ R. Calcular x para que su módulo sea 1.

Solución
Multiplicando y dividiendo por el conjugado del denominador,

1 + ix (1 + ix)2 1 − x2 2x
= 2
= 2
+ i.
1 − ix 1+x 1+x 1 + x2
El módulo es
s 2 s s
1 − x2 2 1 + 2x2 + x4 (1 + x2 )2
 
2x
ρ= + = = = 1,
1 + x2 1 + x2 (1 + x2 )2 (1 + x2 )2

de modo que el módulo es 1 independientemente del valor de x.


2x
El argumento es ϕ = arc tg .
1 − x2

PROBLEMA 16.8

3(1 + i)(3 − 3i)
Calcular el módulo y argumento de w = √ .
(−2 − 2 3i)2

279
Solución
Aplicamos las propiedades del módulo y argumento del producto y cociente
de números complejos. Tenemos ası́:
√ √ √ √
3|1 + i| · |3 − 3i| 3 · 2 · 12 3 6
|w| = √ = √ = .
| − 2 − 2 3i|2 ( 16)2 8

√ √
arg w = arg 3 + arg(1 + i) + arg(3 − 3i) − 2 arg(−2 − 2 3i)
= 0 + π/4 − π/6 − 2(π + π/3) = −31π
12 .

PROBLEMA 16.9
2(3 − 2i) + (2 + i) − 5 − i
Hallar el módulo y argumento de z = .
2(2 + i) − (3 − 2i) + 3 − i

Solución
Simplificamos en primer lugar dicho número:

6 − 4i + 2 + i − 5 − i 3 − 4i
z= = .
4 + 2i − 3 + 2i + 3 − i 4 + 3i
De este modo, √
|3 − 4i| 32 + 42
|z| = =√ = 1.
|4 + 3i| 42 + 32
Además,
3 − 4i (3 − 4i)(4 − 3i) 12 − 16i − 9i − 12
z= = = = −i,
4 + 3i 42 + 32 25
de modo que Arg z = 3π/2 y arg z = 3π/2 + 2kπ, k ∈ Z.

PROBLEMA 16.10

Calcular z = (1 + i)(1 + i 3)(cos ϕ + i sen ϕ).

Solución
Calculamos el módulo y argumento de cada factor:

|1 + i| = 2, arg(1 + i) = π/4;

280
√ √
|1 + i 3| = 2, arg(1 + i 3) = π/3;
| cos ϕ + i sen ϕ| = 1, arg(cos ϕ + i sen ϕ) = ϕ.


El módulo del producto será el producto de los módulos |z| = 2 2, y el
argumento la suma de los argumentos arg z = π/4 + π/3 + ϕ.

PROBLEMA 16.11
Expresar en forma binómica x + iy los complejos

a) 5 cis π/6, b) 8√cis 165◦ ,


c) 2 cis 800◦ , d) 6 cis 9π/4.

Solución
Utilizamos en todos los casos la fórmula cis ϕ = cos ϕ + i sen ϕ.

a) 5 cis π/6 = 5(cos π/6 + i sen π/6) = 5 3/2 + i · 5/2.
b) 8 cis 165◦ = 8(cos 165◦ + i sen 165◦ ) = −8 cos 15◦ + i sen 15◦ .
c) 2 cis 800◦ = 2 cis(4π + 80◦ ) = 2 cos 80◦ + 2i sen 80◦ .
√ √ √ √
d) 6 cis 9π/4 = 6 cis(2π + π/4) = 6(cos π/4 + i sen π/4) = 3(1 + i).

PROBLEMA 16.12
Expresar en forma binómica
5
(1 + i)4 − (1 − i)4

(2 + 3i)(1 − i)
E= + .
(1 − i)3 + (1 + i)3 (1 − 2i)(2 − i)

Solución
Simplificamos por separado los distintos términos:

(1 + i)4 − (1 − i)4 = [(1 + i)2 − (1 − i)2 ] · [(1 + i)2 + (1 − i)2 ] = 4i[2 + 2i2 ] = 0;
(1 − i)3 + (1 + i)3 = 2 + 6i2 = −4;
(2 + 3i)(1 − i) = 5 + i;
(1 − 2i)(2 − i) = −5i.

5+i −1 + 5i
En definitiva, E = = .
−5i 5

281
PROBLEMA 16.13
Expresar en forma binómica los siguientes complejos:
3 + 2i
a) (4 + 3i)3 , b) ,
3 − 2i
1 (4 − 3i)(2 + 3i)
c) , d) .
(4 + 2i)(3 − 2i) 5 − 3i

Solución
a) Por la fórmula de Newton,

(4 + 3i)3 = 1 · 43 + 3 · 42 (3i) + 3 · 4(3i)2 + (3i)3


= 64 + 144i − 108 − 27i = −44 + 117i.

b) Multiplicando y dividiendo por el conjugado del denominador,


3 + 2i (3 + 2i)2
=
3 − 2i (3 − 2i)(3 + 2i)
9 + 12i − 4 5 12
= = + i.
9+4 13 13

c) Operando como en el apartado anterior,


1 1
=
(4 + 2i)(3 − 2i) 12 − 8i + 6i + 4
1 16 + 2i 4 i
= = 2 2
= + .
16 − 2i 16 + 2 65 130

d) Operamos nuevamente como en los casos anteriores,


(4 − 3i)(2 + 3i) 8 + 12i − 6i + 9
=
5 − 3i 5 − 3i
17 + 6i (17 + 6i)(5 + 3i) 67 81
= = = + i.
5 − 3i 34 34 34

PROBLEMA 16.14
Escribir en la forma trigonométrica los siguientes números com-
plejos:

a) −1, √ b) √1 + i,
c) 1 − 3i, d) 3 + i,
e) −i, f) −5 − 12i.

282
Solución
a) Como | − 1| = 1 y arg(−1) = π, entonces

−1 = cos π + i sen π.


b) Como |1 + i| = 2 y arg(1 + i) = π/4,

1 + i = 2(cos π/4 + i sen π/4).

√ √ √
c) El módulo√y argumento son |1 − 3i| = 1 + 3 = 2, arg(1 − 3i) =
arc tg(− 3) = −π/3. Ası́:

1 − 3i = 2[cos(−π/3) + i sen(−π/3)].

√ √ √ √
d) En este caso, | 3+i| = 3 + 1 = 2 y arg( 3+i) = arc tg(1/ 3) = π/6,
de donde, √
3 + i = 2[cos(π/6) + i sen(π/6)].

e) Es evidente que | − i| = 1 y arg(−i) = 3π/2, con lo que:

−i = cos(3π/2) + i sen(3π/2).


f) En este caso, | − 5 − 12i| = 25 + 144 = 13 y, como tg ϕ = 12/5, resulta
que arg(−5−12i) = π+arc tg(12/5), por ser negativas las componentes
del número complejo. En definitiva,

−5 − 12i = 13[cos(π + arc tg(12/5)) + i sen(π + arc tg(12/5))].

PROBLEMA 16.15
Expresar en la forma x + iy y r cis ϑ los complejos

1−i 3−i 3 − 4i
a) , b) √ , c) .
1+i 3+i 24 + 7i

Solución
1−i (1 − i)2 −2i
a) = 2
= = −i = cis(3π/2).
1+i 1−i 2
√ √ √ √
3−i ( 3 − i)2 2 − 2 3i 1 3
b) √ = 2
= = − i.
3+i 3−i 4 2 2

283
1 √3 √ !

1 3
Como − i = 1 y arg − i = −π/3, tenemos:

2 2 2 2
√ √
3−i 1 3
√ = − i = cis(−π/3).
3+i 2 2

3 − 4i (3 − 4i)(24 − 7i) 44 117


c) = 2 2
= − i.
24 + 7i 24 + 7 625 625
El módulo y argumento son
r 2
44 117 44 + 1172
625 − 625 i = = 0,2 y ϑ = arc tg(−117/44),

6252

de modo que

3 − 4i 44 117
= − i = 0,2 cis arc tg(−117/44).
24 + 7i 525 525

PROBLEMA 16.16
Representar en forma trigonométrica el número complejo
(1 + i)(1 − i)2 i
z= √ .
1 3
2 + 2 i

Solución

Por las propiedades del módulo, tenemos:


√ √
|1 + i| · |1 − i|2 · |i| 2 · ( 2)2 · 1 √
|z| = 1 √3
= = 2 2.
2 + 2 i 1

Como el argumento del producto es la suma de los argumentos,


√ !
1 3
arg z = arg(1 + i) + 2 arg(1 − i) + arg i − arg + i
2 2
= π/4 + 2(3π/2 + π/4) + π/2 − π/3 = 4π + π/4 − π/3 = 4π − π/12.


En definitiva, z = 2 2[cos(−π/12) + i sen(−π/12)].

284
PROBLEMA 16.17
Si z = cis ϑ = cos ϑ + i sen ϑ, expresar los números
1 1
a) , b) ,
1+z 1−z
2z 1−z
c) , d) .
1 − z2 1+z

Solución
a) Multiplicamos numerador y denominador por 1 + z:

1 1+ z 1 + cos ϑ − i sen ϑ 2 cos2 ϑ/2 − 2i sen ϑ/2 cos ϑ/2


= 2
= =
1+z |1 + z| (1 + cos ϑ)2 + sen2 ϑ 2(1 + cos ϑ)
2 cos ϑ/2(cos ϑ/2 − i sen ϑ/2) 1
= = (1 − i tg ϑ/2).
2 · 2 cos2 ϑ/2 2

b) Análogamente al apartado anterior,

1 1− z (1 − cos ϑ) + i sen ϑ 2 sen2 ϑ/2 + 2i sen ϑ/2 cos ϑ/2


= = =
1−z |1 − z|2 (1 − cos ϑ)2 + sen2 ϑ 2(1 − cos ϑ)
2 sen ϑ/2(sen ϑ/2 + i cos ϑ/2) 1
= = (1 + i cotg ϑ/2).
2 · 2 sen2 ϑ/2 2

c) Como en los casos anteriores, multiplicamos y dividimos por el conjugado


del denominador:
2z 2z(1 − z 2 ) 2z − 2z z 2 2z − 2 z
= = =
1 − z2 2
|1 − z | 2 |1 − z 2 |2 |1 − z 2 |2
2 cos ϑ + 2i sen ϑ − 2 cos ϑ + 2i sen ϑ 4i sen ϑ
= 2 2 2
=
|1 − cos ϑ + sen ϑ − 2i cos ϑ sen ϑ| |1 − cos 2ϑ − i sen 2ϑ|2
4i sen ϑ 4i sen ϑ
= 2 2
=
(1 − cos 2ϑ) + sen 2ϑ 1 + cos 2ϑ − 2 cos 2ϑ + sen2 2ϑ
2

4i sen ϑ 4i sen ϑ
= = = i cosec ϑ.
2(1 − cos 2ϑ) 4 sen2 ϑ

d) Procedemos como en los casos anteriores. Ası́:

1−z (1 − z)(1 + z) 1+ z−z−zz z−z


= = =
1+z |1 + z|2 |1 + z|2 |1 + z|2
cos ϑ − i sen ϑ − cos ϑ − i sen ϑ −2i sen ϑ −i sen ϑ
= 2
= 2
= .
4 cos ϑ/2 4 cos ϑ/2 1 + cos ϑ

285
PROBLEMA 16.18
3 − 2ai
Dado el número complejo z = , determinar el valor de a ∈ R
4 − 3i
para que el cociente sea real y calcular dicho cociente.

Solución
Si multiplicamos y dividimos por el conjugado del denominador,
 
(3 − 2ai)(4 + 3i) 12 + 9i − 8ai + 6a 12 + 6a 9 − 8a
z= = = + i.
25 25 25 25

9 − 8a
Para que sea real, debe cumplirse que = 0 con lo que a = 9/8 y
25
12 + 6(9/8)
entonces z = = 3/4.
25

PROBLEMA 16.19
Determinar dos números complejos cuya suma sea 4 y su producto
8.

Solución
Basta resolver el sistema z1 +z2 = 4, z1 z2 = 8. Como z2 = 4−z1 , al sustituir,
obtenemos:

2 4 ± −16
z1 (4 − z1 ) = 8 =⇒ z1 − 4z1 + 8 = 0 =⇒ z1 = = 2 ± 2i.
2
La respuesta es pues z1 = 2 + 2i, z2 = 2 − 2i.

PROBLEMA 16.20
Caracterizar
  los números complejos w que verifiquen la relación
a + bw
Im = Im w, (a, b, c, d ∈ R).
c + dw

Solución
Aplicando la fórmula 2i Im w = w − w, tenemos la siguiente ecuación:

286
 
a + bw a + bw a + b w
Im = Im w ⇐⇒ − =w− w
c + dw c + dw c + d w
ac + ad w + bcw + bdw w − ac − bdw w − adw − bc w
⇐⇒
|c + dw|2
(bc − ad)(w − w)
= = w − w ⇐⇒ bc − ad = |c + dw|2 ó w = w
|c + dw|2

⇐⇒ |dw + c| = bc − ad ó w ∈ R

bc − ad
⇐⇒ |w + c/d| = ó w ∈ R.
|d|

Como se observa por el resultado obtenido,


√ una solución es la circunferencia
bc − ad
de centro el punto −c/d y radio y otra solución es el eje real.
|d|

PROBLEMA 16.21
Determinar un número complejo a + ib cuyo cuadrado sea igual a
su conjugado.

Solución
La condición que se establece es la siguiente:

(a + ib)2 = (a2 − b2 ) + 2iab = a − ib.

Igualando las partes reales e imaginarias, resulta el sistema de ecuacio-


nes:
a2 − b2 = a, 2ab = −b,

que, al resolver, da lugar a

b = 0 =⇒ a2 = a =⇒ a = 0 ó a = 1;
b 6= 0 =⇒ 2a = −1 =⇒ a = −1/2 =⇒ 1/4 − b2 = −1/2

=⇒ b2 = 3/4 =⇒ b = ± 3/2.

En definitiva, se obtienen las cuatro soluciones


√ √
0, 1 + 0i, −1/2 + i 3/2, −1/2 − i 3/2.

287
PROBLEMA 16.22
Hallar un par de enteros p y q tales que (3+7i)(p+iq) sea imaginario
puro.

Solución
Al hacer que la parte real sea cero obtenemos:

(3p − 7q) + (7p + 3q)i = λi =⇒ 3p − 7q = 0 =⇒ p = 7, q = 3,

pues, por hipótesis, p y q son enteros.

PROBLEMA 16.23
Determinar dos números complejos, sabiendo que su suma es 1+4i,
su cociente es imaginario puro y la parte real de uno de ellos es
−1.

Solución
Sean z1 = a + bi y z2 = c + di ambos números. Las condiciones propuestas
dan lugar a las siguientes ecuaciones:

(a + c) + (b + d)i = 1 + 4i;
a + bi
= λi;
c + di
a = −1.

De lo anterior se deduce que

a + c = 1 =⇒ c = 2;
b + d = 4;
(a + bi)(c − di) ac + bd cb − da
λi = = 2 + 2 i
c2 + d2 c + d2 c + d2
ac + bd
=⇒ 2 = 0 =⇒ ac + bd = 0 =⇒ bd = 2.
c + d2

Del √ 2, b + d = 4,√se obtienen las soluciones b = 2 − 2, d =
sistema bd = √
2 + 2 y b = 2 + 2, d = 2 − 2. En definitiva, las soluciones son
√ √
z1 = −1 + (2 − 2)i, z2 = 2 + (2 + 2)i;
√ √
z1 = −1 + (2 + 2)i, z2 = 2 + (2 − 2)i.

288
PROBLEMA 16.24
Hallar la relación que debe existir entre los parámetros a, b, c y d
para que la ecuación de coeficientes complejos z 2 + (a + ib)z + (c +
id) = 0 tenga una raı́z imaginaria pura.

Solución
Sean x1 y x2 las raı́ces de la ecuación dada y escribimos x1 = im, x2 = p+qi.
De la relación entre los coeficientes y las raı́ces, resulta el sistema:

p = −a (1)
x1 + x2 = p + (m + q)i = −(a + bi) =⇒
m + q = −b (2)

−mq = c (3)
x1 · x2 = −mq + mpi = c + di =⇒
mp = d (4)

Del sistema resultante eliminamos los parámetros m, p y q, con lo que obte-


nemos:
De (1) y (4):
m = d/p = −d/a. (5)

De (3) y (5):
q = −c/m = −c · (−a/d) = ac/d.
Sustituyendo estas dos últimas expresiones en (2):

−d/a + ac/d = −b

lo que implica que −d2 + a2 c = −abd.

PROBLEMA 16.25
¿Bajo qué condiciones el cociente de un número complejo con
su conjugado es un número real? ¿Qué condición deben cumplir
dos números complejos para que su cociente sea imaginario puro?

Solución
Si llamamos z = a + ib, la condición z/ z ∈ R equivale a:

z z z2 a2 − b2 + 2abi
· = 2 = ∈ R ⇐⇒ 2abi = 0 ⇐⇒ a = 0 ó b = 0.
z z |z| a2 + b2

289
Para que el cociente de dos complejos sea imaginario puro basta que la
diferencia de los argumentos entre el dividendo y el divisor sea de 90◦ . De
otra forma, como

a + bi (a + bi)(c − di) ac + bd (bc − ad)i


= 2 2
= 2 + 2 ,
c + di c +d c + d2 c + d2
el cociente será imaginario puro cuando ac + bd = 0.

PROBLEMA 16.26
En el cuerpo de los complejos, resolver el sistema:

(1 + i)z − iu = 2 + i
(2 + i)z + (2 − i)u = 2i.

Solución
Restando miembro a miembro ambas ecuaciones, obtenemos que z + 2u =
−2 + i, de donde z = −2 + i − 2u.
Al sustituir este resultado en la primera ecuación, resulta:

(1 + i)(−2 + i − 2u) − iu = 2 + i =⇒ (−2 − 3i)u = 2 + i + (1 + i)(2 − i)


5 + 2i (5 + 2i)(−2 + 3i)
=⇒ u = =⇒ u =
−2 − 3i (−2 − 3i)(−2 + 3i)
16 11
=⇒ u = + i.
13 13

De aquı́ se obtiene que el valor de z es:

16 + 11i 13(−2 + i) − 32 − 22i −58 − 9i


z = −2 + i − 2u = −2 + i − 2 · = = .
13 13 13

PROBLEMA 16.27
Sea z = a + ib, a, b ∈ R. Demostrar que existen m, n, p ∈ R tales que
(a2 + b2 )(a2 + b2 + 2a + 1)
= p + mz + nz 2 .
a2 − b2 + a − (1 + 2a)bi

290
Solución
Teniendo en cuenta que a2 + b2 = z z y que z 2 = a2 − b2 − 2abi, podemos
escribir:
(a2 + b2 )(a2 + b2 + 2a + 1) z z(z z + z + z + 1)
=
a2 − b2 + a − (1 + 2a)bi z2 + z
z z(z + 1)( z + 1)
= = z(z + 1) = z 2 + z.
z( z + 1)

Haciendo pues p = 0, m = 1, n = 1, se obtiene lo deseado.

PROBLEMA 16.28
Expresar el producto (x2 + 1)(y 2 + 1)(z 2 + 1) como suma de dos
cuadrados.

Solución
Llamando z1 = x + i, z2 = y + i, z3 = z + i, podemos escribir:

(x2 + 1)(y 2 + 1)(z 2 + 1) = |z1 |2 |z2 |2 |z3 |2 = |z1 z2 z3 |2 .

Si escribimos en forma binómica el producto z1 z2 z3 , obtenemos:

z1 z2 z3 = (x + i)(y + i)(z + i) = (xyz − x − y − z) + i(xy + yz + zx − 1),

con lo que

|z1 z2 z3 |2 = (xyz − x − y − z)2 + (xy + yz + zx − 1)2 .

PROBLEMA 16.29
Calcular la suma de las siguientes progresiones:
a) 1 − i + i2 − i3 + · · · + (−i)n .
b) 1 − 2i + 4i2 − 8i3 + · · · + (−2i)n−1 .
c) 1 + 2i + 3i2 + 4i3 + · · · + nin−1 .

291
Solución
a) Se trata de una progresión geométrica de razón −i y cuya suma es:

1 − (−i)n+1 [1 − (−i)n+1 ](1 − i) 1 − (−i)n+1 − i − (−i)n+2


S= = = .
1 − (−i) (1 + i)(1 − i) 2
En concreto,

1−i+i+1


 2 =1 si n = 4k
 1−i+1−i = 1 − i

si n = 4k + 1
2
S = 1−i−i−1
 2

 = −i si n = 4k + 2
 1−i−1+i = 0

si n = 4k + 3.
2

b) En este caso, se trata de una progresión geométrica de razón −2i y su


suma es
1 − (−2i)n [1 − (−2i)n ](1 − 2i) 1 − 2i − (−2i)n − (−2i)n+1
S= = = .
1 − (−2i) (1 + 2i)(1 − 2i) 5

c) Llamando Sn a la suma de la progresión, si multiplicamos por i y resta-


mos, resulta:

Sn = 1 + 2i + 3i2 + 4i3 + · · · + nin−1


iSn = i + 2i2 + 3i3 + 4i4 + · · · + nin
1 − in
Sn (1 − i) = 1 + i + i2 + i3 + · · · + in−1 − nin = − nin ,
1−i
1 − in nin
=⇒ Sn = − .
(1 − i)2 1 − i

PROBLEMA 16.30
Factorizar los siguientes polinomios:
a) x2 − 2x + 10.
b) a2 − 8ab + 20b2 .
c) z 2 + z + 1 + i.

Solución
a) Teniendo en cuenta la fórmula x2 + y 2 = (x + iy)(x − iy), escribimos la
expresión dada como suma de cuadrados:

x2 −2x+10 = x2 −2x+1+9 = (x−1)2 +32 = [(x−1)+3i][(x−1)−3i].

292
b) Procediendo análogamente al apartado anterior,

a2 − 8ab + 20b2 = a2 − 8ab + 16b2 + 4b2 = (a − 4b)2 + (2b)2


= [(a − 4b) + 2bi][(a − 4b) − 2bi].

c) Debido a que z 2 + 1 = (z + i)(z − i), tenemos:

z 2 + z + 1 + i = (z + i)(z − i) + z + i = (z + i)(z − i + 1).

PROBLEMA 16.31
Descomponer el polinomio P = 2x3 −(5+6i)x2 +9ix+1−3i, sabiendo
que admite una raı́z real.

Solución
Llamamos α a dicha raı́z real. Al sustituirla en la ecuación y separar en
parte real e imaginaria, obtenemos:

2α3 − 5α2 + 1 + i(−6α2 + 9α − 3) = 0.

Esto indica que α es raı́z común de las ecuaciones 2α3 − 5α2 + 1 = 0 y


−6α2 + 9α − 3 = 0. Resolviendo este sistema se obtiene que α = 1/2 y
dividiendo P por el factor 2x − 1, resulta en definitiva que:

P = (2x − 1)(x − 2i − 1)(x − i − 1).

PROBLEMA 16.32
Probar que cualquier raı́z de la ecuación z 3 +3z+5 = 0 tiene módulo
mayor que la unidad.

Solución
Si α es raı́z de la ecuación, α3 + 3α = −5 =⇒ |α3 + 3α| = | − 5| = 5.
Suponiendo que |α| ≤ 1, por la desigualdad triangular,

5 = |α3 + 3α| ≤ |α|3 + 3|α| ≤ 1 + 3 = 4,

lo que es absurdo. Esto indica que efectivamente |α| > 1.

293
PROBLEMA 16.33
Mediante la transformación√w = az + b los números z1 = 6 + 2i,
z2 = 8 + 2i y z3 = 7 + (2 + 5√ 3)i se transforman en w1 = −2 + 5i,
w2 = −2 + 7i y w3 = −(2 + 5 3) + 6i, respectivamente. Hallar a y b.

Solución
Aplicando la trasformación a z1 y z2 , tenemos:

w1 = az1 + b =⇒ −2 + 5i = a(6 + 2i) + b; (1)


w2 = az2 + b =⇒ −2 + 7i = a(8 + 2i) + b. (2)

Restando estas ecuaciones, tenemos que −2i = −2a =⇒ a = i. Al sustituir


este valor en (2):

−2 + 7i = 8i − 2 + b =⇒ b = −i.

La transformación buscada es entonces w = i(z − 1) y se puede comprobar


que también transforma z3 en w3 .

B. POTENCIA Y RAÍZ DE NÚMEROS COMPLEJOS.

Debido a la fórmula eix = cos x + i sen x, podemos representar todo número


complejo z = r(cos ϕ + i sen ϕ) en forma exponencial como z = reiϕ , lo
que permitirá definir en forma sencilla las operaciones de potencia y raı́z de
números complejos.
A partir de las operaciones de suma y producto ya definidas, tenemos las
siguientes fórmulas para calcular la potencia y raı́z n-ésimas de números
complejos.
1) Fórmula de Moivre. Dado z = r(cos ϕ+i sen ϕ) ∈ C, si n ∈ N, entonces
z n = rn (cos nϕ + i sen nϕ), o bien z n = rn einϕ .

2) Dado z = r(cos ϕ + i sen ϕ) ∈ C, si n ∈ N, entonces n z tiene exactamente
n soluciones:

n

n
ϕ + 2kπ
z= r(cos ϑ + i sen ϑ), donde ϑ = , k = 0, 1, . . . , n − 1.
n

294
Gráficamente, el resultado anterior indica que la raı́z n-ésima de un núme-
ro complejo tiene n soluciones cuyos afijos son los vértices de un polı́gono
regular.

PROBLEMA 16.34
Demostrar la fórmula de Moivre: Si z = r(cos ϕ + i sen ϕ) ∈ C,
entonces z n = rn (cos nϕ + i sen nϕ), ∀n ∈ N.

Solución
Aplicaremos el método de inducción.
Si n = 1, es evidente pues z 1 = z = r(cos ϕ + i sen ϕ).
Si suponemos que la fórmula es cierta para un cierto n, probaremos que
z n+1 = rn+1 [cos(n + 1)ϕ + i sen(n + 1)ϕ]:

z n+1 = z n · z = rn (cos nϕ + i sen nϕ) · r(cos ϕ + i sen ϕ)


= rn+1 (cos nϕ · cos ϕ − sen nϕ · sen ϕ) + i(cos nϕ · sen ϕ + sen nϕ · cos ϕ)
 

= rn+1 [cos(n + 1)ϕ + i sen(n + 1)ϕ].

PROBLEMA 16.35
Probar que todo número complejo z = r(cos ϕ + i sen ϕ) tiene exac-
tamente n raı́ces de orden n:

n
√ ϕ + 2kπ
z = n r(cos ϑ + i sen ϑ), donde ϑ = , k = 0, 1, . . . , n − 1.
n

Solución
Supongamos que w = s(cos ϑ + i sen ϑ) es una raı́z n-ésima de z. Entonces,
por la fórmula de Moivre,

wn = z ⇐⇒ sn (cos nϑ + i sen nϑ) = r(cos ϕ + i sen ϕ).

Al igualar los módulos y argumentos, tenemos:



sn = r =⇒ s = r; n

ϕ + 2kπ
nϑ = ϕ + 2kπ =⇒ ϑ = , para algún k ∈ Z.
n

295
Para ver el número de raı́ces que se obtienen, habrá que determinar cuántos
ϕ + 2kπ
valores de ϑ distintos se obtienen. Para eso basta elegir k tal que
n
esté comprendido entre 0 y 2π:

ϕ + 2kπ
0≤ < 2π ⇐⇒ −ϕ ≤ 2kπ < −ϕ + 2nπ
n
−ϕ −ϕ
⇐⇒ −1 < ≤k< + n < n ⇐⇒ 0 ≤ k < n.
2π 2π

PROBLEMA 16.36
¿Es cierta en el campo complejo la implicación x3 = y 3 =⇒ x = y ?
¿Puede ser real la raı́z de un número complejo con parte imagina-
ria no nula?

Solución
En el campo complejo, a diferencia del real, no es cierta la implicación
x3 = y 3 =⇒ x = y debido a que la raı́z cúbica tiene tres soluciones.

Tampoco es cierto que, si z = a + bi con b 6= 0, entonces n z ∈ R porque la
potencia n-ésima de un número real es siempre real.

PROBLEMA 16.37
Si z = eiϑ , demostrar que z n + z −n = 2 cos nϑ y z n − z −n = 2i sen nϑ.

Solución
Como |z| = 1, z· z = 1 =⇒ z = 1/z. Además z n = z −n . Entonces z n +z −n =
z n + z n = 2 Re(z n ).
Ahora bien, como z n = einϑ = cos nϑ + i sen nϑ, Re(z n ) = cos nϑ, luego
z n + z n = 2 cos nϑ.
Análogamente, como Im(z n ) = sen nϑ, resulta que

z n − z −n = z n − z n = 2i Im(z n ) = 2i sen nϑ.

296
PROBLEMA 16.38

Calcular la sexta potencia de z = 2 + 2i/ 3.

Solución
- Forma binómica. Aplicamos la fórmula del binomio de Newton:
√ √ √ √
(2 + 2i/ 3)6 = 26 + 6 · 25 · 2i/ 3 + 15 · 24 · (2i/ 3)2 + 20 · 23 · (2i/ 3)3
√ √ √ 4096
+15 · 22 · (2i/ 3)4 + 6 · 2 · (2i/ 3)5 + (2i/ 3)6 = − .
27

- Forma trigonométrica. Como



√ 2/ 3 √
|z| = (4/ 3) y tg ϑ = b/a = = 1/ 3 =⇒ ϑ = π/6,
2
deducimos, por la fórmula de Moivre, que
 6
6 4 4096
z = √ (cos(π/6) + i sen(π/6) = (cos π + i sen π).
3 27

PROBLEMA 16.39
Desarrollar las siguientes expresiones:
a) (1 + i)n , n ∈ Z+ .
b) (x − i)5 , x ∈ R.

Solución
a) Aplicando la fórmula del binomio de Newton,
         
n n n 2 n n n
(1 + i)n = + i+ i + ··· + in−1 + i
0 1 2 n−1 n
             
n n n n n
= 1− + − + ... + i n − + − ... .
2 4 6 3 5

b) Aplicando la misma fórmula anterior,


         
5 5 5 5 4 5 3 2 5 2 3 5
(x − i) = x + x (−i) + x (−i) + x (−i) + x(−i)4
0 1 2 3 4
 
5
(−i)5 = x5 − 10x3 + 5x + i −5x4 + 10x2 − 1 .
   
+
5

297
PROBLEMA 16.40
 10
1+i
Calcular el módulo y el argumento de w = .
1−i

Solución
Por la fórmula de Moivre,

1 + i 10 1 + i 10 |1 + i| 10 √ √ 10
|w| = = = = ( 2/ 2) = 1.

1−i 1 − i |1 − i|
Como además

(1 + i)(1 + i) 10 1 + i + i − 1 10
   
w= = = i10 = −1,
(1 − i)(1 + i) 2

resulta que arg w = π.

PROBLEMA 16.41
Calcular z = (1 + i)n y escribirlo en forma binómica para el caso
n = 25.

Solución

Como |1 + i| = 2 y arg(1 + i) = π/4, entonces

z = 2n/2 (cos πn/4 + i sen πn/4).



Si n = 25, cos 25π/4 = cos(6π+π/4) = cos π/4 = 2/2 ası́ como sen 25π/4 =

2/2. Resulta entonces que
√ √
(1 + i)25 = 225/2 ( 2/2 + i 2/2) = 212 (1 + i).

PROBLEMA 16.42
 6
1 + sen α + i cos α
Simplificar la expresión .
1 + sen α − i cos α

298
Solución

Multiplicando y dividiendo por el conjugado del denominador y teniendo en


cuenta que sen α = cos(π/2 − α) y cos α = sen(π/2 − α), resulta:

1 + sen α + i cos α (1 + sen α + i cos α)(1 + sen α + i cos α)


=
1 + sen α − i cos α (1 + sen α)2 + cos2 α
(1 + sen α)2 − cos2 α + 2i cos α(1 + sen α)
=
2(1 + sen α)
2
(1 + sen α) − (1 − sen α)(1 + sen α) + 2i cos α(1 + sen α)
=
2(1 + sen α)
(1 + sen α) − (1 − sen α) + 2i cos α
= = sen α + i cos α
2
= cos(π/2 − α) + i sen(π/2 − α) = cis(π/2 − α).

De este modo, la base tiene módulo 1 y argumento π/2 − α, con lo que la


potencia n-ésima será cis n(π/2 − α) y, en particular, para n = 6:

cis 6(π/2 − α) = cis(3π − 6α) = cos(3π − 6α) + i sen(3π − 6α)


= cos(π − 6α) + i sen(π − 6α) = − cos 6α + i sen 6α.

PROBLEMA 16.43
Calcular [(sen a − sen b) + i(cos a − cos b)]n .

Solución

Aplicando las fórmulas

a+b a−b a+b a−b


sen a − sen b = 2 cos sen y cos a − cos b = −2 sen sen ,
2 2 2 2

resulta:

[(sen a − sen b) + i(cos a − cos b)]n


a−b n
 
a+b a−b a+b
= 2 cos · sen − 2i sen · sen
2 2 2 2
 
a−b a+b
= 2n senn cis −n · .
2 2

299
PROBLEMA 16.44
Calcular sen 3ϕ en función de sen ϕ.

Solución
Debido a la fórmula de Moivre sabemos que (cos ϕ + i sen ϕ)3 = cos 3ϕ +
i sen 3ϕ. Desarrollando el primer miembro de la igualdad anterior, e igua-
lando las partes real e imaginaria, obtenemos:

(cos ϕ + i sen ϕ)3 = cos3 ϕ + 3i cos2 ϕ sen ϕ + 3i2 cos ϕ sen2 ϕ + i3 sen3 ϕ
= cos3 ϕ − 3 cos ϕ sen2 ϕ + i(3 cos2 ϕ sen ϕ − sen3 ϕ);
=⇒ cos 3ϕ = cos3 ϕ − 3 cos ϕ sen2 ϕ,
sen 3ϕ = 3 cos2 ϕ sen ϕ − sen3 ϕ
= 3(1 − sen2 ϕ) sen ϕ − sen3 ϕ = 3 sen ϕ − 4 sen3 ϕ.

Análogamente podemos expresar cos 3ϕ en función de cos ϕ.

PROBLEMA 16.45
1
Sabiendo que 2 cos α = z + , obtener 2 cos nα.
z

Solución
Despejando z de la ecuación, tenemos:
1
z+ = 2 cos α =⇒ z 2 − 2z · cos α + 1 = 0
z p p
=⇒ z = cos α ± cos2 α − 1 = cos α ± − sen2 α = cos α ± i sen α.

Operamos en primer lugar con el signo +; el inverso de z es:


1 1 cos α − i sen α
= = = cos(−α) + i sen(−α).
z cos α + i sen α cos2 α + sen2 α
Aplicando la fórmula de Moivre, obtenemos:

z n = (cos α + i sen α)n = cos nα + i sen nα,


1
= [cos(−α) + i sen(−α)]n = cos(−nα) + i sen(−nα) = cos nα − i sen nα.
zn

300
Sumando estas dos últimas igualdades, resulta:

1
zn + = 2 cos nα.
zn
Operando con el signo − se llega al mismo resultado.

PROBLEMA 16.46
√ √
1+ 5
◦ ◦ 5−1
Probar que cos 36 = y cos 72 = .
4 4

Solución
Debido a la fórmula de Moivre,

(cos 36◦ + i sen 36◦ )5 = cos(5 · 36◦ ) + i sen(5 · 36◦ ) = cos π + i sen π = −1 + i · 0.

Llamaremos por comodidad c = cos 36◦ y s = sen 36◦ y calculamos la parte


real e imaginaria de (c + is)5 :

(c + is)5 = c5 + 5ic4 s + 10i2 c3 s2 + 10i3 c2 s3 + 5i4 cs4 + i5 s5


= (c5 − 10c3 s2 + 5cs4 ) + i(s5 − 10c2 s3 + 5c4 s).

De aquı́ se deduce que 0 = s5 − 10c2 s3 + 5c4 s y, usando el hecho de que


s2 = 1 − c2 , resulta:

0 = s5 − 10(1 − s2 )s3 + 5(1 − s2 )2 s = 16s5 − 20s3 + 5s


=⇒ 0 = s(16s4 − 20s2 + 5) =⇒ 16s4 − 20s2 + 5 = 0
√ √
2 20 ± 400 − 320 5± 5
=⇒ s = =
s 32 √
8
5± 5
=⇒ s = ± .
8

Debido a que ◦ ◦ ◦ ◦ ◦
√ 30 < 36 < 45 , se deduce que sen 30 < s < sen 45 , es decir
1/2 < s < 2/2. s De los valores obtenidos para s, el único que verifica esta
√ √ √
5− 5 2 2 5− 5 3+ 5
acotación es s = , de donde c = 1 − s = 1 − =
8 8 8
y
s √ s √ s √ √ √
3+ 5 6+2 5 1 + ( 5)2 + 2 5 1+ 5
c=+ =+ =+ = .
8 16 16 4

301
Por otra parte,

cos 72◦ = cos(2 · 36◦ ) = cos2 36◦ − sen2 36◦ = 2 cos2 36◦ − 1
√ √ √
(1 + 5)2 −2 + 2 5 −1 + 5
= −1= = .
8 8 4

PROBLEMA 16.47
sen 4ϑ
Demostrar que = 8 cos3 ϑ − 4 cos ϑ.
sen ϑ

Solución
Por la fórmula de Moivre, (cos ϑ + i sen ϑ)4 = cos 4ϑ + i sen 4ϑ.
Por otra parte, por la fórmula del binomio de Newton,

(cos ϑ + i sen ϑ)4 = cos4 ϑ + 4 cos3 ϑ(i sen ϑ) + 6 cos2 ϑ(i sen ϑ)2
+4 cos ϑ(i sen ϑ)3 + (i sen ϑ)4 = cos4 ϑ − 6 cos2 ϑ sen2 ϑ
+ sen4 ϑ + i(4 cos3 ϑ sen ϑ − 4 cos ϑ sen3 ϑ).

Igualando la parte imaginaria, resulta que

sen 4ϑ
sen 4ϑ = 4 cos3 ϑ sen ϑ − 4 cos ϑ sen3 ϑ =⇒ = 4 cos3 ϑ − 4 cos ϑ sen2 ϑ
sen ϑ
sen 4ϑ
=⇒ = 4 cos3 ϑ − 4 cos ϑ(1 − cos2 ϑ) = 8 cos3 ϑ − 4 cos ϑ.
sen ϑ

PROBLEMA 16.48
Expresar en forma binómica la suma
1 1 1
S =1+ + 2
+ ··· + .
1 + i (1 + i) (1 + i)28

Solución
Como los términos de la suma siguen una progresión geométrica, su suma
es
1 1
(1+i)29
−1 (1+i)29
−1 i
S= 1 = −i
= − i(1 + i).
1+i − 1 1+i
(1 + i)28

302
Por otra parte, debido a la fórmula de Moivre,

(1+i)28 = [ 2(cos π/4+i sen π/4)]28 = 214 (cos 28π/4+i sen 28π/4) = −214 .

Sustituyendo en el resultado anterior, se deduce que

S = −2−14 i − i − i2 = 1 − i(2−14 + 1).

PROBLEMA 16.49
1+i n 1−i n
   
Siendo f (n) = √ + √ , calcular f (1), f (2), f (3) y f (4),
2 2
y demostrar que f (n + 4) = −f (n).

Solución

1 + i 1+i 1−i 1+i
Como √ = 1, Arg √ = π/4 y √ = √ , entonces

2 2 2 2
 n
1+i nπ
f (n) = 2 Re √ = 2 cos .
2 4
En particular,
√ √
f (1) = 2, f (2) = 0, f (3) = − 2, f (4) = −2.

Por otra parte,


(n + 4)π  nπ 
f (n + 4) = 2 cos = 2 cos + π = −2 cos nπ/4 = −f (n).
4 4

PROBLEMA 16.50
√ √
Calcular 1 − i y 5 + 12i, expresando el resultado en la forma
binómica.

Solución

a) Si escribimos 1 − i en forma binómica, obtenemos:
√ √
1 − i = x+iy =⇒ |1−i| = 2 = |x+iy|2 = x2 +y 2 y 1−i = x2 −y 2 +2ixy.

De aquı́ se deduce el sistema de ecuaciones:



x2 − y 2 = 1, x2 + y 2 = 2, 2xy = −1,

303
cuya solución es
s√ s√
√ √ 2+1 2−1
2x2 = 2 + 1, 2y 2 = 2 − 1 =⇒ x = ± , y=± .
2 2

Como 2xy = −1, las únicas soluciones son


s√ s√ s√ s√
2+1 2−1 2+1 2−1
−i , − +i .
2 2 2 2

b) Análogamente al apartado anterior,



5 + 12i = x + iy =⇒ 5 + 12i = x2 − y 2 + 2xyi =⇒ x2 − y 2 = 5, 2xy = 12,

25 + 144 = x2 + y 2 =⇒ x2 + y 2 = 13,
2x2 = 18, 2y 2 = 8 =⇒ x = ±3, y = ±2.

Teniendo en cuenta que xy = 6, las soluciones son,

3 + 2i, −3 − 2i.

PROBLEMA 16.51
√ √
Hallar i y 1 + i.

Solución
a) Como el módulo y el argumento principal de i son |i| = 1 y Arg i = π/2,
√ √ π/2 + 2kπ
tenemos que | i| = 1 y arg i = , para k = 0, 1.
2
Las dos raı́ces son entonces
√ √
π π 2 5π 5π 2
y1 = cos +i sen = (1+i), y2 = cos +i sen =− (1+i).
4 4 2 4 4 2

b) En este caso, el módulo y el argumento principal de 1 + i son 2 y
arc tg 1 = π/4, respectivamente. Esto indica que el módulo y argu-
√ π/4 + 2kπ
mento de las raı́ces cuadradas de 1 + i son 4 2 y , (k = 0, 1),
2
respectivamente. Tenemos entonces:

4

4
y1 = 2(cos π/8 + i sen π/8), y2 = 2(cos 9π/8 + i sen 9π/8).

304
PROBLEMA 16.52
r
√ 1
Hallar −16 − 30i y escribir en forma binómica .
3 − 4i

Solución

a) Si llamamos −16 − 30i = x+iy, tenemos que −16−30i = x2 −y 2 +2xyi,
de donde x2 −y 2 = −16 y 2xy = −30. Al resolver el sistema, obtenemos
los valores x = −3, y = 5 y x = 3, y = −5, lo que da las dos soluciones
−3 + 5i y 3 − 5i.

b) Multiplicando y dividiendo por el conjugado del denominador, obtene-


mos:
r s r
1 3 + 4i 3 + 4i
x + iy = = = .
3 − 4i (3 − 4i)(3 + 4i) 25

3 + 4i
Elevando al cuadrado, = x2 − y 2 + 2xyi, lo que da lugar al sis-
25
tema
x2 − y 2 = 3/25, 2xy = 4/25.

Al resolver este sistema, obtenemos las soluciones (2+i)/5 y (−2−i)/5.

PROBLEMA 16.53

3+i
Calcular las raı́ces cúbicas de z = √ expresándolas en forma
− 3+i
módulo-argumental.

Solución
√ √ √ √
( 3 + i)(− 3 − i) −2 − 2 3i 1 3i
En forma binómica es z = √ √ = =− − .
(− 3 + i)(− 3 − i) 4 2 2
p
Como |z| = (1/4) √ + (3/4) = 1 y el afijo de z está en el tercer cuadrante,
arg z = π + arc tg 3 = 4π/3, podemos escribir z = cos(4π/3) + i sen(4π/3).
Las raı́ces cúbicas de z son

3
2kπ + 4π/3 2kπ + 4π/3
z = cos + i sen , k = 0, 1, 2.
3 3

Los valores que se obtienen son y1 = 14π/9 , y2 = 110π/9 , y3 = 116π/9 .

305
PROBLEMA 16.54
Hallar los números complejos cuyo cubo sea igual al cuadrado de
su conjugado.

Solución
Sea z = m(cos w + i sen w) el número buscado (y suponemos z 6= 0). De
acuerdo con la fórmula de Moivre, será z 3 = z 2 si y sólo si

m3 (cos 3w +i sen 3w) = m2 (cos 2w −i sen 2w) = m2 [cos(−2w)+i sen(−2w)].

Igualando sus módulos y argumentos, resulta que m3 = m2 y 3w = 2kπ −


2w, de lo que se deduce que m = 1 y 5w = 2kπ. De aquı́ resultan cinco
soluciones:
cos 0 + i sen 0, cos(2π/5) + i sen(2π/5),
cos(4π/5) + i sen(4π/5), cos(6π/5) + i sen(6π/5), cos(8π/5) + i sen(8π/5).
Dichas soluciones corresponden precisamente a las raı́ces de la ecuación z 5 −
1 = 0.

PROBLEMA 16.55
a) Efectuar la operación
s
[a(cos α + i sen α)]2 · [b(cos β + i sen β)]4
E= .
[c(cos γ + i sen γ)]3
b) Si en el apartado a) damos los valores a = 2, b = 3, c = 1, α = 0,
β = 30◦ , γ = 40◦ , hallar los posibles valores de E .

Solución
a) Teniendo en cuenta la fórmula de Moivre y el producto y cociente de
complejos en forma módulo-argumental, obtenemos:
r
a2 b4
E = [cos(2α + 4β − 3γ) + i sen(2α + 4β − 3γ)]
c3
ab2
 
2kπ + (2α + 4β − 3γ) 2kπ + (2α + 4β − 3γ)
= √ cos + i sen , k = 0, 1.
c c 2 2

b) Como 2α + 4β − 3γ = 0◦ y ab2 /c c = 18, es E = 18(cos kπ + i sen kπ) =
18 cos kπ.
Para k = 0, E = 18 y para k = 1, E = −18.

306
PROBLEMA 16.56
Sabiendo que la suma de las raı́ces n-ésimas de la unidad es igual
a cero, probar que
1 + cos 72◦ + cos 144◦ + cos 216◦ + cos 288◦ = 0;
sen 72◦ + sen 144◦ + sen 216◦ + sen 288◦ = 0.

Solución
Las raı́ces quintas de la unidad son cos(2kπ/5) + i sen(2kπ/5), para k =
0, 1, 2, 3, 4, es decir

1, cos 72◦ +i sen 72◦ , cos 144◦ +i sen 144◦ , cos 216◦ +i sen 216◦ , cos 288◦ +i sen 288◦ .

Como su suma es cero, también sus partes real e imaginaria son cero. De
aquı́ se deducen las igualdades propuestas.

PROBLEMA 16.57
Si z1 y z2 son las raı́ces de una ecuación de segundo grado con
coeficientes reales, demostrar que z1n + z2n es real para cualquier
entero n. Si la ecuación es en particular z 2 − 2z + 2 = 0, calcular
z1n + z2n .

Solución
Como los coeficientes son reales, tenemos los siguientes resultados posibles
en función del discriminante de la ecuación ∆:
i) Si ∆ > 0, hay dos soluciones reales distintas.
ii) Si ∆ = 0, hay una solución real doble.
iii) Si ∆ < 0, hay dos soluciones que son complejos conjugados.
Ası́ pues, si ∆ ≥ 0, como z1 , z2 ∈ R, es evidente que z1n + z2n ∈ R.
Si ∆ < 0, tenemos que

z 1 = z2 =⇒ z1n + z2n = z1n + z n1 = z1n + z1n = 2 Re z1n ∈ R.

Las raı́ces de la ecuación z 2 − 2z + 2 = 0 son



2± 4−8 2 ± 2i
z= = =⇒ z1 = 1 + i, z2 = 1 − i.
2 2

307
Para probar que α = z1n + z2n = (1 + i)n + (1 − i)n es real, basta comprobar
que α = α. Ahora bien,
n n
α = (1 + i) + (1 − i) = (1 − i)n + (1 + i)n = α,

de donde se deduce el resultado.

PROBLEMA 16.58
Si a y b son raı́ces de la ecuación z 2 sen2 ϑ − z sen 2ϑ + 1 = 0, de-
mostrar que an + bn = 2 cos nϑ cosecn ϑ.

Solución
Calculamos las raı́ces de la ecuación:
√ √
sen 2ϑ ± sen2 2ϑ − 4 sen2 ϑ sen 2ϑ ± 4 sen2 ϑ cos2 ϑ − 4 sen2 ϑ
z = 2
=
p2 sen ϑ 2 sen2 ϑ
2 2
sen 2ϑ ± 4 sen ϑ(cos ϑ − 1) sen 2ϑ ± 2i sen2 ϑ
= =
2 sen2 ϑ 2 sen2 ϑ
sen ϑ cos ϑ ± i sen ϑ2 cos ϑ ± i sen ϑ
= 2
= = cotg ϑ ± i.
sen ϑ sen ϑ

Llamamos entonces a = cotg ϑ + i y b = cotg ϑ − i, que son comple-


jos conjugados. Escritos en forma trigonométrica, como su módulo es r =
p
1 + cotg2 ϑ = cosec ϑ y el argumento de a es ϕ = arc tg(1/ cotg ϑ) = ϑ,
resulta que a = cosec ϑ(cos ϑ + i sen ϑ) y b = cosec ϑ(cos ϑ − i sen ϑ). Por la
fórmula de Moivre, sus potencias son

an = cosecn ϑ(cos nϑ + i sen nϑ), bn = cosecn ϑ(cos nϑ − i sen nϑ),

de donde an + bn = 2 cosecn ϑ cos nϑ como se querı́a probar.

PROBLEMA 16.59
Teniendo en cuenta que cos α = Re(eiα ) y recordando la fórmula de
la suma de los términos de una progresión geométrica, calcular en
Xk
forma simplificada C = 2n cos 3n.
n=1

308
Solución
k
X
Llamando S = 2n sen 3n, obtenemos:
n=1

C + iS = 2(cos 3 + i sen 3) + 22 (cos 6 + i sen 6) + · · · + 2k (cos 3k + i sen 3k)


2k+1 e3(k+1)i − 2e3i
= 2e3i + 22 · e6i + · · · + 2k e3ki =
2e3i − 1
2k+1 cos 3(k + 1) − 2 cos 3 + i[2k+1 sen 3(k + 1) − 2 sen 3]
= .
−1 + 2 cos 3 + 2i sen 3

Multiplicando numerador y denominador de la última expresión por el con-


jugado del denominador y separando la parte real del número complejo
resultante, se obtiene finalmente:

2k+2 cos 3k − 2k+1 cos 3(k + 1) + 2 cos 3 − 4


C= .
5 − 4 cos 3

PROBLEMA 16.60
Determinar las ecuaciones de segundo grado cuyas raı́ces son:
a) cis 120◦ , cis 240◦ .
b) cis π/n, cis(−π/n).
c) a(cos nϑ + i sen nϑ), a(cos nϑ − i sen nϑ).

Solución
a) Escribimos en primer lugar las raı́ces en forma binómica.

√ √
cis 120◦ = cis 2 · 60◦ = (cos 60 + i sen 60)2 = (1/2 + i 3/2)2 = −1/2 + i 3/2;
√ √
cis 240◦ = cis 2 · 120◦ = (−1/2 + i 3/2)2 = −1/2 − i 3/2.

El producto y la suma de dichas raı́ces son

cis 120 · cis 240 = cis 360 = 1 y cis 120 + cis 240 = −1/2 + −1/2 = −1,

con lo que la ecuación buscada es x2 + x + 1 = 0.


b) Debido a que

r1 = cis π/n = cos π/n + i sen π/n,


r2 = cis(−π/n) = cos(−π/n) + i sen(−π/n) = cos π/n − i sen π/n,

309
resulta que

r1 + r2 = 2 cos π/n y r1 · r2 = cis(π/n − π/n) = cis 0 = 1,

con lo que la ecuación es x2 − 2x cos π/n + 1 = 0.


c) Análogamente a los casos anteriores, observando que las raı́ces dadas r1
y r2 son complejos conjugados, tenemos que

r1 · r2 = a2 y r1 + r2 = 2a cos nϑ,

de modo que la ecuación pedida es x2 − 2ax cos nϑ + a2 = 0.

PROBLEMA 16.61
Resolver la ecuación z 4 + 16 = 0.

Solución
De z 4 + 16 = 0, se deduce que
p
z 4 = −16 = 16(cos π + i sen π) =⇒ z = 4
16(cos π + i sen π)

 
4 π + 2kπ π + 2kπ
= 16 cos + i sen ,
4 4

√ √
donde
√ k√= 0, 1, 2, 3.√Para √
cada valor √
de k se
√ obtiene z1 = 2+ 2i, z2 =
− 2 + 2i, z3 = − 2 − 2i y z4 = 2 − 2i.

PROBLEMA 16.62
Resolver las ecuaciones:
a) z 2 + 4z + 29 = 0.
b) z 2 + 2iz + 1 = 0.
c) z 4 + z 2 + 1 = 0.
d) 8z 3 − 12z 2 + 10z − 3 = 0.

Solución
a) Al despejar z, se obtiene directamente,

−4 ± 16 − 116 −4 ± 10i
z= = =⇒ z1 = −2 − 5i, z2 = −2 + 5i.
2 2

310
b) Como en el caso anterior,

−i ± −1 − 1 √ √
z= =⇒ z1 = i(−1 + 2), z2 = −i(1 + 2).
1

c) Si llamamos w = z 2 , la ecuación se escribe como w2 + w + 1 = 0 y sus


raı́ces son:
√ √ √ √
−1 ± 1 − 4 −1 ± i 3 −1 i 3 −1 i 3
w= = =⇒ w1 = + , w2 = − .
2 2 2 2 2 2
Las soluciones de la ecuación original son las raı́ces cuadradas de w1
y w2 . Como sus módulos y argumentos son:
p
|w1 | = |w2 | = 1/4 + 3/4 = 1, arg w1 = 2π/3, arg w2 = 4π/3,

sus raı́ces cuadradas son:


   
2π/3 + 2kπ 4π/3 + 2kπ
z1 = cis , z2 = cis , k = 0, 1,
2 2

que, en forma trigonométrica, dan las soluciones:

z11 = cos π/3 + i sen π/3 , z12 = cos 4π/3 + i sen 4π/3,
z21 = cos 2π/3 + i sen 2π/3 , z22 = cos 5π/3 + i sen 5π/3.

d) Dividimos la ecuación por 8 y hacemos el cambio z = z 0 + 1/2, y resulta:

1 3 3 1 2
     
3 5 3 5 1 3
z3 − z2 + z − = 0 ⇐⇒ z 0 + − z0 + + z0 + − =0
2 4 8 2 2 2 4 2 8
3 3 1 3 3 3 5 0 5 3
⇐⇒ z 03 + z 02 + z 0 + − z 02 − − z 0 + z + − =0
2 4 8 2 8 2 4 8 8
03 z0 0 02 1
⇐⇒ z + = 0 ⇐⇒ z z + =0
2 2
√ √
0 0 i 2 0 i 2
⇐⇒ z1 = 0, z2 = , z3 = − .
2 2

Al deshacer el cambio de variable, obtenemos en definitiva que:


√ √
z1 = 1/2, z2 = (1/2) + i 2/2, z3 = (1/2) − i 2/2.

PROBLEMA 16.63
Resolver 2z 2 + (−1 + i)z + 3 + i = 0.

311
Solución
Despejando z tenemos:
p √
−(−1 + i) ± (−1 + i)2 − 4 · 2(3 + i) 1 − i ± 1 − 1 − 2i − 24 − 8i
z = =
√ 4 4
1 − i ± −24 − 10i
= .
4

Si llamamos ahora −24 − 10i = x + iy, resulta:

−24 − 10i = (x + iy)2 =⇒ x2 − y 2 = −24, 2xy = −10.

Además, igualando
√ los módulos de ambos complejos, tenemos la ecuación
x2 + y 2 = 242 + 102 = 26.
El sistema x2 − y 2 = −24, x2 + y 2 = 26 tiene las cuatro soluciones x =
±1, y = ±5. Como se debe verificar además que 2xy = −10, las únicas
soluciones posibles son x = 1, y = −5 y x = −1, y = 5.
En definitiva,
1 − i ± (1 − 5i) 2 − 6i 1 3i 4i
z= =⇒ z1 = = − , z2 = = i.
4 4 2 2 4

PROBLEMA 16.64
Resolver la ecuación z 3 − 3iz − 5(1 + i) = 0, haciendo z = x + iy .

Solución
Al sustituir z = x + iy y separar el resultado en parte real e imaginaria,
resulta el sistema de ecuaciones:

x3 − 3xy 2 + 3y − 5 = 0, 3x2 y − y 3 − 3x − 5 = 0.

Para resolver este sistema, efectuamos la resta de ambas ecuaciones, con lo


que:

x3 + y 3 − 3xy 2 − 3x2 y + 3y + 3x = 0
⇐⇒ (x + y)(x2 − xy + y 2 ) − 3xy(x + y) + 3(x + y) = 0
⇐⇒ (x + y)(x2 + y 2 − 4xy + 3) = 0.

Al sustituir en la primera ecuación la condición x + y = 0, obtenemos la


ecuación 2x3 + 3x + 5 = 0 en la que x = −1 es la única solución real. Esto

312
produce la raı́z z1 = −1 + i de la ecuación dada. Efectuando la división,
se obtiene una ecuación de segundo grado cuyas raı́ces son z2 = 2 + i y
z3 = −1 − 2i.

PROBLEMA 16.65
Determinar los números reales a y b de manera que z = 1 + i sea
raı́z de la ecuación z 5 + az 3 + b = 0.

Solución
Debemos desarrollar la expresión Q = (1 + i)5 + a(1 + i)3 + b e igualarla a
cero:

Q = 1 + 5i + 10i2 + 10i3 + 5i4 + i5 + a + 3ai + 3ai2 + ai3 + b = 0


=⇒ (1 − 10 + 5 + a − 3a + b) + i(5 − 10 + 1 + 3a − a) = 0
=⇒ b − 2a = 4, 2a = 4 =⇒ a = 2, b = 8.

PROBLEMA 16.66
1+i
Resolver la ecuación = e2x .
1−i

Solución
Multiplicando numerador y denominador por 1 + i tenemos:

1+i (1 + i)2 2i π π
e2x = = = = i = cos + i sen = eiπ/2
1−i 1 − i2 2 2 2
=⇒ 2x = i(π/2 + 2kπ) =⇒ x = i(π/4 + kπ), k ∈ Z.

(Recordamos que, en el campo complejo, ea = eb =⇒ a = b + 2kπ, k ∈ Z.)

313
C. EJERCICIOS PROPUESTOS.

1. Escribir en la forma trigonométrica r(cos ϑ+i sen ϑ) los siguientes


números complejos:

a) 3 − 2 − i.
p √ !

q
2− 3 1
Resp.: z = 2 2 − 3 − −i· p √ .
2 2 2− 3


b) 2 − 1 + i.
√ !

q
2−1 i
Resp.: z = 4 − 2 2 p √ +p √ .
4−2 2 4−2 2

z1 − z2
2. Hallar el módulo y argumento de u = si |z1 | = |z2 | = 1,
1 − z 1 z2
z1 6= z2 .
Resp.: |u| = 1, arg u = arg z1 .

3. Simplificar las expresiones:


(a + i)3 − (a − i)3
a) .
(a + i)2 − (a − i)2
Resp.: (3a2 − 1)/2a.

1 + i 1 − 2i
b) − .
1−i 1+i
Resp.: (1 + 5i)/2.

(−2 + i)3
c) .
1 − 3i
Resp.: (−7 + i)/2.


−1 − i 2
d) √ .
( 2 − i)4

Resp.: (5 + i 2)/27.

314
4. Calcular (2 − 3i)4 .
Resp.: −119 + 120i.

√ √
5. Calcular ( 3 − i)n y ( 3 + i)n .
−nπ n nπ
Resp.: z1n = 2n cis , z2 = 2n cis .
6 6

6. Expresar como suma de cuadrados la expresión (a2 + b2 )(c2 + d2 ).


Resp.: (ac − bd)2 + (bc + ad)2 .

7. Hallar las ecuaciones de segundo grado cuyas raı́ces son:


√ √
a) 3 + 5, 3 − 5.
Resp.: x2 − 6x + 4 = 0.

b) −3 + i, −3 − i.
Resp.: x2 + 6x + 10 = 0.

√ √
c) 2 + 3i, 2 − 3i.
Resp.: x2 − 4x + 7 = 0.

8. Factorizar los siguientes polinomios:


a) 5x2 + 4y 2 .
√ √
Resp.: ( 5x + 2iy)( 5x − 2iy).

b) x2 + xy + y 2 .
 √ √ 
Resp.: (x + y/2) + 3iy/2][(x + y/2) − 3iy/2 .

9. Resolver las ecuaciones:


a) 4z 2 − 12z + 25 = 0.
Resp.: z1 = (3/2) + 2i, z2 = (3/2) − 2i.

b) z 2 + iz = 2.
√ √
Resp.: z1 = 7/2 − i/2, z2 = − 7/2 − i/2.

315
c) z 3 + 10z 2 + 37z + 42 = 0.
√ √
Resp.: z1 = −2, z2 = −4 + 5i, z3 = −4 − 5i.

10. Resolver las ecuaciones:


a) z 2 − (2 + 2i)z + (4 + 2i) = 0.
Resp.: z1 = 1 + 3i, z2 = 1 − i.

b) z 2 − (2 + 2i)z + 2i = 0.
Resp.: z1 = z2 = 1 + i.

11. Hallar las raı́ces cuartas de −i.


Resp.: r1 = cis 3π/8, r2 = cis 7π/8, r3 = cis 11π/8, r4 = cis 15π/8.


12. Calcular las raı́ces sextas de 4 3 + 4i.
√ √ √
Resp.: z1 = 2 cis π/36, z2 = 2 cis 13π/36, z3 = 2 cis 25π/36,
√ √ √
z4 = 2 cis 37π/36, z5 = 2 cis 49π/36, z6 = 2 cis 61π/36.

n+5


n
2
13. Calcular lı́m zn siendo zn = n+ 1+ i.
n→∞ n
Resp.: lı́m zn = 1 + e2 · i.
n→∞

316

Você também pode gostar